FF (Assignment Q)

Lakukan tugas rumah & ujian kamu dengan baik sekarang menggunakan Quizwiz!

Difference between pitting edema vs brawny edema vs weeping edema?

pitting = soft brawny = hard weeping = crying

What is systolic dysfunction?

Compromised contractile function of the ventricles causing reductions in the SV, CO, and EF. Pts with systolic dysfunction will usually present with compromised EF less than 40%

Which functional activities causes FAI?

Activities that involve hip flex and hip IR (sit/stand, tying shoes, pivot horse riding, kicking a ball)

What happens if you come back down to normal altitude from higher altitude? (coming back to the plains)

HR: normal BP: normal CO: increases SV: increases

Heberden's nodes vs Bouchard's nodes Which condition is it related to?

Heberden's = DIP enlargement Bouchard's = PIP enlargement OA

Rules for Biofeedback summary -electrode placement -intensity

Weak muscle -Wide electrode placement -High sensitivity "WHeak: wide/high" Tight/Constricted muscle -Close electrode placement -Low sensitivity

What are the shoulder flexion muscles?

Pec Major Anterior deltoid Coracobrachialis Biceps (long and short head)

What is atrial tachycardia?

-100-250 BPM -Around 2 atrial contractions (P waves) before ventricle contraction (QRS complex) (look at slide 24 on FF#9) -NOT AN EMERGENCY AS LONG AS VENTRICLES DOES ITS JOB

What are the scapular depressor muscles?

Pec major Pec minor Lower trap

What are the scapular protraction muscles?

Pec major Pec minor Serratus Anterior

What is norm ROM for toe out?

7-10 degrees

Decreased sensitivity to pain?

Hypalgesia

Runs well Can go upstairs foot over foot (reciprocal stair climbing) Active, restless, tantrums Jumps with 2 feet Catches large ball a. 2 years b. 3 years c. 4 years d. 5 years

a. 2 years

· What cord does the ulnar nerve come from?

o Medial cord

What are the 2 hallmark signs of Right sided heart failure?

jugular venous distention peripheral edema

· Which nerve roots does the axillary nerve come from?

o C5/6

What is Hypertensive crisis?

Systolic over 180 and/or diastolic over 120, with patients needing prompt changes in medication if there are no other indications of problems, or immediate hospitalization if there are signs or organ damage

Saddle: Effects of Forward/Backward position

Backward saddle position -greater work of HS muscle -less stress on the knee Forward saddle position -greater work of rec fem -greater patello-femoral stress

What causes heart failure?

Cardiac muscle dysfunction: altered systolic and/or diastolic activity of the myocardium that develops from an abnormality within the cardiac structure or function. Ex: HTN, CAD, cardiac dysrhythmias, valve abnormalities, pericardial pathology, cardiomyopathies

What is Right sided heart failure?

Direct insult to the RV caused by conditions that increase PA pressure. Increased pressure within the PA increases the afterload, placing greater demand on the RV and causing it to go into failure. Blood is not effectively pumped from the RV and backs up into the RA and venous vasculature, producing 2 hallmark peripheral signs (jugular venous distention and peripheral edema)

Which part of the lung is cleared in a R sidelying position on a DECLINED bed position arms at sides?

Left upper lobe: Singular segment

2 tests for Appendicitis?

McBurney's Point/Sign -palpate at RL quadrant -(+) pain felt at RL quadrant Rovsing's Sign -palpate LL quadrant -(+) pain felt at RL quadrant Blumberg Sign/Rebound tenderness -used for PERITONITIS -palpate RL quadrant and quickly release -(+) pain felt at RL quadrant

What are the scapular downward rotation muscles?

Rhomboid major/minor Levator scap Pec minor Mid trap "MR Linkin Park likes to go DOWN"

What are the downward rotation muscles?

Rhomboids Levator scap Pec minor Mid trap

Tests for high ankle sprain

Syndesmosis Squeeze (Hopkin's) Test -anterior tibiofibular ligament Kleiger's (ER Stress) Test -deltoid ligament -syndesmosis injury/high ankle injury if pain over anterior or posterior tibiofibular ligaments and interosseous membrane

Two DPT students are reviewing the literature for the effects of Swiss ball exercises on LBP outcomes . According to levels of evidence, which studies provide the BEST evidence for support of use of Swiss ball exercises? A. Clinical case report B. Multicenter randomized controlled trials C. Clinical case series D. Cross Sectional studies

B. Multicenter randomized controlled trials

A 45 year old female patient is being evaluated in an outpatient clinic by a physical therapist. The PT performs the test shown in the picture. A positive result of the test likely indicates the presence of which of the following conditions? (Look at image on slide 18 on FF#25) A. Radial collateral ligament tear B. Carpal tunnel syndrome C. De Quervain disease D. Restricted ulnar deviation

C. De Quervain disease ST is Finkelstein

78 year old patient has acute exacerbation of COPD. The patient has a FEV1 of 70% with FEV1/FVC<70% and has shortness of breathing during ambulation. According to the GOLD classification, the patient would MOST likely be classified as: A. Very severe B. Severe C. Moderate D. Mild

C. Moderate COPD GOLD Classification: FEV1 >80 = mild 50-80 = moderate 30-50 = severe <30 = very severe

A student is in the autonomous stage of the NPTE preparation. Which of the following combinations is MOST appropriate in this stage? A. Block practice/summary feedback B. Block practice/continuous feedback C. Random practice/summary feedback D. Random practice/continuous feedback

C. Random practice/summary feedback

A PT is performing cranial nerve testing on a 55-year-old male patient. When observing the patient's right pupillary response to light, the PT observes that there is no constriction of the right pupil but the left pupil does constrict. Which cranial nerve is the MOST likely cause of this impairment? A. Left Oculomotor nerve (CN III) B. Right Optic nerve (CN II) C. Right Oculomotor nerve (CN III) D. Left Trochlear nerve (CN IV)

C. Right Oculomotor nerve (CN III) -1 pupil constricts so its not optic nerve issue (take out B) -No constriction in the same eye that is tested so its a ipsilateral issue (R CN3 issue)

Nerve roots associated with each DTR reflex?

C5/6 = Biceps C5/6 = Brachioradialis C7/8 = Triceps L2-4 = Patellar L5/S1 = Medial hamstrings S1/2 = Achilles

A 50-year-old male fainted in the office and was transferred to the ER for further evaluation. The medical history includes DM II and HTN. The arterial blood gas results are as follows: Blood pH - 7.43, PaCO2 - 28 mmHg and HCO3 - 18 mEq/L. This is known as: A. Partially compensated respiratory alkalosis. B. Uncompensated respiratory alkalosis. C. Compensated metabolic alkalosis. D. Compensated respiratory alkalosis.

D. Compensated respiratory alkalosis. pH is within normal = compensated

A patient complains of constant back pain that radiates upward toward the thorax and anteriorly into the abdominal region. MRI and X-ray show negative findings. The PT should suspect potential source of the pain is due to? A. Disc herniation B. Spondylolisthesis C. Diaphragm D. Kidney

D. Kidney -Diaphragm referes pain to the L shoulder and doesn't refer pain anteriorly

What is FEV1 and what % is normal from total FVC

Forced expiratory volume in 1 second 70% or more from total FVC

What are the ROM norms for each joint (hip/knee/ankle) during gait?

Hip: Stance = 0-30 flex 0-20 hyperext Swing = 20-30 flex Knee: Stance = 0-40 flex Swing = 0-60 flex Ankle: Stance = 0-10 DF, 0-20 PF Swing = 0-10 PF Running ROM = 1.5-2 xROM

Decreased sensitivity to sensory stimuli?

Hypesthesia

What is de Quervain's tenosynovitis? Which special test is used to dx?

Inflammation/degeneration of Extensor Pollicis Brevis and Abductor Pollicis Longus tendons at first dorsal compartment "Apple Eats Peanut Butter" Finkelstein: tuck in thumb and wrist ulnar deviation

Knee Ligament Orientation

Summary of ligament Orientation in the KNEE: - ACL: Oblique superior and laterally - PCL: Oblique medially and anterior-superiorly - LCL: Oblique inferior and posteriorly - MCL: Oblique anterior and inferior

What is the function of CN9?

Taste from posterior tongue Sensation from posterior tongue Sensation from oropharynx Salivation (parotid gland)

What is the function of CN7?

Taste of anterior tongue Muscles of facial expression Dampens sound (stapedius) Tearing (lacrimal gland) Salivation (submandibular and sublingual glands)

Pressure in palm on ulnar side of hand causes flexion of fingers causing strong grip a. Palmar grasp reflex b. Plantar grasp reflex c. Rooting reflex d. Moro reflex

a. Palmar grasp reflex 28 weeks of gestation to 9 months

Rides tricycle Stands on 1 foot briefly Jumps off step Hops on one foot Gallops Kicks ball Understands sharing Climbs on playground equipment a. 2 years b. 3 years c. 4 years d. 5 years

b. 3 years

Rotation of head causes extremity of face side to ext and extremities on skull side to flex a. Symmetrical tonic neck reflex b. Asymmetrical tonic neck reflex c. Tonic labyrinthine reflex d. Galant reflex

b. Asymmetrical tonic neck reflex Birth to 6 months

A patient complains of persistent wrist pain after painting a house few weeks ago. The patient demonstrates signs and symptoms consistent with de Quervain's tenosynovitis. An appropriate special test to confirm the diagnosis is a. Phalen's test. b. Finkelstein's test. c. Froment's sign. d. Cozen's test.

b. Finkelstein's test. Finkelstein's test is used to detect stenosing tenosynovitis of the APL and EPB. Phalen's Test is used to detect carpel tunnel syndrome. Froment's sign is used to examine weakness in the AP and short head of the FPB muscles, indicates an ulnar nerve entrapment at the elbow or at the wrist. (grabbing paper with thumb) Cozen's test is used to examine lateral epicondylitis at the elbow.

Begins to walk unassisted Transitions in/out of squatting Creeps up/down steps Transitions floor to stand Begins self-feeding Reaches with supination, neat pincer grasp, can release, build a tower of two cubes Searches for hidden toys Suspicious of strangers Plays patty-cake and peekaboo Imitates a. 7 months b. 8-9 months c. 10-15 months d. 20 months

c. 10-15 months

Hops on 1 foot several times Stands on tiptoes Relates to friends a. 2 years b. 3 years c. 4 years d. 5 years

c. 4 years

Pusher Syndrome Which side would the pt be leaning towards? Which side should the PT sit on with treating the impairment?

-Use good side to lean TOWARD the weaker side -body leaning toward the more affected side Intervention -PT can sit on pt's LESS INVOVLED side and instruct the pt to "LEAN OVER TO ME" so you have the pt actively do it -pt can be positioned with the LESS INVOLVED side next to a wall and instruct the pt to "LEAN TOWARD THE WALL" -The pt should be fully involved in problem solving -Ex: PT should ask questions like "what direction are you tilted? and what direction do you have to move in order to achieve vertical?"

Symptoms of Dementia

-impaired higher level functions likes language, perception -personality changes -memory deficits -deterioration of judgement, orientation -confusion -normal consciousness

5 classifications of Hoehn and Yahr for PD?

1 = minimal or absent disability, unilateral symptoms 2 = minimal bilateral or midline involvement, no balance involvement 3 = impaired balance, some restrictions in activity 4 = all symptoms present and severe; stands and walks only with assistance 5 = confinement to bed or wheelchair "1 day Hoehn and Yahr were walking and Hoehn noticed his 1 arm started hurting" "2nd day he noticed both arms hurting" "3rd day he noticed his balance was off" "4th day he needed assistance from Yahr with standing and walking" "5th day his condition was so bad, he needed to be confined to a bed/WC"

Inability to recognize weight?

Abarognosis

Pain produced by a non-noxious stimulus?

Allodynia

Inspiration muscles?

External intercostals

Which nerve roots does the femoral nerve stem from?

L2-L4

Which nerve root affected with sluggish knee jerk DTR?

L3

Rule for knee bolt for AKA amputee?

LIME Lateral heal/ IR bolt Medial heal/ ER bolt

What are the shoulder IR muscles?

Pec major Lats Teres major Subscap

What does S3 and S4 heart sounds indicate?

S3 = CHF S4 = MI or HTN

Difference between lymphangitis/lymphadenitis/lymphadenopathy?

SUMMARY: Lymphangitis : Lymph vessels, Lymphadenitis: Inflammation of lymph nodes, Lymphadenopathy: Enlargement of lymph node

What is VO2 max?

peak oxygen uptake

Which nerves are responsible for the Corneal reflex (both afferent and efferent)?

sensory/afferent = Trigeminal nerve (CN 5) motor/efferent = Facial nerve (CN 7) S5M7

What is a stenotic valve? What is a regurgitant valve?

stenotic valve has impaired opening regurgitant valve has impaired closing

The PT is reviewing the lab reports and sees the following values: HCO 3 (Bicarbonate ion) = 18.1 mEq /L, PaC02 (arterial carbon dioxide) = 39.9 mm Hg and pH = 7.20. The BEST diagnosis for this patient would be: A. Uncompensated Metabolic acidosis. B. Uncompensated Respiratory alkalosis. C. Uncompensated Respiratory acidosis. D. Uncompensated Metabolic alkalosis.

A. Uncompensated Metabolic acidosis.

A patient reports feeling off balanced when walking on a crowded beach during the day where he has to look out for people as he walks. The Physical Therapist should suspect that the patient would have difficulty in which of the following conditions during testing with the CTSIB? A. Condition 2 (ECSS) B. Condition 6 (VCMS) C. Condition 4 (EOMS) D. Condition 5 (ECMS)

B. Condition 6 (VCMS) Rationale: -walking on a beach in a CROWDED AREA involves visual conflict/sway-reference

The PT is observing the gait of a patient with right trans femoral amputation. The therapist notices excessive right plantar flexion at heel strike. What could be the possible cause for this foot slap? A. Plantar flexion bumper too rigid B. Plantar flexion bumper too soft C. Heel cushion is too rigid D. Excessive inset of the foot

B. Plantar flexion bumper too soft

A 30-year-old patient is unable to control descent when moving from standing to sitting position. Which of the following interventions would be MOST appropriate to address this problem? A. Hamstring strengthening with concentric contractions B. Quadriceps strengthening with eccentric contractions C. Quadriceps strengthening with concentric contractions D. Gluteus medius strengthening with eccentric contractions

B. Quadriceps strengthening with eccentric contractions

Following surgery of the right hip, a patient ambulates as shown in the picture. As part of the intervention, the PT opts to include FES to help improve the gait pattern. Stimulation should be initiated for: (look at slide 15 on FF#11) A. Right abductors during swing on the right. B. Right abductors during stance on the right. C. Left abductors during stance on the left. D. Left abductors during swing on the left.

B. Right abductors during stance on the right. You want it to be most functional as possible (FES)

A 35-year-old female presents with swelling in the lower extremities. Upon examination, the PT palpates skin surfaces that are uneven with indentations and knotted fat structures. Additional findings are fat build ups around the pelvis, buttocks, hip, and a negative stemmer's sign. What is the patient's diagnosis? A. STAGE 3 lipedema B. STAGE 2 lipedema C. STAGE 1 lymphedema D. STAGE 4 lipedema

B. STAGE 2 lipedema

A PT is examining a 15 months old patient with Down syndrome. The patient has neck pain, limited neck motion, and decreased strength of extremities as compared to the last PT session. DTR of upper limb are 3+. The MOST likely cause of these symptoms is: A. Upper motor neuron signs seen in Down syndrome. B. Spinal cord impingement due to Atlanto-axial joint subluxation. C. Increased tone and reflexes seen in Down syndrome. D. Atlanto-axial joint subluxation causing lemniscal impingement.

B. Spinal cord impingement due to Atlanto-axial joint subluxation. -Down Syndrome is a LMN disease (DOWN = LOWER) -AA joint subluxation caused from Down Syndrome -Down syndrome is not a UMN disease -D would be sensory symptoms avoid rotation and aggressive exercise with Down Syndrome pts

A PT needs to check the walking gait of patient who suffered from CVA and decides to record a video in slow motion. The 3 hamstring muscles contract eccentrically during which phase of gait cycle? A. Initial Swing B. Terminal Swing C. Midstance D. Terminal Stance

B. Terminal Swing HS fire eccentrically to control the knee in ext during terminal swing to prepare for initial contact

How will COPD patients present?

Barrel chest History of smoking Cough

A 47-year-old mine-worker was admitted to acute care for burn injury. The PT observes that the anterior surface of the patient's trunk and both anterior lower extremities were affected. Which of the following BEST represents the percentage of body surface area involved? A. 18% B. 27% C. 36% D. 45%

C. 36%

A 42-year-old female patient presents with chief complaints of asymmetrical weakness of lower and upper extremities. There is presence of muscle fasciculations and impaired speech, but her sensations seem relatively intact. Her husband mentions that she randomly starts laughing or crying for "no reason at all". Which of the following signs and symptoms should the therapist LEAST likely expect to be positive in this case? A. Presence of memory impairments B. Presence of foot slap while walking C. Cervical flexor muscle weakness D. Reduced vital capacity

C. Cervical flexor muscle weakness laughing and crying = pseudobulbar effects ALS involves cervical neck EXTENSORS not flexors

All of the following diagnoses you would include fall prevention as a part of plan of care EXCEPT? A. Parkinson's Disease B. Multiple Sclerosis C. Complete Spinal Cord Injury D. Peripheral Neuropathy

C. Complete Spinal Cord Injury

A 78 year old male has an acute exacerbation of COPD. The patient has an FEV1 of 60% with FEV1/FVC<70%. During the examination, the patient appeared to have SOB during ambulation. According to the GOLD classification, the patient would MOST likely be classified as: A. Very severe B. Severe C. Moderate D. Mild

C. Moderate -look at FEV1

A PT examination reveals posterior superior iliac spine (PSIS) is low on the left; anterior superior iliac spine (ASIS) is high on the left. Interventions should MOST likely include: A. Stretching the right hip flexors to correct right anterior rotated innominate B. Strengthening of left hip flexors to correct left posterior rotated innominate C. Stretching the left hip extensors to correct left posterior rotated innominate D. Strengthening the right hip extensors to correct right posterior rotated innominate

C. Stretching the left hip extensors to correct left posterior rotated innominate Why not B? Stretching BEFORE strengthening!

2 Types of BPPV?

Canalithiasis -Otoconia crystals have become dislodged and moving around with movement and are hanging out in the canal which causes the fluctuation in nystagmus -Latency within (1-40 sec) -Short in duration (<1 min) -Nystagmus: fluctuates in intensity -More common on NPTE Cupulolithiasis -Immediate due to the otoconia crystals that are stuck to the cupula -Persistent in duration -Nystagmus: no change in intensity -Poor prognosis

What is difference between concurrent validity vs face validity?

Concurrent: test performed and compared to the gold standard test and results are matched Ex: HR measured by peripheral pulses and compared with ECG readings "CONCURrent Validity: When you ask someone to CONCUR with you, you're best asking the person with more knowledge to CONCUR with you. This person would be considered the GOLD STANDARD. (Catch Me If You Can when Leonardo DiCaprio asks his fellow doctors if they CONCUR since they have a higher knowledge)" Face: test appears to measure what it is intended to measure "FACE validity: When a patient's FACE appears angry, then they'll react angry. When a patient's FACE appears happy, then they'll react happy. (at face value)"

Touch sensation experienced as pain?

Dyesthesia

EKG chart interpretation: Hypocalcemia/Hypercalcemia Hypokalemia/Hyperkalemia Hypomagnesemia/Hypermagnesemia (look at slide 62 on FF#29)

Hypercalcemia: Hyper = High; Calcemia = Milk. You'll see QT interval shortening. If you're drinking a lot of MILK (Calcium) then you'll become a grown adult soon and reduce the amount of time you spend as a QT (cutie) kid. Hypocalcemia: Hypo = Low; Calcemia = Milk. You'll see QT interval prolonged primarily by lengthening the ST segment. If you are STubborn and don't drink your MILK (calcium), then you won't become a grown adult and you remain a QT (cutie) kid for a longer period of time.

What is positive for egophony?

If the sound sounds like "A" when asking the person to say "E" EA sports

Difference between Intra-rater reliability vs Inter-rater reliability vs Test-retest reliability?

Intra: test performed by 1 person ex: MMT/goni by 1 person Inter: test performed by 2 or more individuals for 1 variable ex: MMT/goni by 2 or more PTs on a person Test-retest: same test to the same individual on 2 occasions

Difference between Lipedema vs Lymphedema?

Lipedema -bilateral condition -cellulitis rare/absent -pain on pressure present -no distal edema in the foot -stemmer sign negative Lymphedema -unilateral OR bilateral but is asymmetric -cellulitis common -pain on pressure absent -distal edema in the foot -positive stemmer sign (stage 2 or 3) More on Lipedema -affects proximal area like butt/thigh -does NOT affect feet -affects skin elasticity which causes sensitivity to touch and pressure, which makes it bruise easily and painful

DD for the 3 Murphy's Signs (image on slide 36 in FF#29)

Murphy's Sign for Acute Cholecystitis -(+) pain with pressure under R subcostal margin with deep breath Murphy's Punch sign for Kidney -PT closes fist and gives a thrust to the 12th vertebra (area above kidney) -(+) = pain and tenderness indicated kidney infection/inflammation Murphy's Sign test for lunate dislocation -pt makes a fist -(+) = middle knuckle in line with ring and index finger

What does it mean if the ST segment is depressed at 2 small boxes beyond the J point and T wave may also be inverted?

Myocardial Ischemia is present (reduced blood flow to the heart)

What are the horizontal Add muscles?

Pec major

Muscles that do ankle eversion/PF

Peroneal longus Peroneal brevis (Superficial peroneal nerve)

Modified Postural Drainage: Pregnancy Kids

Pregnancy: -Avoid supine and Right s/l position because it compresses the IVC causing decreased CO Kids: -Use 2 fingers instead of whole hand because hands might be too big for underdeveloped lungs -if less than 1 y.o use 1 finger for postural drainage

Requirements needed for WC, Door frame size, and Ramp Rules

Push handles to floor = 36'' Arm rest to floor = 30'' Total length = 46'' Total width = 23" Door Width: -Minimum = 32" -Ideal = 36" Reach Forward HIGH 48" LOW 15" SIDE is 24" Rotate on WC rules: -To rotate 90⁰ you need 36" -To rotate 180 you need 60" -To rotate 360 you need 60" x 60" Ramp rules 8.3 % ramp 1 : 12 Normal Adult seat depth 16" Normal Adult seat width 18" Normal Adult seat height 20" Hemiplegic 17.5 "The ADA requires that all business and public use wheelchair ramps adhere to a 1:12 slope ratio, meaning that for every inch of vertical rise there must be 12 horizontal inches (one foot) of ramp. If, for example, you had a stairway with a vertical rise of 21″, you would need 21′ of ramp. Additionally, ADA stipulates that no ramp can run longer than 30 consecutive feet without a rest or turn platform."

Scoliosis: How would R side scoliosis effect the following muscles? QL Lats External obliques

R side curvature -muscles on the R side will be lengthened -muscles on the L side will be shortened

Convert RPE to METS

RPE vs METS: 42234. These numbers stand for how many numbers in the RPE scale are being included in that section. RPE 4(6,7,8,9): METS <2 ————— RPE 2(10,11): METS 2-3 ____ RPE 2(12,13): METS 3-6 ____ RPE 3(14,15,16): METS 6-8 _____ RPE 4(17,18,19,20): METS >8

What is correlation?

Relationship between 2 variables (ONLY AND ALWAYS 2) (lower case "r" is the symbol for correlation) Can be negative or positive correlation ex: Number of hours and NPTE scores: person who studied 2 hours/day got a 500, person who studied 4 hours/day got a 700, person who studied 6 hours/day got a 800 ex: BMI = weight over height (as weight increases, BMI increases as well)

2 tests to tests Vestibulocochlear nerve (CN8)

Rinne test Weber test Rinne's Normal - AC>BC, conductive loss - BC>AC -BC is worse because ideally you want to hear through the sound waves (AC) and not through physical touch (BC) Weber's - U/L Conductive loss - lat to affected side, U/L Sensorineural- lateralization to normal side. -Conductive hearing loss: sound appears louder (lateralization on the affected side) due to something compacting the unaffected side (lateralization on the same effected side) -Sensorineural hearing loss: sound appears louder (lateralization) on the unaffected side due to involvement of the inner ear, cochlea, or auditory nerve on the affected side (lateralization opposite to the affected side) "If the pt is complaining about the L ear and it's lateralized to the L then it is conductive hearing loss" "If the pt is complaining about the L ear and it's lateralized to the R then it is sensorineural hearing loss"

What are the stages of lymphedema?

Stage 0: -No outward swelling -Asymptomatic Stage 1: Reversible stage -Elevation reduces swelling -No tissue fibrosis -Swelling is soft or pitting Stage 2: Spontaneously Irreversible -Fibrosis of tissue -Brawny/hard swelling -Swelling is no longer pitting -Positive stemmer sign -Frequent infections may occur Stage 3: Lymphostatic elephantiasis -Positive stemmer sign -Significant increase in limb volume -Typical skin changes noted (hyperkeratosis, papillomas, deep skin folds) -bacterial and fungal infections of the skin and nails more common

What is total lung capacity (TLC)?

TV + IRV + ERV + RV Max volume of air the lungs can contain

What muscles are concentrically contracting when performing shoulder ext from 180 degrees to 90 degrees?

Triceps Lats Teres major Posterior delt

What causes Bishop's Hand or Benediction Hand Deformity?

Ulnar nerve palsy: Presents with wasting of hypothenar muscles of the hand 4th and 5th finger remains in flexion when asked to open hands

Which part of the lungs are cleared in a SITTING position with back in a reclined position?

Upper lobes: Apical segment

Gait related impairments for Multiple Sclerosis

Weakness -causes problems such as foot drop/toe drag Vaulting -compensatory technique involving heel raise of the stronger leg to make it easier for the weaker swing leg to clear through -Hip hike/Trunk lean/Circumduction Spasticity -muscle tightness or spasticity can interfere with gait Balance problem -swaying and drunken type of gait known as ataxia Sensory deficit -numbness in the feet leading to sensory ataxia Fatigue -can lead to increased gait problems

Ascends stairs to step to pattern (2 feet on each step) Running more coordinated Jumps off bottom step Plays make believe Throws ball overhand a few feet a. 7 months b. 8-9 months c. 10-15 months d. 20 months

d. 20 months

Rolls prone to side, supine to side Sits with support No head lag in pull to sit Bilateral reaching with forearm pronated when trunk supported Ulnar palmar grasp Laughs out loud a. 1 month b. 2 months c. 3 months d. 4 months

d. 4 months

· Which nerve roots does the median nerve stem from?

o C6/7/8

What is Dupuytren's contracture?

Contracture of palmar fascia, fixed flexion of the MCP and PIP (closed fist)

What is CRPS-NOS (not otherwise specified)

Specific injury or lesion has not been determined as the cause of symptoms

Posterior Cord Syndrome

"Post malone who has MS, late syphilis infection and has a vitamin B12 deficiency lost his special sensation and has poor balance and proprioception now" -caused by vitamin B12 deficiency/MS/late syphilis infection -decreased proprioception, balance, movement coordination -decreased sensation/vibration/fine touch

What is the MAXIMUM current density that should be used to initiate iontophoresis when the current amplitude is 10 milliamperes and the conductive surface area is 20 cm2? 1. 0.2 milliamperes/cm2 2. 0.5 milliamperes/cm2 3. 5 milliamperes/cm2 4. 20 milliamperes/cm2

2. 0.5 milliamperes/cm2 1. To calculate current density, the current amplitude is divided by conductive surface area; therefore, 10/20 = 0.5, not 0.2. 2. To calculate current density, the current amplitude is divided by conductive surface area; therefore, 10/20 = 0.5. 3. To calculate current density, the current amplitude is divided by conductive surface area; therefore, 10/20 = 0.5, not 5. 4. To calculate current density, the current amplitude is divided by conductive surface area; therefore, 10/20 = 0.5, not 20.

Which of the following ankle-brachial index values suggests arterial calcification? 1. 0.6 2. 0.9 3. 1.0 4. 1.4

4. 1.4 1. An ankle-brachial index of 0.6 is suggestive of moderate peripheral artery disease. This result would warrant referral but does not suggest arterial calcification. 2. An ankle-brachial index of 0.9 is a normal finding. 3. An ankle-brachial index of 1.0 is a normal finding. 4. Ankle-brachial index measures above 1.1 indicate arterial calcification. The calcification limits the compressibility of the artery.

A patient underwent surgical repair of a SLAP lesion 10 days ago. Which of the following interventions is LEAST appropriate at this time? A. Active isometric contraction of the biceps B. Passive humeral rotation with shoulder in scapular plane C. Pendulum exercises within the tolerated range D. Active isometric contraction of the triceps

A. Active isometric contraction of the biceps

A 45 year old male patient had BKA and is using a prosthesis. The PT notices that the patient demonstrates a certain gait deviation caused by the keel being too short. Which of the following gait deviation is MOST likely to be seen? A. Early knee flexion B. Delayed knee flexion C. Forward trunk bending D. Lateral trunk bending

A. Early knee flexion keel too short knee would compensate by flexing too early People who wear shoes that are too big flex their knees earlier than expected

A PT is treating a patient with burn injuries on the volar aspect of his hand and moderate level of edema. Which of the following positioning strategies will be MOST APPROPRIATE in order to avoid deformities and formation of contractures? A. Extension of proximal and distal IP joints B. Flexion of PIP and extension of DIP joints C. Flexion of PIP and DIP joints D. Extension of PIP and flexion of DIP joints

A. Extension of proximal and distal IP joints -For hand burn injuries all joints in ext except for MCP and thumb ABD (intrinsic plus position)

A physical therapist working in an intensive care unit. What is the correct sequence for donning the personal protective equipment? A. Gown, mask, gloves B. Gown, googles, gloves C. Mask, googles, gown D. Mask, gown, gloves

A. Gown, mask, gloves

A patient comes to PT clinic following a left below knee amputation. Assuming the black dot in the middle was the patient's COM before amputation, which of the following location is MOST likely to the new COM after amputation. A. Moves higher on the right side (Yellow dot) (look at slide 6 on FF#13) B. Moves lower on the right side (Red dot) C. Moves higher on the left side (Blue dot) D. Moves lower on the left side (Green dot)

A. Moves higher on the right side (Yellow dot) COM would move towards the heavier side

A patient is instructed to perform the exercise shown in the picture at home, however patient is unable to complete the movement. The patient's lower extremity limitation is MOST likely caused by which of the following conditions? A. Passive insufficiency of the quadriceps B. Active insufficiency of the quadriceps C. Active insufficiency of the hamstrings D. Passive insufficiency of the iliopsoas Picture is person doing a standing quad stretch

A. Passive insufficiency of the quadriceps

A physical therapist is examining a 41 year old male patient in an out patient clinic. Which of the following sound is PT auscultating? (look at slide 16 on FF#10) A. Vesicular sounds B. Mitral valve C. Pulmonary valve D. Tracheal sounds

A. Vesicular sounds Bronchial= heard in large airways in anterior chest near 2nd and 3rd intercostal space Bronchovesicular= posterior chest between scapula Vesicular= most lung fields

A 12 year old girl has a structural right thoracic idiopathic scoliosis. The clinical features you would expect to find include a high: (Look at image on slide 23 on FF#25) A. right shoulder, a prominent right scapula and a left hip that protrudes B. left shoulder, a prominent left scapula and a right hip that protrudes C. right shoulder, a prominent left scapula and a right hip that protrudes D. left shoulder, a prominent right scapula and a left hip that protrudes

A. right shoulder, a prominent right scapula and a left hip that protrudes How would a R Thoracic scoliosis effect the LE? -R hip is lower causing the RLE to be longer causing pronation

A PT is working for a home health company and mean annual income for all PTs is $100,000. Last year, the physical therapist's salary was 2 standard deviations more than the mean. What percentage of his salary falls ABOVE $100,000. A. 68% B. 47.5% C. 95% D. 34%

B. 47.5% Key word = "MORE THAN THE MEAN" 68% = range of 1 SD 95% = range of 2 SD 34% = above 1 SD

A patient demonstrates collapse of the medial arch and over pronation during stance phase. Which muscle should be strengthened to support the medial arch, and assist in decelerating pronation during stance? A. Anterior Tibialis B. Posterior Tibialis C. Soleus D. Abductor Hallucis

B. Posterior Tibialis Post tib is primary muscle for medial arch

DD: BPPV UVH Meniere's disease Acoustic Neuroma

BPPV -Disorder arising from a problem in the inner ear -Vertigo with head change position -Vertigo lasts <1 min -Nystagmus -Nausea with/without vomiting UVH -Balance system in your inner ear, peripheral vestibular system, is not working properly -Resting (spontaneous) nystagmus -Oscillopsia -Disequilibrium -Postural instability Meniere's disease -Recurrent and usually progressive vestibular disease -Tinnitus -Deafness -Sensation of fullness in the ear -Vertigo Acoustic Neuroma -Tumor on vestibular nerve leading from inner ear to brain -Hearing loss on one side -Tinnitus in the affected ear -Unsteadiness -Loss of balance -Dizziness -Facial numbness

A PT examines the output from a single lead electrocardiogram of a patient in an outpatient clinic. The ECG strip is shown in picture below. PT should determine the heart rate of the patient as (Look at slide 9 on FF #9 lecture) A. 110 beats per minute B. 70 beats per minute C. 80 beats per minute D. 50 beats per minute

C. 80 beats per minute 8 QRS complexes. Add a ZERO next to it and you get 80 as your HR. 8 x 10 = 80 Count the number or QRS complexes and multiply by 10

A PT is examining a 45 old patient who presents with weakness and tingling in the lower extremities over the past 2 weeks. Which of the following examination findings would MOST likely confirm diagnosis of Guillain Barré syndrome? A. Hypertonicity in the affected muscles B. Positive Babinski sign C. Diminished tendon reflexes D. Presence of clonus with rapid passive foot dorsiflexion

C. Diminished tendon reflexes Key words: weakness/GBS Diff between MS and GBS? MS can happen in any area. GBS is polyneuropathy with a distal to proximal progression MS damages the central nervous system. That's the brain and spinal cord. GBS damages the peripheral nervous system. GBS is a LMN lesion "Lower the volume of your GameBoy" ALS = ALL (both UMN and LMN lesion)

A 34 year old male has been diagnosed with a complete C5- C6 spinal cord injury. Patient used a power chair to commute from the parking lot to the PT clinic. While performing a transfer, the MOST important consideration is to A. Make sure he is using his shoulder extensors to support his upper body B. Make sure his legs are supported by PT aide C. Make sure his power chair is turned off D. Make sure to use the sliding board

C. Make sure his power chair is turned off -Make sure questions involve SAFETY of the pt, yourself, and others. First thing you do is to turn the power WC off to insure safety -If its a manual WC you need to move it out of th way -MAKE SURE QUESTION = FOCUS ON SAFETY

A physical therapist is facilitating an in service on BLS by educating on the Do's and Don'ts of adult high-quality CPR. Which of the following SHOULD the rescuer do during the training? A. Include pauses in between compressions for greater than 10 seconds B. Compress chest to depth of less than 2 cm C. Minimize pauses in between compressions D. Compress chest at a rate of 123/min

C. Minimize pauses in between compressions B: should be 2 inches or 5 cm D: should be 100-120 2 breaths after 30 compressions, each breath delivered over 1 second, each causing chest to rise

A 58-year-old patient is lifting a wine box using a stoop lift at a winery. Which of the following is the MOST significant factor in increasing compression forces on the spine in addition to the weight of the wine box? A. The height of the wine box from the ground B. Performing the lift with the lumbar spine in a neutral position C. The distance of the wine box from the base of the spine D. The muscle strength of the lower extremities

C. The distance of the wine box from the base of the spine

A 58 year old patient recently suffered an above knee amputation and underwent surgery. A PT is examining the wound site and sees an exudate. Which of the following findings indicates an infected wound and should be reported to be physician? A. Dark red blood B. Bright red blood C. Viscous yellow exudate D. Serosanguineous exudate

C. Viscous yellow exudate infected wound = yellow/green serosanguinous = normal recovery is happening

ACL tear

-Twisting or hyperextension of the knee MOI -Sensation of their knee "popping" or "giving out" as the tibia subluxes anteriorly -Instability in the involved knee and the inability to walk without assistance -Immediate swelling (acute hemarthrosis) ST for ACL -Anterior drawer = 90 degrees flex -Lachman = 20-30 degrees of flex

A patient is taking anticoagulants. Which of the following side effects should the therapist be concerned about during the patient's exercise? 1. Ecchymosis 2. Deep vein thrombosis 3. Dehydration 4. Hypotension

1. Ecchymosis 1. Anticoagulants prevent clots. A potential side effect is ecchymosis due to the increased risk of bleeding. (Vallerand, p. 39; Taber's p. 323) 2. Anticoagulants are used to prevent deep vein thromboses (Vallerand, p. 39). 3. Anticoagulants affect clotting factors and do not directly affect body fluid volume levels; therefore, dehydration would not be the highest concern for this patient (Vallerand, p. 638). 4. Hypotension is not a side effect of anticoagulants. Bleeding or bruising would be the highest concern for this patient (Vallerand, p. 639).

A physical therapist examines a right-handed patient who demonstrates a rounded upper back and protracted shoulders. The patient may be at INCREASED risk for developing which of the following conditions in the left upper extremity? 1. Thoracic outlet syndrome 2. Carpal tunnel syndrome 3. Paresthesia in an ulnar nerve distribution 4. Lateral epicondylalgia

1. Thoracic outlet syndrome 1. Anterior scalene or pectoralis minor tightness from this posture can impinge the neurovascular bundle, causing thoracic outlet syndrome (pp. 321-322). 2. Overuse injury of the wrist is unrelated to this postural presentation (p. 795). 3. The ulnar nerve is well protected above the elbow. Injury most often occurs at the elbow or wrist. (pp. 725, 813) 4. Overuse injury of the elbow is unrelated to this postural presentation (p. 720).

A patient who has hypothyroidism is MOST likely to exhibit which of the following signs or symptoms? 1. Ptosis 2. Muscle ache 3. Dysphagia 4. Tachycardia

2. Muscle ache 1. Ptosis is not a common symptom of hypothyroidism. 2. Muscle ache (myalgia) is a common musculoskeletal symptom of hypothyroidism. 3. Dysphagia is not a common symptom of hypothyroidism. 4. Bradycardia, not tachycardia, is a common symptom of hypothyroidism.

A PT is examining a patient who is exhibiting different BP readings from bilateral arms, as well as a drop SBP. The patient's extremities are cold to the touch, and a bulge in the abdominal area is also present. Which of the following should the PT do FIRST? A. Start medical response team B. Documents finding and referred to physician C. Compress the bulge and elevate the limb D. Notify nurse to change BP cuff as there is an error in measurement of BP

A. Start medical response team

Which of the following locations of pain is MOST consistent with bladder infection? 1. Groin 2. Sacral area 3. Lower buttocks 4. Suprapubic area

4. Suprapubic area 1. Groin pain is incorrect. Groin pain is associated with upper urinary tract problems, such as kidney or ureter infection. 2. Sacral pain is incorrect. Sacral pain is associated with colon cancer and colitis. 3. Lower buttocks pain is incorrect. Lower back pain, not lower buttocks pain, is associated with bladder infection. 4. Pain generated by the bladder typically manifests in the suprapubic area and the lower back.

A study is looking at effects of different exercise options for a patient with knee OA. The study compared pool exercise, treadmill walking, and ankle weights on knee pain in 30 adults. An appropriate statistical test to assess the effectiveness of treatment in three groups is: A. Spearman's rho B. Two paired t tests C. ANOVA D. Chi square

C. ANOVA Spearman's rho = Correlation Two paired t tests = 2 group equal ANOVA = 3 group Chi square = 2 group non-equal

A patient's electrocardiogram shows a new ST segment elevation from baseline and a sinus rhythm of 68 beats per minute. What is the MOST likely diagnosis? A. Bradycardia B. Low blood pressure C. Acute myocardial infarction D. Congestive heart failure

C. Acute myocardial infarction

A PT is ambulating a 75 year old male who is in recovering from a prostatectomy procedure. The patient complains of palpitations, shortness of breath and fatigue. What should be the interpretation and immediate action according on the ECG strip shown below? (look at image on slide 26 on FF#9) A. Ventricular fibrillation; Call for a defibrillator B. Premature ventricular contractions; Take him to his bed and monitor for changes in ECG. C. Atrial Fibrillation; Stop exercise and report to the physician. D. 3rd degree heart block; Activate emergency.

C. Atrial Fibrillation; Stop exercise and report to the physician. Premature ventricular contractions = no P wave

A PT is treating a male patient in an acute care setting. Which of the following laboratory values should a physical therapist be concerned about in a patient who is taking anti coagulants? A. White blood cell count of 8,000 cells per microliter B. Red blood cell count of 5.5 million cells per microliter C. International normalized ratio of 5.5 D. Erythrocyte sedimentation rate of 15 mm/hr

C. International normalized ratio of 5.5 Summary: -Normal INR is 0.9-1.1 , on patients with anticoagulant INR is 2-3. 5.5 is too high and can lead to thinning of blood and too much bleeding. Normal RBC count: men - 4.7 to 6.1 million cells per microlitre (cells/mcL) women - 4.2 to 5.4 million cells/mcL. -Heparin, Aspirin, Warfarin, Coumadin should not be used together because if combined can lead to too much blood thinning (Anticoagulation) causing excess bleeding

Mnemonic for incomplete SCI syndrome

Central Cord Syndrome -MUD-E -Motor more involved than sensory -UE > involvement than LE -Distal > proximal -E = cervical Ext injury (review mega review video) Brown-Sequard Syndrome -Brown- POT -Pain & Temperature effected on Opposite side

What is the cremasteric reflex and which nerve does it test?

Contraction of cremaster muscle (scrotum/testis elevate) after stroking same inner thigh (+)Absence signifies: SCI and corticospinal lesion

LE PNF patterns

D1 flex = FADER ex: put pants/socks on, cross legs D1 ext = EABIR ex: getting out of car D2 flex = FABIR D2 ext = EADER D1 flex/ext mimics normal gait and soccer kick "1 ball in soccer" D2 flex/ext mimics Karate kick "Make your table, start with FADER, opposite ABD/ADD and IR/ER"

What outside factors effects HR?

High altitude (mountains) Low altitude (water/aquatic) Age Hot/cold weather

2 Types of Foot Orthosis

Longitudinal Arch (Scaphoid pad) -Used for Pes Planus/flat foot UCBL -Controls hindfoot valgus/calcaneal eversion/forefoot ABD -Reduces subtalar motion -3 point counterforces

What type of sensations is the Anterolateral Spinothalamic Pathway in charge of?

Non discriminative sensations: Pain Temperature Tickle Itch Sexual sensations

Abnormal sensation such as numbness, prickling, or tingling, without apparent cause?

Paresthesia

-Chronic progressive neurodegenerative disorder -Depletion of dopamine from the substantia nigra

Parkinson's Disease

Wheelchair Propulsion: Propulsive Phase vs Recover Phase

Propulsive phase -Begins with initial hand contact on push rim -Ends with release of hand contact from push rim Recovery phase -Begins with release of hand contact from push rim -Ends with initial hand contact with push rim

What is GERD?

Reflux of gastric content of the gastroduodenal contents into the esophagus Symptoms -heart burn: 30 min AFTER eating and at night lying down -dysphagia -sour taste: from regurgitation of acids -hoarseness of voice -atypical pain of the head and neck Complications -Aspiration pneumonia, asthma -Esophagitis

What is the J point of an EKG?

The point where the S wave turns into the ST segment, point of reference for interpreting the ST segment

Wong Baker FACES Pain Rating Scale

Used for kids who can't express themselves 0 = very happy, no hurt 1 = hurts just a little bit 2 = hurts a little more 3 = hurts even more 4 = hurts a whole lot 5 = hurts as much as you can imagine (just know 0 is happy and 5 is bad)

A fireman was brought to the ER with burns on his anterior chest and trunk, anterior right arm, and bilateral anterior legs. Which of the following accurately describes the area of distribution of the burn? Select one: a. 54% b. 40.5% c. 20.5% d. 36%

b. 40.5%

Touching skin along spine from shoulder to hip causes lateral flex of trunk to same side of stimulus a. Symmetrical tone neck reflex b. Asymmetrical tonic neck reflex c. Tonic labyrinthine reflex d. Galant reflex

d. Galant reflex 30 weeks gestation to 2 months

What is gait cycle/stride/step?

gait cycle = IC to IC of the same foot stride length = 1 complete gait cycle Step = beginning of an event by 1 limb until the beginning of the same event with the contralateral limb 2 steps = 1 stride = 1 gait cycle

· Which nerve roots does the musculocutaneous nerve come from?

o C5/6/7

· What cord does the radial nerve come from?

o Posterior cord

· What is ape hand deformity?

o Thenar atrophy and thumb in the plane of the hand, caused by median nerve entrapment

A patient with sharp pain at the ankle. According to the patient, the ankle was injured while playing basketball last week. When performing the pictured test on the patient, the PT finds that the patient's ankle remains in neutral position. This would indicate: (Look at image on slide 20 on FF#25) A. Medial Meniscus tear B. Anterior talofibular ligament injury C. Achilles tendon rupture D. Calcaneofibular ligament tear

C. Achilles tendon rupture ST is Thompson Test for Achilles tendon rupture -Meniscus tear = use McMurray's/Thessaly -ATFL = use Anterior Drawer Test for ankle -CFL = use Talar Tilt Test Grade 1= SARO at BEGINNING or range. Grade 2= LARO before tissue resistance. Grade 3= LARO up to tissue resistance. Grade 4= SARO AT tissue resistance. Grade 5= thrust. Grade 1 & 2= pain. Grade 3 & 4= to increase ROM

A physical therapist observes that a patient walks in to the clinic with an abnormal gait. On examination, PT identifies that the patient has a shorter step length on the right lower extremity. Which of the following problems is MOST likely the cause of the gait dysfunction? A. Right Iliopsoas contracture B. Left Gluteus Maximus contracture C. Left Iliopsoas Contracture D. Left gluteus medius weakness

C. Left Iliopsoas Contracture -L hip ext is limited from a tight L iliopsoas which causes limited R hip flex COWS = Contracture opposite/Weakness same

The PT is treating a 43 year old male who sustained left hip fracture. The patient is having difficulty gaining hip flexion ROM. How much minimum hip flexion should the PT try to obtain so that the patient can ambulate with a normalized gait pattern? A. 15 degrees B. 25 degrees C. 40 degrees D. 50 degrees

B. 25 degrees

What happens to your HR/BP/CO/SV after you get used to the altitude change (acclimatization)?

HR: increases BP: normal CO: normal SV: decrease

Central Cord Syndrome

"50 CENTral had whiplash from MVA accident, peed his pants because he couldn't open the bathroom door due to his weak UE and also got frost bite because he couldn't feel the cold door and now he can't feel pain." -bladder dysfunction -UE weakness > LE -Pain/Temp loss -damage by cervical hyperext

Belly crawls Quadruped creeping Moves quadruped to sitting Side-sitting Pulls to stand through kneeling of furniture Cruises sideways, can stand alone Reaches with closest arm, radial digital grasp, radial palmar, three jaw chuck grasp, and inferior pincer grasp with thumb and forefinger Can transfer objects from one hand to the other a. 3 month b. 5 months c. 7 months d. 8-9 months

d. 8-9 months

What is Sheuermann's disease?

Abnormal degree of kyphosis due to ANTERIOR vertebral WEDGEING, and multiple Schmorl nodes Juvenile thoracic kyphosis

How to predict HR?

Age predicted -HR max = 220 - age Karvonen's formula -HR reserve multiplied by the percentage of intensity plug resting HR -Range = 60-80% of HR max -asked for on the NPTE -Ex: Karvonen's Formula: Eg: Age is 60 , Resting heart rate is 80 HR max: 220-60 = 160 HR min: 160-Rhr = 80

Sensation experienced at a site remote from point of stimulation?

Allesthesia

Which 2 pathways are pathways for somatic sensory?

Anterolateral Spinothalamic Pathway Dorsal Column Medial Lemniscal Pathway

Contraindications for Ultrasound?

Anywhere you have compromised sensation/reduced circulation/cancerous/sensitive regions (metal is okay)

What is a premature atrial contraction?

Atria contracts (p wave) before ventricle finishes relaxing (t wave) ex: you see the p wave interrupting the t wave (look at slide 24 on FF#9) -NOT AN EMERGENCY AS LONG AS VENTRICLES DOES ITS JOB

Which lobe is Broca's area and Wernicke's area located? Which division (upper or lower) of the brain are each area located?

Broca's = frontal lobe/upper Wernicke's = temporal lobe/lower

A 36 year old patient experiences limitation with left side bending and left rotation at the C5 6 junction. What is the MOST appropriate intervention? A. Apply PA glide at left C5 B. Apply AP glide at left C5 C. Apply PA glide at right C5 D. Apply PA glide at right C6

C

Painful, burning sensations, usually along the distribution of a nerve?

Causalgia

Pressure sensitive vs Pressure tolerate areas for BKA

FAFA: Pressure sensitive areas Fibular nerve Anterior tibial crest Fibular head Anterior tibia MDPT: Pressure tolerant areas Medial tibia Distal end Patellar tendon Tibial and fibular shaft

How would asthma effect pulmonary function tests??

FEV1 decreased RV and FRC increased due to air trapping

Normal End Feels

Hard -Bone to Bone -stiff sesnsation that is painless -Ex: elbow ext Soft -there is yielding compression (mushy feel) -Ex: elbow and knee flex Firm -Toward the end of ROM there is an elastic resistance -Ex: ER or shoulder

Saddle Height: Effects of Higher seat vs Lower seat height

Higher seat height -greater knee ext -greater work of hamstrings muscles -less lateral displacement of the knee (better for early rehab that focuses on knee ext ROM) Lower seat height -greater knee flex -decrease of performance -greater stress on the knee

Hoffman vs Homan

Hoffman = (+) flick DIP and thumb flexes = UMN lesion Homan = pain with squeezing calf = DVT

Current terms

Intensity -amount of current delivered per unit area (how much) Frequency -how many times per time -ex: how many times did the current touch the skin Pulse Duration -amount of time the current is delivered -ex: how much time the current actually STAYED on the skin Duty Cycle -on time / (on time + off time) -should be less than 50% MVIC -Maximal voluntary isometric contraction ex: MMT

What is Kehr's sign?

Kehr's sign is the occurrence of acute pain in the tip of the shoulder due to the presence of blood or other irritants in the peritoneal cavity when a person is lying down and the legs are elevated. Kehr's sign in the left shoulder is considered a classic symptom of a ruptured spleen. Steve Kehr: "S"teve ("S"pleen) as a LEFTY NBA player, so think KEHR = LEFT shoulder and Spleen

Which part of the lung is cleared in a PRONE position on a neutral bed with arm raised?

Lower lobes: Superior segment

What is the normal value for hemoglobin in adults male/female?

Males: 13-18 g/dL Females: 12-16 g/dL

Drugs that Regulate Blood Cholesterol: Drugs that lowers blood cholesterol? SE of the drug?

Niacin SE -requires lab test to check LFT, Physician's prescription for dosage

List the shoulder add muscles

Rhomboid major/minor Lats Teres major Subscap

What are the cardinal signs of PD?

TRAP -Tremor -Rigidity -Akinesia/Bradykinesia -Postural instability Early signs: loss of sense of smell, constipation, sleep disorders

What is the difference between 2nd degree: type 1 and type 2 heart block?

Type 1 (Wenckebach): -PR interval gets progressively longer until a QRS is dropped -Pattern can be predicted (less severe) Type 2 (Mobitz): -PR intervals are constant and a QRS is dropped surprisingly -Pattern can NOT be predicted (more severe) -disease of the Bundles of His and Purkinje fibers (dealing with the ventricle)

Which part of the lungs are cleared in SITTING position with back flexed over pillow?

Upper lobes: Posterior segment

What are the scapular elevator muscles?

Upper trap Rhomboid major/minor Levator scap

Professional Responsibility: PT vs PTA

Utilization -PT is directly responsible for the actions of the PTA related to pt/client management -The PTA may perform selected physical therapy interventions under the directions and at least general supervision of PT -In all practice settings, the performance of selected interventions by PTA must be consistent with safe and legal PT practice

Difference between Vtach and Vfib?

Ventricular tachycardia has three or more PVCs and a rate of greater than 100bpm; ventricular fibrillation is chaotic electrical activity with only fibrillatory waves

Scoliosis: Rule for direction of vertebral body and spinous process?

Vertebral body goes toward the CONVEX side Spinous process goes toward the CONCAVE side

Which system of the following impairments of balance would pt become unstable when the sensory input they rely on is taken away?

Vision -2/3/5/6 -because eyes are closed or conflict between vision and vestibular system in these conditions Somatosensory -4/5/6 -because foam/compliant surface is present Vestibular -5/6 -Both somatosensory and vision is not available

A physical therapist is examining a patient who is recovering from a motor vehicle accident that injured the left forearm. The patient appears to have damaged the posterior interosseous nerve (branch of radial nerve). Which of the following motions will be MOST impaired? a. Forearm pronation b. Ulnar deviation c. Wrist flexion d. Wrist extension

d. Wrist extension The major disability associated with radial nerve injury is a weak grip, which is weakened because of poor stabilization of the wrist and the finger joints. In addition, the patient demonstrates an inability to extend the thumb, the wrist, and the elbow, as well as the proximal phalanges.

Explain what happens to your knee and hip and pelvis when you pronate the foot

knee: genu VALGUM hip: coxa VARUS pelvic: anterior rotation (hip drops and moves forward causing the pelvis to move with it anteriorly)

· What cords does the median nerve come from?

o Medial and Lateral cords

What is lymphatic fluid comprised of?

water and protein that is transported back into venous circulation

Symptoms of Huntington's Disease?

• 35 and 55 years • Affects muscle coordination and leads to cognitive decline, psychiatric problem • Involuntary jerking or writhing movements (Huntington's chorea) • Muscle problems, such as rigidity or muscle contracture (dystonia) • Slow or abnormal eye movements • Impaired gait, posture and balance • Difficulty with the physical production of speech or swallowing

5 stages of pressure ulcers? (look at NDNQI pdf to see different examples)

• Stage 1 - reddened area that does not go away. • Stage 2 - first 2 layers of skin, superficial in nature. • Stage 3 - subcutaneous fat may be visible. • Stage 4 - down to the bone and including the bone. • Unstageable - is related to not visualizing the wound base because of necrotic tissue. Unstageable: Full-thickness tissue loss in which the base of the ulcer is covered by slough (yellow, tan, gray, green, or brown) and/or eschar (tan, brown, or black) in the wound.

A researcher is collecting data on 100 tennis players with overhead injuries. Out of the 100 patients with rotator cuff injuries, 60 patients had a positive special test, and 40 had a negative special test. What is the sensitivity of the special test? A. 20% B. 80% C. 40% D. 60%

D. 60% Sensitivity = TP / (TP + FN) Specificity = TN / (FP + TN)

A PT examines a patient who complains of foot pain while running. The examination shows that the patient has excessive foot pronation. Which of the following would be the MOST appropriate orthotic insert? A. A lateral forefoot post under the fifth metatarsal head B. A lateral rearfoot post under the calcaneus placing it in an everted position C. A middle post just proximal to the third metatarsal head D. A medial post just proximal to the first metatarsal head

D. A medial post just proximal to the first metatarsal head

3 ways to diagnose MS?

MRI -usually 2 separate lesions in the CNS that are distinct CSF analysis -elevated immunoglobulins and presence of oligoclonal lgG bands, slight protein elevation Evoked potential -decreased nerve conduction velocity -common symptom of weak hand muscles and clumsiness dropping things due to demyelination

Venous filling time test

Normal = 15 second >15 second = arterial disease <15 seconds = venous disease (Rubor dependency used for Arterial Insufficiency)

List the functions of all the CN

Olfactory = smell Optic = vision Oculomotor = eye movement/pupil constriction/accommodates lens Trochlear = inward eye movement Trigeminal = facial, corneal, and tongue sensation/mastication/dampens sound (lensor tympani) Abducens = abduction of eye Facial = facial expression/taste of anterior 2/3 of tongue/dampens sound/tearing/salivation Vestibulocochlear = balance/hearing Glossopharyngeal = taste and sensation of posterior 1/3 of tongue/sensation from oropharynx/salivation Vagus = Thoracic and abdominal viscera/muscles of larynx and pharynx/decreases HR/increases GI mobility Spinal Accessory = Head movement (SCM and trapezius) Hypoglossal = tongue movement and shape

What is CRPS and what are the s/s?

Painful condition where individual experiences hypersensitivity to pain and inflammation in a certain region of the body, motor impairment, and emotional/psychological responses

Postural Drainage: Key Positions -Upper Lobes -Lower Lobes

Upper Lobes: Anterior segment = Supine lying on a bed with pillows under knees Posterior segment = Sitting on a chair, leaning FORWARD over a folded pillow Apical segment = Sitting in a recliner, leaning slightly backward Lower Lobes: Superior segment = Prone lying flat on bed Lateral segment = L or R sidelying, 1/4 turn supine from prone Anterior segment = Supine lying with foot end elevated Posterior segment = Prone lying with foot end elevated

What structures are impinged under FAI?

capsule ligaments pectineus piriformis femoral nerve/artery

Head dropping into extension suddenly for a few inches causes arms to abduct with fingers open, then cross trunk into adduction; cry a. Palmar grasp reflex b. Plantar grasp reflex c. Rooting reflex d. Moro reflex

d. Moro reflex 28 weeks of gestation to 5 months

What should you do with a pt with atrial flutter and atrial fibrillation?

flutter = stop or slow down exercise fibrillation = stop and refer to physician (MORE SEVERE)

Position Strategies to Reduce Common Malalignments: S/L on more affected side

(take them out of flexion synergy) More Affected UE -Scap = PROTRACTED -Shoulder = FORWARD/SLIGHTLY ABDUCTED/ER -Elbow = EXTENDED -Forearm = SUPINATED -Wrist = NEUTRAL -Fingers = EXTENDED -Thumb = ABDUCTED More Affected LE -Hip = EXTENDED -Knee = FLEXED and supported by pillows Alternate position -slight hip and knee FLEX with pelvic PROTRACTION

What is atrial flutter?

-250-350 BPM -Multiple atrial contractions (P waves) before ventricle contraction (QRS complex) -Sawtooth appearance (look at slide 25 on FF#9) Atrial fluTTer: Make that TT noise with your mouth. Do it over and over. IT sounds like an electric SAW gearing up. So think SAWtooth. Also make that noise TT because you're putting your tongue against your TEETH to make that noise so you can think saw TOOTH appearance

A patient presents to outpatient clinic after recovering from a right CVA. The patient still has some residual effects of left hemiplegia, including spasticity. What is the BEST intervention for spasticity? Select one: 1. Quick stroking 2. Prolonged icing 3. Rapid vestibular stimulation 4. Quick stretch

2. Prolonged icing Answer: Prolonged icing is an inhibitory technique which can help decrease spasticity. It decreases spasticity by decreasing muscle spindle discharge and decreasing gamma motor neuron activity. Other options like quick stroking, quick icing, rapid vestibular stimulation are neuro facilitatory treatment interventions for flaccid muscles to help stimulate them to contact.

A patient presents with increased pigmentation and reports that they have been fatigued however they improve with rest. Additionally, the Physical Therapist notices that the patient has a blood glucose level of 60 mg/dL and blood pressure of 102/72 mmHg. The Physical therapist should suspect that the patient has which of the following? 1. Hyperthyroidism 2. Hypothyroidism 3. Addison Disease 4. Hypoparathyroidism

3. Addison Disease With hypothyroidism, there is elevated blood pressure, delayed glucose uptake & decreased glucose absorption and no increase in pigmentation. With Hypoparathyroidism, we can see pigmented skin but changes in blood pressure and glucose levels are generally not seen. With Hyperthyroidism, we would notice fatigue, hyperpigmentation, low blood pressure but it does not cause hypoglycemia. Common clinical presentations of Addison Disease include dark pigmentation of the skin, low blood pressure, progressive fatigue which improves with rest and hypoglycemia. Addison disease is also known as primary adrenal insufficiency.

A patient being examined by a physical therapist for bilateral shoulder pain and weakness is noted to have undiagnosed exophthalmos. Further screening for which of the following diseases would be MOST appropriate? 1. Cushing syndrome 2. Wilson disease 3. Graves disease 4. Addison disease

3. Graves disease 1. Cushing syndrome does not produce protruding eyes. Cushing syndrome is secondary to increased intake of glucocorticoids. (pp. 500-501) 2. Wilson disease does not produce protruding eyes. Wilson disease is characterized by Kayser-Fleischer rings around the iris of the eye (from copper deposits). (p. 537) 3. Graves disease, a form of hyperthyroidism, can cause fluid accumulation behind the eyeball, resulting in protruding eyes (p. 484). 4. Addison disease does not produce protruding eyes. Addison disease is associated with decreased adrenocortical secretions. (p. 498)

A physical therapist is performing gait training with a patient who has right hemiparesis. The patient displays extensor tone in the stance and swing phases of gait. Which of the following is the MOST appropriate orthotic recommendation? 1. Solid ankle-foot orthosis set in 10° of plantar flexion 2. Floor reaction ankle-foot orthosis 3. Hinged plastic ankle-foot orthosis with plantar flexion stop 4. Posterior leaf spring orthosis

3. Hinged plastic ankle-foot orthosis with plantar flexion stop 1. The ankle-foot orthosis should be set in slight dorsiflexion to discourage the extension synergy, which includes ankle plantar flexion (p. 635) 2. A floor-reaction ankle-foot orthosis encourages knee extension, which is consistent with an extension synergy (p. 1296). 3. A hinged plastic ankle-foot orthosis with plantar flexion stop provides the greatest control for the patient and limits the amount of plantar flexion, thereby decreasing the extension synergy (p. 1294). 4. A posterior leaf spring orthosis does not provide adequate control for a patient with increased extensor tone and is not recommended in the presence of hypertonia (p. 1293).

Which of the following statements BEST explains why a patient with reports of dyspnea would benefit from the position shown in the photograph? (Image of man hunched forward with elbows resting on knees) 1. The accessory respiratory muscles are relaxed in the position shown. 2. The position increases the thoracic cavity volume. 3. The position allows increased strength of contraction of the diaphragm. 4. The position encourages pursed-lip breathing and emphasizes expiration.

3. The position allows increased strength of contraction of the diaphragm. 1. The accessory muscles are generally activated in this position, because the arms are supported, and, therefore, the accessory muscles can work on the rib cage to assist the respiratory muscles. The accessory muscles are performing their role and not "resting." Therefore, this is an incorrect option. 2. This is incorrect, because the diaphragm is pushed upward and not downward when the intraabdominal pressure rises, as in this position. 3. When the person leans forward as in the photograph, the intraabdominal pressure rises and lengthens the diaphragm as it is raised upward. This allows the strength of the contraction to increase. Due to the improved length-tension relationship, the diaphragm has an improved strength of contraction. 4. This position does not have any impact on pursed-lip breathing and expiration. Both these actions can be performed in any position and not necessarily in the position shown.

A physical therapist is examining a patient following open reduction internal fixation of the distal tibia and fibula. The physical therapy examination reveals active range of motion of the involved ankle from 10° of dorsiflexion to 0° of plantar flexion. What phase of gait is MOST likely affected? 1. Heel strike (initial contact) 2. Midstance 3. Toe off (preswing) 4. Acceleration (initial swing)

3. Toe off (preswing) 1. At heel strike (initial contact), the ankle is in a neutral position, so only having 10° of movement at the ankle would not affect this phase of gait. 2. The ankle will move from plantar flexion to dorsiflexion during midstance, but the greatest plantar flexion occurs at toe off (preswing). 3. The greatest amount of plantar flexion is needed in the ankle during the toe off (preswing) phase of gait. The patient has 0° of plantar flexion, so this phase of gait would be most affected. 4. The ankle remains in a neutral position during acceleration (initial swing). Since the patient has 10° of dorsiflexion, this phase of gait would not be affected.

A 25 year old female has been suffering from widespread pain with several tender points in the upper body which increases with stress. The Physical Therapist suspects the patient has Fibromyalgia. Which of the following intervention would be LEAST effective? Select one: 1. Deep breathing and relaxation exercises 2. Aerobic exercises at moderate intensity 5 days a week 3. Trigger point massage 4. Resistive exercises with therabands

3. Trigger point massage Answer: Trigger points are tender knot like areas in the muscles. In Myofascial syndrome there is radiating pain on palpation with pressure. They can be released with a trigger point massage. But here the patient has Fibromyalgia and the tender points seen in this condition are not similar in pathophysiology to trigger points seen in myofascial syndrome. Trigger point massage is not helpful as they are absent in this condition. Light to moderate aerobic exercise, light resistance exercise, relaxation/breathing techniques are all helpful interventions for fibromyalgia

Examination of a patient with balance dysfunction reveals the following: Romberg test: positive Gait: wide-based, slow, with decreased trunk rotation Loss of balance when asked to turn head while walking No sign of ataxia Based on these findings, which of the following diagnoses isMOSTlikely correct? 1. Benign paroxysmal positional vertigo 2. Cerebellar lesion 3. Unilateral vestibular lesion 4. Mononeuropathy of the sural nerve

3. Unilateral vestibular lesion 1. Patients with benign paroxysmal positional vertigo will most likely not have a positive finding on the Romberg test. 2. Patients with cerebellar lesions are more likely to exhibit ataxic gait. 3. Patients with a unilateral vestibular lesion will experience vertigo, postural instability, oscillopsia, and disequilibrium. The wide-based gait is an attempt to minimize trunk rotation and movement of the head, which can increase sensory conflict and vertigo in the patient with a unilateral vestibular lesion. 4. Mononeuropathy of the sural nerve is unlikely to cause a balance deficit.

A patient with complete C6 tetraplegia reports a 3-day history of increased lower extremity spasticity, left shoulder pain, and fever. Which of the following conditions is MOST likely responsible for these findings? 1. Decreased respiratory function 2. Autonomic dysreflexia 3. Urinary tract infection 4. Deep vein thrombosis

3. Urinary tract infection 1. Pulmonary dysfunction is a long-standing issue in patients who have C6 quadriplegia and would not cause the recent change in status (O'Sullivan, p. 863). 2. Autonomic dysreflexia results in hypertension, not left shoulder pain or fever, and has a rapid onset (O'Sullivan, p. 862). 3. Urinary tract infection is correct because spasticity can be influenced by internal factors such as a urinary tract infection (O'Sullivan, p. 862). Also, a urinary tract infection can be associated with fever and can refer pain to the ipsilateral shoulder. (Goodman, p. 364) 4. Symptoms of deep vein thrombosis include swelling, erythema, and heat in a localized area (O'Sullivan, p. 609). These symptoms are not described in the stem.

A patient with chronic obstructive pulmonary disease would MOST likely have a DECREASED value in which of the following pulmonary function tests? 1. Residual volume 2. Total lung capacity 3. Vital capacity 4. Functional residual capacity

3. Vital capacity 1. The amount of air remaining within the lungs when the expiratory reserve volume has been exhaled is called the residual volume (Goodman, p. 1723). Chronic obstructive pulmonary disease causes increases in residual volume due to air trapping (O'Sullivan, pp. 435-440). 2. At full inspiration, the lungs contain their maximum amount of gas. This volume of air is called the total lung capacity (Goodman, p. 1723). Total lung capacity is increased with chronic obstructive pulmonary disease due to destruction of alveolar walls (O'Sullivan, pp. 436-438). 3. Chronic obstructive pulmonary disease is a combination of airway narrowing, parenchymal destruction, and pulmonary vascular thickening. Vital capacity is an indicator of the amount of air that is expelled after the patient takes a maximal inspiration and follows that with a maximal expiration (Goodman, p. 1723). Chronic obstructive pulmonary disease causes a decrease in vital capacity (O'Sullivan, pp. 436-441). 4. Functional residual capacity is the amount of air in the lungs at the end of quiet exhalation (Goodman, p. 1723). Chronic obstructive pulmonary disease causes an increase in functional residual capacity due to air trapping (O'Sullivan, pp. 439-441).

Peds milestones mnemonics to remember

4-5 comes before 6-7 months. So P comes before S. 4-5 Months roll from Prone to Supine. 6-7 roll from Supine to Prone months is supine in SL. If you DOUBLE the time to 6 months, you can DOUBLE the roll so baby can now go supine to prone 3 months is supine in SL. If you DOUBLE the time to 6 months, you can DOUBLE the roll so baby can now go supine to prone 6 months: sits alone without support. Draw two eyes on top of you 6 and it looks like the baby is sitting there without support and hands-free 7 Months: 7 looks like an arrow directing. So you can think trunk rotation since the arrow is pointing you from one direction to another(right to left) Months 8-9: Eight rhymes with date. When you date you can STAND ALONE and you can CRUISE in your car Month 10-15: Walk unassisted since you're in DOUBLE DIGITS now and you can be in DOUBLE LE SUPPORT to WALK. TWO digits in 10, so you can use TWO fingers for pincer grasp and tower of TWO cubes Prone to supine - 2-3 months Head control pull to sit - 3-4 months Sitting 6 months Independent sitting with good trunk control 7 months Creeping 7-8 months Crawling 10 months Standing with support 8-10 months Walking 15 months Stairs step to: 2 years Step by step alternate; 2.5-3 years Tricycle 3 years Standing on tip toes 4 years

A patient has a fracture of the third metacarpal that occurred 2 months ago. The patient currently has heat, swelling, erythema, and tenderness at the fracture site. Which of the following courses of action would be MOST appropriate at this time? 1. Apply a compression glove. 2. Administer continuous ultrasound. 3. Instruct in active range of motion exercises. 4. Contact the physician.

4. Contact the physician. 1. Heat, swelling, erythema, and tenderness at the fracture site are signs of a nonunion fracture; a compression glove would not be an appropriate treatment. 2. Heat, swelling, erythema, and tenderness at the fracture site are signs of a nonunion fracture; continuous ultrasound may increase inflammation. 3. Heat, swelling, erythema, and tenderness at the fracture site are signs of a nonunion fracture; active range of motion may make the fracture worse. 4. Heat, swelling, erythema, and tenderness at the fracture site indicate a potential nonunion fracture. A referral to a physician is indicated.

Which of the following interventions is MOST important to perform in conjunction with manual lymphatic drainage? 1. Muscle strengthening 2. Muscle stretching 3. Joint mobilization 4. Diaphragmatic breathing

4. Diaphragmatic breathing 1. While strengthening exercise is recommended for lymphedema, it is not required for manual lymphatic drainage to be successful. 2. Muscle stretching is important for maintaining flexibility and range of motion, it is not required for manual lymphatic drainage. 3. While there are times when joint mobilization is indicated with this diagnosis, it is not a requirement for a successful outcome with manual lymphatic drainage. 4. Diaphragmatic breathing is necessary to create changes in intrathoracic pressure that results in increased lymphatic drainage in the thoracic lymphatic duct. It is recommended to be performed after manual lymphatic drainage.

A home health patient who recently had a three-vessel coronary artery bypass graft describes experiencing bilateral lower extremity swelling, leg pain, and shortness of breath, especially when lying down. The patient MOST likely has which of the following diagnoses? 1. Deep vein thrombosis 2. Myocardial infarction 3. Pulmonary embolism 4. Heart failure

4. Heart failure 1. A deep vein thrombosis corresponds to the leg pain and possibly swelling, but symptoms would typically be unilateral. (pp. 65-66) 2. A myocardial infarction corresponds to shortness of breath but typically does not result in acute swelling. A myocardial infarction could result in the development of heart failure, which would result in these symptoms, but this is a secondary result, not a primary result. (pp. 50-51) 3. A pulmonary embolism would result in shortness of breath, usually not changed by position, and typically cardiac arrest. (p. 66) 4. Typical signs of heart failure include dyspnea, paroxysmal nocturnal dyspnea, orthopnea, and peripheral edema. (p. 97)

A patient who walks with an antalgic gait reports hip pain, loss of appetite, and night sweats. The patient has a low-grade fever. Which of the following tests is MOST important to perform to confirm the diagnosis? 1. Squat test 2. Patrick test 3. McBurney test 4. Iliopsoas muscle test

4. Iliopsoas muscle test 1. The squat test is used to screen for a musculoskeletal source of hip pain, such as a fracture (p. 626), however does not take into account the patient's constitutional symptoms that may reflect pain of systemic origin. 2. The Patrick test assesses hip pain of musculoskeletal origin (p. 616) but does not take into account the patient's constitutional symptoms that may reflect pain of systemic origin. 3. The McBurney test is a test for appendicitis. Pain resulting from appendicitis usually begins in the umbilical region and may be localized to the right lower quadrant but is less likely to refer pain to the hip. (pp. 319-320) 4. The iliopsoas muscle test is a screening test for psoas abscess. A patient who has a psoas abscess will most likely have hip pain, a low-grade fever, antalgic gait, and night sweats. (pp. 312-313)

A physical therapist is examining a patient who reports anterior knee pain during activity. The patient's resisted knee extension is strong but painful. Palpation elicits tenderness over the patient's patellar tendon, but no structural faults are noted. Which of the following protective devices is MOST appropriate to facilitate a return to sports for the patient? 1. Hinged knee brace 2. Knee sleeve with lateral buttress 3. Thrust unloader brace 4. Infrapatellar strap

4. Infrapatellar strap 1. A hinged knee brace does not protect the patellar tendon. It is commonly used with medial collateral ligament or lateral collateral ligament injuries. (Magee, p. 758) 2. Buttresses are used to manage malalignments; this patient has no structural faults (Chui, p. 328; Magee, pp. 823-824). 3. Unloader braces are used to manage osteoarthritis symptoms by decreasing compressive loads in the joint compartment. Varus and valgus alignments are used to help with medial or lateral collapse. (Chui, p. 304) 4. An infrapatellar strap is most appropriate for treating patellar tendinitis in the absence of a patellar tracking problem (Magee, p. 762).

A patient with a brainstem infarction resulting in left lateral medullary syndrome is MOST likely to demonstrate which of the following symptoms? 1. Hemiparesis in the left side of the body 2. Hemiparesis in the right side of the body 3. Loss of discriminative touch and proprioception in the left side of the body 4. Loss of pain and temperature sensation in the right side of the body

4. Loss of pain and temperature sensation in the right side of the body 1. Ipsilateral hemiparesis is not a characteristic of lateral medullary syndrome. Ipsilateral symptoms may be loss of pain in the face. 2. Contralateral hemiparesis is not a characteristic of lateral medullary syndrome. Medial medullary syndrome is associated with contralateral hemiparalysis. 3. Ipsilateral loss of discriminative touch and proprioception is not a characteristic of lateral medullary syndrome. Ipsilateral symptoms may be loss of pain in the face. 4. Contralateral loss of pain and temperature is associated with lateral medullary syndrome.

A 17-year-old patient sustained a knee injury while playing soccer 2 weeks ago. The patient reports falling and landing with the right knee fully flexed. Since the injury, the patient reports the knee has had intermittent catching in motion and has been painful in full flexion. The patient MOST likely has which of the following diagnoses? 1. Osgood-Schlatter disease 2. Anterior cruciate ligament tear 3. Patellofemoral syndrome 4. Meniscal tear

4. Meniscal tear 1. Osgood-Schlatter disease is an apophysitis of the tibial tubercle. It is not due to a traumatic incident and is commonly seen in pubescent individuals with a report of pain and tenderness over the tibial tubercle. (p. 820) 2. Hyperflexion is not a common mechanism of an anterior cruciate ligament tear. Locking sensation is not a symptom of an anterior cruciate ligament tear. (pp. 721-722) 3. Patellofemoral pain syndrome is used to describe many disorders around the knee that result in anterior knee pain. The patient would report pain in the anterior portion of the knee that worsens during impact-loading activities. (pp. 820-825) 4. The symptoms of catching and pain at end range flexion are considered indicative of a meniscal tear (pp. 787-794).

A patient has a Stage 3 pressure injury on the left heel. The physical therapist notes that the wound contains 80% tan-yellow necrotic tissue. The wound is round, measures 3.8 cm in diameter, has a depth of 2 mm, and produces moderate amounts of drainage. The BEST initial emphasis for intervention for the first week of treatment would be to decrease which of the following measurement parameters? 1. Depth 2. Diameter 3. Exudate 4. Necrotic tissue

4. Necrotic tissue 1. Depth might actually increase as the necrotic tissue is removed. (p. 152) 2. Diameter might actually increase as the necrotic tissue is removed. (p. 152) 3. While managing wound exudate to prevent maceration is important (p. 175), removal of necrotic tissue to prevent spread of infection and facilitation of wound healing should be the primary goal in the first sessions of treatment (pp. 149-150). 4. Necrotic tissue must be decreased in order to make room for granulation tissues and enable wound healing. (pp. 149-150)

A patient reports a burning sensation along the medial aspect of the knee and lower leg. The sensation is increased during resisted hip adduction and knee flexion. The patient MOST likely has which of the following conditions? 1. Meralgia paresthetica 2. Neuropathy of the obturator nerve 3. Complex regional pain syndrome 4. Neuropathy of the saphenous nerve

4. Neuropathy of the saphenous nerve 1. Meralgia paresthetica is described as burning pain along the anterolateral aspect of the thigh that increases with hip extension (not hip adduction as described in the stem) (Dutton, p. 910). 2. The obturator nerve has no cutaneous innervation to the lower leg (Moore). Pain would reside in the adductor thigh compartment (Dutton, p. 910). 3. Complex regional pain syndrome of the lower extremity typically involves the foot and ankle. There are multiple characteristics present, such as pain, allodynia, hyperalgesia, abnormal vasomotor response, and abnormal sudomotor (sympathetic nervous system) activity. (Dutton, p. 268) 4. The saphenous nerve is cutaneous and supplies skin on medial aspect of leg and foot (Moore). Entrapment can cause pain at the medial side of knee. The pain is described as burning pain that increases with knee flexion and hip adduction. (Dutton, pp. 934-935)

An inpatient who is being seen in the physical therapy department vomits material with the appearance of coffee grounds. While reviewing the patient's medical record, the physical therapist notes that the patient recently started taking heparin. Which of the following conditions is the MOST likely cause of the coffee-ground emesis? 1. Diverticulitis 2. Gastroesophageal reflux 3. Ulcerative colitis 4. Peptic ulcer

4. Peptic ulcer 1. Diverticulitis refers to the presence of outpouchings in the wall of the colon or small intestine. Symptoms include passing of blood in the stool but do not include coffee-ground emesis. (p. 892) 2. Gastroesophageal reflux disease is defined as an inflammation of the esophagus due to a backward flow of gastric juices. The clinical symptoms include painful swallowing, heartburn, and dysphagia. Vomiting is not a classic symptom. (p. 869) 3. Ulcerative colitis is a chronic inflammatory disorder of the mucosa and submucosa of the colon. It is characterized by diarrhea and rectal bleeding. Symptoms do not include coffee-ground emesis. (pp. 883-885) 4. Signs of peptic ulcer disease include weakness, diaphoresis, epigastric pain, and coffee-ground emesis. Coffee-ground emesis results from bleeding that has slowed or stopped, with conversion of red hemoglobin to brown hematin by gastric acid. Heparin is an anticoagulant and is associated with a higher risk of gastrointestinal bleeding. (p. 878)

Which of the following strategies for intervention would be MOST appropriate for a patient with moderate Alzheimer disease and urinary incontinence? 1. Continuous catheterization 2. Functional exercises 3. Biofeedback 4. Prompted voiding

4. Prompted voiding 1. Continuous catheterization is usually used for patients who have a neurological deficit that directly affects urinary function, such as a spinal cord injury, and places the patient at risk for developing a urinary tract infection (Lazaro, p. 424). 2. Functional exercises that incorporate appropriate activation of pelvic floor muscles are recommended for patients with stress urinary incontinence, however, require correct activation of the muscles and may be challenging to teach to patients with a moderate cognitive deficit (Avers, pp. 515-516). 3. Biofeedback can be used as an adjunct to functional exercises, but the patient must be cognitively aware to use biofeedback (Avers, p. 517). 4. Prompted voiding is used for patients who may have cognitive deficits and require reminders of regular intervals for voiding (Avers, p. 520).

An 18-month-old child with developmental delay collapses into a rounded "C"-shaped posture of forward flexion when placed in a sitting position. Which of the following techniques is most appropriate INITIALLY to improve the child's posture? 1. Seat the child in a chair with an abduction wedge. 2. Apply approximation through the spinal joints. 3. Present toys from above so that the child must look up. 4. Provide postural correction at the pelvis.

4. Provide postural correction at the pelvis. 1. An abduction wedge addresses hip alignment, not forward flexed posture. 2. Without the pelvis in proper positioning, this strategy is ineffective. 3. Without the pelvis in proper positioning, this strategy is ineffective. 4. This technique will address the posterior pelvic tilt associated with "C"-shaped posture.

A patient with undiagnosed type 2 diabetes is MOST likely to have which of the following signs or symptoms? 1. Overall decrease in urinary output 2. Reported shakiness after exercise 3. Increased heart rate variability with exercise 4. Stocking-glove numbness of the lower extremities

4. Stocking-glove numbness of the lower extremities 1. Increased, not decreased urinary output is a symptom of undiagnosed or poorly controlled type 2 diabetes (p. 403). 2. Shakiness after exercise is a common complaint of hypoglycemia. Undiagnosed individuals would tend to be hyperglycemic. Exercise is likely to further raise blood glucose levels in patients with undiagnosed type 2 diabetes. (p. 407) 3. Individuals with type 2 diabetes often show reduced heart rate variability due to dysfunction of the parasympathetic and sympathetic nervous system (p. 404). 4. Since multiple nerves are affected with diabetic neuropathy, a stocking-and-glove pattern of loss of sensation is present rather than an alteration in sensation that corresponds to a nerve root or peripheral nerve distribution (pp. 405-406).

Which of the following individuals is authorized to have access to the medical records of a patient who is in the intensive care unit after a motor vehicle accident? 1. The patient's attorney, while preparing a civil suit related to the accident 2. The human resource representative at the patient's employer, inquiring about a return-to-work date 3. The patient's brother, who is providing financial support to the patient's family 4. The patient's friend, who is named in the durable power of attorney for health care

4. The patient's friend, who is named in the durable power of attorney for health care 1. Written consent from the patient or the patient's power of attorney would be required before releasing the patient's medical record to an attorney. 2. Written consent from the patient or the patient's power of attorney would be required before releasing the patient's medical record to a human resource representative. 3. Written consent from the patient or the patient's power of attorney would be required before releasing the patient's medical record to a family member. 4. The patient executing a durable power of attorney may appoint a friend as his or her health care decision-maker. This person would have access to the patient's medical record.

The manager of a work-hardening clinic has data showing that 42% of the clinic's patients did not return to work at their previous level of function. To improve this percentage, which of the following areas would be the MOST appropriate focus for a quality improvement study? 1. Sources of patient referrals 2. Speciality certification of the clinic's therapists 3. Types of diagnoses 4. Treatment plans of care

4. Treatment plans of care 1. In this case, referral sources are not likely to affect outcomes, but the practice must be aware of how access to physical therapy (direct access, practice without referral or self-referral) may influence reimbursement for services provided (pp. 194-195). 2. Care should be based in the science and evidence for the treatment approach. Specialist certification may provide improved plans of care, but how the plan is executed is more likely to affect outcomes. (p. 110) 3. If the problem for quality improvement was rate of return to work, then all diagnoses would be considered a factor, not specific types. 4. Quality is defined as the degree to which an organization increases the likelihood of desired outcomes. Quality depends on the integration of several factors: what happens to the patient, the care delivered by the clinician, the organization's management, and the external environment where policies are made. In this situation, outcomes are most likely to improve by focusing on the plans of care. (p. 111)

A 6-year-old patient who has Legg-Calvé-Perthes disease is MOST likely to have which of the following gait deviations? 1. Vaulting 2. In-toeing 3. Toe-walking 4. Trendelenburg

4. Trendelenburg 1. Legg-Calvé-Perthes disease affects the hip and is not typically associated with vaulting. Vaulting is typically seen with hemiparetic limbs. 2. Legg-Calvé-Perthes disease affects the hip and is not typically associated with in-toeing. Further, limited range of motion into hip medial (internal) rotation is common in patients who have Legg-Calvé-Perthes disease, making in-toeing even less likely. 3. Legg-Calvé-Perthes disease affects the hip and is not typically associated with toe-walking. Toe-walking can be caused by short ankle plantar flexors or excessive tone. This is not seen in Legg-Calvé-Perthes disease, which typically only affects the muscles of the hip. 4. A positive Trendelenburg sign secondary to hip pain and/or hip abductor weakness is very common in children who have Legg-Calvé-Perthes disease. Although Legg-Calvé-Perthes disease can affect children of nearly any age, it is most common among boys ages 4 to 8 years.

A patient with a recent prostatectomy has urinary incontinence. Loss of support to which of the following structures is MOST likely contributing to the incontinence? 1. Ureter 2. Ductus deferens 3. Levator ani 4. Urethra

4. Urethra 1. The ureters connect the kidneys to the bladder and are not in proximity to the prostate gland. 2. During the pelvic part of its course, the ductus deferens maintains direct contact with the peritoneum; no other structure intervenes between them. A vasectomy is the procedure in which the ductus deferens is ligated. It is not affected by prostatectomy. 3. The levator ani muscle is a skeletal muscle that attaches to the pelvis and is not DIRECTLY impacted by removal of the prostate. When the prostate is removed, there will be an expected period of incontinence lasting up to 6 weeks. Strengthening of the pelvic floor muscles can be a compensatory strategy for the loss of the support previously supplied by the prostate. 4. Prostatectomy indicates removal of the prostate gland. The prostate gland provides mechanical support to the urethra between the bladder neck and the penis.

28-year-old woman, post-partum 3 months is working out in a gym with a therapist. PT has decided to start training her on therapeutic ball. With the patient sitting on the ball, which direction should the ball be displaced to train her abdominal muscles? A. Anteriorly B. Posteriorly C. Sitting on the ball and flexing and extending knee D. Training on ball not recommended

A. Anteriorly -abdominals are contracted with a posterior pelvic tilt which it promoted with the ball anteriorly

A PT is ordered to examine a 76 year old patient who has suffered a recent stroke. The occupational therapist informs the PT that the patient has apraxia. She cannot drink from the bottle on command. However, she can point out to the bottle and verbalize the purpose of the bottle. From this information, what sort of apraxia does this patient have? How should the PT approach treatment? A. Ideomotor apraxia; The PT should speak in short concise sentences. B. Ideational apraxia; The PT should always give the patient 3-step commands C. Ideomotor apraxia; The PT should always give the patient 3-step commands D. Ideational apraxia; The PT should speak in short concise sentences.

A. Ideomotor apraxia; The PT should speak in short concise sentences. Should NOT do 3 step commands because the pt can't do things on command

Arterial insufficiency vs Venous insufficiency

Arterial insufficiency: a lack of adequate blood flow to a region or regions of the body Venous insufficiency: inadequate drainage of venous blood form a body part, usually resulting in edema and/or skin abnormalities and ulcerations

In the picture, the therapist is putting pressure on the patient's head. Which of the following patient responses will demonstrate a behavioral or psychological problem? A. The patient reports fear of falling as the therapist increases the pressure B. The patient reports severe back pain C. The patient reports severe pain on the forehead D. The patient showed turns the head to the right side

B. The patient reports severe back pain

A patient with a diagnosis of SCI is having difficulty learning how to transfer from mat to wheelchair. The patient just cannot seem to get the idea of how to coordinate this movement. The MOST effective use of feedback during early motor learning is to: A. Provide feedback only after a brief delay. B. Focus on knowledge of results and visual inputs. C. Focus on proprioceptive inputs. D. Focus on knowledge of performance

B. Focus on knowledge of results and visual inputs. -KR is better for early motor learning -KR before KP

A 6-year-old male child is referred to PT. The mother is concerned that child is using his hands to climb up their legs in order to stand up. Which of the following condition the PT can suspect? (image of kid doing Gower's sign) A. Cerebral palsy B. Muscular Dystrophy C. Spina bifida D. Down Syndrome

B. Muscular Dystrophy

A patient underwent surgical repair of Full Thickness Rotator cuff tear 6 days ago. PT wants to perform passive and assisted movement. Which position will be MOST appropriate for the patient? A. Patient in sitting with arm abducted 70 degrees with slight flexion. B. Patient is supine with arm abducted 45 degrees with slight flexion. C. Patient in sitting with arm abducted 15 degrees with slight flexion. D. Patient in supine with arm abducted 90 degrees with slight flexion.

B. Patient is supine with arm abducted 45 degrees with slight flexion.

Head/neck and Trunk Lift What PNF patterns is your R and L UE going in to when lifting from Right to Left?

-Going from feet to the air -Lead arm is the Weak arm -Assist arm is the Strong arm When you lift from Right to Left: -Lead arm starts from D2 ext into D2 flex -Assist arm starts from opposite wrist into D1 flex

Rules for electrode placement and sensitivity for biofeedback?

1 specific muscle = closely spaced electrodes multiple/more muscle activity = wide spread electrodes weak/flaccid muscle = high sensitivity spastic muscle = low sensitivity

When is Apgar screening test performed for newborns?

1, 5, 10 minutes after birth Continues every 5 minutes if infant is having difficulties

Distinguish between 1st degree/2nd degree/3rd degree heart block?

1st degree = delay in conduction -increased PR interval (> 0.2 second) (large box) 2nd degree = partially blocked conduction -increased (type 1) or consistent (type 2) PR interval before a QRS drop, then cycle is repeated ex: Delayed American Airlines flight says its delayed 2 hours, then 4 hours, then 10 hours, then the flight is dropped to the next day 3rd degree = fully blocked conduction

Which system is affected if swaying is found with each Clinical Test of Sensory Interaction on Balance (CTSIB)?

2 = Somatosensory affected 4 = Vision affected 5 = Vestibular affected

Which lead should you look at on a EKG image for the NPTE exam?

2nd lead and cleanest image with no labels

Types of Lifts and Transfers

3 Person LIft/Carry -Used to transfer a pt from a stretcher to a bed or treatment plinth Ex: football field injury 2 Person Lift -Used to transfer pts of different heights or surfaces or transferring to the floor Dependent squat pivot transfer -Used to transfer a pt who cannot stand independently, but CAN bear some weight through the trunk and LE Hydraulic Lift -Device used for dependent transfers when the pt is obese, there is only one therapist available to assist with the transfer or pt is totally dependent

Different Types of Point Gait with AD

3 Point -3 points of contact at the same time (2 feet and crutch) 2 Point -2 point of contact at the same time (L foot with R crutch or R foot with L crutch) 4 Point -4 points of contact SEPARATELY -slowest form

A 48 year old male patient reports of shortness of breath and complains of fatigue. During the examination, the PT examines patient's heart sounds before starting an exercise program. Which valve is being auscultated in the picture? (slide 29 FF #8) A. Tricuspid valve B. Pulmonary valve C. Mitral Valve D. Aortic Valve

A. Tricuspid valve

A physical therapist is observing a 67 year old female patient walk, and immediately notes a forward trunk lean. A LIKELY contributing cause for this deviation is: A. Excessive Ankle Dorsiflexion B. Quadriceps Spasticity C. Ankle Plantar Flexion Weakness D. Ankle Plantar Flexion Contracture

D. Ankle Plantar Flexion Contracture Pt is inn anterior pelvic tilt causing them to balance their COG by forward trunk lean "Pt has ankle PF contracture, knee hyperextension, anterior pelvic tilt, anterior trunk lean"

Distinguish content validity vs construct validity

Content validity: test should measure specifically what the pt problem is ex: for testing balance, BERG balance CAN be done (it's one of the many options to test balance) Construct validity: test should measure what its supposed to measure ex: goni HAS TO and is the ONLY test that measures ROM

Shoulder: 4 Functional Muscle Groups?

Cuff Protectors/Stabilizers -Rotator cuff (SITS) -Long head of biceps Scapular Positioners -Scapular-thoracic muscles GH "Powerers" -Deltoid UE Propellers -Thorax to Humerus

Purpose of Diazepam/Nitroglycerin/Heparin/Lipitor?

Heparin -anticoagulant/blood thinner for DVT Nitroglycerin -dilate coronary arteries -used to prevent or relieve angina symptoms Diazepam -used to relax skeletal muscles and decrease muscle spasm Lipitor -anti-cholesterol agent used to manage hypercholesterolemia and mixed dyslipidemias -used for fat

Temperature settings for: Hot pack Paraffin bath Contrast bath

Hot pack -165-170 F Rx temperature -6-8 layers -Hot packs peak heat is in first 5 min, during this time pt is at greatest risk of burns -treatment time = 20-30 min Paraffin bath -Rx temp = 125-127 F -Rx time = 15-20 min -Used on wrist and hands or feet with are irregular distal areas -Used before exercise mobilization Contrast bath -Immerse involved limb in 100-111 F water for 4 min -Then immerse involved limb in 55-65 F water for 1 min

Increased sensitivity to pain?

Hyperalgesia

Increased sensitivity to sensory stimuli?

Hyperesthesia

Iontophoresis: Negative Ions from medicine and Indications

I SAD = SACM Iodine = Sclerotic scars Salicylate = Analgesic Acetate = Calcium deposits Dexamethasone = MSK inflammation I(eye)odine = Sclerotic scar (eye scar) SALIcylate = ANAlgesic (Sali and Ana) ACetate = CAlcium deposit (AC-CA) dexaMethasone = Msk inflammation -Anode = Positive -Cathode = Negative

What is positive for bronchophony?

If the sound is CLEAR when asking the pt to say "99" I got 99 problems but a BRONCHOPHONY ain't one

List the shoulder ER muscles

Infraspinatus Teres minor

How would cystic fibrosis effect pulmonary function tests?

Increased RV and FRC (RV + ERV) Decreased FEV1

3 main skin receptors?

Krause end bulbs "Santa Krause (cold)" Ruffini endings = detects heat "Dog's warm breath goes RUFFRUFF" Pacinian corpuscles = detects pressure and vibration sensory modalities "Pachinny pressure/vibration"

What are the 2 special signs associated with MS?

Lhermittes' Sign -neck flexion causes electric shock sensations down spine Uhtoff's Phenomenon -increased neurological symptoms in response to heat (body gets overheated from hot weather, exercise, fever, or sauna, and hot tubs)

Low or high stretch bandages used for phase 1 treatment lymphedema? What stage should compression garments be used during the day for lymphedema pts?

Low stretch When transitioning to a maintenance phase of treatment from phase 1

Waddell Sign

Organic pain -pain caused by tissue damage, any illness or injury is organic pain Non-organic pain -abnormal illness behaviors exhibited by pts who have depression, emotional disturbance, or anxiety states Presence of 3 of 5 Waddell signs have been correlated significantly with disability (look at slide 25 on FF #28)

What is FRC (functional residual capacity)?

Residual volume + Expiratory Reserve Volume Air left in the lungs after regular exhalation

What is Gamekeeper's thumb?

Sprain/rupture of the ulnar collateral ligament of MCP joint of first digit

How to stress tibial nerve/sural nerve/common peroneal nerve? What test is this used for?

TED: tibial nerve: eversion/DF SID: sural nerve: inversion/DF PIP: common peroneal nerve: inversion/PF SLR

What is confidence interval?

Standard deviation that measures the variability of data (bell curve) 34 13 2

Drugs that Regulate Blood Cholesterol: Drugs that lowers high cholesterol? SE of the drug?

Statins: -Lipitor (atorvastatin) -Pravachol -Zocor -Livalo -Crestor SE Rhabdomyolysis -dark colored urine -muscle pain -soreness -stiffness

Inability to perceive heat?

Thermanalgesia

Which CN is responsible for anterior 2/3 tongue sensation?

Trigeminal nerve (CN5)

Which parts of the lungs are cleared in a SUPINE bed flat position?

Upper lobes: Anterior segment

What are the scapular upward rotation muscles?

Upper trap Serratus Anterior Lower trap

Fremitus

Vibration that is produced by the presence of secretion in the airways -More air in the area = decreased fremitus -More secretions in the area = increased fremitus fremitus = vibration, the more vibration you feel on the pts back when you ask the pt to say 99, the more secretions they have Fremitus - vibration that is produced by the voice or by secretions or increased tissue density in the airys Increased fremitus = increase density in the lung spaces (consolidation/collapse) Examples: Pneumonia Tumor or mass CF Bronchitis Decrease in Fremitus Pneumothorax Hemothorax Pleural effusion Emphysema

What is ERV (expiratory reserve volume)?

Volume of air that can be exhales in excess if tidal breathing

What is IRV (inspiratory reserve volume)?

Volume of air that can be inspired when needed

Repair of a SLAP Lesion

When biceps tendon is detached, progress cautiously -limit PROM or AROM elevation of UE to 60 degrees for first 2 weeks -limit same movement to 90 degrees at 3-4 weeks -only perform PROM humeral rotation with shoulder in SCAPULAR plane for first 2 weeks -progress to 30 degrees of ER and 60 degrees of IR at 3-4 weeks -AVOID tension in the biceps (elbow ext with shoulder ext) for first 4-6 weeks -AVOID active biceps contraction for 6 weeks

Rules for ABG: compensate/uncompensated/partially compensated

Within normal pH = Compensated None of 3 are normal = Partially Compensated Either CO2 or HCO3 is out of norm range = Uncompensated

Touch on cheek causes turning head to same side with mouth open a. Palmar grasp reflex b. Plantar grasp reflex c. Rooting reflex d. Moro reflex

c. Rooting reflex 28 weeks of gestation to 3 months

A PT is assessing a patient with a chief complaint of low back pain and a diagnosis of L2-L3 nerve compression. Which of the following gait deviations at the hip and pelvis are NOT to be expected in this patient? Select one: a. Trunk lurches backward and toward the unaffected stance leg from heel off to mid swing b. Posterior tilt of the pelvis during initial swing c. Semicircle movement of the hip during swing—combining hip flexion, hip abduction and forward rotation of the pelvis d. Excessive hip flexion at initial and mid swing

d. Excessive hip flexion at initial and mid swing Rationale: L2-L3 nerve compression leads to weak hip flexors. Hence excessive hip flexion will not be seen at the initial and mid-swing phase of the gait cycle. Options A: Hip flexion is passively generated by a backward movement of the trunk Option B: Abdominals are used during initial swing to advance the swing leg Option C: Semicircular movement combining hip flexion, hip abduction, and forward rotation of the pelvis to compensate for hip flexor weakness.

Rule for hard and soft bumpers for heel cushion?

hard heel cushion/stop = knee flexion soft heel cushion/stop = knee extension

· Which muscles are innervated by the axillary nerve?

o Deltoid o Teres minor

What is the abdominal reflex and what does it test?

Contraction of the superficial abdominal muscles when stroking abdomen lightly (+) Asymmetry: signifies UMN lesion on the absent side

Muscles that do ankle eversion/DF

Peroneal tertius Extensor digitorum longus (Deep peroneal nerve)

Anterior Cord Syndrome

"ANThony went to a headbangers concert and now has bilateral UE/LE loss and can't feel pain or temperature" -cervical flexion injury -bilateral motor loss -pain/temp/light touch loss -below level of lesion

Safety and Precautions: Hickman Catheter vs Swan Ganz Catheter

"Arterial Line: Arterial BP, ABG PICC Line: Administer drugs, food Hickman Line: CVP, Blood analysis Swan Ganz: Balloon in right side of heart"

Brown Sequard Syndrome

"Brown got stabbed on his L side and lost all strength and special sensation on his L side but his R side had only pain and temperature loss" -ipsilateral motor loss -ipsilateral tactile discrimination, vibratory, proprioceptive loss -ipsilateral spastic paralysis -contralateral pain and temp loss -caused by stab wound

Safety and Precautions: Ergonomic Guidelines Workstation Recommendations

-18-20 inch monitor -Monitor display should be 10 degrees below the horizontal and 20 inches away from eyes -Wrist rest should match the front edge of the keyboard for maximal comfort -Use a mouse that contours the hand -Space under desk should be at least 30 inches wide, 19 inches deep, 27 inches in height, should be 2-3 inches between the top of the thighs and desk

Vertebrobasilar Artery Syndrome: Locked in Syndrome

-Basilar artery infarct of bilateral pons -Rapid progression from hemiparesis to tetraplegia -CN paralysis (CN 5-12) -Rapid progression from dysarthria to anarthria -Result = pt cannot move or speak BUT is alert and oriented

CVA at PCA symptoms

-Contralateral homonymous hemianopsia -Memory deficits -Visual agnosia -Prosopagnosia (difficulty naming people on sight) -Central post-stroke (thalamic) pain "Pt name Patrick has contralateral homonymous hemianopsia, memory deficits, visual agnosia, prosopagnosia, and central post stroke pain"

CVA at MCA symptoms

-Most common location for ischemic stroke and most likely on the NPTE -Contralateral motor AND sensory loss involving the UE and face (UE>LE) -Contralateral homonymous hemianopsia (can only see on one half of each eye on same side) -Dominant side (Left CVA) = Aphasia (Broca's & Wernicke's) -Nondominant side (Right CVA) = Perceptual problems (unilateral neglect, depth perception, spatial relations, agnosia, apraxia) "Pt named Larry has aphasia and motor/sensory loss at the R UE" = has a L CVA "Pt name Rachel has difficulty with perceiving the world and neglects her L side, has trouble with depth perception, spatial relations, recognizing objects/textures (agnosia), and imitating movement (apraxia)" = has a R CVA

Intervention goals for MS pts?

-Optimize mobility -Maximize physical function -Minimize loss -Fatigue and heat sensitivity combined with weakness and spasticity, often limit people with MS from getting enough exercise -Consider morning (AM) exercises due to decreased core temperature and fatigue and Submax intensity (50-70%)

What is preeclampsia?

-Pregnancy induced acute hypertension after the 20th week of gestation (usually not within 1st trimester) -Increase in protein in urine, hyperreflexia, edema, visual disturbances, headache, sudden weight gain seen -A blood pressure reading greater than 140/90 and a second abnormal blood pressure reading FOUR HOURS after the first confirms the diagnosis -THIS IS AN EMERGENCY SITUATION

What is Aphasia?

-Pts with lesions involving the cortex of the dominant hemisphere (L hemisphere) demonstrate speech and language impairments -Acquired communication disorder caused by brain damage and is characterized by an impairment of language comprehension, formulation, and use -Global aphasia = indicative of extensive damage and involves both production and comprehension of language

RLA Level 5 Interventions

-Quiet environment -Highly structured functional tasks -Give control to the pt when safe and appropriate -Give options -Use cues/external direction -Use AD -Treatment plans should include using a daily planner

Position During Ambulation on Level Ground

-Stand behind and slightly toward the pt's involved LE -PT's hand should be nearest to the pt to grasp under the back of the gait belt with FOREARM SUPINATED -PT should move forward in step with the pt; PT's forward foot moves with the AD

What causes Clubbed nails?

-hypertrophy of underlying soft tissue -respiratory or cardiac problems like COPD, emphysema, congenital heart defects, cor pulmonale

Rigidity (PD)

-increased resistance to passive movement that affects both agonists and antagonists -cogwheel rigidity: jerky, ratchet like resistance to passive movement as muscles alternately tense and relax. It occurs when tremor coexists with rigidity -lead pipe rigidity: a sustained resistance to passive movement, with no fluctuations -rigidity is often asymmetrical, especially in the early stages of PD. Typically affects proximal muscles first, especially the shoulders and neck, and it progresses to involve muscles of the face and extremities

During the test in the photograph, the patient demonstrates hyperreflexia. What is the MOST likely cause of this finding? (image of patellar reflex test) 1. A spinal cord tumor at L1 2. A herniated nucleus pulposus at L4 3. Femoral nerve impingement 4. A cauda equina lesion

1. A spinal cord tumor at L1 1. Pressure on the spinal cord at L1 would cause signs of upper motor neuron lesions below this level (pp. 1392-1393). 2. A weak or absent patellar reflex is expected with nerve root L4 compression (pp. 1392-1393). 3. Peripheral nerve injuries, such as a femoral nerve impingement, would result in a diminished patellar reflex (pp. 94-95). 4. Peripheral nerve injuries, such as a cauda equina lesion, would result in a diminished patellar reflex (pp. 135-136).

Inability to localize a sensation?

Atopognosia

What is the COPD GOLD Classification FEV1?

COPD GOLD Classification: FEV1 >80 = mild 50-80 = moderate 30-50 = severe <30 = very severe

On examining a patient with a decreased trunk side bending to the left, the PT finds a left closing restriction between L2 L3 vertebrae. What is the MOST appropriate intervention to improve the restriction? A. Apply PA glide on L2 transverse process on the right side B. Apply PA glide on L4 transverse process on the right side C. Apply PA glide on L4 transverse process on the left side D. Apply PA glide on L3 transverse process on the left side

D

What is CRPS type 1?

Develops from noxious event Spontaneous pain or allodynia/hyperalgesia Edema, vascular abnormalities Non-nerve origin

What type of sensations is the Dorsal Column Medial Lemniscal Pathway in charge of?

Discriminative touch: Stereognosis (recognize form/shape by touch) Tactile pressure Barognosis (recognize weight) Graphestesia (recognize writing on skin by touch) Texture recognition Kinesthesia (awareness of position and movement) 2 point discrimination Proprioception Vibration

Expiration muscles?

Internal intercostals Quadratus lumborum Rectus abdominis

Rules for High Voltage Pulse Galvanic Current (HVPC)

Negative electrode -used to promote healing of inflamed or infected wounds Positive electrode -used to promote healing of wounds without inflammation Pulse frequency is same for both negative and positive = 60-125 PPS 5 days/week 45-60 min "inflammation is bad - negative; proliferation is good - positive"

Neuropraxia vs Axonotmesis vs Neurotmesis

Neuropraxia -segmental demyelination -temporary sensory symptoms "Covering of nerve is hurt" Axonotmesis -loss of axonal continuity but connective tissue coverings remain intact -Wallerian degeneration distal to lesion "Just axon is cut, but nerve is fine" Neurotmesis -complete severance of nerve fiber with disruption of connective tissue covering -muscle fiber atrophy and sensory loss "whole nerve is cut"

Difference between Nominal vs Ordinal and is it quality or quantity?

Nominal is data in GROUPS: male/female, blood type, type of breath sounds, types of arthritis ex: M&Ms Ordinal is data measured in RANKS ex: MMT, levels of assistance, joint laxity grades, starbucks cups Quality (NO quality) NOn-parametric data

Different types of scars?

Normal scar- Flat and similar to skin color Hypertrophic scar- Hypertrophic scarring would describe a healed wound with thick fibrous tissue that remains within the original wound border (multiple layers of scars in the same area) Keloid- Keloid scarring is a condition in which excessive scar tissue grows outside of the original margins of the wound (multiple layers of scars outside of the original area) "K"eloid is "K"razy, as it goes wild and outside of the original wound margin. Which type of burn is has hypertrophic/keloid scarring? -Deep partial thickness "Hong Kong Department of Police"

Which part of the lung are cleared in a L sidelying position on a DECLINED bed position?

Right middle lobe

Which muscles are being used with R lateral deviation of the mandible?

R lateral pterygoid L medial pterygoid (review video) "LATERAL" deviation —> matches "LATERAL" pterygoid so you know it's the IPSILATERAL MUSCLE since it's the SAME word so SAME side. Now you know the opposite side (Contralateral) will be the MEDIAL pterygoid.

Rule for lymphatic drainage?

Summary - Stroking: Distal to Proximal - Decongestion: Proximal to Distal - Exercise: Proximal to Distal

What is heart failure?

Syndrome characterized by impaired cardiac pump function, resulting in inadequate systemic perfusion and an inability to meet the body's metabolic demands

2 most common conditions with RA pts?

Ulnar drift (ulnar deviation) Palmar subluxation at the MCP

Explain what happens your knee and hip and pelvic when you supinate the foot

knee: genu VARUS hip: coxa VALGUM pelvis: posterior rotation (hip rises and moves posteriorly causing the pelvis to move with it posteriorly)

A patient reports leg pain that begins after 3 minutes of walking and is relieved by lying down and hanging the leg off the side of the bed. Which of the following tests or measures should the physical therapist use to identify the degree of dysfunction? 1. Ankle-brachial index 2. Dynamic Gait Index 3. Visual analog scale 4. Angina scale

1. Ankle-brachial index 1. The patient exhibits signs of vascular intermittent claudication, which is associated with arterial insufficiency. The ankle-brachial index (ABI) compares or indexes the ankle blood pressure to the arm blood pressure and is a noninvasive measure of arterial blood supply. (Bryant, p. 194) 2. Dynamic Gait Index is a test of ability to modify gait in response to task demands, not for arterial insufficiency (Shumway-Cook). 3. Visual analog scales are used to estimate perceptions of pain and do not indicate degree of organic dysfunction (Bryant, p. 391). 4. An angina scale is used to quantify chest pain and is not relevant for the condition described in the patient scenario (Goodman).

A patient being evaluated for a C4-C5 herniated disc also reports having periodic dizziness. Which of the following actions should the physical therapist perform INITIALLY? 1. Ask questions about the precipitating factors for the dizziness. 2. Ask questions about sensory changes. 3. Assess deep tendon reflexes of the upper extremities. 4. Assess the integrity of the vertebral artery system.

1. Ask questions about the precipitating factors for the dizziness. 1. Dizziness is associated with disorders in many different systems as well as drug reactions. A more specific definition will lead the physical therapist to which system should be investigated in more detail. 2. The problem of dizziness should be further clarified before going on to another area, such as screening for changes in sensation. 3. After taking a complete history, the therapist should then perform an upper quarter screening, which would include assessing deep tendon reflexes of the upper extremities. 4. For the patient with dizziness, the integrity of the vertebral artery should be tested during the specific tests and measures section of the examination, which follows the history and systems screening process.

Which of the following integumentary system observations is consistent with skin cancer? 1. Dark, raised spot that bleeds with minimal contact 2. Redness, scaling, and edema of the skin 3. Red fluid-filled vesicles that vary in size 4. Ring-shaped pigmented patches covered with scales

1. Dark, raised spot that bleeds with minimal contact 1. These observations are classic signs of malignant melanoma (p. 435). 2. These observations are signs of contact dermatitis (p. 423). 3. These observations are signs of herpes zoster (p. 351). 4. These observations are signs of tinea corporis, also known as ringworm (p. 428).

A patient reports constant bilateral posterior calf pain with itching. The examination reveals a body temperature of 98.8°F (37.1°C), normal temperature of the lower extremities, and tenderness to palpation and crusting of the skin in the affected areas. Which of the following conditions is MOST likely present? 1. Dermatitis 2. Cellulitis 3. Atrophie blanche 4. Superficial venous thrombosis

1. Dermatitis 1. Symptoms of dermatitis include lack of fever, itching, normal temperature of the lower extremities, inflammation, tenderness to touch, presence of vesicles and crusting, and either unilateral or bilateral distribution (Bryant, pp. 178, 211). 2. Although the patient's limb is painful, the other reported findings are not consistent with cellulitis. Signs of cellulitis include fever, elevated temperature of the lower extremities, erythema, inflammation, tenderness, one or a few bullae, no lesions, and unilateral distribution. (Bryant, p. 237) 3. Atrophie blanche is characterized by white, scar-like areas associated with pain. Affected areas present as smooth white plaques (Bryant, p. 210). 4. Superficial venous thrombosis is characterized by pain and tenderness along the affected vein, usually the saphenous vein. Crusting of the skin is not a feature of this condition. (Goodman, pp. 647, 649)

A patient had a total hip arthroplasty with a constrained (cemented) prosthesis using a posterolateral approach. Which of the following recreational activities has the LEAST risk of causing prosthetic loosening or dislocation for this patient? 1. Distance freestyle swimming 2. Water skiing 3. Stationary rowing 4. Playing racquetball

1. Distance freestyle swimming 1. High-impact sports are contraindicated for this patient. Hip positioning into end-range flexion, adduction, and medial (internal) rotation is also contraindicated. Freestyle swimming does not involve either of these two concerns. 2. Water skiing involves both high-impact activity and the possibility of positioning in end-range flexion, both of which are contraindicated for this patient. 3. Stationary rowing involves positioning in end-range flexion, which is contraindicated for this patient. 4. Playing racquetball involves high-impact activity, which is contraindicated for this patient.

Avascular necrosis of the hip is caused by multiple etiologies resulting in an impaired blood supply to the femoral head. Which of the following ROMs are decreased in the hip with avascular necrosis? Select one: 1. Flexion, internal rotation, abduction 2. Flexion, internal rotation, adduction 3. Flexion, external rotation, abduction 4. Flexion, external rotation, adduction

1. Flexion, internal rotation, abduction Answer: "FABIR" (F flexion, AB abduction, IR internal rotation) all have a decreased ROM at the hip with avascular necrosis. Avascular necrosis has symptoms including pain in the groin and/or thigh, and tenderness with palpation at the hip joint

A physical therapist is using lumbar mechanical traction with the patient in supine position for a patient who weighs 180 lb (81.6 kg). The traction weight is initially set to 60 lb (27.2 kg). The patient reports not being able to feel any pull. Which of the following actions is MOST appropriate for the therapist to take? 1. Increase the pull to approximately 50% of the patient's body weight. 2. Switch to manual traction. 3. Substitute 40 lb (18.1 kg) of pull in prone position. 4. Discontinue the treatment.

1. Increase the pull to approximately 50% of the patient's body weight. 1. A traction force of 30% to 50% of the patient's weight is necessary to cause effective segmental vertebral separation. This patient could have a force up to 90 lb (40.8 kg). (Cameron, p. 387; Placzek) 2. Manual traction would be less likely to produce the needed force (Cameron, p. 396). 3. More weight, not less weight, is needed. Prone position could be considered, depending on the examination findings (Cameron, p. 387; Placzek). 4. Increasing the traction force should be considered prior to discontinuing the treatment. Traction should be discontinued if symptoms peripheralize. (Cameron, p. 380)

A patient has sensory deficits to pin-prick and muscle weakness in the median nerve distribution. Based on these findings, which of the following should be expected with testing of median nerve conduction velocity and latency? 1. Increased latency and decreased nerve conductive velocity 2. Increased latency and increased nerve conductive velocity 3. Decreased latency and decreased nerve conductive velocity 4. Decreased latency and increased nerve conductive velocity

1. Increased latency and decreased nerve conductive velocity 1. The clinical examination suggests compromise of the median nerve. Latency would be increased and would be inversely proportional to conduction velocity. 2. Velocity equals distance/time. The time is latency. Therefore, velocity and time always vary inversely. 3. Velocity equals distance/time. The time is latency. Therefore, velocity and time always vary inversely. 4. When a nerve is damaged, the electrical signals take more time to get around the blocked area, so velocity decreases.

While playing recreational baseball, an individual is hit with a baseball just anterior to the ear. Later that evening, the person is drooling from the left side of the mouth and has noticeable drooping of the face on the left side. Which of the following cranial nerves is MOST likely involved? 1. Left facial (CN VII) 2. Right facial (CN VII) 3. Left trigeminal (CN V) 4. Right trigeminal (CN V)

1. Left facial (CN VII) 1. The facial nerve (CN VII) exits the skull at the stylomastoid foramen (auditory canal), which is the anterior aspect of the ear. It is responsible for innervating all muscles of facial expression on the ipsilateral side of the face. Damage to this nerve results in flaccid paralysis, or hypotonia of the facial muscles. 2. The right facial nerve (CN VII) controls the muscles on the right side of the face. The patient has flaccidity on the left side. 3. The trigeminal nerve (CN V) exits the skull at the superior orbital fissure, the foramen rotundum, and the foramen ovale. The trigeminal nerve (CN V) is responsible for sensation to the ipsilateral face and motor function of the ipsilateral muscles of mastication. 4. The trigeminal nerve (CN V) exits the skull at the superior orbital fissure, the foramen rotundum, and the foramen ovale. The trigeminal nerve (CN V) is responsible for sensation to the ipsilateral face and motor function of the ipsilateral muscles of mastication. This is a peripheral injury so the Rule of 17 doesn't apply (only central lesion injuries)

A patient is referred to physical therapy for right shoulder pain. Which of the following findings suggests that physical therapy intervention may not be appropriate? 1. Pain that subsides with right sidelying 2. Tenderness to palpation at the origin of the biceps tendon 3. Trigger points at the right rib 2-3 intercostal space 4. Pain with resisted shoulder lateral (external) rotation

1. Pain that subsides with right sidelying 1. Right sidelying often increases musculoskeletal pain but may decrease pleural friction/irritation and thus may decrease visceral pain (Goodman). 2. Musculoskeletal pain is often tender to palpation or pressure (Magee, pp. 8-9, 61). 3. Trigger points are the most common musculoskeletal cause of chest pain (Goodman). 4. Resisted testing or contraction of the involved muscle may increase pain of a musculoskeletal origin (Goodman,).

Iontophoresis should be used with precaution for a patient who has which of the following conditions? 1. Peripheral neuropathy 2. Raynaud disease 3. Degenerative joint disease 4. Chronic edema

1. Peripheral neuropathy 1. Iontophoresis should be used with precaution in patients who have conditions involving decreased sensation (Cameron, p. 276; Bellew, p. 323). 2. Raynaud disease is not a precaution or contraindication for iontophoresis (Cameron, p. 276). 3. Degenerative joint disease may be associated with joint pain and inflammation, both of which could be indications for iontophoresis (Bellew, p. 319). 4. Chronic edema may be an indication for iontophoresis (Bellew, p. 319).

A patient who has bicipital tendinopathy is MOST likely to experience pain with which of the following maneuvers? 1. Resisted shoulder flexion with the forearm supinated and the elbow flexed to 15° 2. Passive shoulder medial (internal) rotation and abduction with 90° of elbow flexion 3. Passive shoulder flexion greater than 90° with application of light pressure on the acromion 4. Resisted shoulder extension with the elbow extended and the shoulder abducted to 90° and medially (internally) rotated

1. Resisted shoulder flexion with the forearm supinated and the elbow flexed to 15° 1. This movement describes the Speed test, which is diagnostic of bicipital tendinopathy (p. 614). 2. This movement is more likely to elicit pain in an individual who has rotator cuff impingement, not bicipital tendinopathy (p. 605). 3. This movement is more likely to elicit pain in an individual who has rotator cuff impingement, not bicipital tendinopathy (p. 605). 4. This movement is more likely to elicit pain in an individual who has supraspinatus tendinopathy, not bicipital tendinopathy (p. 605).

Which of the following medical conditions or medications does NOT cause secondary osteoporosis? Select one: 1. Hypothyroidism 2. Chronic renal failure 3. GI diseases 4. Glucocorticoids

1. Hypothyroidism Answer: Hypothyroidism actually causes an increase in bone density. Secondary osteoporosis (develops owing to other medical conditions) including: GI diseases; hyperthyroidism; chronic renal failure; excessive alcohol consumption; use of certain medications such as glucocorticoids. Osteoporosis is detected radiographically by cortical thinning, osteopenia (increased bone radiolucency), trabecular changes, and fractures.

Which of the following recommendations would be MOST appropriate to include in the INITIAL session for a patient who has chronic pain? 1. Use cognitive behavioral therapy techniques. 2. Use self management techniques independently. 3. Instruct the patient to limit all activity. 4. Instruct the patient to perform high intensity aerobic activity.

1. Use cognitive behavioral therapy techniques. 1. Patients may benefit from cognitive behavioral therapy to establish pain coping skills (p. 1112). 2. Using self management techniques independently is a recommendation. However, a patient is not expected to be independent during the initial session. (pp. 1112-1115) 3. Instruction in continued activity is important as exercise is a key part of chronic pain management (p. 1115). 4. Aerobic activity should be performed but at a low to moderate intensity for patients who have chronic pain (p. 1114).

Glossopharyngeal breathing is an appropriate technique to teach a patient who has: 1. a high cervical spinal cord injury. 2. emphysema. 3. cystic fibrosis. 4. asthma.

1. a high cervical spinal cord injury. 1. Glossopharyngeal breathing is a means of increasing pulmonary function when there is severe weakness of the muscles of inspiration, as found in a patient with a higher cervical spinal cord injury (O'Sullivan). 2. While weakness of some muscles is associated with emphysema, emphysema is primarily an obstructive lung disease (Goodman, p. 796). Glossopharyngeal breathing will not assist patients with emphysema. 3. Cystic fibrosis is a secondary obstructive disorder due to copious, thick secretions (Goodman, p. 821), not due to muscle weakness. Glossopharyngeal breathing will not assist patients with cystic fibrosis. 4. Asthma is a chronic inflammatory condition of the airways (Goodman, p. 799) and is not due to muscle weakness. Glossopharyngeal breathing will not assist patients with asthma.

When taking a patient's pulse, which of the following numerical pulse amplitude classifications represents a normal finding? 1. 1+ 2. 2+ 3. 3+ 4. 4+

2. 2+ 1. 1+ is a diminished pulse and is the result of reduced stroke volume and ejection fraction and/or increased vascular resistance. 2. 2+ is considered normal under resting conditions. 3. 3+ is a moderately increased pulse amplitude and is associated with increased stroke volume and ejection fraction. 4. 4+ is a markedly increased pulse amplitude and is associated with increased stroke volume and ejection fraction.

A patient who had a myocardial infarction 5 days ago is referred for a low-level treadmill test. The patient reports having had several episodes of mild angina at rest, after meals, and during the night since being hospitalized. Which of the following actions is MOST appropriate for the physical therapist? 1. Proceed with the usual low-level protocol, because mild angina is common this soon after a myocardial infarction. 2. Defer testing the patient, because the symptoms suggest unstable angina after a myocardial infarction. 3. Perform the test at a lower-than-usual workload, because the symptoms suggest unstable angina after a myocardial infarction. 4. Defer testing the patient, because 5 days after a myocardial infarction is too soon to begin physical exertion.

2. Defer testing the patient, because the symptoms suggest unstable angina after a myocardial infarction. 1. This patient has signs of unstable angina, and this warrants immediate medical attention (O'Sullivan). 2. Unstable angina warrants immediate medical attention (O'Sullivan). 3. Unstable angina warrants immediate medical attention (O'Sullivan). 4. Five days after a myocardial infarction is not too soon for exercise testing. Testing should be performed before discharge, but the patient has signs of unstable angina, which warrant immediate medical attention. (ACSM; O'Sullivan)

A home health physical therapist is working with a patient who had a myocardial infarction 2 weeks ago. The patient reports interrupted sleep, increased swelling of the feet, and shortness of breath. The patient's heart rate is 120 bpm and respiratory rate, 28 breaths/minute. Auscultation reveals crackles in both lung bases. The therapist should suspect: 1. acute congestive heart failure. 2. pneumonia in bilateral lower lobes. 3. atelectasis. 4. renal failure.

1. acute congestive heart failure. 1. These signs and symptoms are consistent with congestive heart failure (Hillegass, p. 97). 2. Pneumonia may result in orthopnea and disrupted sleep, but it would not cause lower extremity edema (Hillegass, p. 143). 3. Atelectasis may be associated with crackles (rales) and shortness of breath, but the other symptoms are not consistent with atelectasis (Hillegass, p. 150). 4. Renal failure may result in lower extremity edema, shortness of breath, and tachypnea but not crackles and tachycardia (Goodman, p. 373).

A patient with diabetes had a right transtibial amputation 5 days ago. An advantage of a rigid removable dressing versus bandaging is that the rigid dressing: 1. allows for early weight bearing. 2. requires less monitoring. 3. allows applied pressure to be varied. 4. is less costly.

1. allows for early weight bearing. 1. A rigid dressing allows for early fitting of a prosthesis and weight-bearing and helps alleviate edema and pain, which is an advantage. 2. A rigid dressing requires more (not less) supervision because there is no access available to the incisions and the dressing is fabricated by the surgeon or prosthetist. As the residual limb heals, sutures are removed, and the limb changes shape, a new cast must be made. 3. Pressure cannot be varied when using a rigid dressing, unlike bandaging with an elastic wrap or shrinker. 4. A rigid dressing requires careful application by a skilled individual and, therefore, may be more costly. An elastic shrinker is a less expensive option.

A therapist is treating a patient who recently had a myocardial infarction. At the beginning of treatment, blood pressure was 120/80 mm Hg and heart rate was 90 beats/min. Midway through treatment, blood pressure was 130/84 mm Hg and heart rate was 105 beats/min. The BEST action for the therapist to take is to: 1. continue with treatment. 2. increase the intensity of treatment. 3. stop the treatment, and notify the physician. 4. decrease the intensity of the next treatment.

1. continue with treatment. 1. Systolic blood pressure is expected to rise in direct proportion to the level of exertion performed. A hypertensive response to low-level exercise (over 160/90 mm Hg) in the patient who is at least 3 days post myocardial infarction may be indicative of cardiac ischemia. Heart rate should increase between 12-24 bpm above the resting heart rate. The patient is showing a normal response to exercise and should continue with treatment. 2. The patient's vital signs show a normal response to exercise. There is no indication to increase activity level. 3. The patient's vital signs show a normal response to exercise. There is no indication to stop treatment. 4. The patient's vital signs show a normal response to exercise. After a recent myocardial infarction, the patient should avoid activities that cause a significant change in vital signs.

A 74 year old retired contractor walks to the treatment room with a prominent backward lean, when he is in stance on the left lower extremity. From this observation, you hypothesize that the MOST LIKELY cause is: A. Weak hip extensors on the left during stance phase B. Weak hip flexors on the left during stance phase C. Weak hip extensors on the right during swing phase D. Weak hip flexors on the right during swing phase

A. Weak hip extensors on the left during stance phase

Which area of the brain does each CN originate from?

CE MI PONS MEDU CE-2 (CN 1,2): cerebrum MI-2 (CN 3,4): mid brain PONS-4 (CN5,6,7,8) MEDU-4 (CN9,10,11,12): medulla

What is MET?

Metabolic equivalents (METs): represents the basic systemic oxygen requirement at rest, roughly 3.5 mL

What is the scapular retraction muscle?

Mid trap

What is Left sided heart failure?

Pathology that occurs with LV insult, reduces the CO leading to a backup of fluid into the LA and lungs. The increased fluid in the lungs produced 2 hall mark signs of left sided heart failure (SOB and cough)

What is cystic fibrosis?

Secretions are thicker causing narrowing/obstruction of airways leading to hyperinflation, infection, and tissue destruction

Types of Selective Debridement vs Non-selective Debridement?

Selective: -sharp debridement (scalpel/scissors/forceps) -enzymatic (topical application) -autolytic (use body's own mechanism) Non-selective: (mainly used when necrotic tissue is >50%) -wet to dry (moistened gauze over necrotic tissue to be completely dried and removed) -wound irrigation (use pressurized fluid to remove necrotic tissue) -hydrotherapy (use whirlpool with agitation to debride wound)

Loss of light touch sensibility?

Thigmanesthesia

What are the upward rotation muscles?

Upper trap SA Lower trap (ULSA)

Skips Kicks ball well Dresses self Swings self on playground swing (pumps legs independently) a. 2 years b. 3 years c. 4 years d. 5 years

d. 5 years

A physical therapist is assessing a patient with a sudden onset of dizziness two days ago. Upon performing the Roll Test, the patient had geotropic nystagmus, stronger on the right side as compared to the left. Which of the following is the MOST APPROPRIATE intervention for this patient? Select one: a. Bar-B-que roll for cupulolithiasis maneuver starting on the R b. Semont maneuver for cupulolithiasis with side lying on the R c. Semont maneuver for canalithiasis with side lying on the L d. Bar-B-que roll maneuver for canalithiasis starting on the R

d. Bar-B-que roll maneuver for canalithiasis starting on the R Rationale: Patient presents with horizontal canal BPPV (Canalithiasis) on the R side since the geotropic nystagmus is stronger on the R. The most appropriate intervention for canalithiasis is the bar-b-que roll with the head turned to the involved side first.

What is considered to be the value for hypoglycemia? Red flag value for fasting blood sugar?

hypoglycemia<60 mg/dL red flag = <70 mg/dL

List the arthrokinematics for each major joint of the body

· Roll/slide o Shoulder: opposite o Elbow: same o Proximal RU: opposite o Distal RU: same o Wrist: opposite o Fingers: same o Hip: opposite o Knee: same o Ankle: opposite

A patient has a resting blood pressure of 120/80 mm Hg. During an activity with a metabolic equivalent level of 4, which of the following blood pressure responses is MOST likely associated with poor left ventricular function? 1. 108/78 mm Hg 2. 129/76 mm Hg 3. 138/83 mm Hg 4. 164/84 mm Hg

1. 108/78 mm Hg 1. A drop in systolic blood pressure greater than or equal to 10 mm Hg with an increase in workload is considered an abnormal response and correlates with left ventricular dysfunction. 2. An increase in systolic blood pressure is a normal response. Diastolic blood pressure may remain the same or increase or drop by 10 mm Hg. This drop in diastolic blood pressure is within normal limits. 3. An increase in systolic blood pressure is a normal response. Diastolic blood pressure may increase or drop by 10 mm Hg. These increases in systolic and diastolic blood pressure are normal. 4. An increase in systolic blood pressure is a normal response. A drop in systolic blood pressure is associated with left ventricular failure.

A patient reports feeling a "pop" in the posterior leg accompanied by sudden onset of pain localized to the middle third of the posterior lower leg. There is no loss of functional strength. This injury is consistent with complete rupture of the: 1. tibialis posterior muscle. 2. popliteus muscle. 3. Achilles tendon. 4. plantaris tendon.

4. plantaris tendon. 1. The tibialis posterior muscle is a significant plantar flexor and invertor of the ankle. Complete rupture of this tendon will significantly affect functional strength evidenced by gait. (p. 355) 2. Rupture of the popliteus tendon will result in decreased knee function and difficulty unlocking the knee from an extended position. (p. 333) 3. Complete rupture at the Achilles tendon will result in the inability to flex the foot and an inability to walk, run, or jump. (pp. 334, 354) 4. Rupture of the plantaris is often felt as a "pop." However, the plantaris is a vestigal tendon that has no significant role in function at the knee or ankle. (p. 355)

Levels of Assistance

7 Complete Independence -timely/safely 6 Modified Independence -device 5 Supervision -cueing/setup/coaxing 4 Min Assist -Pt participation = 75% or more 3 Mod Assist -Pt participation = 50-74% 2 Max Assist -Pt participation = 25-49% 1 Dependent -Pt participation less than 25% *NPTE can phrase the question as pt participation or PT participation, so just subtract PT participation from 100% and you will get pt participation

Which of the following motions will close the right lumbar facet joints from a neutral position? A. Lumbar extension, right side flexion, left rotation B. Lumbar extension, left side flexion, right rotation C. Lumbar flexion, right side flexion, left rotation D. Lumbar flexion, left side flexion, right rotation

A (FLX: opening: SB away/rotate towards) (EXT: closing: SB towards/rotate away)

The manual therapy technique appropriate to correct a bilateral closing restriction of T2 on T3 is: A. Central PA glide on the spinous process of T3 while stabilizing T2. B. Central PA glide on the spinous process of T2 while stabilizing T3. C. Unilateral PA glide on the right transverse process of T3 while stabilizing T2. D. Unilateral PA glide on the left transverse process of T3 while stabilizing T2.

A (bar is CLOSING so BOTTOMS UP) (bar is OPENING so POP THE TOP)

A 30 year old male patient presents with right lower abdominal pain PT performs the "pinch an inch" test result is positive Which condition below is MOSTLY related to the symptoms described? A. Appendicitis B. Diverticulitis C. Crohn's disease D. Irritable bowel syndrome

A. Appendicitis -Pinch an inch test specific for appendicitis -appendix located on the R side Diverticulitis -L side pain Crohn's Disease -R side -abdominal pain not associated Irritable bowel Syndrome -L side pain

A patient with pleural effusion is being ambulated by a physical therapist. Which of the following precaution should the therapist keep in mind? A. Collection bottle should be kept below the level of inserted tube B. Collection bottle should be kept below the level of urinary catheter C. Collection bottle should be removed before ambulating the patient D. Collection bottle should be held by the PT while the patient ambulates

A. Collection bottle should be kept below the level of inserted tube Rationale: -collection bottle should still be inside when pt is ambulating

What are the Code of Ethics?

Altruism -PT shall adhere to the core values of the profession and act in best interest of the pts over PT's interest Autonomy -PT shall provide all necessary information to allow pts to make informed decisions about care Professional Judgement -PT shall be accountable for making sound professional judgements Beneficence -An act of charity, mercy, and kindness with a strong connotation of doing good to others including moral obligation. All professionals have the foundational moral imperative of doing right

What is the bulbocavernous reflex and which nerves are you testing for?

Anal sphincter contraction in response to squeezing the glans penis or tugging on an indwelling Foley catheter Reflex mediated by S2-4 It is the first reflex to come after SCI, indicates that the body is out of spinal shock

What is ANOVA? Is it parametric or non-parametric?

Analysis of Variance: Comparing 2 or more independent groups ex: comparing who does spinal mobs the best: AT vs PT vs chiro Parametric (equal)

Guarding Techniques for Stairs: Ascending vs Descending

Ascending -PT should be positioned to the pt's WEAKEST side -Grasp the gait belt with one hand; be prepared to use your other hand to control the trunk -Advance your feet up one step after the pt has advanced one step, but maintain your feet in an anterior-posterior position Descending -PT in front and to the side of the pt in the area where there is the least protection -Grasp the gait belt with one hand and the handrail -Do NOT allow the pt to develop momentum when descending the stairs

A patient diagnosed with COPD becomes short of breath when walking 10 feet with a walker. Which of the following techniques would MOST likely increase the distance the patient is able to walk without becoming short of breath? A. Incentive spirometry B. Pacing C. Diaphragmatic breathing D. Segmental breathing

B. Pacing -teaching the pt taking breaks and going slow to pace themselves Incentive Spirometry -used for respiratory issues or CABG Diaphragmatic breating -used for normal breathing Pursed breathing -used for COPD Segmental breathing -used for restrictive or atelectasis

A 52-year-old female complains of R forearm/wrist discomfort. The patient had undergone mastectomy 1 year ago. During examination, the PT observes that the patient has increased swelling that does not change with arm elevation and initial signs of fibrotic changes. This PT identifies the stage of lymphedema as: A. Stage 1 B. Stage 2 C. Stage 3 D. Stage 4

B. Stage 2

A 32 year old healthy male is working out on a stationary bike in an outpatient physical therapy clinic. After the first 4 minutes of constant load, sub maximal exercise, VO2 reaches steady state, indicating that: A. Levels of lactic acid in the blood has reached steady state. B. The ATP demand is being met aerobically. C. The exercise can be continued for 4 hours without further increase in heart rate. D. The respiratory rate is insufficient to meet the ATP demand.

B. The ATP demand is being met aerobically. Steady state = volume of oxygen coming in is steady Key words: stationary bike, constant-load, sub-maximal exercise VO2: volume of oxygen consumed What happens if ATP demand is not being met aerobically? When Oxygen is not available, the cell is forced to produce energy (=ATP) through ANAEROBIC processes, that produce much less energy (about 15 times less), than AEROBIC processes.

Malignant Tumors: Basal Cell Carcinoma Squamous Cell Carcinoma

Basal Cell Carcinoma: -Long sun exposure, face, fair skinned -Raised patch, rolled border, indented center/thickened area of skin Squamous Cell Carcinoma: -Sun exposed areas: face, neck, back of hand -Poorly defined margins; flat, red area, ulcer or nodule, grows quickly

What muscle are eccentrically contracting when performing shoulder ext from 180 degrees to 90 degrees?

Biceps Coracobrachialis Brachialis Anterior delt

What does Cozen's test and Mill's test for and how are they different?

Both test for lateral epicondylitis (tennis elbow) Cozen = MMT (wrist in ext and resist pressure) Mills = MLT (wrist in flex and stretch)

Intervention technique used to TREAT Posterior SCC BPPV for HEP?

Brandt-Daroff Exercise -Hold position for 30 seconds or until vertigo stops -Repeated 10-20 times, 3 x/day until pt is without vertigo for 2 consecutive days -Pt rotates 45 degrees to affected side and S/L toward unaffected side and switches back and forth

During exams week, the physical therapist observes the posture of a 29-year-old female patient. The PT notices that she has a forward head posture. Which of the following characterizes forward head posture? A) Extension of lower cervical spine B) Flexion of upper cervical spine C) Increase in the resistance moment arm of the head D) Decrease in the resistance moment arm of the head

C) Increase in the resistance moment arm of the head

A patient is seen after an uncomplicated vaginal delivery of her third child. During the treatment, the patient begins to complain of a headache and visual disturbances, and suddenly develops a seizure. Which condition is MOST likely? A. Preeclampsia B. Gestational Diabetes C. Eclampsia D. Ectopic Pregnancy

C. Eclampsia -happens after pregnancy Preeclampsia -happens DURING pregnancy -HA and visual disturbance also common but NO SEIZURES Gestational diabetes -diabetes only during pregnancy -after the mother gives birth the diabetes goes away -Babies born early to mothers with gestational diabetes may experience Respiratory Distress Syndrome — a condition that makes breathing difficult. Low blood sugar (hypoglycemia). Sometimes babies of mothers with gestational diabetes have low blood sugar (hypoglycemia) shortly after birth. Ectopic pregnancy -pregnancy happening outside the normal structural changes

On examination of a 45-year-old male patient, the PT observes that the patient is able to clench his teeth but is unable to smile, whistle or close his eyes. The patient reports that most of the sounds around him seem unpleasant and too loud. Which nerve is affected and what other findings can help the PT confirm the diagnosis? A. Oculomotor nerve; Absence of corneal reflex. B. Trigeminal nerve; Absence of sensation on the face. C. Facial Nerve; Absence of taste on anterior 2/3rd of the tongue D. Vestibulocochlear nerve; loss of balance and equilibrium

C. Facial Nerve; Absence of taste on anterior 2/3rd of the tongue -able to clench teeth = CN 5 is okay -unable to smile, whistle, close eyes = CN 7 affected -sound unpleasant and too loud = hyperacusis = CN 7 affected -oculomotor deals with pupillary light reflex -corneal reflex is CN 5S/7M

A PT is treating a patient with a diagnosis of ALS. The patient has weakness of all extremities and gets fatigued very easily while doing ADLs. His main goal is to maintain mobility and function as much as possible. Which of the following will be LEAST appropriate for this patient? A. Recommending soft foam collar for neck B. Taking frequent breaks during activities C. Recommend KAFO and walker for ambulation D. Slow, prolonged stretches and ROM exercises for UE and LE

C. Recommend KAFO and walker for ambulation -Muscles are getting denervated so you don't' want to overwork the muscles. KAFO would aggravate the extremity which would be detrimental for function -Soft neck collar is fine because they have weak cervical neck extensors

A 58 year old male underwent a total hip arthroplasty on the left side using a posterolateral approach. The PT is working on a gait training program for the patient. The therapist should instruct the patient to hold the crutch or cane in the: A. Left hand to decrease activity in the left hip abductors. B. Left hand to facilitate activity in the left hip abductors. C. Right hand to decrease activity in the left hip abductors. D. Right hand to facilitate activity in the left hip abductors.

C. Right hand to decrease activity in the left hip abductors. Muscle needs time to repair so you want to decrease activity in the L hip abd, the cane helps support this muscle during this stage of rehab

Concept: Need the gravity to assist in the drainage. A physical therapist positions a patient in sidelying in preparation for postural drainage activities. Which lung segment would be indicated based on the patient's position? A. posterior basal segment of the lower lobes B. apical segment of the upper lobes C. lateral basal segment of the lower lobes D. anterior segment of the upper lobes

C. lateral basal segment of the lower lobes Posterior basal segment = prone position Apical segment = sitting with trunk backwards on a recliner Anterior segment = supine

When performing the following test on a patient, the therapist finds that the patient's thigh remains on the plinth and her knee is somewhat extended. This would indicate tightness of: (Look at image on slide 21 on FF#25) A. the illiopsoas B. the hamstrings C. the rectus femoris D. the sartorius

C. the rectus femoris ST is Thomas Test -Sartorius would be hip in ABD and ER What test would confirm Rectus Femoris tightness? -Ely's test in prone

Treatment for vertical torsion: up/down beating nystagmus) for BPPV pts?

Canalith Repositioning Maneuver: "Epley Maneuver"

What is Kruskal Wallis test? Is it parametric or non-parametric?

Comparing 3 or more groups (cousin/friend of ANOVA) Non-parametric (unequal)

SLAP tear vs Bankart

Coraco-acromial arch: -A key site of pathology and an active treatment site -SLAP lesion superior labrum tearing or pulling away by the biceps insertion which is under the Subacromial arch Bankart -inferior labrum involvement Hill-Sachs lesion -is an injury that occurs secondary to an anterior shoulder dislocation. The humeral head 'collides' with the anterior part of the glenoid, causing a lesion, bone loss, defect and deformity of the humeral head. This may cause a change loss of range of motion, feelings of instability and pain.

A 6-month-old child was referred to physical therapy for right torticollis. The MOST effective method to stretch the muscle is by positioning the head and neck into: A. Flexion, left side-bending, and left rotation. B. Extension, right side-bending, and left rotation. C. Flexion, right side-bending, and left rotation. D. Extension, left side-bending, and right rotation.

D. Extension, left side-bending, and right rotation.

A physical therapist is administering iontophoresis for the management of a sclerotic scar in a patient with adhesive capsulitis. To administer this treatment, which ion and polarity should be used? A. Zinc, negative pole. B. Acetate, positive pole. C. Copper, positive pole. D. Iodine, negative pole.

D. Iodine, negative pole.

A PT is evaluating a 34 year old female patient with a vague diagnosis of low back pain. The patient displays a positive Thomas test. Which sub phase of the gait cycle will MOST likely show limitation in the hip range of motion? (picture shows tight iliopsoas with Thomas test) A. Loading response B. Initial contact C. Mid-stance D. Terminal stance

D. Terminal stance

UE PNF patterns

D1 flex = FADER = eat it D1 ext = EABIR = put it away D2 flex = FABER = put up the sword D2 ext = EADIR = put the sword back down "Make your table start with FADER, opposite ABD/ADD"

Muscle action for Flexor Digitorum Profundus (FDP)?

DIP flex

What is CRPS type 2?

Develops after nerve injury Not limited to territory of injured nerve Edema; skin blood flow abnormality Abnormal sudomotor activity

Special test used for FAI?

FADIR (flexion and IR stresses impingement)

Which CN is responsible for anterior 2/3 taste?

Facial nerve (CN7)

Neonatal Reflexes

Galant reflex = lateral flexion toward stimulus 0-2 months "Cowboy gallantly went on his horse causing lateral flexed trunk" Rooting reflex = mouth open toward stimulus 0-3 months "Mouth open to a vegetable root" Moro reflex = cervical ext causes crying and UE flex/add response 0-4 months "Moro raised his arms up and was crying " Asymmetrical tonic reflex = face side UE ext and skull side UE flex 0-5 months "Heisman trophy is asymmetrical" Tonic labyrinthine reflex = prone flex tone and supine ext tone 0-6 months "baby likes to be in flexed position in prone"

Other pathologies that cause referred pain to the R shoulder?

Gall bladder, stomach, pancreas, and small intestines Pancoast Tumor (upper lung tumor) -causes referred pain in the C8-T2 nerve distribution (mimics TOS) Blood in the abdominal cavity secondary to liver trauma -refers pain to the R shoulder Pancreatic cancer -if head of pancreas is involved, refers pain to the R shoulder

Which CN is responsible for posterior 1/3 taste AND sensation?

Glossopharyngeal (CN9)

What are the different grades of joint mobs used?

Grade 1: -Small amplitude of movement -Performed at beginning of range (painful) Grade 2: -Large amplitude movement -Performed within the range, but not reaching the limit (painful) Grade 3: -Large amplitude rhythmic oscillations -Performed "UP TO" the limit of the available motion -Stressed into the tissue resistance (pain free) Grade 4: -Small amplitude rhythmic oscillations -Performed "AT" the limit of the available motion -Stressed into the tissue resistance (pain free) Grade 5: -Small amplitude high velocity thrust technique -Performed to snap adhesions at the limit of range (pain free)

What are the CV and Respiratory effects of aquatic therapy? (HR/BP/SV/CO/VC)

HR: decreases BP: decreases SV: increases CO: increases VC: decreases (Ability of lungs to expand, hydrostatic pressure of water restricts your lungs to expand fully causing VC to decrease)

What are the initial changes with HR/BP/CO/SV with altitude changes?

HR: increases BP: increases CO: increases SV: no change

Hernia types? Referred pain location? Hernia contraindications?

Hiatal hernia causes shoulder pain "the nosy stomach neighbor that peaks in to the esophagus" -usually causes L sided pain Femoral hernia causes lateral pelvic wall pain Inguinal hernia causes groin pain Umbilical hernia causes pain around the umbilical ring in the mid to lower abdomen "Mostly LEFT sided pain" Hernia contraindications: -AVOID intra-abdominal pressure -AVOID Valsalva maneuver. There is already a hole or cavity developing, the Valsalva maneuver will cause the patient to have more problems

What is a pericardial friction rub?

High pitched leathery and scratchy sound resulting from inflammation of the pericardial sac that may be caused from trauma, infection, tumor, collagen diseases, anticoagulant, and MI

What is high/moderate/fair/low correlation?

High: 0.76 to 1.0 Moderate: 0.51 to 0.75 Fair: 0.26 to 0.50 Low: 0.00 to 0.25 Think of it in quarters

Abnormal ECG signs relation: Hyperkalemia Hypokalemia Hypercalcemia Hypocalcemia

Hyperkalemia = increased T wave Hypokalemia = U wave/Inverted T wave Hypercalcemia = QT shortened Hypocalcemia = QT longer

Hypoglycemia vs Hyperglycemia

Hypoglycemia -glucose <70 -very hungry symptoms -shakiness/dizziness/fainting/ Hyperglycemia -glucose >300 -DM2 > 300 -DM1 with ketone bodies is >180 -symptoms after you eat -weakness/dry mouth/urination/excessive thirst/fruity odor "Hot and dry, blood sugars high" "Cold and clammy, better get them some candy"

Settings for US? -intensity -frequency -pulsed/continuous -head diameter

Intensity -1.5 W = chronic -0.8 W = acute Frequency -superficial = 3 Mhz -deep = 1 Mhz Pulsed or Continuous -Chronic cases = use 100% continuous -Acute cases = use 20% pulsed Head diameter smaller areas (hand/fingers) = 2 cm regular/larger areas = 5 cm

Difference between Interval vs Ratio and is it quality or quantity?

Interval is data that does NOT have a true zero ex: temp in Celsius/Farenheit (0 degrees C/F does not represent the complete absence of temperature) Ratio is data that HAS a true zero ex: ROM, Nerve conduction velocity, distance walked, BP, BMI, force, velocity (rati0) Quantity RIP (Ratio Interval -> Parametric data) Research Is Pain

Rules for above knee prosthesis (low walls vs high walls)?

Low wall = weak muscles ex: low anterior wall = weak quads lower lateral wall = weak abductors High wall = tight muscles ex: high anterior thigh wall = tight hip flexors which pulls pelvic anteriorly

Which part of the lung is cleared in a L/R sidelying position on a DECLINED bed position arm raised?

Lower lobes: Anterior basal segment

What is Anal reflex and which nerve are you testing for?

Reflex contraction of the external anal sphincter when stroking around the anus Afferent: pudendal nerve Efferent: S2-4

Difference between S3 and S4 heart sound?

S3 = "ventricular gallop" ventricular filling, associated with heart failure (associated with LV failure) -common in high level athletes S4 = "atrial gallop" abnormal, ventricular filling and atrial contraction (associated with MI or chronic HTN)(LV hypertorphy)

What are the different types of non-articulated foot orthosis? (slide 17 FF#13)

SACH foot = allows DF/PF but no medial/lateral support SAFE foot = provides medial lateral support SEATTLE foot = uses keels (imagine springy toes) to store energy while walking used in late stance Microprocessor foot = uses computer chip to read what motion is needed while walking

Treatment for spastic/flaccid bladder

Spastic bladder: -Spastic bladder is AT or ABOVE T12 and can be treated with suprapubic tapping. Flaccid bladder: -Flaccid is BELOW T12 and can be treated with Valsalva or Crede's maneuver (manual pressure on the bladder)

Stages of Cancer

Stage 0: -This stage describes cancer in situ, which means "in place." Stage 0 cancers are still located in the place they started and have not spread to nearby tissues. This stage of cancer is often highly curable, usually by removing the entire tumor with surgery. Stage I: -This stage is usually a small cancer or tumor that has not grown deeply into nearby tissues. It also has not spread to the lymph nodes or other parts of the body. It is often called early-stage cancer. Stage II and Stage III: -In general, these 2 stages indicate larger cancers or tumors that have grown more deeply into nearby tissue. They may have also spread to lymph nodes but not to other parts of the body. Stage IV: -This stage means that the cancer has spread to other organs or parts of the body. It may also be called advanced or metastatic cancer.

Stages of Lipedema

Stage 1 -skin surface smooth -subcutaneous fat thickened -fat structure FINE-KNOTTED Stage 2 -skin surface uneven -fat structure coarsely KNOTTED Stage 3 -Tissue additionally coarser and harder -LARGE LOBED DEFORMING FAT LOBES Stage 4 -Additional severe lipolymphedema

Brunnstrom's Stages of Recovery

Stage 1 = flaccidity -no active limb movement Stage 2 = beginning of minimal voluntary movement -in synergy, with associated reactions -increased tone Stage 3 = voluntary control of movement synergy (SPASTICITY AT PEAK) -further increased tone to peak level Stage 4 = MOVEMENT OUTSIDE OF SYNERGY -decreased tone Stage 5 = increased complex movement -greater independence from limb synergies Stage 6 -individual joint movement -coordinated movement Stage 7 -normal function

Best auscultation point for breath sounds? What are the borders of the answer?

Triangle of auscultation Borders: Scapula Lower trap Lats The triangle of auscultation is a small area of the back where three muscles (trapezius, latissimus dorsi, and rhomboideus major) converge. This area is near the scapula and becomes enlarged when a person leans forward with arms folded across the chest. When a physician places a stethoscope on the triangle of auscultation, the sounds of the respiratory organs can be clearly heard.

Which CN is most likely affected with a pt having difficulty looking down when going downstairs?

Trochlear nerve (CN4)

Type 1 vs Type 2 Diabetes?

Type 1 • Pancreas produces no insulin • Diagnosed mostly at childhood, but can be any age • Symptoms-polyphagia (extreme hunger), weight loss, polyuria, polydipsia (extreme thirst), blurred vision and dehydration • KETOACIDOSIS (body breaks down fat too fast because it's lacking glucose for energy/fuel which turns in to ketones which causes your blood to become too acidic which increases risk of coma/death) = Keto/Metabolic Acidosis "May was very HUNGRY (polyphagia) and THIRSTY (dehydration), after her meal she went to the bathroom and said she URINATED ALOT (polyuria). She told me she has BLURRY VISION sometimes when watching demon slayer too long. It's important that she takes her insulin because if she doesn't she's at risk of ketoacidosis and may be but into a coma/death" Type 2 • Body's resistance to insulin/defective insulin • Occurs secondary to other dysfunction • Symptoms-Similar to Type 1 with no occurrence of ketoacidosis

What is type 1 error vs type 2 error?

Type 1 error: false positive ex: Dr tells you you're pregnant but you're really NOT pregnant Type 2 error: false negative ex: Dr tells you you're NOT pregnant but you're really pregnant

What is the rule for weak vs tight hip flexor muscles in regards to gait?

Weakness will cause a decrease in ROM in the same motion Tight/contracture muscles will cause a decrease in ROM in the opposite motion COWS Contracture Opposite Weakness Same (only applies to hip flexor muscles during gait)

Which of the following findings is MOST important to assess in a patient who is 2 days post radical mastectomy (radical mastectomy is removal of one or two breasts, chest muscles and lymph nodes to treat breast cancer)? a. Limited range of motion in the involved upper extremity b. Atrophy in the involved upper extremity c. Chest wall adhesions d. Contractures of the elbow flexors

a. Limited range of motion in the involved upper extremity After mastectomy, it is common that the patients experience temporary and sometimes long-term loss of shoulder mobility. It can be due to incisional pain, muscle guarding or any other factors. Chest wall adhesions can also develop after surgery, but it cannot be seen after two days of surgery.

You are treating a patient in inpatient rehabilitation with deficits in pupillary light reflex in the right eye and a failure to look at the nose with the right eye. When testing lateral deviation of the eye, the patient is able to follow the stimulus. No deficits noted in chewing. Which anatomical location is MOST likely affected? Select one: a. Midbrain b. Cerebellum c. Pons d. Medulla

a. Midbrain Rationale: Cranial nerves IV and III are located in the midbrain. Cranial nerve III is the oculomotor nerve, and damage to the oculomotor nerve causes ptosis (drooping eyelid), dilation of the pupil, and loss of accommodation of the light reflex. Cranial nerve IV is the trochlear nerve. The trochlear nerve's motor innervation is to the superior oblique muscle which is responsible for rotating the eye down and in.

A pt's EKG report describes the presence of significant Q waves. This finding is suggestive of which of the following conditions? a. myocardial infarction b. supraventricular tachycardia c. premature atrial complex d. atrial fibrillation

a. Myocardial infarction -A prominent, pathological Q wave is indicative of a transmural myocardial infarction b. Supraventricular tachycardia -Supraventricular tachycardia is associated with P waves that can be merged into T waves. The QRS complexes are typically identical in shape and normal in duration c. Premature atrial complex -Indicated by an R wave that is close to the preceding R wave, and the P wave of the early beat is shaped differently from normal P waves d. Atrial fibrillation -Characterized by inconsistent, irregular R-R intervals

You are evaluating a 37 y.o. secretary in an outpatient clinic who presents with right upper extremity pain shooting into the hand. On examination, the therapist performed the test as shown in the image and the patient complained of pain, weakness, and tingling in the hand. To confirm the diagnosis, the therapist can perform which of the following additional tests? (image is pt performing Roos test) Select one: a. Positive Adson's test b. Positive Neer's test c. Positive Roos test d. Positive Anterior Apprehension test

a. Positive Adson's test The test shown in the image is Roos test which is used for the diagnosis of thoracic outlet syndrome. To confirm the diagnosis, the therapist can perform Adson test which is one of most common methods of testing for thoracic outlet syndrome. Anterior apprehension test is used for anterior shoulder instability; Neer's test is used for impingement.

Extension of cervical joint produces ext of UE and flex of LE; Flexion of cervical joint produces flex of UE and ext of LE a. Symmetrical tonic neck reflex b. Asymmetrical tonic neck reflex c. Tonic labyrinthine reflex d. Galant reflex

a. Symmetrical tonic neck reflex 6-8 months

POE, WB on forearms Elbows in line with shoulders, head elevated to 90 degrees Head in midline in supine, hands on chest Increased back extension with scapular adduction in supported sitting Takes some weight with toes curled in supported standing Coos, chuckles Optical and labyrinthine head-righting present Which developmental age? a. 1 month b. 2 months c. 3 months d. 4 months

c. 3 months

Can maintain quadruped Pivots on belly; moves body in circle while prone Pivot prone (prone extension) position Assumes sitting from quadruped Trunk rotation in sitting Recognizes tone of voice May show fear of strangers a. 3 month b. 5 months c. 7 months d. 9 months

c. 7 months

A pediatric PT is evaluating a 2 month old child. The PT pulls the child from a supine to seated position using the child's forearms. The child extends the trunk in response to this stimuli. Which of the following statements is the MOST accurate? Select one: a. The child has an normal traction reflex b. The child has a normal moro reflex c. The child has an abnormal traction reflex d. The child has an abnormal moro reflex

c. The child has an abnormal traction reflex Rationale: This question describes the traction reflex. The traction reflex is tested by pulling a child from supine to sit. A normal response is to fully flex and grasp. An extensor response would be considered abnormal.

A patient complains of frequent episodes of tingling on the medial aspect of the calf and ankle especially on forward flexion. On evaluation, the knee jerk is found to be weak along with weakness of tibialis anterior and extensor hallucis muscles and a limited straight leg raise with negative prone knee bend. A disc herniation at which of the following levels would cause these symptoms? Select one: a. L2-L3 b. L4-L5 c. L5-S1 d. L3-L4

d. L3-L4 Rationale: L3 would have sluggish knee jerk, a positive prone knee bend and pain on full flexion. Paresthesias would be present on inner knee and anterior lower leg L4 lesions would be associated with weakness of tibialis anterior, extensor hallucis along with paresthesias in the medial aspect of calf and ankle, weak knee jerk and a limited straight leg raise. L5 lesions would be associated with weakness of extensor hallucis, peroneals, gluteus medius, ankle dorsiflexors, hamstrings along with paresthesias on the lateral aspect of leg, medial three toes with decreased ankle jerk and painful crossed straight leg raise. S1 lesions would cause paresthesias along the lateral two toes, lateral foot, lateral leg to knee, plantar aspect of foot along with weakness of the calf and hamstrings, wasting of gluteals, peroneals, plantar flexors and limited straight leg raise. L4 nerve root is affected in this case. At the lumbar level between L3-L4:Nerve root exiting- L3Nerve affected in stenosis- L3But with herniation, it is the L4 nerve root that is involved. Because of the course of the nerve root as it exits, the L4 disc (between L4 and L5) only rarely compresses the L4 nerve root; it is more likely to compress the L5 nerve root. (Image attached for better understanding).In this case the symptoms aggravated on forward bending- indicating it was a disc pathology. So for L4 to be affected, the herniation should have taken place at L3-L4 level.

What are the effects of Beta blockers?

reduces HR and contractility lowers the myocardial oxygen demand (-olol) Action of epinephrine and norepinephrine? Epinephrine, more commonly known as adrenaline, is a hormone secreted by the medulla of the adrenal glands. Strong emotions such as fear or anger cause epinephrine to be released into the bloodstream, which causes an increase in heart rate, muscle strength, blood pressure, and sugar metabolism. consider how someone reacts to an Epinephrine injection (EPI pen) during an allergic reaction. That will help you remember the body reaction.

What is Guillain-Barre Syndrome (GBS)?

• Autoimmune response as the patient's antibodies attack its own peripheral nerves • Syndrome is typically observed after respiratory infection, GI infection characterized by diarrhea, surgery, vaccination or childbirth. • LMN disease • Rapid onset of weakness from DISTAL to PROXIMAL with BILATERAL symptoms peaking at 2-3 weeks, but progressing no greater than 4 weeks • Severity ranges widely, 70% of cases have full recovery within 3-6 months B in GBS = always BILATERAL symptom involvement

Which of the following conditions are malignant tumors of the connective tissue? 1. Fibroma and chondroma 2. Fibrosarcoma and chondrosarcoma 3. Papilloma and adenoma 4. Adenocarcinoma and basal cell carcinoma

2. Fibrosarcoma and chondrosarcoma 1. This option lists two types of benign tumors of the connective tissue. 2. This option lists two types of malignant tumors of the connective tissue. 3. This option lists two types of benign tumors of the epithelium/skin. 4. This option lists two types of malignant tumors of the epithelium/skin.

Which of the following clinical features is a CONTRAINDICATION to intermittent compression? 1. Past history of deep vein thrombosis 2. Impaired sensation 3. Local infection 4. Peripheral neuropathy

3. Local infection 1. Recent or acute deep vein thrombosis is a contraindication, but a past history or having a predisposition to deep vein thrombosis is not. 2. Impaired sensation is a precaution. 3. Infection may spread as a result of compression, and, therefore, infection is a clear contraindication. 4. Peripheral neuropathy is not a contraindication; it is a precaution.

A patient with a C4 complete ASIA A SCI requires pulmonary hygiene and education on accessory muscle use. Which accessory muscles should the Physical Therapist focus on to assist the patient in developing and strengthening for compensatory measures to improve respiration? 1. Intercostals, abdominals, upper trapezius, scalenes, rhomboids 2. Scalenes, levator scapula, intercostals, erector spinae muscles 3. Scalenes, cervical extensors, sternocleidomastoid, upper trapezius 4. Omohyoid, masseter, lateral pterygoid, sternocleidomastoid, and upper trapezius

3. Scalenes, cervical extensors, sternocleidomastoid, upper trapezius Answer: The neurological levels of spinal cord injury and muscles of respiration are as follows. C1-C2: SCM, upper trapezius, cervical extensors. C3-C4: partial diaphragm, scalenes, levator scapulae. C5-C8: Diaphragm, pectoralis major and minor, serratus anterior, rhomboids, latissimus dorsi. T1-T5: some intercostals, erector spinae. T6-T10: Intercostals and abdominals.

A patient reports right lower quadrant pain. When the patient is in the supine position with hips and knees flexed 90°, deep palpation in the right lower quadrant reproduces the pain. What is the MOST appropriate action for the physical therapist to do NEXT? 1. Inform the patient about possible diverticulitis. 2. Test rectus abdominis strength. 3. Stretch the right hip flexor muscles. 4. Contact the physician about possible iliac vein thrombosis.

3. Stretch the right hip flexor muscles. 1. Diverticulitis is more likely to present with left-sided rather than right-sided lower quadrant pain (Goodman & Heick, p. 319). 2. The test position described in the question is for the iliopsoas muscle. Rectus abdominis symptoms with palpation would be more centered in the abdomen; therefore, there is no indication that the rectus abdominis muscle should be evaluated. (Goodman & Heick, p. 313) 3. This is the test position to identify dysfunction of the iliopsoas muscles; therefore, stretching the right hip flexor, which is the muscle action of the iliopsoas, is the appropriate action. (Goodman & Heick, p. 313) 4. Thrombosis that occludes the iliac vein produces posterior lower leg swelling, pain or tenderness, dilation of superficial veins, and pitting edema. The patient does not have these signs or symptoms. (Goodman & Fuller, p. 649)

patient presents to the ED post TBI. On reading the patient's notes, the Physical Therapist observes the following vitals: BP 130/90; HR 105 bpm, RR 20 breaths/min, GCS 5/15. Which of the following presentations would be MOST appropriate for this evaluation to be accurate? Select one: 1. Patient opens eyes in response to pain, incomprehensible sounds present, patient extends limb in response to pain 2. Eyes do not open in response to pain, no verbal response in patient, withdraws limb in response to pain 3. Eyes open spontaneously, patient is confused patient flexes limb in response to pain 4. Eyes open to pain, incomprehensible sounds, no movement of limbs in response to pain

4. Eyes open to pain, incomprehensible sounds, no movement of limbs in response to pain Answer: Eyes open to pain (2) plus incomprehensible sounds(2) plus no movement of limbs in response to pain(1)=5.

A patient who has meralgia paresthetica has been referred to physical therapy. Which of the following clinical features is MOST likely to be assessed by the physical therapist during the examination? 1. Strength of the adductor longus 2. Strength of the quadriceps femoris 3. Sensation of the superior medial aspect of the thigh 4. Sensation of the lateral aspect of the thigh

4. Sensation of the lateral aspect of the thigh 1. The adductor longus is innervated by the obturator nerve, and strength testing would assess the motor integrity of this nerve. Meralgia paresthetica does not involve the obturator nerve. 2. The quadriceps femoris is innervated by the femoral nerve, and strength testing would assess the motor integrity of this nerve. Meralgia paresthetica does not involve the femoral nerve. 3. Meralgia paresthetica is an entrapment or injury to the lateral femoral cutaneous nerve, a purely sensory nerve. This injury affects sensation to the lateral thigh. Sensory testing of the superior medial aspect of the thigh would be an assessment of the ilioinguinal nerve. 4. Meralgia paresthetica is an entrapment or injury to the lateral femoral cutaneous nerve, a purely sensory nerve. This injury affects sensation to the lateral thigh. Sensory testing of this region is the most appropriate assessment.

What is ventricular tachycardia?

PVCs that occur 3 or more in a row is called ventricular tachycardia

List BMI scale

<18.5 = Underweight 18.5-24.9 = Normal 25-29.9 = Overweight 30-34.9 = Obese 35< = Extremely obese

A patient presents with foot pain during running. The PT noted the patient has excessive foot pronation and would like to provide an orthotic that could help relieve pain. Which of the following orthotic would MOST LIKELY benefit this patient? A. Forefoot medial post under first metatarsal head B. Cushion heel C. Forefoot lateral post under fifth metatarsal head D. Posterior leaf spring

A. Forefoot medial post under first metatarsal head C would be for excessive foot supination

Decreased flexion Momentary head elevation with minimal forearm support Tracks a moving object with head rotation Reciprocal and symmetrical kicking Positive support and primary walking reflexes in supported standing Hands fisted with indwelling thumb most of the time Neonatal reaching Alert, bright expression Which developmental age? A. 1 month B. 3 months C. 5 months D. 7 months

A. 1 month

A physical therapist is examining a 25 year old male who had a recent fall while fixing the roof. The patient seems distracted and the PT has difficulty maintaining the patient's attention. Which area of the brain is MOST likely injured? A. Frontal lobe B. Temporal lobe C. Parietal lobe D. Occipital lobe

A. Frontal lobe -main primary motor cortex -decision making -CEO of the organization key words: recent fall, distracted, difficulty maintaining attention

After evaluating a baseball player, the PT concludes that there is insufficient deceleration of the shoulder during throwing. Which of the following strengthening techniques will be MOST effective in improving control of deceleration of the shoulder? A. Eccentric exercises of the teres minor and infraspinatus B. Eccentric exercises of the teres major and pectoralis major C. Concentric exercises of the teres minor and infraspinatus D. Concentric exercises of the teres major and pectoralis major

A. Eccentric exercises of the teres minor and infraspinatus Anytime you need to slow down movement, you need eccentric control of the opposing muscle group. So in this case the baseball player was performing IR but needed eccentric control from the ER.

A physical therapist is visiting a 32-year old female with diagnosis of BPPV in the clinic. What Techniques are the best way to treat Benign Paroxysmal Positional Vertigo? A. Epley Maneuver and Semont Maneuver B. Canalith Repositioning Maneuver and Gaze Stability Exercises C. Brandt- Daroff Exercises and Gaze Stability Exercises D. Epley Maneuver and Habituation Exercises

A. Epley Maneuver and Semont Maneuver

Which of the following activities should be the PRIMARY emphasis of a PT treatment program for a child who has athetoid cerebral palsy with abnormal, involuntary movement? A. Facilitating co-contraction patterns and encouraging control in voluntary movement B. Preservation of strength and muscle tone C. Facilitating use of primitive reflexes to perform fine motor skills D. Prevention of contractures and determine the best method of mobility

A. Facilitating co-contraction patterns and encouraging control in voluntary movement You want to encourage voluntary control

A 25-year-old gymnast has LBP that started about 3 weeks ago. Her pain symptoms increases on standing, bending and decrease on sitting. On Examination SLR was found negative. Which of the following will be the MOST LIKELY finding on her radiographs? A. Forward slippage of one of the vertebrae on the vertebra below it B. Posterio-lateral slip of the vertebral disc C. Degenerative changes with presence of osteophytes D. Regional osteopenia

A. Forward slippage of one of the vertebrae on the vertebra below it -look at DD chart on slide 49 FF#29

A patient with right hemisphere damage would MOST likely have: A. Left unilateral neglect and difficulty reaching for objects presented on the Left side of the body. B. Left unilateral neglect and difficulty reaching for objects presented in the midline of the body. C. Difficulty crossing midline to reach objects on the Right side of the body. D. Difficulty stabilizing gaze and locating objects on the right side of the body.

A. Left unilateral neglect and difficulty reaching for objects presented on the Left side of the body.

A patient complains of back pain radiating to his lower limbs on bending his trunk forward with relief on coming to erect posture. What's the best intervention? A. Lumbar Traction with 25 percent body weight in prone B. Lumbar traction with 50 percent body weight in supine C. Lumbar traction with 55 percent body weight in prone D. Lumbar traction with 70 percent body weight in supine

A. Lumbar Traction with 25 percent body weight in prone -disc (HNP) issue due to pain with flexion so you want them in prone -ideal is 25-50% BW -55% BW is too high

A patient is semi-conscious and opens his eyes to pain. The best motor response elicited is withdrawal and he is producing incomprehensible sounds. Which of the following is the MOST APPROPRIATE indicator of the level of consciousness and severity of injury on the Glasgow Coma Scale (GCS)? A. The patient has a GCS score of 8/15 indicative of severe brain injury B. The patient has a score of 5/15 indicative of severe brain injury C. The patient has a score of 12/15 indicative of mild brain injury D. The patient has a score of 10/15 indicative of moderate brain injury

A. The patient has a GCS score of 8/15 indicative of severe brain injury Mnemonic: GCSEVM456

A 45 year old male with a BMI of 38 kg/m 2 is enrolled in a 6 week fitness training program. Which is the MOST appropriate measure to assess change in fitness from pre and post fitness training? A. The time it takes for the heart rate to return to baseline B. Resting respiration rate at pre training C. Rating on a Wong baker scale D. Increase in blood pressure during exercise

A. The time it takes for the heart rate to return to baseline

A PT is examining a 46 year old patient who underwent a radical mastectomy. The patient has developed lymphedema on the right UE. Which of the following statements is CORRECT intervention in this patient? A. The trunk and axilla should be decongested first followed by arm and hand B. Manual lymphatic drainage should involve proximal to distal stroking C. The hand should be decongested first followed by trunk and axilla D. Manual lymphatic drainage is contraindicated in post mastectomy

A. The trunk and axilla should be decongested first followed by arm and hand

A 45-year-old patient been walking 3 days/week for 20 minutes for the past 3 weeks. When progressing the exercise program, which of the following modifications will MOST likely accomplish the weight-loss goal? A. Walk 4 days/week at current walking speed and increase the duration to 45 minutes B. Increase the walking speed and keep the duration at 20 minutes. C. Walk 6 days/week and decrease the duration to 10 minutes. D. Decrease the walking speed and increase the duration at 25 minutes.

A. Walk 4 days/week at current walking speed and increase the duration to 45 minutes Remember FITT principle guideline -Frequency first -Intensity second -Time third -Type fourth

A group of PT students are compiling an injury prevention program for adolescent soccer players. The independent t test was used as the statistical tool. How would statistically significant results be presented if alpha was set at .05? A. p <.05 B. p >.05 C. r >.05 D. R2 (squared) <.05

A. p <.05 Key word = "significant" ex: 5% chance that you will not pass but there is a 5% chance of making an error in my decision. If P value is less than 5% (p<0.05) that means there were less errors and the results are SIGNIFICANT which is good. Results are meaningful If P value is greater than 5% (p>0.05) that means there were more errors and the results are INSIGNIFICANT which is bad. Results are NOT meaningful

A therapist is performing this test on the patient while examining the hip and knee. What interpretation can be made regarding the muscle length? A. Normal one and two joint Hip flexor B. Shortness of two joint Hip flexor and normal one joint Hip flexor C. Shortness of both one joint and two joint hip flexor D. Shortness of one joint Hip flexor and normal two joint hip flexor

B. Shortness of two joint Hip flexor and normal one joint Hip flexor -iliopsoas is a 1 joint muscle -rec fem is a 2 joint muscle

A 45 year old patient reveals drooping of the shoulder, rotatory winging of the scapula, an inability to shrug the shoulder, and complaints of aching in the shoulder. Based on these findings, the cause of these symptoms would MOST LIKELY be due to: A. Weakness of serratus anterior. B. A lesion of the spinal accessory nerve. C. A lesion of the long thoracic nerve. D. Weakness of lower trapezius muscle.

B. A lesion of the spinal accessory nerve. -rotary winging = UT weakness

Which of the following is NOT an expected sign seen in a patient with facial nerve (CN VII) palsy. A. Hyperacusis B. Absence of sensation in the anterior 2/3 of the tongue C. Absence of corneal reflex D. Decreased lacrimation

B. Absence of sensation in the anterior 2/3 of the tongue Key word: sensation Sensation of anterior 2/3 of tongue is Trigeminal nerve (CN 5) A/C/D are all apart of Facial nerve (CN 7) Hyperacusis = increased sensitivity to sound

A patient referred for vestibular rehab presents with spontaneous nystagmus that can be suppressed with visual fixation, oscillopsia, and a loss of gaze stabilization. Additional symptoms found during examination are disequilibrium, an ataxic wide-based gait with consistent veering to left. Based on these findings, the patient is MOST likely to have which condition? A. BPPV B. Acute UVH C. Acoustic neuroma D. Meniere's disease

B. Acute UVH -A not correct because BPPV can NOT be suppressed with visual fixation

The PT will measure distance between spinous process to inferior angle at 0, 60 and 100 abduction at three different time points. One of the MOST important factors for the success of this study is: A. Face validity B. Intra rater reliability C. Inter rater reliability D. Content validity

B. Intra rater reliability (only 1 PT testing)

A 74 year old retired contractor walks to the treatment room. Patient presents with backward trunk lean during the swing phase of gait. What is the MOST likely cause and associated movement pattern seen? A. Ipsilateral gluteus maximus weakness B. Ipsilateral hip flexor weakness C. Ipsilateral gluteus medius weakness D. Ipsilateral vastus medialis muscle weakness

B. Ipsilateral hip flexor weakness Trunk ext would compensate in order for the LE to swing Rule for hip muscles during swing phase is opposite -backward trunk lean = anterior hip muscles

A 58 year old patient presents to a PT clinic for regular annual check up. The patient has a history of prostate cancer which was recently diagnosed with a benign neoplasm. Which of the following statements BEST describes the condition of the cancer? A. The cancer has spread to bone and will need to be removed. B. The patient has a harmless growth of cells and patient should have regular check ups with the physician C. The patient has a harmful growth of cells and should be treated for malignant tumors. D. The patient will need chemotherapy and radiation immediately.

B. The patient has a harmless growth of cells and patient should have regular check ups with the physician -benign = harmless

Treatment method for horizontal canalithiasis? How to perform technique?

BBQ Roll/Lempert/Baloh Maneuver -Hold each position for 15 seconds -Pt starts in 20 degrees of cervical flex and head turned toward affected side -Head neutral -Head rotated 90 degrees toward nonaffected side -Pt turned into S/L position with head maintained at 90 degree rotation -Pt sit up right with head maintained in 90 degree rotation

A 60 years old male is undergoing cardiac rehabilitation post complicated MI in the hospital. On day 2, the PT wants to progress the patient to sitting. Which of the following is an appropriate INITIAL task for this patient? A Make the patient stand and do weight bearing activities. B Make the patient sit on the upright chair during the visitors time C Make the patient sit on reclining chair and check vitals D Patient is not ready for upright posture yet.

C Make the patient sit on reclining chair and check vitals -SAFETY FIRST = CHECK VITALS!!!

2 Types of CVA: Ischemic vs Hemorrhagic Stroke

CVA: sudden loss of neurological function caused by an interruption in blood flow to the brain Ischemic: -80-85% of CVAs -clot or blockage from result of a build up of atherosclerotic plaque, followed a thrombus or embolism -symptoms may progress in the first 72 hours as infarction and cerebral edema increase Hemorrhagic: -bleeding into the brain as a result of rupture, leakage, or trauma

Two PTs perform a test on a patient using the Oswestry Disability Index. The patient visit is Monday (M) and Wednesday (W). One PT reports scores of 29 and 30 on M & W; other PT scored 36 and 37 (M&W). This is indicative of a problem in: A. Concurrent validity B. Intrarater reliability C. Interrater reliability D. Construct validity

C. Interrater reliability

When observing a patient ambulating, a PT notes that the patient's gait has the following characteristics : inability to initiate and terminate movement, decreased stride length, short bilateral step length, decreased arm swing and trunk rotation. This gait pattern is often observed in patients with diagnosis of: A. CVA B. ALS C. Parkinson's Disease D. Multiple Sclerosis

C. Parkinson's Disease

A 39 year old comes to an outpatient clinic complaining of hip problems. The PT notices a drop of the left hip during right midstance. The MOST LIKELY cause of this impairment can be the injury of: A. Right Inferior gluteal nerve B. Right Femoral nerve C. Right Superior gluteal nerve D. Right Obturator nerve

C. Right Superior gluteal nerve R weak glute med is supplied by superior gluteal nerve Trendelenburg = weak glute med on the stance leg side Glute max supplied by inferior gluteal nerve Femoral nerve supplies anterior side (quads) Obturator nerve supplies medial side (Adductors)

The displayed graph shows trunk lean in obese patients. According to this graph, which of the following statements BEST describes the relationship between trunk lean and waist circumference? (Image on slide 7 FF #31) A. There is strong positive relationship between the two variables. B. There is weak negative relationship between the two variables. C. There is a strong negative relationship between the two variables. D. There is weak positive relationship between the two variables.

C. There is a strong negative relationship between the two variables >0.5 of Rsquared is strong

Which of the following patients will be absolutely contraindicated for an aquatic therapy session with a physical therapist? A. A 22-year male athlete with an open wound covered with occlusive dressing B. A 40-year female with lymphedema post mastectomy but with a fear of water C. A 55-year patient post stroke with a gastrostomy tube D. A 50-year patient with respiratory disorder and vital capacity <1 liter

D. A 50-year patient with respiratory disorder and vital capacity <1 liter

The patient in the picture below shows no reflex activity in the first test but normal in the second test. Which is the cranial nerve involved? (1st picture = corneal reflex) (2nd picture = jaw jerk reflex) A. Afferent fibers of trigeminal nerve and efferent fibers of oculomotor nerve. B. Afferent fibers of facial nerve and efferent fibers of trigeminal nerve C. Afferent fibers of oculomotor and efferent fibers of facial nerve D. Afferent fibers of trigeminal nerve and efferent fibers of facial nerve.

D. Afferent fibers of trigeminal nerve and efferent fibers of facial nerve. Corneal reflex: sensory/afferent = Trigeminal nerve (CN 5) motor/efferent = Facial nerve (CN 7)

A PT instructed a patient in home postural drainage positioning. The PT visits the home and finds the patient trying to perform drainage for the anterior apical segments of upper lobe in supine. What is the best way for the PT to correct the position? A. Change the position to prone lying on a bed with two pillows under the pelvis B. Change the position to sitting on a chair, leaning forward over a folded pillow C. Keep the position to supine on a bed with pillows under the knees D. Change the position to sitting in a recliner, leaning slightly backward

D. Change the position to sitting in a recliner, leaning slightly backward

A PT treating a patient who complains of sharp abdominal pain. The PT palpates by pushing in left lower quadrant, but the pain is elicited in right side. Which of the following is MOST appropriate intervention? A. Apply a moist hot pack to the back and ask him to consult his physician B. Encourage the patient to ignore the pain and do stretching C. Give the patient some pain killers and water D. Have him lie down, remain as still as possible and notify the physician immediately

D. Have him lie down, remain as still as possible and notify the physician immediately (+) for Rovsing's Sign for Appendicitis

A PT is performing cranial nerve tests on a patient. Which of the following is LEAST appropriate to assess the integrity of the facial nerve? A. Ask the patient to show their teeth B. Ask the patient to puff out both cheeks C. Apply sugar or saline solution to the anterior part of the tongue and ask the patient to identify D. Have patient clench teeth and hold against resistance

D. Have patient clench teeth and hold against resistance Tests for trigeminal nerve (CN5): mastication A/B/C all tests facial nerve (CN7)

A physical therapist is examining ECG of a patient who is taking diuretics from last 2 years. An inverted T wave (U wave) is seen in which condition? A. Hypercalcemia B. Hypocalcemia C. Hyperkalemia D. Hypokalemia

D. Hypokalemia

During intervention for a patient with weak tibialis anterior muscles, the PT decides to use FES to improve ambulation. Stimulation should be initiated for the weak muscles during which phase of the gait cycle? A. Mid stance to terminal stance B. Initial contact to mid stance C. Loading response to mid stance D. Initial swing to mid swing

D. Initial swing to mid swing If question was focusing more on preventing foot slap/drop rather than functional aspect the answer would be B

While evaluating the gait cycle of a 68 year old male patient, the PT observes right pelvic hike during the swing phase of the right gait cycle. Which of the following condition is LEAST likely to cause the problem? A. Reduced right hip flexion B. Inadequate right knee flexion C. Lack of right ankle dorsiflexion D. Right ankle plantar flexor weakness

D. Right ankle plantar flexor weakness

A 44 year old male patient is being evaluated by a PT. The patient reports referred pain in the left shoulder with diagnosis of a positive Kehr's sign. Which of the following is NOT a potential cause of a positive Kehr's sign. A. Recent laparoscopy B. Intra abdominal bleeding C. Rupture of the spleen D. Trauma to head of pancreas

D. Trauma to head of pancreas -head of pancreas = R shoulder pain -Kehr's sign is pain at the tip of the shoulder due to blood/air/etc in the abdominal/peritoneal cavity -A/B/C all cause pain to the L shoulder -laparoscopy is a diagnostic surgery that examines the organs in the abdominal cavity and has risk of introducing air in the abdominal cavity Kehr's sign is the occurrence of acute pain in the tip of the shoulder due to the presence of blood or other irritants in the peritoneal cavity when a person is lying down and the legs are elevated. Kehr's sign in the left shoulder is considered a classic symptom of a ruptured spleen. Steve Kehr: "S"teve ("S"pleen) as a LEFTY NBA player, so think KEHR = LEFT shoulder and Spleen

Distinguish Facet Arthropathy vs Spondylolisthesis

Facet -pain with ext and IPSI rotation Spondylolisthesis -pain with ext, IPSI SB, and contra rotation

Interventions for UVH: Adaptation

Gaze Stabilization Exercises (GSR): -coordinate eye and head movements to maintain gaze stability (reset gain of VOR to 1) GSR VOR x1 Viewing -Target is stable -Head movements while eyes stay focused on target GSR VOR x2 Viewing -Progression of x1 Viewing -Target and Head/Eyes move in opposite directions

A 40-year-old male patient has been diagnosed with Spondylolisthesis, a condition where the body of the involved lumbar vertebra slips anteriorly upon the body of the vertebra below. What is the most important recommendation for him? a. Avoid knee flexion and adduction b. Avoid hyperextension at the spine c. Avoid posterior pelvic tilt d. Avoid tying shoes or picking things from the floor

b. Avoid hyperextension at the spine In spondylolisthesis, repetitive extension and torsion activities exacerbate the pain of the patient. So, it will be advisable to avoid hyperextension activities. Flexion exercise and stabilization activities are helpful for the patient.

A patient describes sharp elbow pain over the origin point of the common extensor tendon of the wrist extensors. The pain is alleviated with rest and the Mill's test is positive. Which of the following disorders is MOST likely present? a. Medial epicondylitis b. Lateral epicondylitis c. Anconeus tendonitis d. Olecranon bursitis

b. Lateral epicondylitis In lateral epicondylitis, a patient complains of pain over the lateral aspect of elbow. The pain is related to activities that involve wrist extension/grasp, as it is the wrist extensors that must contract during grasping activities to stabilize the wrist. Diffuse achiness and morning stiffness are also common complaints. And Cozen's test, Mill's test will be positive.

A physical therapist is assessing a patient in a skilled nursing facility with RLE Lymphedema. The patient has a past medical history of cardiac arrhythmia. Objective assessment reveals fibrosis of the RLE, frequent infections, and a positive Stemmer's Sign. Which of the following is MOST likely true? Select one: a. The patient has congestive heart failure b. The patient is in stage 2 lymphedema c. The patient is in stage 0 lymphedema d. The patient is in stage 4 lymphedema

b. The patient is in stage 2 lymphedema Rationale: The pt is in stage 2 lymphedema. Stage 2 lyphedema is spontaneously irreversible lymphedema and is characterized by a (+) Stemmers Sign and brawny edema. Pts in this stage have fibrosis

A patient complains of waking up several times at night from severe "pins and needles" in the right hand. On awakening, the hand feels numb for half an hour and fine hand movements are impaired. The therapist's examination reveals sensory loss and paresthesias in the thumb, index, middle, and lateral half of the ring finger and reduced grip and pinch strength. Some thenar atrophy is present. Based on these examination findings, the MOST appropriate diagnosis is: a. Thoracic outlet syndrome b. Ulnar nerve entrapment. c. Carpal tunnel syndrome d. Pronator teres syndrome

c. Carpal tunnel syndrome. The characteristic features of carpal tunnel syndrome include intermittent pain and paresthesias in the median nerve distribution of the hand and, muscle weakness/ paralysis is also present. The symptoms of CTS are typically worse at night due to the position of wrist flexion typically adopted during sleep and can be associated with morning stiffness. The chief complaint of TOS is diffuse arm and shoulder pain, especially when the arm is elevated beyond 90 degrees. Ulnar nerve entrapment has different levels of affection and it will cause weakness in the muscles supplied by the ulnar nerve. Pronator teres syndrome -The patient typically complains of an insidious onset of pain that is usually felt on the anterior aspect of the elbow, radial side of the palm, and the palmar side of the first, second, third, and half of the fourth digits. Unlike CTS, there is no Tinel's sign at the wrist, and there are no nocturnal symptoms.

A patient with a diagnosis of Parkinson's Disease is referred to an outpatient PT clinic. He is beginning to show signs of bilateral involvement, but has had no falls so far. Which of the following is the MOST APPROPRIATE stage of Hoehn-Yahr classification? Select one: a. Hoehn and Yahr stage I b. Hoehn and Yahr stage III c. Hoehn and Yahr stage II d. Hoehn and Yahr stage V

c. Hoehn and Yahr stage II Rationale: Symptoms of rigidity and postural instability are cardinal symptoms of Parkinson's disease. Parkinson's disease is classified with the Hoehn and Yahr scale. Hoehn and Yahr Level II indicates minimal bilateral or midline involvement with no balance impairment.

Supported upright position with soles of feet on firm surface causes reciprocal flexion/extension of legs a. Palmar grasp reflex b. Plantar grasp reflex c. Rooting reflex d. Walking (stepping) reflex

d. Walking (stepping) reflex 38 weeks of gestation to 2 months

What are Amyotrophic Lateral Sclerosis characteristics?

• LMN pathology Muscle weakness, hyporeflexia, hypotonicity, atrophy, muscle cramps, fasciculations • UMN pathology Spasticity, pathological reflexes, hyperreflexia, muscle weakness • Bulbar Bulbar muscle weakness, dysphagia, dysarthria, sialorrhea, pseudobulbar affect • Respiratory Respiratory muscle weakness (inspiratory and expiratory), dyspnea, exertional dyspnea • Frontotemporal dementia Related impairments (e.g., loss of insight, emotional blunting) • Cognitive impairments (e.g., attention deficits, deficits in cognitive flexibility) • Behavioral impairments (e.g., irritability, social disinhibition)

What is Huntington's Disease (HD)?

• Neurodegenerative GENETIC (hereditary) disease. There is degeneration of the basal ganglia and cerebral cortex. • An autosomal-dominant illness caused by an expansion of the CAG repeats on the short arm of chromosome 4, is clinically characterized by a combination of movement disorders, cognitive decline and behavioral changes.

A patient is referred to physical therapy with a diagnosis of adhesive capsulitis. During the initial evaluation, the patient is found to have limited shoulder range of motion, decreased tolerance to weight-bearing, recurrent night pain, and increased deep tendon reflexes. Which of the following courses of action is BEST for the physical therapist? 1. Contact the patient's referring physician. 2. Initiate small-amplitude oscillations performed at the beginning of the range of motion. 3. Perform pulsed ultrasound to the anterior aspect of the shoulder. 4. Delay therapy for 1-2 weeks until the pain decreases.

1. Contact the patient's referring physician. 1. These measures do not fit with the diagnosis of adhesive capsulitis and might indicate a more serious pathological condition. Signs and symptoms of metastases include decreased tolerance to weight-bearing, change in deep tendon reflexes, and change in sleep habits. 2. Pain control mobilizations are not contraindicated, but it is more important to get the patient back to the physician for reevaluation. 3. Ultrasound should not be performed until a definitive diagnosis can be determined. 4. The physical therapist should take action to refer the patient the physician for reevaluation as soon as possible.

Dementia vs Delirium

-Dementia is gradual and irreversible -Delirium is sudden and reversible -Delirium is typically caused by acute illness or drug toxicity (sometimes life threatening) and is often reversible. -Dementia is typically caused by anatomic changes in the brain, has slower onset, and is generally irreversible.

Which of the following changes in cardiopulmonary function is MOST likely to occur in a pregnant woman in her third trimester of pregnancy? 1. Increase in oxygen consumption 2. Decrease in respiratory rate 3. Decrease in cardiac output 4. Decrease in heart rate

1. Increase in oxygen consumption 1. Cardiac output increases 30% to 60%, and oxygen consumption increases 15% to 20% during pregnancy. 2. Respiratory rate does not change during pregnancy. 3. Cardiac output increases 30% to 60%, and oxygen consumption increases 15% to 20% during pregnancy. 4. Heart rate increases during pregnancy.

A healthy, older, sedentary individual has normal vital signs at rest. During maximal aerobic exercise, which of the following values should demonstrate MINIMAL change? 1. Stroke volume 2. Diastolic blood pressure 3. Venous oxygen content 4. Systolic blood pressure

2. Diastolic blood pressure 1. Stroke volume increases linearly with acute exercise (Avers, p. 401). 2. During endurance exercise systolic blood pressure increases, but diastolic blood pressure should not change significantly. (Hillegass, p. 532) 3. Even in the elderly population, a great amount of oxygen is extracted from working muscles, and venous oxygen changes significantly with exercise (Avers, p. 402). 4. Systolic blood pressure increases in direct proportion to the increase in exercise intensity. Also, a rise in systolic pressure must occur as the result of increased cardiac output required to sustain exercise. (Hillegass, p. 532)

Which of the following BEST describes the huffing technique of airway clearance? 1. Cough while holding a pillow against the chest. 2. Forcefully exhale while keeping the mouth open. 3. Forcefully exhale through pursed lips. 4. Take two normal breaths, then cough firmly on the third exhalation.

2. Forcefully exhale while keeping the mouth open. 1. Coughing while holding a pillow against the chest refers to splinting the incision and is most commonly used by surgical patients to reduce pain while coughing (p. 545). 2. Huffing consists of taking a deep inspiration followed by forceful exhalation without closing the glottis. Keeping the mouth open facilitates this. (p. 545) 3. Exhaling through pursed lips is a technique for dyspnea, not airway clearance (p. 550). 4. Huffing involves taking one deep inspiration, not two normal breaths (p. 545).

During auscultation of the chest wall, a physical therapist notes an area of decreased breath sounds. Which of the following additional findings would support the suspicion of consolidation? 1. Tympanic sound with percussion 2. Increased fremitus 3. Hypertrophy of accessory muscles 4. High-pitched wheezes

2. Increased fremitus 1. A tympanic sound with percussion may be heard over a hyperinflated chest (p. 528). 2. Increased fremitus is palpated in the presence of an increase in secretions (p. 525). 3. A person with chronic obstructive pulmonary disease might exhibit increased muscle activity in the respiratory accessory muscles (p. 190). This would not typically be observed in a person with an increase in secretions. 4. The adventitious sound associated with consolidation would be crackles, not wheezing (pp. 521-522).

A patient sustained a distal radius fracture involving the Lister tubercle 6 weeks ago. During a physical therapist's assessment, no limitation in passive range of motion in the thumb (1st digit) interphalangeal and metacarpophalangeal joints is found. What active thumb (1st digit) motion is MOST likely to be affected? 1. Interphalangeal flexion 2. Interphalangeal extension 3. Metacarpophalangeal flexion 4. Metacarpophalangeal adduction

2. Interphalangeal extension 1. The flexor pollicis longus acts on interphalangeal flexion. Although its proximal attachment is on the radius, it does not wrap around the Lister tubercle. (pp. 750, 772) 2. The extensor pollicis longus brevis wraps around the Lister tubercle and attaches distally on the dorsal side of the distal phalange of the thumb (1st digit). If a patient has a fracture to the distal radius and no other soft tissue restrictions are noted, then interphalangeal extension would be affected. (pp. 754, 772) 3. The flexor pollicis brevis acts to achieve thumb (1st digit) metacarpophalangeal flexion. The flexor pollicis brevis does not wrap around the Lister tubercle. (pp. 754, 772) 4. The adductor pollicis acts to achieve thumb (1st digit) metacarpophalangeal adduction. The adductor pollicis does not wrap around the Lister tubercle. (pp. 754, 772)

A patient complains of back pain that radiates to his buttock and leg when bending his trunk forward however the pain centralizes when he moves to an erect posture. What's the best initial intervention? Select one: 1. Lumbar traction with 25 percent BW in supine 2. Lumbar traction with 25 percent body weight in prone 3. Lumbar traction with 50 percent BW in prone 4. Lumbar traction with 50 percent body weight in supine

2. Lumbar traction with 25 percent body weight in prone Answer: The symptoms suggest a disc pathology. Treatment force of lumbar traction for disc protrusion is 25 percent BWT in prone position. For joint distraction it is 50 percent BWT in prone

A patient's electrocardiogram report describes the presence of significant Q waves. This finding is suggestive of which of the following conditions? 1. Premature atrial complex 2. Myocardial infarction 3. Supraventricular tachycardia 4. Atrial fibrillation

2. Myocardial infarction 1. A premature atrial complex is indicated by an R wave that is close to the preceding R wave, and the P wave of the early beat is shaped differently from normal P waves (p. 317). 2. A prominent, pathological Q wave is indicative of a transmural myocardial infarction (pp. 330-331). 3. Supraventricular tachycardia is associated with P waves that can be merged into T waves. The QRS complexes are typically identical in shape and normal in duration. (p. 319) 4. Atrial fibrillation is characterized by inconsistent, irregular R-R intervals (p. 321).

A 24-month-old child who has a genetic disorder and developmental delay is able to sit independently but can easily be displaced and demonstrates emerging protective reactions and slow trunk-righting reactions in sitting position. Which of the following functional activities is BEST for short-term physical therapy for the child? 1. Pulling up to standing position 2. Obtaining a toy when placed out of reach 3. Holding a toy at midline with both hands 4. Transitioning from sitting to quadruped position

2. Obtaining a toy when placed out of reach 1. Pulling to stand is too difficult for a short-term physical therapy goal for this child, who is functioning at a 5-month to 6-month level. This activity would require intact protective extension and trunk righting. (p. 61) 2. This child is at the second stage of sitting (around 5 months) and would benefit from activities that challenge trunk righting, sitting balance, and protective reactions. These are within the child's ability to achieve, because they are items in the third stage of sitting, which usually occurs around age 6-7 months. (pp. 56-57) 3. No position is noted. This task could be achieved in a supine position. Holding a toy within the center of mass in sitting position is also a skill that is likely already achieved in the second stage of sitting, at age 5-6 months. (p. 57) 4. Transitioning from sitting to quadruped position is a task that is generally achieved in the third quarter of the first year, so it is not an appropriate goal of short-term physical therapy for a child who is in the 5-month to 6-month range of development, which is consistent with second stage of sitting. (p. 61)

Which of the following home programs is MOST appropriate for a patient who has chronic lateral epicondylalgia? 1. Using a forearm cuff to increase loading on the extensor tendons 2. Performing exercises for wrist strength and stretching 3. Administering iontophoresis with dexamethasone (Decadron) and lidocaine (Xylocaine) 4. Doing friction massage of the brachioradialis tendon

2. Performing exercises for wrist strength and stretching 1. A forearm cuff is thought to decrease the muscle loading (p. 723). 2. Lateral epicondylalgia is caused by overuse of the wrist extensors that originate on the lateral epicondyle of the humerus, especially the extensor carpi radialis brevis. If the lateral epicondylalgia is at a chronic stage, conditioning of the extensor muscles and sustained grip activities will be most effective in long-term management. (pp. 720, 723) 3. Iontophoresis would not be appropriate for a home program and is more appropriate for acute lateral epicondylalgia (p. 722). 4. Friction massage of the brachioradialis would not be appropriate since the extensor carpi radialis brevis muscle is usually the muscle that is affected (pp. 720, 722).

An outpatient physical therapist is working with a patient who suddenly demonstrates repetitive and rhythmic head and neck movements and has an unfocused stare. Which of the following actions should the therapist perform FIRST? 1. Call emergency medical services. 2. Remove objects from the immediate area. 3. Place a tongue depressor in the patient's mouth. 4. Hold the patient's head and neck still.

2. Remove objects from the immediate area. 1. Seizures are usually finite in duration. Unless the individual develops status epilepticus (which is not described in the stem), calling emergency medical services is not the first action to perform. 2. A patient who is having a seizure normally only needs protection from injury in the environment. 3. It is not appropriate to insert a tongue protector to protect the tongue from injury. This is more likely to harm the patient. 4. The physical therapist could harm the patient by trying to restrict movement.

Which of the following conditions is a CONTRAINDICATION for mechanical traction of the cervical spine? 1. Muscle spasm 2. Rheumatoid arthritis 3. Hypomobility of the spine 4. Decreased upper extremity sensation

2. Rheumatoid arthritis 1. Gentle intermittent traction may assist in decreasing muscle spasm and the resultant spinal compressive forces. (Cameron, p. 378) 2. Joint capsules, ligaments, and bones are fragile in patients with rheumatoid arthritis (RA). A patient with RA is subject to atlantoaxial subluxation or to developing instability next to areas of hypomobility. (Bellew) 3. Longitudinal traction force provides a gliding separation of the facets, general capsular stretch, and opening of the intervertebral foramen. Traction can improve range of motion. (Cameron, p. 378) 4. Reduction of neurological deficits may result from improved conduction in large-diameter myelinated afferent and efferent nerve fibers. Mechanical traction can release pressure on nerve fibers, thereby increasing nerve conduction. (Cameron, p. 377)

Which of the following conditions is relieved by flexion and aggravated by extension? Select one: 1. Muscle strain 2. Spinal stenosis 3. Osteoporosis 4. Disc herniation

2. Spinal stenosis Answer: Spinal stenosis (narrowing of the spinal canal) is aggravated by extension while flexion relieves it. Flexion widens the spinal canal causing relief for spinal stenosis whereas extension makes it worse. Spondylolisthesis is also aggravated with extension and relieved by flexion. Osteoporosis can cause vertebral fractures when performing flexion and flexion + rotation exercises due to the compression of already porous vertebral bodies.Disc herniation is aggravated by flexion and ipsilateral lateral flexion; and relieved with extension. Muscle strain is aggravated with standing/bending as the muscle is recruiting/activation but is relieved with sitting.

An appropriate long-term goal for a young adult with a C6 spinal cord injury and no additional complicating factors is: 1. propelling a standard wheelchair on level surfaces with moderate assistance. 2. completing sliding board transfers to a level surface independently. 3. performing bed mobility using a bed rail and loops with maximum assistance. 4. performing lower body dressing independently.

2. completing sliding board transfers to a level surface independently. 1. A patient who has a C6 spinal cord injury can independently (without requiring moderate assistance) propel a manual wheelchair. 2. A patient who has a C6 spinal cord injury can independently perform sliding board transfers. 3. A patient with this level of spinal cord injury is expected to achieve independence to some assistance with bed mobility using adaptive equipment 4. A patient with this level of spinal cord injury is expected to need assistance and adaptive equipment when performing lower body dressing.

A physical therapist has been treating a female patient with low back pain for 6 weeks. The patient reports that the pain seemed to lessen earlier in the course of treatment but has intensified again in the past week. To help identify a systemic cause for the pain, the therapist should ask the patient whether the pain: 1. decreases when she lies down. 2. corresponds with her menstrual cycle. 3. increases when she is driving to work. 4. increases when she has a bowel movement.

2. corresponds with her menstrual cycle. 1. Decreased pain with rest is more likely to indicate a mechanical cause of pain. 2. Pain from endometriosis often is cyclic in nature and corresponds with the menstrual cycle. 3. Pain with driving is more likely to indicate a mechanical cause of pain. 4. Pain with bowel movement is more likely to indicate a mechanical cause of pain.

A basketball player is practicing 15 shots at the goal line and then 15 layups in preparation for the upcoming game. The player is MOST LIKELY demonstrating which form of practice and stage of learning? Select one: 1. Random practice, associative stage 2. Random practice, cognitive stage 3. Blocked practice, associative stage 4. Distributed practice, cognitive stage

3. Blocked practice, associative stage Answer: With blocked practice the patient will repeat the same task in series to improve performance as is being demonstrated in this scenario. The basketball player is practicing a set of 15 three-pointers and then 15 lay-up shots. With random practice the patient would have performed a variation of shots in random order to improve performance, cognition and retention. During the cognitive stage the patient is learning what to do while in the associative stage the patient is learning how to do the task and relies on proprioceptive feedback

After the use of a cold pack during the previous session, a patient reports having developed raised, red patches of skin that lasted for 24 hours in the area. The physical therapist should take which of the following actions? 1. Add a towel layer to reduce cold intensity. 2. Contact the physician. 3. Discontinue cryotherapy. 4. Replace the cold pack with a different cold modality.

3. Discontinue cryotherapy. 1. The response is indicative of wheals; it is not indicative of a medical emergency. While adding a layer would decrease the sensitivity, the best indicated action is to document the response and discontinue the modality. (p. 40) 2. The response is indicative of wheals, which is a sign of cold hypersensitivity. The indicated action is to document the response and discontinue the modality. It is not indicative of a medical emergency. (p. 40) 3. The response is indicative of wheals, which is a sign of cold hypersensitivity. The indicated action is to document the response and discontinue the modality. (p. 40) 4. It would not be correct to substitute another cold modality. The indicated action is to document the response and discontinue the modality. (p. 40)

A patient is seen in physical therapy after an arthroscopic debridement of the knee. Which of the following describes the optimal parameters for electrical stimulation to address an extensor lag noted during the examination? 1. Duty cycle of 50%; current intensity high enough to produce 40% of maximal voluntary isometric contraction; 20 minute total treatment time 2. Duty cycle of 20%; current intensity high enough to produce 40% of maximal voluntary isometric contraction; 20 minute total treatment time 3. Duty cycle of 20%; current intensity high enough to produce 60% of maximal voluntary isometric contraction; 10 minute total treatment time 4. Duty cycle of 50%; current intensity high enough to produce 60% of maximal voluntary isometric contraction; 10 minute total treatment time

3. Duty cycle of 20%; current intensity high enough to produce 60% of maximal voluntary isometric contraction; 10 minute total treatment time 1. A duty cycle of 50% is too high and the current intensity is not optimal. 2. The current intensity is not optimal. Greater than or equal to 50% is recommended for strengthening. 3. For strengthening with electrical stimulation the current intensity should be maximal, so 60% is the better answer when compared with 40% of maximal voluntary contraction. Also, the duty cycle should allow for the rest period to be 5 times as long as the hold cycle, so a 20% duty cycle is best. One treatment session should include at least 10 contractions, so a 10 minute treatment time is appropriate. 4. The duty cycle is too high in this example; more rest time is needed for muscle recovery.

A physical therapist is examining a patient who has multiple sclerosis. The patient sits for 12 hours/day with good spinal and pelvic alignment in a standard wheelchair with a foam cushion. The patient sleeps in a sidelying position. The patient's family reports that the patient has recently gained a considerable amount of weight. Which of the following areas is/are at GREATEST risk for skin breakdown? 1. Lateral malleoli 2. Sacrum 3. Greater trochanters 4. Scapulae

3. Greater trochanters 1. The skin over the malleoli is unlikely to be under pressure when the patient is sitting in the wheelchair. 2. Because the patient sits with good alignment, skin breakdown over the sacrum is not likely to occur. 3. With the patient's recent weight gain, the wheelchair armrests and/or frame may be putting pressure on the skin over the greater trochanters. The greater trochanters are also at risk when the patient is in sidelying position. 4. Because the patient sits with good alignment, skin breakdown over the scapulae is not likely to occur.

Which of the following laboratory values should a physical therapist monitor when treating a patient who is taking warfarin (Coumadin)? 1. Hemoglobin 2. Red blood cell count 3. International normalized ratio 4. Erythrocyte sedimentation rate

3. International normalized ratio 1. Hemoglobin would not be changed by anticoagulant medication. Hemoglobin values measure the oxygen-carrying capacity of the red blood cells. (p. 1713) 2. Red blood cell counts are not changed by anticoagulants. The red blood cell count is a method used to assess the oxygen-carrying capacity of the blood. (p. 1712) 3. Warfarin (Coumadin) is an anticoagulant. The physical therapist must be aware when a patient is taking an anticoagulant so that treatment can be modified if there is an increased risk of hemorrhage. The international normalized ratio (INR) was developed to provide results that would not vary between laboratories. Therapeutic anticoagulation requires an INR of 2 to 3. As the INR increases above these values, the risk of bleeding during activity is increased. (pp. 1712-1713) 4. The erythrocyte sedimentation rate is not expected to be affected by anticoagulant medication. The erythrocyte sedimentation rate is used to identify inflammatory or necrotic processes. (p. 1715)

A 55 year-old female arrived at an outpatient clinic with complaints of nausea and dizziness during positional changes including getting in and out of bed. When performing the dix hall pike test, the Physical Therapist notices a persistent up beating torsional nystagmus on the left side which lasted for a duration of 1 min 20 seconds. The Physical Therapist findings are MOST consistent with which of the following diagnosis? Select one: 1. Right posterior SCC canalisthesis 2. Right posterior SCC cupulolithiasis 3. Left posterior SCC cupulolithiasis 4. Left posterior SCC canalisthesis

3. Left posterior SCC cupulolithiasis Answer: The patient most likely presents with left posterior SCC cupulolithiasis as there is an up beating nystagmus and the duration of the nystagmus is >1 min(persistent).Canalithiasis is usually transient (transient nystagmus: duration <1 min) and superior/anterior SCC will present with down beating.

A 55 year-old female arrived at an outpatient clinic with complaints of nausea and dizziness during positional changes including getting in and out of bed. When performing the dix hall pike test, the Physical Therapist notices a persistent up beating torsional nystagmus on the left side which lasted for a duration of 1 min 20 seconds. The Physical Therapist findings are MOST consistent with which of the following diagnosis? Select one: 1. Right posterior SCC canalisthesis 2. Right posterior SCC cupulolithiasis 3. Left posterior SCC cupulolithiasis 4. Left posterior SCC canalisthesis

3. Left posterior SCC cupulolithiasis Rationale: The patient most likely presents with left posterior SCC cupulolithiasis as there is an up beating nystagmus and the duration of the nystagmus is >1 min(persistent).Canalithiasis is usually transient (transient nystagmus: duration <1 min) and superior/anterior SCC will present with down beating. Reference: Physical Rehabilitation by Susan O Sullivan

During the examination of a patient who went on a hiking trip 1 week ago, a physical therapist notes neck stiffness and notices a red rash with partial central clearing on the patient's upper extremity. The therapist should suspect that the patient has which of the following conditions? 1. Meningitis 2. Herpes zoster 3. Lyme disease 4. Retropharyngeal abscess

3. Lyme disease 1. The history of the patient is inconsistent with the classic history of meningitis (pre-existing respiratory infection) (p. 231). 2. Herpes zoster (shingles) occurs in a dermatologic distribution and is characterized by burning pain (pp. 255-256). 3. The history of this patient (hiking trip), symptoms (neck stiffness), and findings (rash) are consistent with Lyme disease (p. 233). 4. Retropharyngeal abscess usually occurs in children younger than 4 years old or is the result of trauma or dental infection (p. 235).

A patient who has a C5 spinal cord injury (ASIA Impairment Scale A) suddenly reports light-headedness and ringing in the ears while sitting upright in a wheelchair. Which of the following conditions is MOST likely present? 1. Angina pectoris 2. Deep vein thrombosis 3. Orthostatic hypotension 4. Autonomic dysreflexia

3. Orthostatic hypotension 1. Angina is cardiac-related chest pain due to ischemia; it is unrelated to ringing in the ears (p. 481). 2. The hallmark sign of deep vein thrombosis is rapid onset of unilateral leg swelling, tenderness, and tightness (p. 607). 3. Orthostatic hypotension is a common complication in patients who have an acute cervical injury. It manifests with dizziness or light-headedness and ringing in the ears when in a vertical position, such as sitting or standing. (p. 876) 4. Autonomic dysreflexia is common in patients who have a spinal cord injury at T6 or above and results in significantly increased blood pressure and a pounding headache (p. 861).

Which of the following strategies would be MOST appropriate when measuring the size and shape of a partial-thickness wound? 1. Hold a tape measure above the wound surface and measure the diameter. 2. Insert a cotton-tipped applicator into the wound at its deepest point and measure the distance from the wound bed to the level of the skin surface. 3. Place a sheet of plastic wrap on the wound and use a marking pen to draw a tracing of the wound's perimeter on the plastic. 4. Place normal saline in the wound, using a measured syringe to fill the wound to the level of the skin surface.

3. Place a sheet of plastic wrap on the wound and use a marking pen to draw a tracing of the wound's perimeter on the plastic. 1. This technique can be used to measure size, but it is not useful for measuring the shape of the wound (Bryant, p. 116). 2. This technique can be used to measure depth but not to measure size and shape. It is not accurate and is difficult to reproduce measurements. A partial-thickness wound has no measurable depth. (Bryant, p. 116) 3. This is the only option in the list that measures size and shape (Hamm, p. 77). 4. This technique is called fluid instillation and is used to measure wound volume rather than size and shape (Bryant, p. 115).

Which of the following findings is MOST likely a negative prognostic indicator for surgical wound healing? 1. Bloody discharge on postoperative day 1 2. Clear and watery discharge on postoperative day 3 3. Purulent discharge on postoperative day 5 4. No discharge and a raised suture line on postoperative day 10

3. Purulent discharge on postoperative day 5 1. With full thickness wounds such as surgical incisions, bleeding is part of the normal process of hemostasis in the acute phase of healing (Bryant, p. 67) 2. Clear and watery discharge on postoperative day 3 is a normal response for a healing surgical wound (Paz, p. 319). 3. A purulent exudate from a surgical wound site is an indication of infection and is a negative prognostic indicator for healing (Paz, pp. 311, 319). 4. No discharge and a raised suture line on postoperative day 10 are consistent with a normal response for a healing surgical wound (Bryant, p. 71).

A 41-year-old patient sustained a grade II injury to the right medial collateral ligament of the elbow 3 weeks ago. A systems review reveals normal cardiopulmonary, integumentary, and neuromuscular status. Which of the following is MOST likely the status of the healing tissue? 1. Well organized and gaining in tensile strength 2. Well organized and capable of withstanding high tensile forces 3. Disorganized but able to withstand high tensile forces 4. Disorganized and unable to withstand high tensile forces

4. Disorganized and unable to withstand high tensile forces 1. Although the repair is gaining in tensile strength, 3 weeks is not enough time for a grade II injury to fully organize. 2. A longer period of time is needed for a well-organized repair in a grade II injury. 3. A disorganized repair is not able to withstand high tensile forces. 4. A grade II ligament injury requires significant repair, and, although the tissue is beginning to organize at 3 weeks, it is not fully organized and is deficient in tensile strength.

A patient has an ulcer on the right lateral malleolus. The ulcer has minimal drainage and measures 1.5 cm × 2.0 cm and 0.5 cm in depth. The patient has an ankle-brachial index of 0.68. Which of the following interventions should be undertaken FIRST? 1. Elevation 2. Compression 3. Hyperbaric oxygen 4. Pressure off-loading

4. Pressure off-loading 1. An ankle-brachial index of 0.68 indicates probable arterial insufficiency (p. 199). Elevation will decrease arterial flow and should not be used. 2. An ankle-brachial index of 0.68 indicates probable arterial insufficiency (p. 199). The wound description does not indicate any edema. Maintained compression would further compromise perfusion in the presence of arterial insufficiency (p. 199). 3. Hyperbaric oxygen can increase oxygen delivery to the wound; however, pressure unloading should occur first to remove the cause of the wound and prevent further breakdown (pp. 199, 362). 4. This wound is over a pressure area. In patients who have vascular disease, the skin and its supporting structures have a decreased ability to endure pressures. Therefore, patients who have vascular disease are at high risk for developing pressure injuries. Off-loading is a first response. (p. 129)

A PT is performing assessing the lung function of a patient with a diagnosis of COPD. The PT would like to assess the amount of air in the lungs after a NORMAL exhalation. Which of the following parameters is the PT referring to? A. Functional residual capacity B. Tidal volume C. Expiratory reserve volume D. Vital capacity

A. Functional residual capacity FRC is the amount of air left after a NORMAL exhalation RV is amount of air left after a FORCEFUL expiration

Self selected speed is measured on a continuous scale in 2 groups of professional and amateur individuals. The data shows markedly skewed distribution. Which of the tests is MOST appropriate to find significant difference in gait speed between two groups? A. Mann Whitney U test B. Analysis of variance C. T test D. Kruskal Wallis test

A. Mann Whitney U test Step 1: how many groups are there? 2 Step 2: is it parametric (equal) or non-parametric/skewed (non-equal)

A PT is treating a patient with a diagnosis of right cerebrovascular accident. On examination, biceps are graded 2 on the Modified Ashworth Scale. Which of the following is the MOST effective intervention to address these findings? A. Prolonged cryotherapy to the left biceps B. Quick icing of the left biceps C. Quick icing of the right triceps D. Neutral warmth to the right biceps

A. Prolonged cryotherapy to the left biceps Rationale: Patient with right CVA with have symptoms on the left side of the body. Grade 2 on the modified ashworth scale is indicative of increase in tone in the biceps on the left side. Quick icing is contraindicated in the agonist muscle group as it increases the tonicity. Prolonged cryotherapy to the hypertonic muscles is the most effective treatment. Neutral warmth and quick icing of the antagonistic muscles can be used but the impairment is not on the right side.

A 35 year old woman comes to PT clinic 3 weeks after normal vaginal delivery. The patients complains of leaking of urine involuntarily during coughing, sneezing, laughing and when climbing stairs. Which of the following is the MOST likely diagnosis? A. Stress incontinence B. Overflow incontinence C. Functional incontinence D. Urge incontinence

A. Stress incontinence

Ankle/Hip Strategies

Ankle strategy: •Sway is small, slow, and near midline, and disturbances of the COM are small and well within the limits of stability. •Muscle activation: Distal to proximal (i.e., gastrocnemius, hamstrings, paraspinals). •Forward sway, gastrocnemius is activated first, followed by hamstrings, then paraspinal muscles. •Backward sway, the anterior tibialis is activated first, followed by quadriceps, then abdominals. Hip Strategy: •Sway is large, faster sway frequencies and larger disturbances of the COM •Muscle activation: Proximal to distal (i.e., abdominals, quadriceps, tibialis anterior). •With forward sway abdominals are activated first, followed by quadriceps. •With backward sway, paraspinal muscles are activated first, followed by hamstrings

A PT administers a series of cranial nerve tests to a patient with a confirmed lower motor neuron disease. Assuming the patient has a lesion impacting the right hypoglossal nerve, which clinical presentation would be most likely? A. right sided tongue atrophy and deviation toward the left with tongue protrusion B. right sided tongue atrophy and deviation toward the right with tongue protrusion C. left sided tongue atrophy and deviation toward the left with tongue protrusion D. left sided tongue atrophy and deviation toward the right with tongue protrusion

B. right sided tongue atrophy and deviation toward the right with tongue protrusion Lick your Lesion: ex: lesion on the right side so I want to deviate my tongue to the right Hypoglossal nerve injury —— > TONGUE will DEViate towards the weakness

A PT is evaluating a patient with tingling sensation on the anterior leg and dorsum of the foot. The PT suspects involvement of the common peroneal nerve. A positive finding on which of the following tests will confirm entrapment of the common peroneal nerve? A. Flexion and adduction of hip with knee extension and ankle dorsiflexion B. Hip flexion, knee extension, dorsiflexion of ankle with inversion of foot C. Knee extension, Hip internal rotation and flexion, inversion of foot with plantar flexion of ankle D. Knee extension, hip flexion, ankle dorsiflexion with everted foot

C. Knee extension, Hip internal rotation and flexion, inversion of foot with plantar flexion of ankle

Rancho Los Amigos Level of Cognitive Functioning

Mnemonic video: https://www.youtube.com/watch?v=SaeFIMuKCHU "New Girl Looks Confused but Cute, wears a CAP"

What is the plantar reflex and which nerve does it test?

Normal: flexion of toes when stroking lateral sole of foot (+) Ext of toes: Babinski present, seen with corticospinal/UMN lesions

Ramp Rules WC Rules Door Rules

Push handles to floor = 36'' Arm rest to floor = 30'' Total length = 46'' Total width = 23" Door Width: -Minimum = 32" -Ideal = 36" Reach Forward -HIGH 48" -LOW 15" -SIDE is 24" Rotate on WC rules: -To rotate 90⁰ you need 36" -To rotate 180 you need 60" -To rotate 360 you need 60" x 60" Ramp rules 8.3 % ramp 1 : 12 Normal Adult seat depth 16" Normal Adult seat width 18" Normal Adult seat height 20" Hemiplegic 17.5 "The ADA requires that all business and public use wheelchair ramps adhere to a 1:12 slope ratio, meaning that for every inch of vertical rise there must be 12 horizontal inches (one foot) of ramp. If, for example, you had a stairway with a vertical rise of 21″, you would need 21′ of ramp. Additionally, ADA stipulates that no ramp can run longer than 30 consecutive feet without a rest or turn platform."

Symptoms associated with Multiple Sclerosis?

Sensory Impairments -focal deficits most common -paresthesia -numbness -proprioception -vibration sense Motor Impairments -UMN disease -loss of orderly recruitment and decreased firing rate of motor neurons -paresis spasticity -increased DTR (UMN) -spasms Pain -Neuropathic pain -Trigeminal neuralgia: severe stabbing pain to one side of face due to demyelination of CN5 -Paroxysmal limb pain -Headache

A physical therapist is guarding a patient who is using an assistive device to come down a flight of stairs. Which of the following is the SAFEST position for the therapist to take to avoid self-injury? 1. In front, with one foot on the step immediately below the patient 2. In front, with both feet on the step immediately below the patient 3. In back, with one foot on the same step as the patient 4. In back, with both feet on the same step as the patient

1. In front, with one foot on the step immediately below the patient 1. The physical therapist should stand in front and to the side of the patient with one foot on the step to which the patient will step and the other foot on the next step down. 2. Standing with both feet on the same step will make the physical therapist unstable if the patient begins to fall. This position does not provide a wide anteroposterior base of support. 3. In back, with one foot on the same step as the patient is not the safest position for the physical therapist, because the therapist may be pulled forward if the patient loses balance forward. 4. Standing with both feet on the same step will make the physical therapist unstable if the patient begins to fall. This position does not provide a wide anteroposterior base of support.

Which of the following procedures should be used to assess a patient's equilibrium? 1. Marching in place 2. Rebound test 3. Heel on shin 4. Finger to therapist's finger

1. Marching in place 1. Equilibrium is a dynamic reaction essential for upright posture and smooth transitional movements. Equilibrium is assessed by testing the body's ability to respond to a change in body position or surface support to maintain body alignment (O'Sullivan, pp. 199-200) Of the options, only marching in place requires a displacement of the center of gravity and a places a demand on the body to maintain upright posture. (Lazaro) 2. The rebound test is a nonequilibrium coordination test, involving contraction of the biceps, sudden release of resistance, and sudden contraction of the triceps to retrain limb movement (O'Sullivan, p. 200). 3. Heel on shin is a nonequilibrium coordination test (O'Sullivan, p. 200). 4. Finger to therapist's finger is a nonequilibrium coordination test (O'Sullivan, p. 199).

What should you do with a pt that has an AV block?: 1st degree 2nd degree type 1 2nd degree type 2 3rd degree

1st degree: continue exercise (not severe) 2nd degree type 1: monitor pt and continue with lower intensity exercise 2nd degree type 2: stop exercise 3rd degree: stop exercise immediately and refer

A Physical Therapist is performing a postural assessment on a patient who presents with hips and knees hyperextended, posterior pelvic tilt, elongated neck flexors. The patient's favored leg appearing longer in standing. The patient MOST likely has which type of deformity? Select one: 1. Round back form of kyphosis 2. Swayback 3. Gibbus deformity 4. Flat back form of Kyphosis

2. Swayback Answer: -Swayback is described as the pelvis shifting posteriorly causing the hips and knees to be hyperextended, a flat back and favored leg appears to be longer in standing with neck flexors elongated. -Round back form of kyphosis is described as hips flexed and knees extended, neck flexors elongated, neutral pelvis. -Flat back form of kyphosis is described as loss of lordosis with pelvis in posterior pelvic tilt, hips and knees hyperextended, forward head posture with increased flexion in the upper thoracic spine.

For a patient who takes a thiazide medication (hydrochlorothiazide), which of the following combinations of signs may indicate an adverse effect of the medication? 1. Agitation, dizziness, dyspnea, and hyperventilation 2. Abdominal cramps, agitation, dyspnea, and numbness 3. Clammy skin, dizziness, hyperventilation, and hypotension 4. Abdominal cramps, clammy skin, hypotension, and numbness

3. Clammy skin, dizziness, hyperventilation, and hypotension 1. Thiazide diuretics increase secretion of sodium, so an adverse drug effect is too much secretion, which leads to hyponatremia (Ciccone, p. 319). Agitation and dyspnea are signs of hypernatremia (Goodman, p. 207). 2. Thiazide diuretics enhance secretion of sodium and potassium in the urine, leading to hyponatremia and hypokalemia (Ciccone, p. 319). The signs given are for hypernatremia and hyperkalemia (Goodman, p. 207). 3. Thiazide diuretics enhance secretion of sodium and potassium in the urine, leading to hyponatremia and hypokalemia (Ciccone, p. 319). Clammy skin and hypotension are signs of hyponatremia, and dizziness and hyperventilation are signs of hypokalemia (Goodman, p. 207). 4. Abdominal cramps and numbness are signs of hyperkalemia (Goodman, p. 207). Potassium is excreted along with sodium in patients who are taking thiazide diuretics, so hypokalemia rather than hyperkalemia may result (Ciccone, p. 319).

A Physical Therapist is treating a patient with Achilles tendonitis. Which of the following is the best exercise for this patient ? Select one: 1. Manually resisted plantarflexion 2. Resisted walking backward 3. Side step down steps 4. Seated rocker board ankle plantarflexion

3. Side step down steps Answer: A resistance training program that emphasizes eccentric loading of the ankle plantarflexors has been shown to decrease pain and improve physical functioning in patients with mid-position Achilles tendinopathy. Eccentric loading, that emphasizes heel-lowering exercises, has been found to be beneficial in the management of insertional Achilles tendinopathy. Active plantarflexion in sitting position would not be best initially in treating Achilles tendonitis. Resisted walking backward with TheraBand would be a concentric activity and not the best exercise option for this patient. Manually resisted plantarflexion is also an open chain exercise. The best exercise option would be to place an eccentric load on the plantarflexor muscles.

Spatial-Perceptual Dysfunction: Agnosia vs Ideational Apraxia vs Ideomotor Apraxia

Agnosia -inability to recognize an object by sight, touch, or hearing Ideational apraxia -inability to perform the a task -Ex: give a pt a toothbrush and they have NOIDEA what to do with the toothbrush, you might see them brush their hair with a toothbrush, they've lost the ability to brush their teeth Ideomotor apraxia -inability to perform the task when asked/"ON COMMAND" to but they can do it normally Ex: you give them a toothbrush but they don't know what to do but you ask them how they got ready this morning and they said they brushed their teeth

How to Ascending/Descending Curb

Ascending -Place front casters up on the curb -Tell pt to lean forward to prevent from falling backward -Push rear wheel up curb with momentum Descending -Descending backwards with forward head and trunk lean to prevent from falling backward -LEAN FORWARD FOR BOTH ASCENDING AND DESCENDING

A physical therapist is performing joint mobilization on a patient in the outpatient clinic. The PT notes the electrocardiogram in the photograph. The physical therapist's INITIAL response should be to: (look on slide 27 on FF#9) A. Continue with joint mobilizations at same intensity B. Continue with joint mobilizations at a lower intensity C. Stop the treatment and monitor ECG for 10 minutes. D. Activate the emergency system or call 911

D. Activate the emergency system or call 911 (Ventricular fibrillation = ventricles beating too fast)

How do you tell the difference between Myocardial ischemia and Myocardial infarction on a EKG chart reading?

Myocardial ischemia = ST segment depression (lack of blood flow = depression/ischemia) "are you DOWN for a SCHEME?" Myocardial infarction = ST segment elevation (slide 21 on FF #9) "if you go FAR you will ELEVATE"

What is preload?

Preload, also known as the left ventricular end-diastolic pressure (LVEDP), is the amount of ventricular stretch at the end of diastole. Think of it as the heart loading up for the next big squeeze of the ventricles during systole

DD: UVH BVH Otitis media Ototoxicity

UVH: -Impairment of balance system in the inner ear, the peripheral vestibular system is not working properly in ONE EAR (viral insults, trauma, vascular events) -Dizziness or vertigo, poor balance, especially with head turns, blurred vision, especially when turning your head quickly -Nausea -Trouble walking, especially outdoors, in dark rooms, or in crowded places BVH: -BOTH EARS affected -Ototoxicity (especially using antibiotics such as gentamicin, streptomycin), meningitis, autoimmune disorder, head trauma, tumor on the 8th CN neuronitis -Oscillopsia (visual blurring with head movements) -Disequilibrium -No nausea -Gait ataxia Otitis Media: -Infection of the MIDDLE EAR causing fever and ear pain Ototoxicity: -Ear poisoning caused by drugs (Aminoglycosides), chemicals

Difference between Cushing's Disease vs Cushing's Syndrome?

• CD --> Pituitary Adenoma - more ACTH (Adrenocorticotropic) secreted by PITUITARY GLAND, stimulates Adrenal gland --> more CORTISOL is released --> increased BP • CS --> Adrenal glands Tumor - Adrenal gland secretes more CORTISOL • Symptoms (common for both): Round " moon" face, excessive facial hair • Fat deposition --> cervical fat pad, truncal obesity; BUFFALO HUMP • Easy bruising and poor wound healing • Weight gain (children show poor growth in height) • Cortisol is a potent anti-inflammatory --> RA and OA Notice in BOTH CD and CS they have an INCREASE of CORTISOL... This means that they will present with same s/s as that increased CORTISOL. The big difference is WHY? CD —> Pituitary adenoma (tumor). CS —> Adrenal gland tumor That CD is the PITS —> Cushing's Disease is the Pituitary tumor. (So you remember CD starts with tumor of pituitary)

Rules for Exercise and Diabetes

• Diabetic patient- Exercise may result in hypoglycemia • Avoid exercise during peak insulin hours (most active 2-4 hours after taking insulin) (if you take insulin at 6PM avoid exercise from 8-10PM) • Insulin is absorbed much more quickly in an active extremity • Provide carbohydrate snack initially, have readily available during exercise. • Do not exercise in extreme cold or hot temperatures • Exercise in the morning is recommended to avoid hypoglycemia resulting from fluctuations in insulin sensitivity

Gait related impairments of PD

• Festinating gait: progressive increase in speed with a shortening of stride. Gait can be anteropulsive (a forward festinating gait) • Difficulty terminating/stopping gait on command Ex: pt can't stop on command when there's an end marker, takes shorter steps past the marker until pt slows down then stops • Freezing of gait (FOG)- sudden abrupt inability to initiate any movement. E.g. Doorways , turns • Difficulty turning: increased steps per turn • Reduced stride length; increased step-to-step variability • Reduced speed of walking • Increased time: double-limb support • Insufficient hip, knee, and ankle flexion: shuffling steps • Insufficient heel strike with increased forefoot loading • Reduced trunk rotation: decreased or absent arm swing • Difficulty with dual tasking: simultaneous motor and/or cognitive tasks • Difficulty with attentional demands of complex environments Good intervention for PD pt experiencing FOG is metronome: using a beat/music to help snap out of the freezing

Legg Perthes' Disease Prognosis (X-ray Image on slide 22 on FF#26 is a dog)

-Age of onset = 2-13 years -Flattening of Femoral Head -Prognosis = better if pt is young "Someone takes a femur bone and CHEWS ON THE FEMORAL HEAD (flattening of femoral head)

Safety and Precautions: Lifting Guidelines

-Always attempt to increase your BOS -Maintain a proper lumbar curve while you lift -Pivot your feet while lifting; do no twist your back -Maintain a slow and consistent speed while lifting

What is a heart murmur? When is systolic murmur present? When is diastolic murmur present?

-abnormal heart sound commonly resulting from valvular disorders due to changes in blood flow around and through altered valve -systolic murmur: between S1 and S2 -diastolic murmur: between S2 and S1

Which of the following examination findings MOST likely indicates a favorable prognosis for a patient who has a posterior lumbar disc herniation? 1. Centralization and ability to restore extension mobility 2. Proximal symptoms and ability to restore flexion mobility 3. Peripheralization with flexion and a positive straight leg raise result 4. Leg pain with extension and a negative crossed straight leg raise result

1. Centralization and ability to restore extension mobility 1. Individuals who have centralization of symptoms and have the ability to restore extension of the spine demonstrate a good prognosis. 2. Proximal symptoms are suggestive of a good prognosis; however, flexion mobility is not a predictor of a good prognosis for patients who have a disc herniation. 3. Neither of these options have been found to have a predictive value in the prognosis of patients who have a disc herniation. 4. Although a negative result on the crossed straight leg raise is favorable, leg pain reproduced with extension is predictive of a poor prognosis.

A patient with no history of trauma has nonradiating low back pain. Lumbar flexion does not reverse the lordosis and is pain-free; lumbar extension increases the symptom. Palpation reveals a step-off in the lower lumbar region. The MOST appropriate treatment for this patient would be: 1. abdominal strengthening. 2. sustained prone positioning on elbows. 3. exaggerated lumbar lordosis in sitting. 4. grade III posteroanterior glide to L5.

1. abdominal strengthening. 1. This patient's signs and symptoms are consistent with spondylolisthesis. Abdominal muscle strengthening and stabilization are key to conservative management. 2. Extension activities are not indicated for a patient with spondylolisthesis. 3. Extension activities are not indicated for a patient with spondylolisthesis. 4. Extension activities are not indicated for a patient with spondylolisthesis.

Following a partial-thickness burn over the entire anterior hip and anterior knee regions, the patient should be positioned with the hip in: 1. 0° of extension and the knee in 0° of extension. 2. 0° of extension and the knee in 20° of flexion. 3. 20° of flexion and the knee in 0° of extension. 4. 20° of flexion and the knee in 20° of flexion.

2. 0° of extension and the knee in 20° of flexion. 1. Burned areas should be positioned to elongate the involved tissues or neutral position of function. The knee should be flexed to prevent an extension contracture. 2. The hip should be neutral to prevent a flexion contracture. The knee should be flexed to prevent an extension contracture. 3. The hip should be neutral to prevent a flexion contracture. The knee should be flexed to prevent an extension contracture. 4. The hip should be neutral to prevent a flexion contracture.

Which of the following conditions would cause a patient to have the GREATEST risk for contracting an infection? 1. Thrombocytopenia 2. Leukopenia 3. Thrombocytosis 4. Leukocytosis

2. Leukopenia 1. Thrombocytopenia occurs when platelets are less than 150,000/mm3. This would not put the patient at risk for infection but it would for spontaneous bleeding (Goodman, Pathology, p. 758; Goodman, Differential Diagnosis, p. 218). 2. In leukopenia, the total number of leukocytes is less than normal, and the individual is at greater risk for infection (Goodman, Pathology, p. 719; Goodman, Differential Diagnosis, pp. 217-218). 3. Thrombocytosis occurs when there is an increase in platelets. Symptoms include increased tendency to clot, splenomegaly, and easy bruising. (Goodman, Differential Diagnosis, p. 218) 4. Leukocytosis is an abnormally high number of leukocytes that generally develops in response to acute infection (Goodman, Pathology, pp. 718-719; Goodman, Differential Diagnosis, p. 217).

Which of the following activities is the HIGHEST level functional activity that a 15-month-old child with L1 myelomeningocele should be expected to achieve independently within 6 months? 1. Rolling 2. Sitting 3. Standing 4. Walking

2. Sitting 1. The child should be able to roll independently but also should be able to sit without assistance. Sitting is considered a higher functional task than rolling. 2. With a lesion below the L1 nerve root level, innervation is intact to all the abdominal muscles and to the iliopsoas, so the child should be able to sit up independently. 3. The child would have innervation to the hip flexors, but would lack innervation to the other lower extremity muscles needed for standing independently. 4. The child would have innervation to the hip flexors, but would lack innervation to the other lower extremity muscles needed for walking independently.

A 15 year-old gymnast walks into the Physical Therapy clinic complaining of constant low back pain. Patient presents with a hyperlordotic curve, retracted bilateral shoulder blades and displaced plumb line. Post examination, the Physical Therapist documents several observations regarding pelvic dysfunction, and pathological standing posture. The patient's pelvis is MOST likely positioned in which of the following? Select one: 1. Nutation with a posterior pelvic tilt 2. Counternutation with posterior pelvic tilt 3. Counternutation with an anterior pelvic tilt 4. Nutation with and anterior pelvic tilt

3. Counternutation with an anterior pelvic tilt Answer: Counternutation relative to the anterior rotation of the ilium ON the sacrum or backward motion of the base of the sacrum out of the pelvis. Water bowl analogy: -when you tip/drink a bowl of water towards you, the bowl is doing a posterior pelvic tilt and your head is going in to nutation -when you tip the bowl of water away from you, the bowl is doing an anterior pelvic tilt and your head is going in to counternutation

A 37-year-old male is currently being treated for a burn that occurred proximal to the elbow joint. That patient has been experiencing moderate edema with few scars. The burn is described as the dermis layer being inflamed, brisk capillary refill, and blisters that are intact, as well as painful. The Physical Therapist should suspect which of the following classifications for the burn described? 1. Subdermal 2. Deep Partial thickness 3. Superficial partial thickness 4. Full thickness

3. Superficial partial thickness Answer: The keywords in the question that describes this type of burn include: moderate edema with very few scaring, inflamed dermis, capillary refill, blisters intact but very painful and sensitive to temperature changes. This describes a superficial partial thickness burn. A deep partial thickness burn has moderate to severe edema with significant scarring present, capillary refill is slow, blisters are broken and the patient will be sensitive to pressure changes. A full thickness burn is described as white, charred, tan with no blanching present. A subdermal burn is charred with subcutaneous tissue evident as well as muscle damage with neurological involvement.

A patient has a burn with marked edema, broken blisters, and waxy discoloration. Which of the following classifications BEST describes this wound? 1. Superficial 2. Full-thickness 3. Superficial partial-thickness 4. Deep partial-thickness

4. Deep partial-thickness 1. A superficial (epidural) burn appears red or erythematous. Slight edema may be present, but blisters will be absent. (p. 1053) 2. A full-thickness burn is characterized by a hard, parchment-like eschar covering the area. The color can be white, charred, tan, mahogany, black, or red. (p. 1055) 3. The most common sign of a superficial partial-thickness burn is the presence of intact blisters. The wound will be bright pink, red, or mottled red. (pp. 1053-1054) 4. A deep partial-thickness burn appears as a mixed red or waxy white color. The surface is wet from broken blisters, and marked edema is the hallmark sign of this burn depth. (p. 1054)

Which of the following nerves are MOST active during the emptying phase of micturition? 1. Sympathetic nerves from the hypogastric plexus 2. Parasympathetic nerves from the hypogastric plexus 3. Sympathetic nerves from the pelvic plexus 4. Parasympathetic nerves from the pelvic plexus

4. Parasympathetic nerves from the pelvic plexus 1. Sympathetic nerves from the hypogastric plexus primarily control blood flow to the bladder, not stimulate bladder contraction. 2. Micturition contractions are stimulated from the pelvic plexus, not the hypogastric plexus. 3. Parasympathetic, not sympathetic, nerves stimulate bladder contractions. 4. Parasympathetic nerves from the pelvic plexus fire in order to contract the bladder and begin emptying.

Which of the following activities would be MOST appropriate to practice to assist a 20-year-old patient who has Duchenne muscular dystrophy in maintaining independence? 1. Stair training using both handrails 2. Gait training using a rolling walker 3. Transfer training using a slide board 4. Power wheelchair training over various surfaces

4. Power wheelchair training over various surfaces 1. The cessation of independent walking in boys with Duchenne muscular dystrophy typically occurs by age 10-12 (Palisano, pp. 250-251) or 13 years (Tecklin, p. 355). Therefore, stair climbing is not likely. 2. The cessation of independent walking in boys with Duchenne muscular dystrophy typically occurs by age 10-12 (Palisano, pp. 250-251) or 13 years (Tecklin, p. 355). Therefore, gait training is highly unlikely to be plausible. 3. The transition to adulthood marks a time of continued progressive disability. Assistance with transfers would be required at this stage. (Palisano, p. 254) 4. A 20-year-old patient with Duchenne muscular dystrophy is likely to require a power wheelchair for functional mobility due to the progressive nature of the disease (Tecklin, p. 362). Typically, by age 14 years, boys who have Duchenne muscular dystrophy are not ambulatory and require power-assisted mobility. The transition to adulthood marks a time of continued progressive disability with a greater reliance on assistive technologies such as a power wheelchair (Palisano, p. 254).

A patient who is an avid runner had insidious onset of anterior leg pain that is localized, sharp, shooting in nature, and especially aggravated with heel strike (initial contact) during walking and running. The pain first occurred after an increase in running speed and hardness of the training surface 2 weeks ago. Which of the following tests and measures is MOST appropriate to determine the cause of the pain? 1. Compartmental pressure testing 2. Joint range of motion and strength 3. Biomechanical gait assessment 4. Vibration and percussion testing

4. Vibration and percussion testing 1. Exertional compartment syndrome presents with subjective symptoms, including exertional leg pain (not the pin-point tenderness as presented in this case), decreased sensation after exertion, and paresthesias (Magee, p. 856). 2. Although these measures should be included in the examination, joint range of motion is usually maintained in persons with a stress fracture and would not be of utmost importance to include in the examination (Magee, p. 856). 3. Although gait assessment may be included as an examination measure, testing for conditions affecting the bones would be inherent at this point, because the patient is having pain with walking. The patient's gait may be altered due to pain at heel strike (initial contact), and an assessment at this point would not reflect the patient's normal gait. (Magee, p. 856) 4. Most individuals with stress fractures report an insidious onset of pain that correlates with a change in equipment or training and is exacerbated by the offending activity. They will have localized bony tenderness and palpable periosteal thickening, especially if they have long-standing symptoms. Some persons have pain at the fracture site with percussion or vibration at a distance from the fracture. Joint range of motion is usually maintained. Persons with a tibial stress fracture will have pin-point tenderness on the tibia and have pain with three-point stress. (Magee, p. 856) If a stress fracture is suspected, a tuning fork may be used at the suspected fracture site to provoke symptoms (Goodman, p. 1031).

A PT notices that a patient is experiencing early toe off during terminal stance in gait. Which of the following identifies a likely cause, AND an appropriate intervention to address that cause? A. Hip flexion contracture, prolonged stretch B. Hip adductor weakness, progressive strengthening C. Gastrocnemius weakness, ultrasound D. Great toe flexion weakness, progressive strengthening

A. Hip flexion contracture, prolonged stretch Hip flexors are tight causing limited hip ext causing early toe off PF contracture would cause early toe off during midstance

A patient's severe knee sprain resulted from medial rotation of the femur on the tibia with simultaneous application of a valgus force while the foot was placed on the ground. Which structures are MOST likely to be involved? A. Medial collateral ligament, medial meniscus, anterior cruciate ligament B. Medial collateral ligament, lateral collateral ligament, posterior cruciate ligament C. Lateral collateral ligament, medial meniscus, anterior cruciate ligament D. Medial collateral ligament, lateral meniscus, posterior cruciate ligament

A. Medial collateral ligament, medial meniscus, anterior cruciate ligament

A 45 year old male patient underwent a surgical procedure on the right knee and has been prescribed crutches. PT is training the patient on the use of crutches. Which of the following is the BEST hand placement for the PT: A. One hand of the PT on the gait belt and the other hand on the patient's shoulder. B. Both hands of the PT on the gait belt. C. One hand of the patient on the gait belt and the other hand of patient on the PT's shoulder. D. Both hands of the PT on the patient's hips.

A. One hand of the PT on the gait belt and the other hand on the patient's shoulder.

A 55 year old male is undergoing cardiac rehabilitation. The PT is educating the patient on symptoms of exertional intolerance. Which of the following symptoms are the signs for exertional intolerance? A. Persistent dyspnea and angina B. Weight loss and leg stiffness C. Numbness in legs and dizziness D. Insomnia and confusion

A. Persistent dyspnea and angina

A 39-year-old comes to an outpatient clinic complaining of hip problems. The physical therapist notices a drop of the left hip during right midstance. The MOST appropriate treatment for this impairment would be: A. Stand on right leg and abduct the left leg B. Stand on left leg and abduct the right leg C. Stand on right leg and flex the right leg D. Stand on left leg and flex the right leg

A. Stand on right leg and abduct the left leg Why not B? closed chain exercise better than open chain exercise and applies more functionally

A PT developed a new test to detect to diagnose a rotator cuff tear. The test-retest correlation was found to be 0.92, the intrarater reliability was 0.91 and interrater reliability was at 0.66. Which of the following statements regarding this test is MOST APPROPRIATE? A. The test results are not as reliable when performed by different therapists and are more reliable when performed by the same therapist B. The test results are reliable when performed by different therapists but are not as reliable when performed by the same therapist C. The test results are not reliable when performed by different therapists or when performed by the same therapist D. The test results are reliable when performed by different therapists and when performed by the same therapist

A. The test results are not as reliable when performed by different therapists and are more reliable when performed by the same therapist

A PT is treating a child in an outpatient clinic. PT notices that the child is unable to do prone on elbows with elbow in line with shoulder. What age would be considered normal for a child to be able to do prone on elbows with elbow in line with shoulder? A. 1 month B. 3 months C. 5 months D. 6 months

B. 3 months

PT is treating a 6-year-old child with autism in outpatient pediatric physical therapy. Which of the following is the MOST appropriate intervention? A. Tumbling on mat to promote core strength. B. Ball toss, hopping, and running with a partner with verbal cues for turn taking. C. Perform activities in a fast-paced environment with long explanations for sequencing. D. Performing standing dance routine in different rooms to improve generalizability of a skill.

B. Ball toss, hopping, and running with a partner with verbal cues for turn taking.

A patient presents with partial and full thickness burns on the chest regions. The PT decides to apply TENS before debridement to modulate pain. Which TENS mode should provide the BEST relief? A. Acupuncture like (low rate) TENS. B. Brief intense TENS. C. Noxious TENS. D. Conventional (high rate) TENS.

B. Brief intense TENS. BRIef tens—> deBRIdement Rationale : Answer: B Brief intense TENS is used to provide rapid-onset, short term relief during painful procedures. The pulse rate and pulse duration are similar to conventional TENS; however the current intensity is increase to the patient's tolerance.

A 45-year-old female presents to PT with a recent diagnosis of MS. Upon initial evaluation, she complains of eye pain that is worsened by eye movement, vision loss in one eye, visual field loss. Which of the following cranial nerves listed below is most likely involved? A. Facial (VII) B. Optic (II) C. Abducens (VI) D. Oculomotor (III)

B. Optic (II) Vision loss and visual field loss involves CN2. With MS CN2 is primarily affected

A PT is treating a patient with complains of chest pain. The PT attempts to assess heart sounds with a stethoscope. Which of the following is true about the first sound during auscultation of the heart A. The first sound is of the closure of the aortic and pulmonic valves B. The first sound is of the closure of the mitral and tricuspid valves C. The first sound is of the opening of the aortic and pulmonic valves D. The first sound is of the opening of the mitral and tricuspid valves

B. The first sound is of the closure of the mitral and tricuspid valves (lub) A is S2 (dub) S1 and S2 are normal S3 and S4 are abnormal

A PT student suffered low back pain due to excessive studying during NPTE week and is using hot packs in the PT clinic. When using hot packs, which of the following scenarios is MOST likely to result in burns? A. Using 6 layers of toweling between the hot pack and the patient B. A treatment time of 30 minutes C. A hot pack heated by immersion in water heated to 205 degrees Fahrenheit D. The patient's skin having too many hair

C. A hot pack heated by immersion in water heated to 205

A 35-year-old male presents with a skin condition as shown in the picture below. There are irregular areas of localized skin edema. Which of the following is the MOST likely diagnosis for this patient? (look at slide 16 FF #14) Difference between each? A. Blisters B. Vesicles C. Wheals D. Pustules

C. Wheals -occurs from cold sensitivity -raised, red patches of skin Blisters = filled with serious fluid and larger than vesicles (has something underneath) Vesicles = filled with fluid as well but is smaller than blisters Wheals = not filled with any fluid (hives) Pustules = filled with puss Bullae = larger than blisters (0.5 diameter)

List the different colors of sputum and what they indicate

Clear: normal Yellow: cold Green: bacterial infection Pink frothy: pulmonary edema due to heart failure Red: bleeding Brown: blood or dirt accumulated Black: fungal infection, smoking

Stages of Motor Learning: Cognitive Associative Automatic

Cognitive: -what to do stage -extrinsic feedback > than intrinsic -knowledge of results 1st then knowledge of performance -feedback after every trial "Cognitive Stage - What do I do? "Trial and Error" - stage with several mistakes and lack of coordination. Pt needs visual cues Demonstrating the task is most effective Cognitive demands are high and high frustration can occur" Associative: -how to do stage -intrinsic feedback -feedback: summed, bandwidth, fading -variable practice "Associative Stage - How do I do it? Known as the "practice and adjustment phase" Pt needs to feel it Kinesthetic learning (proprioception), intrinsic and extrinsic feedback" Automatic: -how to succeed "Autonomous Stage - How do I master it? Known as the "fine tuning phase" Pt is able to self-correct and movements are habitual with little cognitive efforts"

Scoliosis

Concave Side -High pelvis (raised) -Tight muscles (shortened) -Spinous Process Rotation -Anterior Rib Distortion -A decrease in vertebral height -Narrowing of the intercostal spaces -Decrease in lung volume Convex Side -Low pelvis -Lengthened muscles -Vertebral body Rotation -Increased vertebral height

All of the following are components of treatment for lymphedema EXCEPT: A. Decongest the trunk quadrants first before addressing the lymphedematous extremity B. Exercise should always be performed with a compression bandage C. Decongesting the proximal portions of the limb first and then working distally with the direction of flow always towards the trunk D. Affected limbs are bandaged with high -stretch compression bandages

D. Affected limbs are bandaged with high -stretch compression bandages

A patient diagnosed with Parkinson's Disease exhibits a forward stooped posture. As part of the patient's care plan the PT selects a number of active exercises that promote improved posture. Which PNF pattern would be the MOST appropriate to achieve the PT's objective? A. D1 Extension B. D1 Flexion C. D2 Extension D. D2 Flexion

D. D2 Flexion We want to open up the chest Theyre in a d2 ext position so we want to do the opposite

A PT is examining a 46-year-old patient who underwent a radical mastectomy with axillary node removal. Which of the following signs would indicate INITIAL development of lymphedema in the patient? A. Non-pitting edema of lower leg and foot B. Shallow wound beds on the forearm C. Atrophy of the biceps muscle D. Decreased flexibility of the fingers

D. Decreased flexibility of the fingers keywords is INITIAL, lymphedema doesn't happen overnight and decreased flexibility of the fingers are an initial symptom Keyword is INITIAL: Option A) Too hard as non-pitting Option B) Shallow wounds is more insufficiency and takes time. Option C) Not initial

A male patient with BMI of 38 kg/m2 presented to an outpatient clinic wit excessive fatigue. The PT observes Ichthyosis on the lower extremities as shown in the picture. What would be the MOST likely cause in this patient? (look at image on slide 10 FF#18) A. Addison's disease B. Graves disease C. Cushing's syndrome D. Hashimoto's disease

D. Hashimoto's disease key words: BMI - obese >30 fatigue Addison's disease = hypo-adrenal Graves disease = hyperthyroidism Cushing's syndrome = hyper-cortisol Hashimoto's = hypothyroidism **Symptoms of hypothyroidism - tends to occur in middle aged women * MOM'S SO TIRED * • Memory loss • Obesity • Malar flush/ Menorrhagia • Slowness (mentally and physically) • Skin and hair dryness • Onset gradual • Tiredness • Intolerance to cold • Raised BP • Energy levels fall • Depression/Delayed relaxation of reflexe

To conduct an study on cervical pain in adults, PT randomly assigns patients to two groups. One group is treated with cervical manipulation and cryotherapy; the second receives only cryotherapy. In this experimental design, cervical manipulation is: A. Continuous variable. B. Dependent variable. C. Discrete variable. D. Independent variable.

D. Independent variable. I for an I Independent variable: -whenever a PT does something its independent (stretching, manual treatment) -whenever a pt has symptoms its dependent variable (pt has pain, limited ROM) Continuous: -any fraction, decimals Discrete -whole numbers eg: numbers of students in class

Lymphedema occurs due to disruption of the normal circulation of lymphatic fluid in the body. All of the following are causes of secondary lymphedema EXCEPT: A. Infection by Wuchereria Bancroffti B. Kaposi's sarcoma C. Traumatic injuries D. Milroy's disease

D. Milroy's disease Milroy's disease (MD) is a familial disease characterized by lymphedema, commonly in the legs, caused by congenital abnormalities in the lymphatic system. Disruption of the normal drainage of lymph leads to fluid accumulation and hypertrophy of soft tissues. Wurchereria Banfcroffti causes filariasis Kaposi's sarcoma (KS) is a type of cancer that can form masses in the skin, lymph nodes, or other organs. The skin lesions are usually purple in color. They can occur singly or in a limited area, or may be widespread. It may worsen either gradually or quickly. Lesions may be flat or raised.

A 50 year old male patient is diagnosed with a second degree heart block type II. What would a physical therapist expect to find on the ECG strip? A. An increase in PR interval lengths with no dropped beats B. No relationship between P waves and QRS complexes C. A gradual increase in PR interval length in all the beats preceding a dropped beat D. Normal PR intervals in all the beats preceding a dropped beat

D. Normal PR intervals in all the beats preceding a dropped beat (2nd degree type 2) A is 1st degree B is 3rd degree C is 2nd degree type 1 SUMMARY TO REMEMBER HEART BLOCKS: SO think of your heart blocks in terms of covid situation: You (P) and your friend (R) go on regular walks First degree- is like when covid hit- you socially distanced- Increase in PR interval Second degree type 1- Cases started going up so you increased social distancing while walking until your friend stopped coming- Progressive increase in PR interval until drop Second degree type 2- cases are going down but still want to maintain normal distance and randomly your friend doesn't show up- Normal PR interval until sudden drop Third degree- due to social distancing you and your friend are used to walking at your own pace- P and R have no relation

Individuals with impaired vestibular system function are most likely to experience a loss of balance on the CTSIB test, when information from the following system/systems is/are altered during testing. A. Spinal Reticular System B. Visual System C. Somatosensory System D. Somatosensory and visual system

D. Somatosensory and visual system

Distinguish between Erb's vs Klumpke's Palsy

Erb's -C5/6 -MOI stretched downward -Loss of shoulder ABD and ER -Policeman's tip deformity Klumpke's -C8/T1 -MOI stretching of arm overhead -Paralysis of the intrinsic of the hand -Claw hand deformity Er6'5 = Erb's

Differentiate between hypomobility/disc displacement with reduction/synovitis/capsulitis in TMJ disorder

Hypomobility -decreased opening -no pain Disc displacement with reduction -clicking sound present Synovitis -decreased opening -no deviation -pain present Capsulitis -decreased opening -deviation present -pain present

How to differentiate between Posterior/Anterior/Horizontal: Canalithiasis/Cupulolithiasis?

Posterior Canalithiasis -(+) DHP with up beating -Nystagmus <1 min -initial side of rotation with nystagmus Anterior Canalithiasis -(+) DHP with down beating -Nystagmus <1 min -initial side of rotation with nystagmus Horizontal Canalithiasis -(+) Supine Roll Test -toward the ground (Geotropic nystagmus) -side affected = more intense nystagmus Posterior Cupulolithiasis -(+) DHP with up beating -Nystagmus >1 min -initial side of rotation with nystagmus Anterior Cupulolithiasis -(+) DHP with down beating -Nystagmus >1 min -initial side of rotation with nystagmus Horizontal Cupulolithiasis -(+) Supine Roll Test -toward the sky/away from ground (Ageotropic nystagmus) -side affected = less intense nystagmus

What are the functions associated with temporal lobe lesion?

Primary auditory cortex Wernicke's aphasia Auditory loss Language comprehension

What is the UE Thrust pattern?

Pt starts in: -UE flexed - shoulder ADD -forearm SUPINATED -hands FLEXED -Have pt punch/thrust forward and out to promote elbow ext "start in taekwondo punch position to superman flying position"

Common Types of MS

Relapse-Remitting MS -Unpredictable attacks which may or may not leave permanent deficits followed by periods of remission -short duration -may remain symptom free for months or years Primary-Progressive MS -Steady increase in disability without attacks -Steady worsening from the start -Do not have periodic relapses and remissions Secondary-Progressive MS -Initial relapsing-remitting MS that suddenly begins to have decline without periods of remission -slow, steady progression - with or without relapses Progressive-Relapsing MS -Steady decline since onset with super-imposed attacks -Steadily worsen from onset -Flare-ups with or without remissions are also present "Roger and Pete played tennis in SPain to gain some PR"

Wheel Chair Propulsion: Turning Sharp Turns Wheelie

Turning -Pushing harder with one hand than other Sharp turns -Pull one side wheel backward and other side forward Wheelie -Pt palces hands back on hand rims, then pulls them forward abruptly and forcefully

Distinguish between BPPV/Acute UVH/Acoustic neuroma/Meniere's disease

UVH (unilateral vestibular hypofunction) -symptoms are resting (spontaneous) nystagmus/oscillopsia (feel that the room is spinning)/dysequilibrium/postural instability/head tilt to one side BPPV -inner ear issue -symptoms are nystagmus/vertigo with change in head position/occasional nausea with or without vomiting/dysequilibrium Meniere's disease -recurrent and usually progressive vestibular disease -symptoms are tinnitus/deafness/vertigo/sensation of fullness in the ear

Types of incontinence?

Urge Incontinence -involuntary contraction of detrusor muscle with a strong desire to void (urgency) and loss of urine as soon as the urge is felt -caused by meds/alcohol/bladder infections/bladder tumor/neurogenic bladder/bladder outlet obstruction Stress Incontinence -Any type of light pressure on the bladder that causes urine to leak -Sneeze/cough/laugh/exercising puts pressure on the bladder Mixed Incontinence -Combination of urge and stress incontinence -Due to weak pelvic floor or other health concerns Functional Incontinence -Any physical or mental impairments keeps you from making it to a bathroom in time -not associated with health issues related to urinary system Overflow Incontience -When your bladder doesn't fully empty, you experience frequent or constant dribbling of urine -If you can't completely empty your bladder, it remains full and the excess overflows and exits the body via the urethra -Often experience UTI due to urine remaining in the bladder and bacteria grows -Common in men with enlarged prostates

WC Measurements Seat Width Seat Depth Seat Height

Width -Add 2 inches to widest measure of hip Depth/Deep -Subtract 2 inches from posterior butt to popliteal fossa -You don't want to sink too deep in the chair so subtract 2 inches Height -20 inches for adults -17.5 inches for hemiplegic -18.75 inches for children "Seat WIDth: ADD 2 inches —> WIDe ADDs to the siding ; Seat DEPth: SUBTRACT 2 inches —> As you go into the ocean DEPTHS, it's SUB level."

Nearly 2 months ago, a patient noticed left shoulder pain after walking the dog. This pain has progressively worsened. The patient now is unable to move the left upper extremity overhead while performing ADLs. An orthopedic surgeon diagnosed the problem as adhesive capsulitis. The MOST effective direction for glenohumeral mobilization for this patient would be: a. Posterosuperior translatory glides b. Posteroinferior translatory glides c. Anterosuperior translatory glides d. Anteroinferior translatory glides

b. Posteroinferior translatory glides Capsular pattern of restriction is present in adhesive capsulitis i.e. lateral rotation> abduction> medial rotation. Clinical studies have demonstrated that posterior glide is more effective than an anterior glide to increase glenohumeral external rotation range of motion (exception to the convex- concave rule) in adhesive capsulitis. Patient's external rotation can be treated with posterior glide and inferior glide are used to resolve abduction. So, posterior inferior glides should be given in this patient.

A PT in treating a patient with a diagnosis of lower cervical level spinal cord injury. While the PT was performing passive hip stretching, the PT noticed that the patient had started sweating profusely and had goosebumps. On checking the vitals, the patient's HR had dropped and BP was higher than normal. Which of the following is the MOST APPROPRIATE intervention for this patient? Select one: a. Bring the patient to sitting position and clamp the catheter b. Keep the patient in the supine position and start CPR c. Bring the patient to a sitting position and remove the abdominal binder d. Stop the treatment and let the patient rest for at least 2 hours before starting the treatment again.

c. Bring the patient to a sitting position and remove the abdominal binder Rationale: Patients with a T6 or higher level of injury are at a risk of Autonomic Dysreflexia (AD) in the presence of a noxious stimuli. The symptoms of AD include profuse sweating, increased spasticity, hypertension, bradycardia, headache, piloerection, blurred vision, etc. The onset of these symptoms should be treated as a medical emergency and in order to lower the BP, the patient must be brought to a sitting position, noxious stimuli, tight clothing and abdominal binder must be removed, and the catheter must be unclamped.

A physical therapy student is studying about falls and objective measures to assess balance and mobility. Which of the following scores represents a higher risk of falls in the elderly? Select one: a. Score of 18 seconds on Timed Up and Go Test b. Score of 25 on Functional Gait Assessment c. Score of 18 on Performance-Oriented Mobility Assessment d. Score of 49 on Berg Balance scale

c. Score of 18 on Performance-Oriented Mobility Assessment Rationale: Performance-Oriented Mobility Assessment: 25-28 = low fall risk 19-24 = medium fall risk < 19 = high fall risk "PUMA's marketing attracts 19 y.o kids" DGI <19 = fall risk Timed Up and Go Test: Most adults can complete the test in <10 seconds. 11-20 seconds is considered within normal limits for all frail elderly individuals. Scores > 30 seconds are indicative of impaired functional mobility. "Old Stephen Curry did bad on the TUGO" Berg's Balance Scale: 41-56 = low fall risk 21-40 = medium fall risk 0 -20 = high fall risk Score of < 45 indicates a greater risk of falling Functional Gait Assessment: Scores of <22/30 are effective in predicting falls in community-dwelling older adults. "I'm going to test Andrew Wiggins on the FGA"

Joe presents to the clinic with a large burn on his forearm. When the therapist tries to touch it, he does not complain of pain on pressure or light touch. Which of the following is the MOST LIKELY presentation of this wound? Select one: a. Mixed red, waxy white appearance with blanching and slow capillary refill b. Erythematous pink with no blisters c. Erythematous with blanching and quick capillary refill d. White or tan with no blanching

d. White or tan with no blanching Rationale: Full thickness burns are white or tan and are usually anesthetic with poor circulation. A: Deep partial thickness burns have broken blisters and are sensitive to pressure but insensitive to light touch and blanch with slow capillary refill. B: Epidermal burns are erythematous with no blisters. C: Superficial partial thickness burns are erythematous and blanch with quick capillary refill.

Head/neck and Trunk Chop What PNF patterns is your R and L UE going in to when chopping wood from Right to Left?

-Always chop AWAY from your body -Lead arm is the Weak arm -Assist arm is the Strong arm When you chop from Right to Left: -Lead arm is the L UE -Assist arm is the R Ue -Lead arm starts from from D1 flex in to D1 ext -Assist arm starts from top of the opposite wrist into D2 ext

TMJ Arthrokinematics?

-TMJ formed by the mandibular condyle (convex) articulating with the TM disc and glenoid fossa of the temporal bone Arthrokinematics -Mandibular depression: the condyle BOTH rolls and slides anteriorly on the TM disc while the disc also slides anterior to maintain a congruent surface with the fossa -Functional ROM of mouth opening = 40mm -TM ligament allows up to 20-25mm of mandibular opening to occur before it becomes tight, after that, the condyle translates anteriorly to allow further opening (15mm) -Mandibular Protrusion: Occurs when both TMJs slide anteriorly -Lateral Excursion: involves the ipsilateral TMJ spinning in place with the contralateral TMJ sliding anterior

Which of the following describes the use of a physical restraint for an older adult living in a long term care facility that can be used by a physical therapist without a physician's order? 1. A wheelchair tray used for proper positioning of the upper trunk that can be removed by the patient 2. A vest restraint while in bed for a patient who may wander away from the residence 3. For a patient who self propels with the lower extremities, elevating the wheelchair seat so the patient's feet do not reach the ground 4. Removing a patient's ambulatory device to control disruptive behavior

1. A wheelchair tray used for proper positioning of the upper trunk that can be removed by the patient 1. A wheelchair tray is a physical support that the patient is able to remove when it is not in use. It does not require a physician order because it is being used for positioning. 2. Use of a vest restraint is not consistent with patients' rights under Omnibus Reconciliation Act (OBRA) because it restricts a patient's mobility and requires a physician order. 3. Elevating the wheelchair seat is not consistent with patients' rights under Omnibus Reconciliation Act (OBRA) because it restricts a patient's mobility and requires a physician order. 4. Removing an ambulatory device is not consistent with patients' rights under Omnibus Reconciliation Act (OBRA) because it restricts a patient's mobility.

The condition shown for the patient's left hand in the photograph is MOST likely caused by entrapment of which of the following nerves? (Image is of Kiloh-Nevin Syndrome: can't flex hallucis IP and index DIP) 1. Anterior interosseous nerve 2. Radial nerve 3. Posterior interosseous nerve 4. Ulnar nerve

1. Anterior interosseous nerve 1. The image shows an anterior interosseous syndrome (Kiloh-Nevin syndrome) in the patient's left hand. The patient is unable to flex the distal phalanx of the thumb and index fingers (1st and 2nd digits) because the anterior interosseous nerve, which supplies the flexor pollicis longus and the radial half of the flexor digitorum profundus, is entrapped. (p. 411) 2. With entrapment of the radial nerve, all extensor muscles of the forearm would be affected (p. 416). The image shows a deficit in the pinch of the thumb and index finger (1st and 2nd digits). 3. Entrapment of the posterior interosseous nerve results in functional wrist drop (p. 416). The image shows a deficit in the pinch of the thumb and index finger (1st and 2nd digits). 4. When the ulnar nerve is affected, the patient cannot fully adduct the little finger (5th digit) and hold the finger abducted and extended (p. 415). The image shows a deficit in the pinch of the thumb and index finger (1st and 2nd digits).

A patient who sustained a traumatic brain injury and is unable to follow commands has been referred for physical therapy evaluation. When the physical therapist arrives at bedside, the patient is agitated. Which of the following actions should the therapist take INITIALLY? 1. Carefully observe the patient's spontaneous behavior. 2. Postpone the assessment until the patient has become calm. 3. Apply soft restraints to calm the patient before assessment. 4. Proceed with the assessment regardless of the patient's agitated state.

1. Carefully observe the patient's spontaneous behavior. 1. Confused-agitated is a state common in patients following traumatic brain injury. Observing the patient without touching the patient will reveal information that is important for the evaluation. (p. 829) 2. The agitated state may exist for some time, and the assessment should not be postponed (p. 829). 3. The patient needs to feel safe. The therapist should model calm behavior. Restraining the patient would produce more fear and agitation. (p. 838) 4. Formal measurements of range of motion and strength are difficult, and the patient is unable to cooperate. The therapist should use only observation and estimate functional abilities. (p. 829)

A patient reports of a band like aching pain in his right upper abdomen and right upper back, which increases after eating a heavy fatty meal. He has a history of peptic ulcers and takes OTC antacids, which have helped in alleviating his pain. Upon palpation under the ribs, the patient stops breathing and winces due to pain. What do you suspect to be the cause of this new presentation? Select one: 1. Cholecystitis 2. Gastritis 3. Nephrolithiasis 4. Acute pancreatitis

1. Cholecystitis Answer: Cholecystitis refers to an inflammation of the gallbladder. Symptoms include right upper abdominal pain, which can refer to right upper back, nausea, vomiting, and occasionally fever. Murphy's sign is seen to be positive in this case, as with palpation pain was elicited.

A patient with myasthenia gravis has been transferred from intensive care to an acute care neurology unit after a myasthenic crisis. Which of the following physical therapy interventions is MOST appropriate for the patient? 1. Deep breathing exercises 2. Instruction in skin inspection 3. Neuromuscular electrical stimulation 4. Treadmill training with progressive incline

1. Deep breathing exercises 1. In the acute care setting, deep breathing and coughing should be encouraged (p. 1698). 2. Patients with myasthenia gravis have intact sensation and do not typically have problems with skin breakdown. The cardinal features are muscle weakness with fatigue. (p. 1697) 3. Because the fundamental defect in myasthenia gravis is failure of neural transmission at the neuromuscular junction, electrical stimulation would not be appropriate (p. 1696). 4. A patient who has myasthenia gravis should avoid strenuous exercise. Frequent rest periods help conserve energy and give muscles a chance to regain strength. (p. 1698)

After evaluating a patient who is a baseball player, a physical therapist concludes that there is insufficient deceleration of the shoulder during throwing. Which of the following strengthening techniques will be MOST effective in improving control of deceleration of the shoulder? 1. Eccentric exercises of the lateral (external) rotators 2. Eccentric exercises of the medial (internal) rotators 3. Concentric exercises of the lateral (external) rotators 4. Concentric exercises of the medial (internal) rotators

1. Eccentric exercises of the lateral (external) rotators 1. Eccentric training is consistent with the eccentric demand of deceleration, and exercise of the posterior shoulder muscles is needed to improve this function. The eccentric contraction of the rotator cuff lateral (external) rotators decelerates the rapid medial (internal) rotation of the shoulder. 2. Although eccentric training is necessary, the medial (internal) rotators do not function in this manner. 3. Concentric exercise is not consistent with functional demand. 4. Concentric exercise is not consistent with functional demand, and the medial (internal) rotators do not require attention in this scenario.

Which of the following activities should be the PRIMARY emphasis of a physical therapy treatment program for a child who has athetoid cerebral palsy? 1. Facilitating cocontraction patterns and encouraging control in voluntary movement gradation 2. Increasing muscle strength using progressive resistive exercises 3. Facilitating use of primitive reflexes to perform gross motor tasks 4. Preventing development of contractures and ensuring full voluntary range of motion

1. Facilitating cocontraction patterns and encouraging control in voluntary movement gradation 1. Athetoid cerebral palsy is characterized by involuntary movements that are slow and writhing. In therapy, the emphasis should be on facilitating cocontraction and encouraging control in voluntary movement. 2. Although strength training is indicated in children with cerebral palsy, a child with athetoid cerebral palsy lacks the control to consistently produce a maximal effort in a controlled movement. Therefore, the focus must first be on gaining control, then on traditional strength training. 3. A goal of therapy would not be to reinforce primitive reflexes. The primary need for this child is to gain controlled movement. 4. Children with athetoid cerebral palsy are less likely to have contractures, compared with children with other forms of cerebral palsy, because of the constant movement of limbs. This would not be the primary emphasis of therapy.

An inability to perform the exercise shown in the photograph is MOST likely to be associated with difficulty performing which of the following shoulder movements? (Pt is performing a plankl) 1. Flexion 2. Extension 3. Horizontal abduction 4. Medial (internal) rotation

1. Flexion 1. The photograph shows strengthening for the serratus anterior (Kisner). It is difficult to raise the arm above the head without the action of the serratus anterior muscle (Lippert). 2. The photograph shows strengthening of the serratus anterior (Kisner). It is difficult to raise the arm above the head without the action of the serratus anterior muscle (Lippert), but extension should not be impacted. 3. The photograph shows strengthening of the serratus anterior (Kisner). It is difficult to raise the arm above the head without the action of the serratus anterior muscle (Lippert), but horizontal abduction should not be impacted. 4. The photograph shows strengthening of the serratus anterior (Kisner). It is difficult to raise the arm above the head without the action of the serratus anterior muscle (Lippert), but medial (internal) rotation should not be impacted.

A patient with an incomplete C8 spinal cord injury reports of burning in the buttocks when sitting in a wheelchair. Which of the following problems with the patient's wheelchair would be MOST likely to contribute to the burning? 1. Footplates are too high. 2. Footplates are too low. 3. Seat is too narrow. 4. Seat is too deep.

1. Footplates are too high. 1. Footrests that are too high shift the patient's weight posteriorly onto the buttocks, leading to pressure and pain (p. 1402). 2. Footrests that are too low shift weight forward onto the thighs. Although this would cause problems due to pressure on the thighs, it would not lead to pain in the buttocks. (p. 1402) 3. A narrow seat will lead to pressure and pain on the lateral pelvis and thighs, not in the buttocks (p. 1399). 4. A long seat will lead to posterior pelvic tilt and kyphotic posture to compensate for the extra depth (p. 1399).

The joint mobilization shown in the photograph will have the GREATEST effect on improving which of the following phases of gait? (Image of anterior glide of tibia) 1. Heel off (terminal stance) 2. Foot flat (loading response) 3. Toe off (preswing) 4. Midswing

1. Heel off (terminal stance) 1. The therapist is performing an anterior glide of the tibia on the femur, which would increase knee extension (Kisner). During heel off (terminal stance), a knee extension of 0° is necessary for normal function (Magee, p. 992). 2. The knee angle in the foot flat (loading response) phase required for normal gait is 15°. The photograph shows an angle greater than 15°. The joint mobilization shown in the photograph helps to restore knee extension. This would not improve the foot flat (loading response) phase of gait. (Magee, p. 992) 3. The knee angle in toe off (preswing) is 40° of flexion. The joint mobilization shown in the photograph helps to improve knee extension, which would not improve toe off (preswing). (Magee, p. 992) 4. No gait deviation is caused by lack of extension during the swing phase. The knee achieves 25° of flexion in this phase. (Magee, p. 992)

Which of the following chronic hormonal responses to exercise may contribute to menstrual dysfunction? 1. High levels of cortisol, low levels of follicle-stimulating hormone (FSH) 2. High levels of cortisol, high levels of follicle-stimulating hormone (FSH) 3. Low levels of cortisol, low levels of follicle-stimulating hormone (FSH) 4. Low levels of cortisol, high levels of follicle-stimulating hormone (FSH)

1. High levels of cortisol, low levels of follicle-stimulating hormone (FSH) 1. It has been proposed that exercise stress chronically elevates cortisol levels in athletes. This in turn suppresses the secretion of gonadotropin-releasing hormone (GnRH), which in turn results in the suppression of luteinizing hormone (LH) and follicle-stimulating hormone (FSH). Low levels of FSH will result in menstrual dysfunction. 2. Although it has been proposed that exercise stress chronically elevates cortisol levels in athletes, this rise in levels of cortisol suppresses, not stimulates, the secretion of gonadotropin-releasing hormone (GnRH), which in turn results in the suppression of luteinizing hormone (LH) and follicle-stimulating hormone (FSH) rather than stimulation of the secretion of these hormones. 3. It has been proposed that exercise stress chronically elevates cortisol levels in athletes, although lower levels of follicle-stimulating hormone (FSH) will result in menstrual dysfunction. 4. It has been proposed that exercise stress chronically elevates cortisol levels as well as lowers follicle-stimulating hormone levels in athletes.

A patient who has diabetes is exercising and has onset of a headache, blurred vision, and slurred speech. Which of the following conditions is the patient MOST likely experiencing? 1. Hypoglycemia 2. Lipogenic effect of insulin 3. Diabetic ketoacidosis 4. Vitamin B12 deficiency

1. Hypoglycemia 1. Headache, blurred vision, and slurred speech represent central nervous system activity specifically due to decreased blood glucose to the brain (p. 534). 2. Lipogenic effects include thickening of the subcutaneous tissues and a loss of subcutaneous fat, resulting in dimpling of the skin (p. 522) 3. Although diabetic ketoacidosis would result in headaches, other signs of ketoacidosis include acetone breath, dehydration, weak and rapid pulse, and Kussmaul respirations progressing to hyperosmolar coma (polyuria, thirst, neurological abnormalities, and stupor) (p. 534). 4. Vitamin B12 deficiency would result in headaches and cognitive changes but would not be the most likely cause in this scenario. Neuropsychiatric conditions such as dementia, ataxia, psychosis, and peripheral neuropathy can develop in the case of B12 deficiency. (p. 715)

A patient with spinal stenosis is referred to physical therapy. The patient reports pain in the right knee, ankle, and foot. While walking, the patient exhibits a mild foot-drop with no evidence of hypertonicity. When weight bearing, the foot and ankle are pronated. Which of the following orthoses is MOST appropriate? 1. Molded ankle-foot orthosis (AFO) 2. AFO with a split stirrup 3. Custom insole with lateral arch supports 4. Metal AFO with the ankle set in 5° of dorsiflexion

1. Molded ankle-foot orthosis (AFO) 1. A custom-molded thermoplastic ankle-foot orthosis provides an intimate fit for control of the extremity, and will permit correction of the patient's foot and ankle position as well as the foot drop. This type of ankle-foot orthosis also is lighter weight and more cosmetic and comfortable to wear than a metal double-upright ankle-foot orthosis with dorsiflexion assist. (p. 1292) 2. Split stirrups are bulky and would not address the pronation malalignment (p. 1292). 3. A custom insole would not address the patient's foot drop. An ankle-foot orthosis is needed to position the foot and ankle to enhance toe clearance during gait. (pp. 1290-1291) 4. This type of ankle-foot orthosis is not the best choice for this patient because it is heavy, and a metal ankle-foot orthosis would not address the pronation malalignment (p. 1296).

A patient's electrocardiogram shows a junctional rhythm. The patient's heart rate is 60 bpm and regular. Which of the following waves will MOST likely be absent from the rhythm strip? 1. P 2. R 3. S 4. T

1. P 1. Junctional rhythm originates from the atrioventricular junction instead of the sinoatrial node, which normally causes the P wave. Therefore, the P wave will be missing. R, S, T waves come from the ventricles after stimulation from the atrioventricular junction and will be unaffected. 2. Junctional rhythm originates from the atrioventricular junction instead of the sinoatrial node, which normally causes the P wave. Therefore, the P wave will be missing. R waves come from the ventricles after stimulation from the atrioventricular junction and will be unaffected. 3. Junctional rhythm originates from the atrioventricular junction instead of the sinoatrial node, which normally causes the P wave. Therefore, the P wave will be missing. S waves come from the ventricles after stimulation from the atrioventricular junction and will be unaffected. 4. Junctional rhythm originates from the atrioventricular junction instead of the sinoatrial node, which normally causes the P wave. Therefore, the P wave will be missing. T waves come from the ventricles after stimulation from the atrioventricular junction and will be unaffected.

A 21 year old male has been receiving Physical Therapy for 3 weeks status post a left knee ACL reconstruction. At this point in time, a goal of the Physical Therapist would MOST likely include which of the following? Select one: 1. Preventing reflex inhibition of muscle 2. Achieving dynamic control of left knee 3. Improving kinesthetic awareness 4. Achieving full-pain free ROM

1. Preventing reflex inhibition of muscle Answer: Preventing reflex inhibition of muscles would be expected during this time frame. The Physical Therapist would want to prevent atrophy from developing in the quadriceps and hamstrings in order to maintain stabilization. During weeks 4-10, the Physical Therapist will primarily focus on improving the kinesthetic awareness during closed chain activities, full pain free ROM, and dynamic control of left knee

A patient who uses an insulin pump is beginning an outpatient aerobic conditioning session. The patient's current blood glucose level is 95 mg/dL (5.3 mmol/L). Which of the following responses is MOST appropriate for the patient? 1. Reduce the insulin infusion dose. 2. Increase the insulin infusion dose. 3. Continue insulin infusion at the current metered dose. 4. Rest for 1 hour, then remeasure blood glucose level.

1. Reduce the insulin infusion dose. 1. One benefit of using an insulin pump is being able to change the insulin delivery. Reducing or suspending the insulin will safely address the decreased blood glucose level (which is less than 100 mg/dL [5.5 mmol/L]) without the need for a carbohydrate snack (pp. 528, 618). 2. Increasing the insulin infusion dose would further decrease the blood glucose level. The patient's blood glucose is already less than normal. (p. 507) 3. Insulin pump delivery should be reduced, not maintained, in a patient whose current blood glucose level is 95 mg/dL (5.3 mmol/L) (p. 520). If the patient continued at the current rate, the insulin level would be too low (p. 507). 4. Rest would not address the problem, given the blood glucose level. Blood glucose less than 100 mg/dL (5.5 mmol/L) is abnormal and could be addressed by adjusting the insulin infusion dose. (p. 507)

Which of the following activities is LEAST effective for improving the gait of patients who have Stage 1 Parkinson disease? 1. Repetitive wall squats 2. Performance of t'ai chi 3. Walking outdoors on uneven ground 4. Dancing with rhythmic auditory cues

1. Repetitive wall squats 1. This activity is unlikely to affect the patient's gait pattern, although these exercises often are indicated for other impairments in Parkinson disease. These activities are static and not placed in context. Strength training is not the most important intervention in Stage 1 Parkinson disease. Postural and balance activities within functional activities are shown to have greater effect, especially in early stages. 2. Activities using visual and auditory stimuli in a context-specific setting have been shown to be effective for improving gait in individuals who have Parkinson disease. 3. Activities using visual and auditory stimuli in a context-specific setting have been shown to be effective for improving gait in individuals who have Parkinson disease. 4. Activities using visual and auditory stimuli in a context-specific setting have been shown to be effective for improving gait in individuals who have Parkinson disease.

Which of the following characteristics is MOST likely to be found in a superficial partial-thickness burn? 1. Shiny appearance 2. Exposed fat 3. Mild erythema without blisters 4. White color without blisters

1. Shiny appearance 1. A superficial partial-thickness burn would be wet, with a shiny and weeping surface, and mottled red in color. 2. A full-thickness burn would have fat exposed. A partial-thickness burn would have a broken epidermis. 3. Mild to severe erythema without blisters would characterize a superficial burn. 4. White color would characterize a full-thickness burn. Partial-thickness burns are mottled red in color.

An infant's family has just been referred to physical therapy for discharge instructions following surgical repair of an L5 myelomeningocele. Which of the following is MOST critical for the family to learn before leaving the hospital? 1. Signs of increased intracranial pressure 2. Passive range of motion exercises for the lower extremities 3. Signs of hip dislocation 4. Strengthening exercises for the lower extremities

1. Signs of increased intracranial pressure 1. Hydrocephalus occurs in 25% or more of children with myelomeningocele, and an additional 60% develop it after surgical closure of their back lesion. Between 80% and 90% of children with hydrocephalus will require a cerebrospinal fluid (CSF) shunt. Even if the child has a shunt, recognizing signs of increased intracranial pressure and shunt dysfunction is critical because these conditions can lead to additional functional and cognitive decline of the child. 2. Passive range of motion is important, but deficits in this area would not be life-threatening. 3. Hip dislocation is unlikely to occur in infancy and is not an immediate emergency as is increased intracranial pressure. 4. Strengthening within the child's ability level is important, but deficits in this area would not be life-threatening.

A patient sustained a T10 complete spinal cord injury. Which of the following wheelchair-to-bed transfer techniques is MOST appropriate for the patient? 1. Sit pivot 2. Sliding board 3. Standing pivot 4. Mechanical lift

1. Sit pivot 1. Patients who have an injury at the T10 level or below have partial to full innervation of trunk musculature and fully innervated upper extremities. They may initially use a sliding board to assist transfers, but in the long term they should be able to independently perform a sit pivot transfer. 2. Patients who have an injury at the T10 level or below have partial to full innervation of trunk musculature and fully innervated upper extremities. They should be able to independently transfer and not need a sliding board in the long term. 3. Patients who have an injury at the T10 level or below have partial to full innervation of trunk musculature and fully innervated upper extremities. They would not have adequate lower extremity strength to perform a standing pivot transfer. 4. Patients who have an injury at the T10 level or below have partial to full innervation of trunk musculature and fully innervated upper extremities. They should be able to independently transfer and not need a mechanical lift.

Which of the following exercise combinations is MOST appropriate for a patient who has a forward head posture? 1. Strengthen the deep cervical flexors and stretch the sternocleidomastoids and upper cervical extensors. 2. Strengthen the deep cervical flexors and sternocleidomastoids and stretch the upper cervical extensors. 3. Strengthen the cervical extensors and stretch the sternocleidomastoids and deep cervical flexors. 4. Strengthen the cervical extensors and sternocleidomastoids and stretch the deep cervical flexors.

1. Strengthen the deep cervical flexors and stretch the sternocleidomastoids and upper cervical extensors. 1. A forward head position is due to decreased cervical retraction (Kisner). Treatment should include deep cervical flexor and shoulder retractor strengthening and cervical extensor and pectoral muscle stretching (Dutton). 2. Treatment for forward head posture should include deep cervical flexor and shoulder retractor strengthening and cervical extensor and pectoral muscle stretching (Dutton). 3. The deep cervical flexors should be strengthened, not stretched (Dutton). 4. The deep cervical flexors should be strengthened, not stretched (Dutton).

A physical therapist is obtaining the medical history of a patient with amyotrophic lateral sclerosis. Which of the following is MOST important to ask about in order to determine the prognosis for this patient? 1. Swallowing difficulties 2. Cognitive deficits 3. Bowel and bladder function 4. Neck pain

1. Swallowing difficulties 1. Patients with an initial onset of bulbar and respiratory weakness tend to have a more rapid progression to death than patients whose weakness begins in the distal extremities. 2. Cognitive deficits are not associated with amyotrophic lateral sclerosis. 3. Sphincter control problems are not a component of amyotrophic lateral sclerosis. 4. Musculoskeletal pain is not predictive for prognosis in amyotrophic lateral sclerosis.

A patient is initiating a pelvic floor strengthening program. Which of the following positions would be considered gravity assisted? 1. Trendelenburg 2. Sitting 3. Quadruped 4. Standing

1. Trendelenburg 1. The Trendelenburg position, with the hips positioned higher than the heart, is considered a gravity-assisted position for this exercise, given the location of the pelvic floor muscles and the action of these muscles when tightened, which pull upward toward the abdomen. 2. The sitting position is a progression to improve strength and proprioception of the pelvic floor muscles. Gravity will be working against the pelvic floor. 3. The quadruped position is a progression to improve strength and proprioception of the pelvic floor muscles. 4. The standing position is a progression to improve strength and proprioception of the pelvic floor muscles. Also, this would be considered the gravity position.

A patient who is a waiter has hand pain when carrying trays on the shoulder. Which of the following nerve tension tests is MOST likely to have a positive result? 1. Ulnar 2. Median 3. Radial 4. Musculocutaneous

1. Ulnar 1. Carrying food trays on the shoulder is similar to the end position of the ulnar nerve tension test. The tension test for the ulnar nerve includes shoulder depression, abduction, and lateral (external) rotation; elbow flexion; forearm pronation or supination; and wrist and finger extension. (pp. 81, 83-84, 434) 2. The median nerve tension test employs elbow extension, but the position of the waiter is more consistent with elbow flexion (81, 433-434). 3. The radial nerve tension test employs elbow extension, and the position of the forearm is low by the side, not reaching overhead (79-80, 434). 4. The musculocutaneous nerve does not innervate the hand (76-77, 434).

Which of the following types of practice is MOST appropriate for long-term motor learning for a patient with a cerebrovascular accident? 1. Varied task practice with variable time intervals 2. Task practice of one activity for 15 minutes with 10-minute rest 3. Practice of a variety of related skills in blocks of 5 minutes 4. Partial task practice with patient-preferred time intervals

1. Varied task practice with variable time intervals 1. Random practice provides a higher level of contextual interference that requires the individual to retrieve practice from memory stores. Research has shown superior long-term effects for random practice due to the higher cognitive processes required. (Lazaro; O'Sullivan p. 373) 2. Massed practice is preferred for individuals with fatigue issues. Although learning occurs with this type of practice schedule, the depth of cognitive processes required is typically not as high as is expected with changes in tasks and environments. (O'Sullivan, p. 373) 3. Blocked practice is practice of one task performed repeatedly without interruption from other tasks. Although this practice allows for motor learning, it is not best for long-term retention due to the lack of variability, which is a hallmark of typical, daily movement. (Lazaro; O'Sullivan, p. 373) 4. Although complex motor skills can be broken into component parts for practice, delaying practice of the integrated tasks can interfere with the transfer effects and learning. Integrated practice is best for learning, particularly with continuous movement. The daily practice of the same task may improve performance, yet may not be best for long-term retention, as it does not allow the individual to retrieve from cognitive stores. (O'Sullivan, p. 374) Patient-preferred intervals will not be the best choice, considering that optimal learning occurs with random practice schedules.

A physical therapist is examining a 4-year-old child with a history of prematurity and developmental delay. To determine if the child has age-appropriate gross motor skills, the therapist's assessment should include: 1. kicking a rolling ball, catching a small ball, and hopping on one foot. 2. kicking a stationary ball, fast walking, and walking with assistance on stairs. 3. dribbling a basketball, riding a bicycle, and skipping. 4. catching a large ball, riding a tricycle, and running short distances.

1. kicking a rolling ball, catching a small ball, and hopping on one foot. 1. Kicking a rolling ball, catching a small ball, and hopping on one foot are gross motor tasks that are most age-appropriate for a 4-year-old. Gross motor developmental assessment at age 4 years should include functional tasks. (Palisano, p. 62; Tecklin, p. 64) 2. Kicking a stationary ball, fast walking, and walking with assistance on stairs are skills that are appropriate for children age 18 months to 3 years (Palisano, p. 62; Tecklin, p. 64). 3. Dribbling a basketball, riding a bicycle, and skipping are skills that are appropriate for children age 5-6 years (Palisano, p. 63; Tecklin, p. 64). 4. Catching a large ball, riding a tricycle, and running short distances are skills that are appropriate for children age 2-3 years (Palisano, p. 62; Tecklin, p. 64).

The BEST exercise level for an aerobic program for weight loss is: 1. less than or equal to 60% of maximum heart rate for 45 to 60 minutes, 5 to 7 days/week. 2. 80% to 90% of maximum heart rate for 45 to 60 minutes, 3 days/week. 3. less than or equal to 60% of maximum heart rate for 15 to 20 minutes, 3 days/week. 4. 80% to 90% of maximum heart rate for 15 to 20 minutes, 5 to 7 days/week.

1. less than or equal to 60% of maximum heart rate for 45 to 60 minutes, 5 to 7 days/week. 1. Initial exercise training for obese individuals should be moderate (40% to 60% of maximum heart rate) and eventually progress to higher intensities (50% to 75%). Frequency should be 5-7 days/week; duration should be 45-60 minutes/session. 2. Eighty percent to 90% of maximum heart rate is too high of an intensity; and 3 days a week is not a high enough frequency. 3. Fifteen to 20 minutes is not a long enough duration; and 3 days a week is not a high enough frequency. 4. Eighty percent to 90% of maximum heart rate is too high of an intensity; 15-20 minutes is not a long enough duration.

The INITIAL rehabilitation program for a patient post coronary artery bypass surgery would consist of: 1. low-intensity walking. 2. lifting light weights. 3. low-level upper extremity ergometry. 4. stationary bike riding.

1. low-intensity walking. 1. Walking in hall is included as an initial activity in a Phase I treatment program (p. 228). 2. Resistance training should not begin until a minimum of 8 weeks after coronary artery bypass surgery (p. 241). 3. Arm activity, such as use of ergometer, is not appropriate for patients immediately after coronary artery bypass surgery. The activity is contraindicated due to incisional precautions. (p. 241) 4. Stationary bike riding is a higher-level aerobic activity that should not be the initial choice after coronary artery bypass surgery. Limitation or restriction of upper body activities, including unloading the upper extremities, is an important element in the initial rehabilitation program for a patient post coronary artery bypass graft. A patient who may lean on the handlebars would potentially place too much pressure on their sternal incision. (p. 241)

A patient with a hypertonic piriformis will also MOST likely have: 1. sacroiliac joint dysfunction. 2. limited hip extension. 3. anterior thigh paresthesia. 4. gluteus maximus weakness.

1. sacroiliac joint dysfunction. 1. Tightness or spasm of the piriformis may have a significant influence on the sacroiliac joint, causing dysfunction (Dutton, p. 89). 2. The piriformis can function as a hip lateral (external) rotator and abductor of a flexed hip. It is unlikely, therefore, to restrict hip extension. (Dutton, p. 1421) 3. The piriformis muscle derives its innervation from the S1-S2 nerve roots, which innervate the posterior thigh (Drake, pp. 451, 547). The anterior thigh is innervated by the femoral nerve (L2-L4), which arises from a different spinal segment. Therefore, hypertonicity and shortness of the piriformis would produce pain or paresthesias in the posterior thigh, not the anterior thigh. 4. One of the six classic findings in piriformis syndrome is gluteal atrophy. This can occur depending on the duration of the condition (Dutton, p. 89); however, it is not the most likely event, compared to sacroiliac dysfunction.

Effective physical therapy intervention for an infant with torticollis should include teaching the caregiver to: 1. supervise prone positioning with the infant's head turned to the same side as the affected muscle. 2. perform cervical range of motion while the infant is sleeping. 3. provide brief, passive stretch to the affected muscles. 4. apply electrical stimulation on the uninvolved side.

1. supervise prone positioning with the infant's head turned to the same side as the affected muscle. 1. Sleeping in prone with tight muscles (sternocleidomastoid) being stretched provides long duration stretch that is optimal for treatment of torticollis (Palisano, p. 197). 2. Range of motion alone will not achieve muscle stretching necessary for treatment of torticollis (Palisano, p. 197). 3. Passive stretch of long duration is needed to effectively stretch muscles involved in torticollis (Palisano, pp. 197, 199-200). 4. Electrical stimulation is contraindicated in the carotid sinus region and is a precaution for the very young (Cameron).

A physical therapist is examining the posterior aspect of the heels of a patient who has darkly pigmented skin. Which of the following findings would indicate the presence of a Stage 1 pressure injury? 1. An intact area of the skin that is black and leathery 2. An intact area of the skin that is warm and purple 3. A shallow crater with a moist wound bed 4. An intact blister with a boggy feel

2. An intact area of the skin that is warm and purple 1. This presentation would indicate the presence of eschar. A pressure injury with eschar cannot be staged (p. 135). 2. This presentation fits the National Pressure Ulcer Advisory Panel's definition of a Stage 1 pressure injury. Warmth and color change indicate pressure damage. Damaged skin may look purple rather than red in people with darkly pigmented skin. Intact skin indicates that the pressure injury is not deeper than Stage 1. (p. 112) 3. A shallow crater with a moist wound bed is characteristic of a Stage 2 pressure injury (p. 135). 4. An intact blister with a boggy feel is characteristic of a suspected deep tissue injury (p. 135).

The device shown in the photograph would be MOST beneficial for a patient who has which of the following conditions? (image of spirometer) 1. Osteoarthritis 2. Ankylosing spondylitis 3. Reiter syndrome 4. Psoriatic arthritis

2. Ankylosing spondylitis 1. Osteoarthritis is a degenerative disease that mainly affects the hands, hips, and knees, causing disability. Manifestations of the disease do not compromise ventilation, and, therefore, use of an incentive spirometer would not be beneficial. (Goodman, p. 1304) 2. Ankylosing spondylitis is a chronic inflammatory disease that leads to restrictive lung dysfunction which causes all lung volumes and capacities to be decreased (Hillegass, pp. 127-128, 165-166). The device in the photograph is an incentive spirometer, which encourages deep breathing and extended inspiration, leading to an increase in ventilatory capacity and chest expansion (Hillegass, pp. 552-554) 3. Reiter syndrome is a form of reactive arthritis that can follow an infection by a microbial pathogen. Symptoms include urethritis, conjunctivitis, and arthritis. There is no compromise to ventilation indicated, and, therefore, use of an incentive spirometer would not be beneficial. (Goodman, pp. 1343-1344) 4. Psoriatic arthritis occurs in 20% of patients who have psoriasis and leads to joint disorders due to the development of inflammatory synovitis. The disease is most likely to affect the distal interphalangeal joints of the hands. Symptoms do not compromise ventilation; therefore, use of an incentive spirometer would not be beneficial. (Goodman, pp. 1341-1342)

Which of the following associated conditions places children with Down syndrome at GREATER risk for frequent respiratory infections? 1. Cervical instability 2. Chest muscle hypotonicity 3. Immune system dysfunction 4. Arrested alveolar development

2. Chest muscle hypotonicity 1. Although cervical instability is associated with Down syndrome, it is more likely to contribute to neurological problems than to respiratory infections (Magee, p. 1017). Structural abnormalities of the airways and lungs and cough deficits related to hypotonia are more likely to be related to increased risk for respiratory infections. 2. Respiratory tract infections are very common secondary to hypotonicity of the chest and abdominal muscles and contribute significantly to morbidity and mortality. Decreased muscle tone compromises respiratory expansion. (Goodman, p. 1163) Cardiopulmonary manifestations include poor ability to clear secretions from both upper and lower airways, resulting in greater susceptibility to pulmonary infections, and postural abnormalities that may lead to restrictive lung dysfunction in older children. 3. Immune system dysfunction in children with Down syndrome is associated with a higher incidence of acute myeloid leukemia than in the general pediatric population (Goodman, p. 1163). This option is incorrect since it is not the most common or likely cause of increased respiratory infections. 4. Arrested alveolar development occurs when the lungs are injured during development and is characteristic of children with bronchopulmonary dysplasia. It is the most common chronic lung disease associated with prematurity (Tecklin, p. 146) but is not associated with Down syndrome.

A 65-year-old patient arrives for outpatient physical therapy. The patient reports swimming earlier that morning. Since then the patient has felt pain down both arms, has had shortness of breath, and has continued to perspire. What should a physical therapist do NEXT? 1. Refer the patient to a physician for magnetic resonance imaging of the cervical spine. 2. Contact emergency medical services. 3. Evaluate the cervical spine and provide stretches to decrease bilateral arm pain. 4. Treat the patient and recommend that the patient follow up with a physician within 1 week.

2. Contact emergency medical services. 1. There is not enough information to support a suspicion of disc injury in the cervical spine that would warrant magnetic resonance imaging. 2. The patient shows signs of having had a myocardial infarction. A myocardial infarction is more apt to occur in the morning, during exertion, and when one is working with the arms overhead (e.g., swimming). The patient demonstrates all of these warning signs, and emergency medical services should be contacted immediately. 3. Radiating upper extremity pain, shortness of breath, and diaphoresis are signs of myocardial infarction. Evaluating the cervical spine and providing upper extremity stretches is inappropriate and would delay emergent medical testing. 4. Based on the reported symptoms and the warning signs for a myocardial infarction, action should be taken immediately.

Which of the following tools is MOST appropriate for assessing progress in an 8-year-old child who has cerebral palsy with spastic quadriplegia and limited mobility? 1. Bayley III 2. Gross Motor Function Measure (GMFM) 3. Peabody Developmental Motor Scales 4. Bruininks-Oseretsky Test of Motor Proficiency

2. Gross Motor Function Measure (GMFM) 1. The Bayley III is a revised version of the Bayley Scales of Infant Development, which is norm referenced for children from birth to 42 months old (Palisano, p. 532; Tecklin, pp. 82, 84-85). 2. The Gross Motor Function Measure was constructed to evaluate progress over time in children with cerebral palsy. The test was developed to evaluate change in motor status and proficiency due to therapeutic intervention. The test has been validated for sensitivity to change in children with cerebral palsy age 5 months to 16 years (Palisano, pp. 15-16, 90; Tecklin, pp. 77-79) 3. The Peabody Developmental Motor Scales is appropriate for children from birth to 72 months of age (Palisano, p. 219; Tecklin, pp. 79-81). 4. The Bruininks-Oseretsky Test of Motor Proficiency is a test of motor function for children 4 to 21 years of age. The test is primarily constructed to assess motor coordination and balance (Tecklin, pp. 81-83). This test will be too advanced for this patient.

A patient receives neuromuscular electrical stimulation to the quadriceps femoris in supine position with 30° of knee flexion. No contraction is elicited, even though the patient reports a strong tingling sensation under the electrodes. Which of the following alterations is MOST appropriate in order to elicit a contraction? 1. Increase in current amplitude 2. Increase in pulse duration 3. Decrease in electrode size 4. Decrease in distance between electrodes

2. Increase in pulse duration 1. Amplitude is the magnitude of current or voltage (pp. 223-224). The patient already reports a strong tingling sensation, and increasing amplitude would increase the sensation. 2. Pulse duration is the time from the beginning of the first phase of a pulse to the end of the last phase of a pulse (p. 223). Most patients find longer pulse durations more comfortable when larger muscles are being stimulated (p. 247). Longer pulses are needed to depolarize motor nerves (p. 228). 3. Current density is the amount of current delivered per unit area (p. 234). Current density is inversely proportional to the size of the electrode; larger electrodes are more comfortable than smaller ones (p. 233). Decreasing electrode size would increase current density. Increasing current density would increase the strong tingling sensation the patient is currently feeling. 4. The distance or spacing between electrodes affects the depth and course of the current. The closer together electrodes are configured, the more superficially the current travels, and conversely, the greater the distance between them, the deeper the current travels (p. 233). The quadriceps femoris is a long muscle; therefore, increasing electrode spacing would be more appropriate.

Based on the findings of the assessment during inspiration shown in the photographs, the patient MOST likely has restriction involving which of the following lung segments? (image shows R lobe is higher than L lobe after inspiration) 1. Left lingula 2. Left lower lobe 3. Right lower lobe 4. Right upper lobe

2. Left lower lobe 1. The image shows the technique for palpating the lower lobes. To assess the right middle lobe and lingular segment, the therapist places his/her widely outstretched fingers of both hands over the posterior axillary folds and his/her palms over the anterior chest wall. 2. The image clearly shows the technique for palpating the lower lobes. The therapist's thumbs and hands do not move the same distance from each other, indicating restriction on the left side. With unilateral restriction, there is evidence of diminished movement on only one side. 3. The image clearly shows the technique for palpating the lower lobes. The therapist's thumbs and hands do not move the same distance from each other, indicating restriction on the left side and not the right side. With unilateral restriction, there is evidence of diminished movement on only one side. 4. The image clearly shows the technique for palpating the lower lobes. To assess the upper lobes, the therapist places his/her palms anteriorly over the first four ribs with the fingertips extended over the trapezius muscle.

A patient reports fatigue, proximal upper extremity weakness, and double vision that increases in intensity as the day progresses. The patient demonstrates bilateral ptosis of the eyelids, difficulty chewing, dysphagia, and inability to raise the eyebrows. Which of the following conditions is MOST likely present? 1. Bell palsy 2. Myasthenia gravis 3. Trigeminal neuralgia 4. Amyotrophic lateral sclerosis

2. Myasthenia gravis 1. Bell palsy would not result in upper extremity weakness, double vision, nor would it affect bilateral eyelids (Lundy-Ekman, pp. 402-403) 2. The loss of function described in the scenario involves multiple cranial nerves. The fact that it increases as the day progresses implies fatigue that is typical of myasthenia gravis. (Goodman, pp. 1696-1698) 3. Trigeminal neuralgia is a dysfunction of the trigeminal nerve (CN V) that produces sharp, severe, stabbing pain in the distribution of one or more branches of the trigeminal nerve (CN V). It does not cause ptosis, dysphagia, and fatigue, which are described in the stem. (Lundy-Ekman, pp. 61-62). 4. While amyotrophic lateral sclerosis can present with bilateral upper extremity weakness and dysfunction of cranial nerves affecting chewing and swallowing, it is not likely to cause double vision, ptosis, nor difficulty raising the eyebrows (Lundy-Ekman, p. 286).

Which of the following findings are associated with the LOWEST risk for a subsequent cardiac event? 1. Left ventricular ejection fraction of 55% and functional capacity of 3 metabolic equivalents (METs) 2. Occasional premature ventricular contractions and functional capacity of 6 metabolic equivalents (METs) 3. Exercise-induced ST segment depression of less than 2 mm and sustained supraventricular tachycardia 4. Exercise-induced ST segment depression of greater than 2 mm and left ventricular ejection fraction of 45%

2. Occasional premature ventricular contractions and functional capacity of 6 metabolic equivalents (METs) 1. An ejection fraction of 55% to 75% is considered normal (O'Sullivan, p. 473). A patient is at moderate risk for increased morbidity and mortality if functional capacity is less than 5-6 metabolic equivalents (Ehrman, p. 62). 2. It is common to have a few premature ventricular contractions in a normal heart (O'Sullivan, p. 496). A patient is at low risk for increased morbidity and mortality if functional capacity is greater than or equal to 6 metabolic equivalents (Ehrman, p. 62). 3. ST segment depression during activity is considered diagnostic of myocardial ischemia (Ehrman, p. 62). Supraventricular tachycardia (SVT) is an electrical conduction abnormality (O'Sullivan, pp. 495-496). ST segment depression during activity is considered diagnostic of myocardial ischemia (Ehrman, p. 62). Furthermore, supraventricular tachycardia (SVT) is an arrhythmia indicative of compromised cardiac function and is an indication to stop physical activity (Ehrman, p. 73) 4. An ejection fraction of 45% is below normal. Normal ejection fraction is between 55% and 75% (O'Sullivan, p. 473). Exercise-induced ST segment depression greater than 2 mm indicates a high risk for a future cardiac event (Ehrman, p. 62).

A patient who has chronic obstructive pulmonary disease becomes short of breath when walking 5 feet (1.5 m) with a rolling walker. Which of the following techniques would be MOST appropriate in order to increase the distance the patient is able to walk without becoming short of breath? 1. Incentive spirometry 2. Pacing 3. Diaphragmatic breathing 4. Segmental breathing

2. Pacing 1. Incentive spirometry uses visual feedback to encourage the performance of sustained maximal inspiration. Incentive spirometry has been used to reduce postoperative pulmonary complications following abdominal or thoracic surgery, but its effectiveness is controversial. (Malone) 2. Using pacing, the patient would learn to work within his or her exercise tolerance. This may mean walking slower or walking with breaks and would allow greater total walking distance without shortness of breath. (O'Sullivan) 3. Diaphragmatic breathing is used to decrease the work of breathing and improve diaphragmatic movement, not to improve exercise tolerance (Hillegass). 4. Segmental breathing is used for patients with chest hypomobility to augment localized lung expansion (Malone). Chest hypomobility is not usually an issue in patients who have chronic obstructive pulmonary disease.

When providing patient education in cardiac rehabilitation, which of the following signs and symptoms of exertional intolerance should the physical therapist emphasize? 1. Anginal pain, insomnia, sudden weight gain, leg stiffness 2. Persistent dyspnea, dizziness, anginal pain, sudden weight gain 3. Persistent dyspnea, anginal pain, insomnia, weight loss 4. Anginal pain, confusion, leg numbness, weight loss

2. Persistent dyspnea, dizziness, anginal pain, sudden weight gain 1. Leg stiffness is not a sign/symptom associated with exercise intolerance among patients undergoing cardiac rehabilitation. 2. The signs and symptoms listed in this option are associated with exercise intolerance among patients undergoing cardiac rehabilitation. 3. Weight loss is not a sign associated with exercise intolerance among patients undergoing cardiac rehabilitation. However, angina and dyspnea are important signs of exercise intolerance. 4. Leg numbness, confusion, and weight loss are not associated with exercise intolerance. However, angina is important to note in the patient who has cardiac dysfunction.

A patient who has pleural effusion is performing segmental breathing exercises. Where should manual counterpressure be applied to encourage expansion of the posterior basal segments of the patient's lower lobes? 1. Lower lateral costal area 2. Posterior lower ribs 3. Anterior midchest 4. Anterior lower ribs

2. Posterior lower ribs 1. Segmental breathing combines breathing control with manual cues to specific areas of the chest wall. The lateral costal area would be a position to treat the lateral basal segments of the lower lobes, not the posterior basal segments. 2. Segmental breathing combines breathing control with manual cues to specific areas of the chest wall. Proper hand placement to encourage posterior basal expansion is over the posterior aspect of the lower ribs. 3. Segmental breathing combines breathing control with manual cues to specific areas of the chest wall. Midchest would be used to treat the anterior segments of the upper lobes, not the posterior basal segments. 4. Segmental breathing combines breathing control with manual cues to specific areas of the chest wall. The anterior lower ribs would be a position to treat the anterior basal segments of the lower lobes, not the posterior basal segments.

A patient reports that they were lifting heavy weights in the gym a few days ago and have new lower abdominal pain. They have noticed that the pain is worse when lying on the left side. The pain subsides when awakening, yet will gradually increase throughout the day. What type of pain should the Physical Therapist determine is occurring? Select one: 1. Gallbladder pain 2. Pseudorenal pain 3. Liver pain 4. Common bile duct pain

2. Pseudorenal pain Answer: With liver pain the patient will report right upper quadrant pain and pain in the liver during exercise. The pain will be constant and not change upon waking or throughout the day. Pseudorenal pain onset is usually acute and associated with a traumatic history such as lifting a heavy object. The patient will report having lower abdominal pain that is worse at night, especially when lying on the affected side. The pain is usually absent when arising but will get progressively worse throughout the day. With common bile duct pain, the patient will report pain in the mid-epigastric with vague discomfort. The pain is constant, mild at first and increases steadily. With gallbladder pain the patient will report right upper quadrant pain and possibly right shoulder pain. They will describe dull aching pain and a sense of fullness in their abdomen or epigastric.

A research team is using information from medical records to compare outcomes between a group of patients who received a 2-month intervention and a group who did not. Which of the following study designs is being used by the researchers? 1. Prospective cohort 2. Retrospective cohort 3. Cross-sectional 4. Case control

2. Retrospective cohort 1. A prospective cohort would require the investigators to follow the patients until the outcome. Because the information was collected from medical records, and the outcome has already occurred, the stem does not describe a prospective cohort study. (p. 287) 2. The patients in this study have already received the exposure (intervention) and have already experienced the outcome. Therefore, this is a retrospective study. (p. 288) 3. A cross-sectional study is used to assess exposure and outcomes at a single point in time. This study abstracted data over a period of time and, therefore, cannot be a cross-sectional study. (p. 280) 4. A case control study classifies people based on whether they had an outcome of interest and then looks retrospectively at different exposures. Because the sample in this study is compared on the basis of exposure (intervention), this study cannot be a case control study. (pp. 282-283)

Which of the following conditions is MOST likely to be associated with the presence of spastic bowel dysfunction in a patient? 1. Myelomeningocele at S2-S3 2. Spinal cord injury at T10 3. Spinal muscular atrophy 4. Cauda equina tumor

2. Spinal cord injury at T10 1. Spina bifida (myelomeningocele) is a disorder of the spinal cord (Lazaro). Patients who have an injury below S2-S4 would have a flaccid bowel, not a spastic bowel (O'Sullivan, p. 865). 2. Spastic bowel occurs in patients who have a spinal cord lesion above S2 (O'Sullivan, p. 865). The T10 level is above S2. 3. Spastic bowel occurs in patients who have a spinal cord lesion above S2 (O'Sullivan, p. 865). Spinal muscular atrophy affects motor neurons and is characterized by limb and trunk weakness (Palisano) 4. Spastic bowel occurs in patients who have a spinal cord lesion above S2 (O'Sullivan, p. 865). Individuals who have cauda equina injuries exhibit areflexic bowel dysfunction (O'Sullivan, p. 860).

During a phone call with a physical therapist regarding a patient referral, the physician who has referred the patient expresses a belief that the patient is fabricating symptoms for secondary gain and asks the therapist to provide documentation to demonstrate this impression. Which of the following responses is BEST for the therapist to provide in this situation? 1. Deciding whether a patient is fabricating symptoms for secondary gain is beyond the scope of physical therapy practice. 2. The physical therapy evaluation will include tests that will specifically assess for nonorganic signs and behavioral symptoms. 3. The patient will be confronted on issues of fabricating symptoms for secondary gain in an attempt to alter the behavior. 4. It is recommended that the patient be referred to a psychologist prior to initiating physical therapy.

2. The physical therapy evaluation will include tests that will specifically assess for nonorganic signs and behavioral symptoms. 1. The therapist can and should act as part of the medical team, which includes determining sincerity of patients to the extent possible and documenting a finding of malingering to assist in appropriate care as needed. To ignore this responsibility would be unfair to the insurance company, the malingering patient (e.g., in case a psychiatric referral is needed), and other patients who could have used the treatment resources used by the malingering patient. 2. All patients should be given the benefit of the doubt until the clinician, with a high degree of confidence, can rule out an organic cause for the pain. A number of tests can be performed to help determine if the symptoms are of a nonorganic nature, but it is very difficult to judge the intent (psychogenic versus intentional exaggeration for gain). 3. The therapist should not initiate patient care with preconceived ideas that are potentially harmful to the patient's success. Expressing such assumptions to the patient will bias the feedback the patient provides because the patient will be defensive, further skewing the results of the evaluative process. 4. As a member of a health care team, a physical therapist can play a role in documentation of malingering, once behavior that is characteristic of malingering is observed. The therapist should not recommend referral to a psychologist prior to performing the initial evaluation.

The patient who has the wound shown in the photograph has received conservative care for 2 months without improvement. To promote wound healing, which of the following treatments is MOST appropriate to add at this time? (image of small diabetic foot ulcer) 1. Antibiotic treatment 2. Total contact casting 3. Daily dressing changes 4. Sharp debridement of the callus

2. Total contact casting 1. Antibiotic treatment is generally not indicated for uninfected wounds. While the foot wound in the photograph is not progressing, there are not signs of infection present. Off-loading pressure areas is important to facilitate wound healing. 2. Total contact casting is the accepted standard for off-loading pressure areas on the plantar surface of the foot. 3. Daily dressing changes may make the total contact casting impractical. Daily dressing changes may prevent a patient from taking advantage of the increased healing rates associated with total contact casting. 4. While periodic callus removal is important, the foot in the photograph does not demonstrate significant callus formation. Long-term pressure relief from total contact casting may help prevent callus formation.

A physical therapist plans to use autolytic debridement to treat a patient's shallow, dry, partly necrotic leg wound. Which of the following dressings is MOST appropriate for this wound? 1. Hydrofiber dressing 2. Transparent film 3. Enzymatic agent 4. Foam dressing

2. Transparent film 1. Hydrofiber dressings are alginate-like in appearance and are activated by moisture in the wound. They are used for wounds with heavy exudate. The wound described is dry. (p. 386) 2. A dry wound will be best autolytically debrided by using a transparent film dressing (p. 380). 3. Topical enzymes are not used in autolytic debridement (p. 356). 4. A foam dressing is best used for a moist, draining wound (p. 384).

During initial examination of a patient who has rotator cuff tendinitis, a physical therapist notices that, during arm elevation, the involved scapula upwardly rotates early, compared to the uninvolved scapula. Passive range of motion of the involved shoulder is normal. Based on the findings, which of the following pairs of muscles BEST stabilizes the scapula to promote a balanced force couple on the involved side? 1. Rhomboids and upper trapezius 2. Upper trapezius and serratus anterior 3. Latissimus dorsi and lower trapezius 4. Latissimus dorsi and serratus anterior

2. Upper trapezius and serratus anterior 1. The upper trapezius is a prime mover in scapular elevation and upward rotation. The rhomboids are prime movers in shoulder retraction and elevation as well as scapular downward rotation. These two muscles are not the best pair to promote a balanced force couple because they are pulling in different directions to accomplish different motions. (p. 136) 2. The upper trapezius muscle pulls up and the lower fibers of the serratus anterior pull outward in a horizontal direction. The net effect is that together the two muscles pull in different directions to accomplish the same motion, which is the scapula rotating upward. (pp. 135, 137-139) 3. The latissimus dorsi does not affect the movement of the scapula significantly. It works to extend, adduct, medially (internally) rotate, and hyperextend the shoulder. The lower trapezius depresses and upwardly rotates the scapula. This answer is incorrect because the two muscles together do not produce a balanced force couple. (pp. 135, 152) 4. Although the serratus anterior is involved in stabilization of upward rotation, the latissimus dorsi does not affect the movement of the scapula significantly (pp. 137, 152).

A patient has significant shortening of tissues around a joint after immobilization. Patient instruction in which of the following interventions would BEST produce rapid, lasting changes in tissue length? 1. Stretching exercises of short duration performed multiple times throughout the day 2. Use of a dynamic splint worn multiple hours throughout the day 3. Stretching exercises of ballistic quality performed multiple times throughout the day 4. Multiple repetitions of range of motion exercises performed to end range

2. Use of a dynamic splint worn multiple hours throughout the day 1. Stretching exercises of short duration would produce changes over time through remodeling, but these changes would take much longer to occur. Creep, on the other hand, would produce lasting changes over a shorter period of time. (pp. 90-91) 2. This option uses the concept of creep. Creep is permanent deformation of tissue through the application of a low magnitude load over a long period of time. The use of a dynamic splint will allow the application of such a load and induce rapid changes over an extended period with minimal tissue damage and inflammation (pp. 90-91). 3. Ballistic stretches are not recommended for individuals after immobilization because they are thought to create significant trauma to atrophied tissues (p. 99). 4. Range of motion exercises are not thought to increase tissue length because they are not sustained long enough to induce lasting changes. They may be used to maintain tissue length but generally do not provide enough stress by themselves to increase tissue length. (p. 62)

During scoliosis screening, the forward bend evaluation is conducted to determine the presence of which of the following clinical features? 1. Excessive thoracic kyphosis 2. Vertebral rotation 3. Lateral spinal deviation 4. Lumbar lordosis

2. Vertebral rotation 1. The forward bend test will show a rotary component of the spine if there is a scoliosis. Excessive thoracic kyphosis is not determined by the forward bend evaluation. Excessive kyphosis should be viewed from the side (pp. 514-515). 2. Scoliosis during forward bending is indicated by a spinal curve. This is called a rib hump and is caused by rotation of the vertebral bodies (pp. 516-517). 3. The purpose of the forward bend test is to assess for the rotary component of the scoliosis, not lateral spinal deviation. Lateral deviation is better assessed with plumbline (pp. 516, 522). 4. The purpose of the forward bend test is to assess for the rotary component of the scoliosis, not lumbar lordosis (p. 522).

A patient is referred to physical therapy with a diagnosis of myositis ossificans in the quadriceps muscle after sustaining a contusion. The MOST appropriate treatment for the patient is: 1. progressive resistance exercises through full range. 2. active range of motion in the pain-free range. 3. passive range of motion into resistance. 4. transverse friction massage.

2. active range of motion in the pain-free range. 1. Initially strengthening should be avoided; the focus should be on range of motion (pp. 1011-1012). 2. Once rehabilitation can begin, initial treatment is geared toward range of motion (pp. 1011-1012. 3. Passive range of motion into resistance should not be done initially in the treatment of myositis ossificans (pp. 1011-1012). 4. Initial treatment is geared toward range of motion (pp. 1011-1012). Transverse friction massage is contraindicated in the presence of acute inflammation, hematomas, debilitated or open skin, and decreased sensation (p. 403). Massage should be avoided early in the treatment of myositis ossificans.

A physical therapist can BEST assess for dysdiadochokinesia by asking the patient to: 1. identify a spoon, a toothbrush, and a comb by touch. 2. bring a toothbrush or a spoon to the mouth repeatedly, using rapid brisk motions. 3. demonstrate how to use a toothbrush, a spoon, or a comb. 4. name familiar objects, such as a toothbrush, a spoon, and a comb.

2. bring a toothbrush or a spoon to the mouth repeatedly, using rapid brisk motions. 1. Inability to identify objects by touch is astereognosis (p. 93). 2. Dysdiadochokinesia is the impaired ability to perform rapid alternating movements, which could be demonstrated by the repeated elbow flexion and extension in repetitively bringing a utensil to the mouth (p. 193). 3. Apraxia is characterized by the inability to perform purposeful movements, such as using some type of utensil (p. 1213). 4. Nonfluent (Broca's) aphasia results in impaired language formation and word-finding difficulties (p. 142).

After 1 week of a progressive resistance exercise training program, an individual demonstrates significant strength gains. The MOST likely explanation for the observed strength gains is: 1. an increased ratio of fast- to slow-twitch fibers. 2. improved neuromuscular recruitment. 3. muscle-fiber hyperplasia. 4. muscle hypertrophy.

2. improved neuromuscular recruitment. 1. Although transformation of type IIB to type IIA fibers occurs in the early weeks of resistance training, transformation from slow-twitch to fast-twitch fibers is unlikely (p. 177). 2. The initial rapid gain in the tension-generating capacity of skeletal muscle is largely attributable to neural responses, including increased recruitment in number of motor units firing and increased rate of synchronization of firing (p. 176). 3. Muscle-fiber hyperplasia is an increase in the number of muscle fibers, and if it occurs, it is in response to heavy resistance training and only accounts for a small percentage of the increase in strength (p. 177). 4. Muscle hypertrophy, or increase in the size of individual muscle fibers, requires an extended period (4-8 weeks) of moderate-intensity to high-intensity resistance training (p. 177). One week is too short of a duration for such change.

After moving a patient from long-sitting into the position shown in the photograph, the physical therapist notes upbeating torsional nystagmus, starting 5 seconds after the patient assumes the position and lasting for 10 seconds. The patient reports vertigo. The desired outcome after successful intervention would be: 1. dislodging of debris from the cupula. 2. removing of debris from the canal. 3. diminished sensitivity to motion. 4. improved stability of gaze with head movement.

2. removing of debris from the canal. 1. To dislodge the debris from the cupula would be the desired outcome for cupulolithiasis, which occurs when otoconia or debris adheres to the cupula, resulting in vertigo and nystagmus that persist as long as the patient is in the provoking position (p. 931). 2. The brief duration of the nystagmus indicates canalithiasis or debris free-floating in one of the semicircular canals on the right. The desired outcome is to remove the debris from the canal. (p. 981) 3. To diminish sensitivity to movement is a desired outcome in unilateral vestibular hypofunction. The patient shows signs of canalithiasis (pp. 982-983). 4. To improve gaze stability is a desired outcome in unilateral vestibular hypofunction. The patient shows signs of canalithiasis (pp. 982-983).

A physical therapist is performing a supine hamstring stretch on a patient who has a T5 level spinal cord injury. The patient describes a pounding sensation in the ear. What should the therapist do FIRST? 1. Roll the patient to the left side. 2. Take the patient's blood pressure. 3. Bring the patient to an upright position. 4. Notify the patient's physician for immediate medical attention.

3. Bring the patient to an upright position. 1. A pounding sensation in the ear in a patient with a spinal cord injury above the level of T6 is indicative of autonomic dysreflexia. With autonomic dysreflexia, the patient's blood pressure will be elevated and the patient should be brought to an upright position to lower the blood pressure. Rolling the patient to the left side would not lower the blood pressure. 2. A pounding sensation in the ear in a patient with a spinal cord injury above the level of T6 is indicative of autonomic dysreflexia. With autonomic dysreflexia, the patient's blood pressure will be elevated and the patient should be brought to an upright position to lower the blood pressure. 3. A pounding sensation in the ear in a patient with a spinal cord injury above the level of T6 is indicative of autonomic dysreflexia. With autonomic dysreflexia, the patient's blood pressure will be elevated and the patient should be brought to an upright position to lower the blood pressure. 4. A pounding sensation in the ear in a patient with a spinal cord injury above the level of T6 is indicative of autonomic dysreflexia. Although autonomic dysreflexia can be a life-threatening condition, notifying the physician would not be the most immediate action to take and would be the appropriate action only if the blood pressure cannot be reduced by other means.

An elderly patient is being evaluated for discharge from an acute care setting. Prior to admission, the patient lived alone on the second floor of a building without an elevator and walked with the use of a cane. Currently, the patient performs bed mobility and stand pivot transfers with moderate assistance. The patient does not want to go to a transitional care facility for rehabilitation, and prefers to go home instead. Which of the following approaches is MOST appropriate for the physical therapist to take? 1. Recommend discharge to home with a home health aide during the day, and order all necessary equipment. 2. Clearly explain to the patient that the physician is the one who determines the type of facility to which the patient will be discharged. 3. Clearly explain to the patient that the goal of discharge to a transitional care facility is for short-term rehabilitation. 4. Recommend discharge to home with home physical therapy and community services, and order all necessary equipment.

3. Clearly explain to the patient that the goal of discharge to a transitional care facility is for short-term rehabilitation. 1. Given that the patient lives on the 2nd floor and still needs assistance for transfers, the best option is discharge to a short-term rehabilitation facility. 2. The physical therapist can educate the patient about discharge planning as part of the treatment team; the physician is a part of the interdisciplinary team. 3. If the physical therapist educates the patient about the need for a short-term placement, the patient may better understand the need to become more independent before going home. 4. Given that the patient lives on the 2nd floor and still needs assistance for transfers, short-term rehabilitation would be the best option.

A patient with a recent onset of steroid-induced myopathy would have the MOST difficulty performing which of the following activities? 1. Writing with a pen 2. Dressing 3. Climbing stairs 4. Rising from a supine position

3. Climbing stairs 1. Distal musculature is typically spared in steroid-induced myopathy until late stages of the myopathy. 2. Although upper extremity proximal musculature is also a component of steroid-induced myopathy, patients usually compensate and continue to perform dressing activities without difficulty. 3. Prolonged use of corticosteroids can lead to corticosteroid-induced catabolism, resulting in inhibition of muscle protein synthesis and muscle weakness. The proximal muscles of the lower and upper extremities are affected first. Patients first report difficulty in climbing stairs. 4. Involvement of the trunk musculature is a late finding in the progression of steroid-induced myopathy.

Which of the following is the normal equilibrium response of a patient tilting side to side while seated on a rocker board? 1. Convexity of the trunk on the upward side with downward side protective extension 2. Concavity of the trunk and shoulder adduction on the upward side 3. Concavity of the trunk and shoulder abduction on the upward side 4. Convexity of the trunk and protective extension on the upward side

3. Concavity of the trunk and shoulder abduction on the upward side 1. The trunk being convex on the upward side is an abnormal response, and protective reactions are seen on upward side, not the downward side. 2. The trunk should curve toward the upside, creating a concavity, but shoulder adduction would not protect against falling. 3. The normal response is curvature of the trunk toward the upward side (creating concavity) with extension and abduction of the extremities on that side. Protective extension includes extension and abduction to support and protect body from falling. 4. The trunk convexity on the upward side is an abnormal response, although protective reactions on the upward side are normal.

A patient with advanced liver disease has developed severe ascites. This condition will MOST likely have which of the following effects on the patient's respiratory system? 1. Increased residual volume 2. Increased functional residual capacity 3. Decreased inspiratory reserve volume 4. Decreased forced expiratory volume in 1 second (FEV1)

3. Decreased inspiratory reserve volume 1. Increased residual volume is seen in obstructive lung disease and not in restrictive lung disease (Hillegass, p. 127). This would not be seen with ascites and would not be present in the patient described. 2. Increased functional residual capacity is seen in obstructive lung disease and not in restrictive lung disease (Hillegass, pp. 128, 191). This would not be seen with ascites and would not be present in the patient described. 3. Ascites is a condition where there is increased fluid in the peritoneal cavity, causing distention (Moore, pp. 445-446). This distention puts increased pressure upon the diaphragm and thoracic cavity, which may lead to a restrictive lung pattern. Decreased inspiratory reserve volume is a component of restrictive lung disease (Hillegass, pp. 127-128). 4. Decreased forced expiratory volume in 1 second (FEV1) is commonly seen in obstructive lung disease, caused by either secretions or bronchospasm (Hillegass, p. 127). These do not result from ascites and would not be present in the patient described.

A physical therapist is conducting a graded exercise stress test of an apparently healthy adult using a treadmill. The test should be discontinued if which of the following events occurs? 1. Heart rate continues to increase throughout the test. 2. Borg rating of perceived exertion is reported as 13/20. 3. Diastolic blood pressure reaches 120 mm Hg. 4. Significant redness of the skin and perspiration are observed.

3. Diastolic blood pressure reaches 120 mm Hg. 1. Monitoring heart rate response to exercise is the purpose of conducting the test, and, therefore, an increasing heart rate is not a reason to stop. Failure of the heart rate to rise with increasing exercise intensity would be a reason to stop. 2. A Borg rating of perceived exertion of 13/20 should not be a reason to stop the test (ASCM, p. 83). Most people reach their subjective limit of fatigue at an RPE of 18-19/20. 3. A diastolic blood pressure of 120 mm Hg is an indicator for ending the test. A diastolic blood pressure greater than 115 mm Hg is too high to continue testing. 4. Significant redness of the skin and perspiration are normal responses to exercise testing. Cyanosis or pallor would be a reason to stop. (ASCM, p. 84)

A nonathletic male patient reports occasional brief palpitations that occur in the absence of pain, dizziness, or light-headedness. The patient has no personal or familial history of heart disease and is otherwise healthy. Which of the following factors is the MOST likely source of the palpitations? 1. Gender 2. Sedentary activity level 3. Excess caffeine intake 4. Cardiac abnormality

3. Excess caffeine intake 1. Palpitations can occur as a result of hormonal changes (i.e., during menopause or with ovulation). Since this is a healthy male patient, hormonal changes associated with gender can be ruled out. 2. Generally, a low-activity-level/nonathletic lifestyle does not cause or increase the likelihood of palpitations. Exercise can both induce and reduce the frequency and onset of palpitations. 3. Palpitations can occur due to diet, particularly with excessive intake of caffeine. Typically, caffeine intake precipitates the palpitations and causes brief palpitations of gradual onset and without any associated pain, dizziness, or light-headedness. 4. Typically, palpitations of cardiac origin are associated with dyspnea, fainting, or severe light-headedness or dizziness. This patient does not have any of these additional symptoms.

A patient is referred to physical therapy with thoracic spine pain. Which of the following data obtained from the patient's history is MOST likely indicative of the presence of an underlying cardiac condition? 1. Sharp mid back pain that increases with lifting of heavy objects 2. Increased pain with deep breathing 3. Feeling of heaviness in the chest 4. Persistent night pain

3. Feeling of heaviness in the chest 1. Sharp pain in the body part being used is a typical characteristic of nerve or nerve root pain (Magee, p. 9). Also, muscle pain will increase when the muscle contracts or is stretched (Magee, p. 8). 2. Pain related to breathing may signal pulmonary problems or may be related to movement of the ribs (Magee, p. 511). 3. A feeling of heaviness in the chest may be a sign of a cardiovascular condition and indicates the need for referral to a physician (Magee, p. 2). 4. Persistent night pain is a classic symptom of cancer, but it may also be present in a patient who has an occult bone fracture. If a musculoskeletal condition can be ruled out, night pain indicates the need for medical consultation. (Goodman, p. 126)

A patient who has emphysema reports a weight gain of 20 lb (9 kg) over the past month. There is 3+ pitting edema in both distal lower extremities. Which of the following tests is MOST important to identify the likely underlying condition? 1. Heart rate 2. Capillary refill 3. Heart auscultation 4. Ankle-brachial index

3. Heart auscultation 1. The patient has concerning symptoms suggestive of heart failure. Heart rate would not be most important measure because right ventricular heart failure is not associated with acute changes to heart rate. Right ventricular failure is due to the sustained elevation in pulmonary arterial hypertension. (Goodman, pp. 593-595) 2. Capillary refill is used to assess for microvascular disease (Hamm, p. 110). This test would not help explain the patient's clinical presentation of pitting edema and weight gain. 3. The patient has concerning symptoms suggestive of heart failure. The classic signs include peripheral pitting edema, weight gain, jugular vein distention, diminished appetite, right upper quadrant discomfort, and a ventricular gallop (S3) heart sound (Hillegass, pp. 87, 98). Assessing the heart sounds would be of value for the medical diagnosis or for determining whether consultation with another health care professional is needed. 4. The ankle-brachial index is performed to assess for arterial disease and is designed to test the potential loss of perfusion in the lower extremities (O'Sullivan, p. 533). Clinical manifestations of arterial disease include pain, pallor, paralysis, diminished pulses, hair loss, and thin shiny skin (Goodman, pp. 639-640). Venous insufficiency is represented by edema, hemosiderin staining of the skin, and normal arterial pulses (Goodman, pp. 655-656). The patient could have secondary venous issues, but the ankle-brachial index would not be indicated as an additional testing measure.

What reaction is MOST likely being assessed in the photograph? (picture of adult holding baby with arms under stomach while baby is in superman flying position with neck extended) 1. Moro reflex 2. Startle reflex 3. Landau reflex 4. Positive support reflex

3. Landau reflex 1. The Moro reflex is elicited by supporting the infant in a supine position, allowing the head to gently drop posteriorly in relation to the trunk, and assessing the response at the arms (O'Sullivan). 2. The startle reflex is elicited by making a sudden loud or harsh noise and results in sudden extension or abduction of the upper extremities and crying (O'Sullivan). 3. The Landau reflex/reaction is elicited by supporting the infant horizontally in prone position. The expected response is that the infant will extend the neck and trunk. (Ropper) 4. The positive support reflex/reaction is elicited by supporting the infant vertically with contact of the feet to a surface and assessing the lower extremity response (O'Sullivan).

A patient has resulting lymphedema after a radical mastectomy on the left side and has an arteriovenous dialysis shunt in the right upper extremity. Which of the following locations would be MOST appropriate to auscultate the patient's blood pressure? 1. Left cubital fossa 2. Right cubital fossa 3. Left popliteal fossa 4. Right femoral triangle

3. Left popliteal fossa 1. Blood pressure should not be monitored in the arm with an arteriovenous shunt. 2. Blood pressure should not be measured in the arm with lymphedema. 3. If both arms are inappropriate for measuring blood pressure, the thigh is an appropriate location with auscultation at the popliteal artery. Alternative sites for measurement in the lower extremity are proximal to the popliteal artery or proximal to the ankle with auscultation of the posterior tibial artery. 4. If both arms are inappropriate for measuring blood pressure, the thigh is an appropriate location with auscultation at the popliteal artery. The femoral triangle is too high for placement of a blood pressure cuff.

A patient has right shoulder pain that extends from the right upper trapezius to the thorax between the scapulae. The patient reports a recent tendency to bruise easily and has dark-colored urine and clay-colored stools. Which of the following organs is MOST likely the etiology of these findings? 1. Heart 2. Kidney 3. Liver 4. Pancreas

3. Liver 1. Cardiac dysfunction can cause right shoulder pain but not the associated symptoms described in the stem (pp. 689, 693). 2. Renal pathology can cause pain that is felt in the shoulder on the same side as the involved kidney but will not cause the associated symptoms described in the stem (pp. 697-698). 3. Liver dysfunction will cause all of the symptoms described in the stem: jaundice, dark urine, clay-colored stool, easy bruising, and right shoulder pain (pp. 341, 689). 4. Pancreatic pain is referred to the left shoulder; pancreatic dysfunction will not produce the associated symptoms described in the stem (pp. 329, 689).

A physical therapist is assisting a patient in rehabilitation following an ankle sprain. The therapist reviews the patient's first attempt in use of a balance board and asks the patient to visualize the movement of the ankle and the balance board anteriorly and posteriorly in preparation for the next attempt. Which form of motor learning is being used? 1. Knowledge of results 2. Knowledge of performance 3. Mental practice 4. Serial practice

3. Mental practice 1. The patient is being asked to visualize the appropriate movement. Knowledge of results is a form of feedback, not a process of visualization (Shumway-Cook, p. 33). 2. The patient is being asked to visualize the appropriate movement. Knowledge of performance is a form of feedback, not a process of visualization (Shumway-Cook, p. 33). 3. The patient is being asked to visualize the appropriate movement. Mental practice is the act of performing a skill mentally before performing it physically (Shumway-Cook, p. 36). 4. The patient is being asked to visualize the appropriate movement. Serial practice is a physical form of motor learning, not a mental form (Goodman, p. 1403).

Which of the following medications is MOST likely to be used to treat the condition of the patient shown in the photograph? (Image is hands of an RA pt) 1. Furosemide (Lasix) 2. Metformin (Glucophage) 3. Methotrexate (Trexall) 4. Atenolol (Tenormin)

3. Methotrexate (Trexall) 1. The patient in the photograph has rheumatoid arthritis (Goodman, p. 1323). Furosemide is a diuretic used for the treatment of hypertension, not the treatment of rheumatoid arthritis (Ciccone, p. 320). 2. The patient in the photograph has rheumatoid arthritis (Goodman, p. 1323). Metformin is indicated for diabetes, not rheumatoid arthritis (Ciccone, p. 519). 3. The patient in the photograph has rheumatoid arthritis. Methotrexate is a disease-modifying antirheumatic drug used in the treatment of rheumatoid arthritis. (Goodman, p. 1323; Ciccone, p. 240) 4. The patient in the photograph has rheumatoid arthritis (Goodman, p. 1323). Atenolol is a beta-adrenergic blocking agent used in the treatment of hypertension or angina, not rheumatoid arthritis (Ciccone, p. 311).

A patient has severe chronic obstructive pulmonary disease and bilateral knee osteoarthritis. During aquatic therapy involving slow walking, which of the following water levels is MOST appropriate for this patient? 1. Knees 2. Hips 3. Navel 4. Shoulders

3. Navel 1. Water level up to the navel would minimize knee compressive force to a greater extent than water level up to the knees, because more loading forces would be removed (p. 297). 2. Water level up to the navel would minimize knee compressive force to a greater extent than water level up to the hips (p. 297). 3. Patients who have knee arthritis often have difficulty and pain when walking on land. Buoyancy, ground reaction forces, and knee joint forces are less during walking in the water, compared to these same forces while walking on land. The greater the amount of water displaced by the body, the less the ground reaction forces experienced, and the less force transmitted up the body through the ankles, knees, hips, and trunk. Therefore, as water depth increases and more of the body is immersed in water, knee compressive force decreases. This allows a patient who has knee arthritis to be able to walk in water with minimal difficulty and pain, even though the patient may be unable to walk without pain and difficulty on land. Water level up to the navel would minimize knee compressive force to a greater extent than water level up to the knees or hips. (pp. 297, 307) 4. A water level to the shoulders could affect the ability of the lungs to expand (p. 297).

Which of the following signs and symptoms are MOST characteristic of herpes zoster? 1. Unilateral scales along the affected dermatome followed by numbness and burning 2. Bilateral scales along the affected dermatomes followed by numbness and burning 3. Pain and paresthesia followed by a unilateral rash along the affected dermatome 4. Pain and paresthesia followed by a bilateral rash along the affected dermatomes

3. Pain and paresthesia followed by a unilateral rash along the affected dermatome 1. Herpes zoster (shingles) has initial symptoms of pain and paresthesia localized to the affected dermatome, followed by vesicular eruptions along a facial, cervical, or thoracic lumbar dermatome. Shingles is often present as a rash, not scales, unilaterally. 2. Herpes zoster (shingles) is initially manifested as paresthesia and pain along the affected dermatome with a rash that forms days later. Shingles is a rash, not scales, that characteristically presents unilaterally. 3. Herpes zoster (shingles) is initially manifested as pain and paresthesia along the affected dermatome with a rash that forms days later. Shingles characteristically presents unilaterally. 4. Herpes zoster (shingles) is characteristically present unilaterally, not bilaterally.

Which of the following options BEST describes the mode of action of angiotensin-converting enzyme (ACE) inhibitors? 1. Reduction of heart rate and increase in cardiac contractility 2. Reduction of heart rate and decrease in cardiac contractility 3. Reduction of peripheral vascular resistance and increase in venous capacitance 4. Reduction of peripheral vascular resistance and decrease in venous capacitance

3. Reduction of peripheral vascular resistance and increase in venous capacitance 1. The mode of action described does not occur with angiotensin-converting enzyme inhibitors (p. 472). 2. The mode of action described is for beta-blockers, which are another medication used to treat hypertension (p. 470). 3. Angiotensin-converting enzyme inhibitors produce vasodilation, reduce peripheral vascular resistance, and increase venous capacitance (p. 472). 4. Angiotensin-converting enzyme inhibitors produce vasodilation, reduce peripheral vascular resistance, and increase venous capacitance (p. 472). Therefore this option is incorrect since it states an action is decreasing venous capacitance.

A patient who has tetraplegia experiences an insidious onset of pain inferior to the glenohumeral joint. Soft tissue swelling, local warmth, and erythema are present, as well as limited range of motion. The patient's serum alkaline phosphatase levels are elevated. Which of the following conditions are MOST likely present? 1. Actinomycosis and osteomalacia 2. Pyogenic infection and cellulitis 3. Septic arthritis and ectopic bone formation 4. Granulomatous infection and thrombophlebitis

3. Septic arthritis and ectopic bone formation 1. Actinomycosis has associated signs and symptoms of chest pain, dyspnea, fatigue, and fever (Ferri, p. 14). Osteomalacia is characterized by painful muscle weakness, bone pain, and tenderness (Goodman, pp. 1227-1228). Alkaline phosphatase levels are elevated. Since the patient is not experiencing chest pain, this is not the correct answer. 2. Alkaline phosphatase is an enzyme produced by bone cells. Increased levels would increase bone formation due to active osteoclasts (Goodman, p. 1232). This does not occur with a pyogenic infection or cellulitis. Cellulitis is associated with redness, tenderness, and edema (Goodman, pp. 700-701). 3. Septic arthritis is characterized by rapid onset, over hours or days, of monoarthritis with a joint that is swollen, red, tender, and warm with limited range of motion due to pain (Goodman, p. 1245). Increases in alkaline phosphatase levels with local edema, heat, and erythema are indicative of heterotopic ossification (ectopic bone formation) (Lazaro; Goodman, pp. 1288-1289). 4. Neither granulomatous infection nor thrombophlebitis are associated with decreased glenohumeral range of motion (Goodman, pp. 230, 646).

A patient has right lower lobe atelectasis following abdominal surgery. Upon auscultation, a physical therapist notes no adventitious breath sounds. Which of the following is MOST appropriate for this patient? 1. Percussion and vibration in left Trendelenburg position 2. Vigorous percussion and vibration with nebulizer 3. Spirometry with segmental breathing exercises 4. Practice of incisional splinting for coughing techniques

3. Spirometry with segmental breathing exercises 1. Percussion and vibration are appropriate for airway clearance. However, the absence of adventitious breath sounds suggests the airway is clear. (pp. 543-544) 2. Percussion and vibration with a nebulizer that delivers medication are appropriate for airway clearance. However, the absence of adventitious breath sounds suggests the airway is clear. (pp. 495, 543-544) 3. Adventitious breath sounds are abnormal sounds such as crackles, wheezes, and stridor. Atelectasis is a collapse of lung tissue. Spirometry and segmental breathing exercises are appropriate interventions for atelectasis without the presence of adventitious breath sounds. (pp. 150, 521) 4. Incisional splinting is appropriate for pain control when coughing after abdominal surgery but would not address the atelectasis (p. 364).

An 8-year-old child who has Duchenne muscular dystrophy would MOST likely have which of the following examination findings? 1. Swayback posture and protraction of the scapulae 2. Hamstrings contractures and excessive hip extension 3. Standing lordotic posture and winging of the scapulae 4. Quadriceps contractures and excessive hip abduction

3. Standing lordotic posture and winging of the scapulae 1. In a swayback posture, the pelvis is posteriorly tilted and the thoracic spine is in relative flexion. In children who have Duchenne muscular dystrophy, the hip flexors become shortened from overuse and the pelvis tilts anteriorly. The thoracic spine begins to move into relative extension to compensate for the pelvic tilt, and the scapulae wing in an attempt to keep the center of mass behind the hip joint. (p. 246) 2. Tensor fasciae latae contractures become increasingly frequent from age 8-10 years in patients who have Duchenne muscular dystrophy. In addition, hip extension range of motion is often limited due to shortness of the hip flexor muscles. (pp. 248-249) 3. No limitations in range of motion are typically noted before 5 years of age in children who have Duchenne muscular dystrophy. By age 8, the deficits listed would be clearly present. The normal lordotic standing posture is increased, and winging of the scapulae is seen as compensation to keep the center of mass behind the hip joint to promote standing ability. (p. 246) 4. In children who have Duchenne muscular dystrophy, the hamstrings become contracted, not the quadriceps (p. 248).

A Physical Therapist is treating a patient who is recovering from TBI and is RLA stage 5. After 30 minutes into the therapy session the patient is not able to focus and starts losing interest in the activity. What is the appropriate response of the Physical Therapist? 1. Take the patient to a different setting with group activities to keep him motivated. 2. Ask the patient to focus and complete the session 3. Take a break and resume the rest of the activity after the break. 4. Stop the activity for the day as he is fatigued.

3. Take a break and resume the rest of the activity after the break. Answer: RLA stage 5 is defined as confused inappropriate. In this stage patients generally cannot focus for more than 30 minutes and quickly lose interest. These patients require frequent breaks to prevent fatigue and loss of attention. Giving a break and resuming the activity is the best way to keep this patient's focus. Group activities can be overwhelming and distracting and not suitable in this stage. Stopping the activity for the day is not required.

In an older adult patient, impaired proprioception may be a result of malabsorption of which of the following nutrients? 1. Potassium 2. Iron 3. Vitamin B12 4. Vitamin E

3. Vitamin B12 1. Potassium helps regulate normal muscle contraction and would affect the heart, intestines, and respiratory tract, but would not primarily affect proprioception (p. 185). 2. Iron deficiency can occur secondary to blood loss, malabsorption, and pregnancy. Iron deficiency may produce symptoms of irritability, lack of exercise tolerance, and headaches, but not impaired proprioception (pp. 714-715). 3. Intrinsic factor often declines with age. Intrinsic factor is a protein that helps the body absorb vitamin B12. When stomach secretions do not have enough intrinsic factor, vitamin B12 is not absorbed well. This results in pernicious anemia and other problems related to low levels of vitamin B12. Inadequate vitamin B12 gradually affects sensory and motor nerves, causing impaired proprioception to develop over time. These neurological symptoms are due to myelin degeneration and loss of nerve fibers in the dorsal and lateral columns of the spinal cord and cerebral cortex. (pp. 714-715) 4. Vitamin E is a fat-soluble vitamin that is an important antioxidant. Lack of vitamin E would not be expected to result in impaired proprioception. (p. 219)

A patient had a brainstem stroke 2 months ago and is currently able to independently walk 65 ft (20 m) over level surfaces with a straight cane and ascend stairs with minimum assistance. Which of the following activities would MOST appropriately challenge this patient's balance during a physical therapy session? 1. Ascending stairs using a single handrail 2. Standing on one leg with eyes closed 3. Walking over uneven terrain 4. Walking 130 ft (40 m) with a straight cane

3. Walking over uneven terrain 1. Practice ascending stairs addresses impaired ability to transfer weight onto a stance limb and lift the opposite dynamic limb (p. 242). Although dynamic balance is involved in this activity, ascending stairs using a single handrail is not more difficult than an activity the patient is currently able to perform and would not be challenging to the patient. 2. Static balance challenges will benefit static postural control (p. 195). 3. Walking over uneven terrain is a good example of an impairment-based, task-specific, functional activity that will improve the patient's dynamic balance (p. 233). 4. Increasing the duration of walking by progressing to longer distances primarily progresses endurance, not balance (p. 244).

A patient with evidence of nerve root involvement at the L4-L5 level would MOST likely have which of the following signs and symptoms? 1. Weak knee flexors, diminished Achilles tendon reflex, and paresthesia at the lateral aspect of the lower extremity 2. Weak foot intrinsics, diminished patellar tendon reflex, and paresthesia at the medial aspect of the knee 3. Weak ankle dorsiflexors, diminished patellar tendon reflex, and paresthesia at the medial calf and ankle 4. Weak ankle plantar flexors, diminished Achilles tendon reflex, and paresthesia at the lateral border of the foot

3. Weak ankle dorsiflexors, diminished patellar tendon reflex, and paresthesia at the medial calf and ankle 1. A decreased Achilles tendon reflex implicates S1. Paresthesias at the lateral aspect of the lower extremity involve L5-S1. 2. Foot intrinsics are innervated at the S1-S2 level. 3. The L4 nerve root exits at the L4-L5 level. Weak ankle dorsiflexors, diminished patellar tendon reflex, and paresthesias along the medial posterior lower leg and ankle are the only combination of signs and symptoms listed that could all be a result of L4 nerve root compromise. 4. This combination of signs and symptoms would occur with nerve root involvement at the S1-S2 level. Nerve root invovlement of L4-L5 = L4 nerve root affected HNP at L4-L5 = L5 nerve affected

While walking on a treadmill during Phase II cardiac rehabilitation following coronary artery bypass surgery, a patient reports the new onset of chest pain and dyspnea. The physical therapist should instruct the patient to: 1. continue walking while the therapist monitors the patient's vital signs. 2. continue walking at 50% slower speed while the therapist calls the physician. 3. cease walking while the therapist reassesses the patient's vital signs. 4. cease walking while the therapist activates the emergency medical system.

3. cease walking while the therapist reassesses the patient's vital signs. 1. Continuing to walk is inappropriate. Angina during exercise should result in termination of the activity. 2. These are indications to terminate exercise and reassess vital signs. Continuing to walk at a lower intensity is inappropriate. 3. An episode of stable angina is an indication to terminate exercise testing and reassess vital signs. 4. These symptoms do not constitute a medical emergency. It is more appropriate to reassess vitals and ask the patient to take nitroglycerin, if prescribed to do so.

An adult patient requests feedback on his health and wellness exercise program. The patient reports a weekly weight loss of 4.4 lb (2 kg)/week for the last 2 months and is utilizing a dietary intake of 900 kcal/day over the same time period. The BEST advice for the physical therapist to provide to this patient is to: 1. decrease dietary intake further to improve ability for greater weight loss. 2. keep intake and weight loss targets the same. 3. increase dietary intake and target a weight loss of no more than 2.2 lb (1 kg)/week. 4. increase dietary intake and exercise to preserve weight loss at 4.4 lb (2 kg)/week.

3. increase dietary intake and target a weight loss of no more than 2.2 lb (1 kg)/week. 1. Reducing the patient's intake further would result in daily caloric intake below the minimal required daily caloric intake and would, therefore, be detrimental to overall health. 2. The patient's current rate of weight loss is more than recommended, and the current caloric intake is lower than recommended. 3. Dietary intake should be adequate to allow weight loss of no more than 1 kg/week. 4. Weight loss of 2 kg/week is greater than recommended.

A physical therapist uses a foam-based dressing in the treatment of a healing pressure injury that is producing a large amount of exudate. The therapist's rationale for using this dressing is to prevent: 1. eschar formation. 2. desiccation. 3. maceration. 4. infection.

3. maceration. 1. Eschar is associated with desiccation. Using a foam dressing will not likely prevent eschar formation and may promote further desiccation. (Baranoski) 2. Foam dressings are likely to cause, not prevent, desiccation. (Baranoski) 3. Foam dressings are primarily indicated for highly-exudative wounds due to their ability to absorb excess fluid to prevent maceration as long as the dressing is changed frequently (Hamm). 4. Not all foam dressings contain an antimicrobial to prevent wound infection, however all foam dressings are highly absorbent (Hamm).

A patient was brought into the emergency department with shortness of breath and a past medical history of atelectasis. The patient currently has severe right-sided chest pain, sharp pain with inhalation, rapid heart rate, cyanosis of the skin and lips, and decreased breath sounds. The patient is suspected to be suffering from a pneumothorax. Which of the following tracheal pathological changes would be observed in a large pneumothorax versus in a massive atelectasis? Select one: 1. A pneumothorax pulls the trachea to the ipsilateral while a massive atelectasis will push the trachea to the contralateral side 2. There is no change in tracheal position for either pathology 3. Both pneumothorax and a massive atelectasis will push the trachea to the contralateral side 4. A pneumothorax pushes the trachea to the contralateral side while a massive atelectasis will pull the trachea to the ipsilateral side

4. A pneumothorax pushes the trachea to the contralateral side while a massive atelectasis will pull the trachea to the ipsilateral side Answer: The patient's signs/symptoms indicate he is suffering from a large pneumothorax and his right lung has collapsed. During such a condition, the trachea will most likely be deviated and pushed to the contralateral side, in this case, it would be observed as deviated towards the patient's left side as opposed to atelectasis pulling the trachea towards the involved side

A patient sustained a cervical hyperextension injury that caused bleeding into the central gray matter of the lower cervical spinal cord. Which of the following descriptions BEST reflects the highest level of function the patient is likely to achieve? 1. Inability to transfer without assistance and use of a motorized wheelchair required for mobility 2. Ability to propel a standard wheelchair independently with upper extremities but inability to walk 3. Independence in bed mobility and self-care activities but use of a sliding board required for transfers 4. Ability to walk but difficulty with distal upper extremity and hand function

4. Ability to walk but difficulty with distal upper extremity and hand function 1. The lower extremities are often spared with a central cord injury; therefore, a motorized wheelchair is an inappropriate assistive device. 2. The patient is more likely to have distal arm weakness, which should affect wheelchair mobility with upper extremities, but the patient may recover the ability to walk. 3. Patients who have central cord syndrome should expect to have difficulty with activities of daily living that require upper extremity function. Since the lower extremities may be spared, the patient might be able to transfer without a sliding board. Because the question asks for the highest level hoped for, this would not be the best choice. 4. This is a description of central cord syndrome. Because the spinal tracts for the lower extremities are positioned more laterally in the spinal cord, upper extremities are more affected than lower extremities. Patients with central cord syndrome typically recover the ability to ambulate with some remaining distal arm weakness. Seventy-seven percent of patients with central cord syndrome will attain ambulatory function, and 42% will attain hand function.

A patient has been prescribed warfarin (Coumadin) following total hip arthroplasty. Which of the following over-the-counter medications listed in the patient's medical history at the first postoperative visit would be of GREATEST concern to a physical therapist? 1. Diphenhydramine (Benadryl) 2. Cetirizine (Zyrtec) 3. Omeprazole (Prilosec) 4. Acetylsalicylic acid (Aspirin)

4. Acetylsalicylic acid (Aspirin) 1. Benadryl is a histamine antagonist and is not listed as a drug that has interactions with warfarin or has anticoagulant effects (p. 442). 2. Cetirizine is a histamine antagonist and is not listed as a drug that has interactions with warfarin or has anticoagulant effects (p. 316). 3. Omeprazole is a proton pump inhibitor and is not listed as a drug that has interactions with warfarin or has anticoagulant effects. It should be avoided when taking antiplatelet medication. (p. 335) 4. Acetylsalicylic acid (Aspirin) and warfarin are both anticoagulants. The most serious interactions with warfarin are those that increase anticoagulant effects and the risk of bleeding. Patients who are taking warfarin should be instructed not to take acetylsalicylic acid (Aspirin) simultaneously without discussing the combination with a physician. Taking both could cause excessive anticoagulation, which could be harmful. (p. 39)

Following anterior cruciate ligament reconstruction using a graft from the patellar ligament, which of the following early interventions will BEST facilitate healing of the patellar ligament? 1. High-load short-arc quads 2. Prolonged immobilization with the knee fully extended 3. Deep squats 4. Active knee range of motion

4. Active knee range of motion 1. Low loads are indicated at this phase of tissue healing, not high loads. High loads may overload the tissue and cause damage. (Dutton, pp. 1000-1001) 2. Prolonged immobilization will lead to adhesions and decreased range of motion, along with weak/unorganized tissue that is less able to tolerate the normal stresses eventually placed upon it (Dutton, pp. 56-57; Kisner, pp. 814-815). 3. Although weight-bearing may provide a low-load stimulus, deep squats are too aggressive at this stage and may cause tissue damage/harm (Dutton, pp. 1000-1001). 4. Active range of motion provides a light load similar to normal stress to facilitate tissue healing during the repair and regeneration phase of tissue healing (Kisner, pp. 91, 815).

A physical therapist examines a patient recovering from a gunshot wound affecting the left deep fibular (peroneal) nerve. To which surface of the lower leg should the therapist apply electrodes for biofeedback therapy? 1. Posteromedial 2. Posterolateral 3. Anteromedial 4. Anterolateral

4. Anterolateral 1. Muscles in the posteromedial aspect of the lower leg are supplied by the tibial nerve (Moore, pp. 756-757). 2. Muscles in the posterolateral aspect of the lower leg are supplied by the superficial fibular (peroneal) nerve (Moore, p. 750). 3. There are no muscles located in the anteromedial aspect of the lower leg (Moore, p. 692). 4. Electrode placement during biofeedback should be as near to the muscle being treated as possible. The deep fibular (peroneal) nerve supplies muscles in the anterior compartment of the leg, most prominently the ankle dorsiflexors and toe extensors. The specific location of these muscles can best be described as anterolateral. (Moore, pp. 749-750; Lippert, p. 357)

A physical therapist is at a community pool leading a class for patients who have arthritis. The therapist observes an adult patient suddenly fall to the ground. Which of the following sequences of actions is MOST appropriate? 1. Assess the airway, breathing, and circulation of the patient. 2. Begin cardiopulmonary resuscitation and activate the emergency response system. 3. Give two quick breaths, activate the emergency response system, and get an automated external defibrillator. 4. Check responsiveness, activate the emergency response system, check pulse, and begin chest compressions.

4. Check responsiveness, activate the emergency response system, check pulse, and begin chest compressions. 1. The physical therapist should check for responsiveness and then activate the emergency medical system. Assessing breathing and pulse would happen after those steps. (Gulli; Fairchild) 2. Resuscitation is initiated after assessing responsiveness and then activating the emergency medical system (Gulli; Fairchild). 3. The physical therapist should check for responsiveness and then activate the emergency medical system. Rescue breathing would occur after determining the patient has a pulse but is not breathing normally. (Gulli; Fairchild) 4. The physical therapist should check for responsiveness and then activate the emergency medical system. Assessing breathing and pulse would happen simultaneously after those steps. Beginning with chest compressions is recommended if the patient is not breathing or only gasping and has no pulse. (Gulli; Fairchild)

During a bedside evaluation, a patient shows a sudden increase in the rate and depth of respirations, followed by a gradual decrease in respirations and periods of apnea. Which of the following respiratory patterns is the patient exhibiting? 1. Hyperventilation 2. Paradoxical breathing 3. Tachypnea 4. Cheyne-Stokes

4. Cheyne-Stokes 1. Hyperventilation is characterized by rapid deep breathing (p. 517). 2. Paradoxical breathing is characterized by inward abdominal or chest wall movements with inspiration and outward movement with expiration (p. 518). 3. Tachypnea is characterized by rapid shallow breathing (p. 517). 4. Cheyne-Stokes respiration is breathing that waxes and wanes cyclically so that periods of deep breathing alternate with periods of apnea (no breathing) (pp. 98, 517, 550).

A patient reports audible clicking in the temporomandibular joint while chewing food. When a physical therapist is palpating the joint, which of the following pairs of active mandible motions would be MOST informative to confirm temporomandibular dysfunction? 1. Elevation and protrusion 2. Elevation and retrusion 3. Protrusion and retrusion 4. Depression and elevation

4. Depression and elevation 1. Reciprocal clicking with mouth opening and closing has good diagnostic utility in diagnosing temporomandibular conditions and in particular anterior disc displacement (pp. 1269-1272). Opening of the mouth, which is depression of the mandible, is the most revealing and diagnostic movement for temporomandibular dysfunction. Elevation of the mandible is mouth closing and primarily assesses the primary muscles that perform mouth closure. Protrusion of the mandible is used to assess tongue thrust conditions and for deviations during protrusion, which can be caused by muscle or disc problems; therefore, it is not a strongly discriminatory test. (pp. 1278-1279) 2. Reciprocal clicking with mouth opening and closing has good diagnostic utility in diagnosing temporomandibular conditions and in particular anterior disc displacement (pp. 1269-1272). Retrusion of the mandible may be painful in patients with an intracapsular injury, but it not as useful as depression of the mandible in diagnosis of temporomandibular dysfunction (pp. 1278-1279). Elevation of the mandible is mouth closing and primarily assesses the primary muscles that perform mouth closure. Opening of the mouth, which is depression of the mandible, is the most revealing and diagnostic movement for temporomandibular dysfunction (pp. 1278-1279). 3. Reciprocal clicking with mouth opening and closing has good diagnostic utility in diagnosing temporomandibular conditions and in particular anterior disc displacement (pp. 1269-1272). Protrusion of the mandible is used to assess tongue thrust conditions and for deviations during protrusion, which can be caused by muscle or disc problems; therefore it is not a very discriminatory test. Retrusion of the mandible may be painful in patients with an intracapsular injury, but it not as useful as depression of the mandible in diagnosis of temporomandibular dysfunction. Opening of the mouth, which is depression of the mandible, is the most revealing and diagnostic movement for temporomandibular dysfunction. (pp. 1278-1279) 4. Reciprocal clicking with mouth opening and closing has good diagnostic utility in diagnosing temporomandibular conditions and in particular anterior disc displacement (pp. 1269-1272). Opening of the mouth, which is depression of the mandible, is the most revealing and diagnostic movement for temporomandibular dysfunction. Elevation of the mandible is mouth closing and primarily assesses the primary muscles that perform mouth closure (pp. 1278-1279).

A physical therapist is examining a patient who has emphysema and no acute symptoms. During auscultation of the lungs, the therapist will MOST likely observe which of the following findings? 1. Wet crackles (rales) 2. Friction rub 3. Low-pitched wheezes (rhonchi) 4. Diminished breath sounds

4. Diminished breath sounds 1. Crackles are present with secretions in peripheral airways (Goodman, p. 774). 2. A friction rub is associated with a large pleural effusion rather than with emphysema (Hillegass, p. 8). 3. Low-pitched wheezes are associated with obstruction, such as bronchospasm (Goodman, p. 774). 4. Diminished breath or absent breath sounds are present with emphysema (Hillegass, p. 194).

A 4-year-old child has maxillary hypoplasia, an elongated mid face, and a short, upturned nose. The child has a short attention span and poor growth. Which of the following interventions would be MOST appropriate for the child? 1. Gait training with a rolling walker 2. Sensory desensitization activities 3. Activities to inhibit spasticity 4. Dynamic balance activities

4. Dynamic balance activities 1. The stem describes characteristics of a child with fetal alcohol syndrome. Severe delays in walking are not characteristic of a child with fetal alcohol syndrome, and it is likely that a child of this age would be walking independently. Since facial and physical features of fetal alcohol syndrome may be subtle, the diagnosis may not be made until the preschool or school-age years when inattention, hyperactivity, and learning problems are more apparent. (Tecklin, p. 158) 2. The stem describes characteristics of a child with fetal alcohol syndrome. Oversensitivity to sensory input is not a characteristic associated with fetal alcohol syndrome. This is more characteristic of children with autism spectrum disorders. (Tecklin, pp. 406, 415-416) 3. The stem describes characteristics of a child with fetal alcohol syndrome. Spasticity is not associated with fetal alcohol syndrome. Fine motor dysfunction, visuomotor deficits, and weak grasp are characteristics of children with fetal alcohol syndrome. (Palisano, p. 426) 4. The stem describes characteristics of a child with fetal alcohol syndrome. Fine motor dysfunction, visuomotor deficits, balance problems, and weak grasp are characteristics of children with fetal alcohol syndrome. Balance activities would benefit the child. (Palisano, p. 426)

A physical therapist is instructing a patient's care provider on how to perform a bed-to-wheelchair transfer. Which of the following teaching strategies would be MOST appropriate? 1. Have the care provider practice transferring the therapist. 2. Have the care provider observe the therapist perform the transfer with the patient. 3. Explain the transfer while demonstrating and have the care provider practice transferring the therapist. 4. Explain the transfer while demonstrating and have the care provider practice transferring the patient.

4. Explain the transfer while demonstrating and have the care provider practice transferring the patient. 1. Having the care provider perform without an explanation of the transfer would be trial-and-error and not the best way to learn (p. 36). 2. Demonstrating a transfer without an explanation would skip the cognitive stage of learning a psychomotor skill (p. 29). 3. The closer the learning experience is to the actual transfer, the more likely that the skill will be learned. Given that transfer skills are very specific to the individual being transferred, it is important that practice with the patient take place. (p. 36) 4. Motor learning occurs first in the cognitive domain. Explaining the task allows for processing of the skill on a cognitive level. In order to progress to the associative phase of learning, where the goal is to fine-tune the skill, one must perform and practice that skill. Given that transfer skills are very specific to the individual being transferred, it is important that practice with the patient take place. (pp. 29, 36)

A patient who had a lumbar fusion 5 years ago now has acute lumbar disc dysfunction. The patient usually has pain rated at 6/10, but pain ratings are occasionally increased to 9/10 with prolonged sitting and standing. The patient also reports pain during sit-to-stand and sit-to-supine transfers. Which of the following factors would have the MOST impact on the thoroughness of the tests and measures section of the examination? 1. Presence of pain with transitional movements 2. Past medical history of lumbar fusion 3. Moderate pain with static positioning 4. High irritability of the patient's acute back pain

4. High irritability of the patient's acute back pain 1. Range of motion measurements can provide accurate information even though the patient is having problems with transitional activities. Pain with testing of range of motion provides information about the dysfunction that is present. 2. Past history of lumbar fusion is not directly related to the current intensity of symptoms and would not limit accuracy of measurements. 3. If a position causes pain so intense the patient cannot maintain the position, a thorough evaluation can be performed by avoiding problematic positions. 4. Acute, highly irritable pain will cause limitations in the performance of evaluation tests and the accuracy of the findings.

A patient with pain on the right side of the face also has tenderness at the right temporomandibular joint. A physical therapist notes deviation of the mandible toward the left at the end of the available range of mouth opening. Which of the following additional findings would be MOST expected? 1. Abnormal jaw reflex 2. Indentation behind the left condyle of the mandible 3. Paresthesia in the facial nerve (CN VII) distribution on the right 4. Hypermobility of the right temporomandibular joint

4. Hypermobility of the right temporomandibular joint 1. The jaw jerk reflex is used to test the trigeminal nerve (CN V), but the question does not indicate neurological deficits (p. 1284). 2. The left temporomandibular joint is not hypermobile, so an indentation would not be expected (pp. 1278-1279). 3. Facial nerve paresthesias are not expected with temporomandibular joint hypermobility. The temporomandibular joint is primarily supplied by three nerves that are part of the mandibular division of the trigeminal nerve (CN V). (p. 1266) 4. A deviation during opening is associated with hypomobility toward the temporomandibular joint deviation and hypermobility contralaterally (pp. 1278-1279).

A patient has a cerebellar dysfunction that affects both the trunk and the limbs. During examination, which of the following findings will MOST likely be observed? 1. Bradykinesia 2. Festinating gait 3. Impairments in muscle force generation 4. Impairments in the timing of muscle activation

4. Impairments in the timing of muscle activation 1. Bradykinesia is associated with Parkinson disease and is not seen with cerebellar disorders (pp. 565-570). Parkinson disease affects the basal ganglia and not the cerebellum (p. 538). 2. Festination of gait is typically seen with persons living with Parkinson disease (p. 540), not persons with cerebellar ataxia. Parkinson disease affects the basal ganglia and not the cerebellum (p. 538). 3. Cerebellar impairments may cause some hypotonia initially, but patients recover normal passive muscle tone and normal reflexes quickly (p. 566). 4. Cerebellar dysfunction is characterized by delays in initiating and timing of movements. Ataxia is a general term used to describe abnormal coordination of movements. It is demonstrated by deficits in speed, amplitude of displacement, directional accuracy, and force of movement (pp. 565, 567). Ataxic movements are thought to occur due to impairments in the timing and duration of muscle activation.

A physical therapist is examining a patient and finds that when the patient's feet are together in standing position, the left iliac crest appears lower than the right. However, when the feet are spread apart, the iliac crests are level. This discrepancy is MOST likely caused by tightness of which of the following muscle groups? 1. Right hip abductors 2. Left hip adductors 3. Right hip adductors 4. Left hip abductors

4. Left hip abductors 1. Tight right hip abductors would not pull the left hip downward; they would tend to raise the left iliac crest. 2. With feet together, there is slack on the hip adductors, so there should not be any effect on the iliac crest height. 3. With feet together, there is slack on the hip adductors, so there should not be any effect on the iliac crest height. 4. The iliac crests are level with the hips abducted (feet spread apart), but the right iliac crest is higher when the hips are adducted (feet together). Tightness in the left hip abductors would be the most likely cause, because the tight left hip abductors would be pulling the left hip downward.

A 10 year-old patient is home from summer camp with generalized right knee pain. His mother reports to the Physical Therapist that he has muscle pain, cramps, disruptive sleep patterns, frequent headaches, and unexplained arthritic pain in his right knee. The Physical Therapist observes a unilateral right eyelid drooping. What pathology should the Physical Therapist suspect? 1. Rheumatoid arthritis 2. Syphilis 3. Osteoarthritis 4. Lyme disease

4. Lyme disease Answer: -Like syphilis, Lyme disease can be described as an imitator, as Lyme disease signs and symptoms mimic those of many other diseases. Symptoms vary widely and may not develop for as long as one month after a bite; in some cases, symptoms do not develop at all. Clinical manifestations of the infection occur in three stages. -Stage 1: the early, localized stage, usually occurs within 5 to 14 days following a tick bite. Flu-like symptoms suggestive of early dissemination such as fatigue, chills, fever, headache, lethargy, myalgias, or arthralgias may also develop early in the course of the infection and may be the only symptom for patients without a rash. -Stage 2: disseminated infection, occurs within weeks to months after the tick bite, particularly to the skin, nervous system, heart, and joints. Skin manifestations include multiple, smaller erythema migraines, similar to the original rash. Neurologic symptoms may be the first to arise and occur in 15% of all cases, most commonly manifested as aseptic meningitis with mild headache, stiff neck, and difficulty with mentation; cranial neuropathies, particularly Bell palsy (involvement of cranial nerve VII); and radiculopathies. Even if untreated, neurologic symptoms may improve or resolve. -Stage 3: late persistent infection, may become apparent months to years after the initial infection. In the United States approximately 60% of individuals who are left untreated develop stage 3 joint symptoms characterized by intermittent or chronic monoarticular (one joint) or oligoarticular (affecting only a few joints) arthritis. This type of arthritis is associated with marked pain and swelling, especially in the larger joints, such as the knees.

Which of the following interventions are components of complete decongestive therapy (CDT) for patients who have lymphedema? 1. Manual lymphatic drainage, iontophoresis, and aquatic exercises 2. Manual lymphatic drainage, ultrasound, and decongestive exercises 3. Manual lymphatic drainage, electrical stimulation, and aquatic exercises 4. Manual lymphatic drainage, compression therapy, and decongestive exercises

4. Manual lymphatic drainage, compression therapy, and decongestive exercises 1. Complete decongestive therapy (CDT) consists of restoration of the skin with follow-up skin care, manual lymph drainage, compression therapy, and decongestive exercise therapy, not iontophoresis. 2. Complete decongestive therapy (CDT) consists of restoration of the skin with follow-up skin care, manual lymph drainage, compression therapy, and decongestive exercise therapy, not ultrasound. 3. Complete decongestive therapy (CDT) consists of restoration of the skin with follow-up skin care, manual lymph drainage, compression therapy, and decongestive exercise therapy, not electrical stimulation. 4. Complete decongestive therapy (CDT) consists of restoration of the skin with follow-up skin care, manual lymph drainage, compression therapy, and decongestive exercise therapy.

A patient is referred to physical therapy after a quadriceps contusion. After good initial progress with rehabilitation, the patient reports sharp pain of the anterior thigh and progressive loss of knee flexion. Which of the following conditions is MOST likely present? 1. Avascular necrosis of the femur 2. Osteochondritis dissecans 3. Slipped capital femoral epiphysis 4. Myositis ossificans

4. Myositis ossificans 1. Avascular necrosis of the femur causes hip pain due the lack of blood to the femoral head. There is usually an insidious onset without trauma to the quadriceps. (pp. 251-252) 2. Osteochondritis dissecans is a necrotic bone lesion with no known cause. The knee, talus, and elbow can be involved. (pp. 1091-1092) 3. Slipped capital femoral epiphysis occurs when the capital femoral epiphysis becomes displaced. The patient is usually an adolescent male with limited hip range of motion and hip and thigh pain. This condition is not associated with quadriceps contusion. It is characterized by a sudden or gradual displacement of the femoral neck from the capital femoral epiphysis, while the head remains in the acetabulum. (pp. 1464-1466) 4. Myositis ossificans is a complication of quadriceps contusion and is caused by heterotropic bone formation on the femur. During quadriceps contraction, the muscle belly rubs across the bone, causing the sharp pain. (pp. 731-732)

A patient reports a recent significant weight loss as a result of diarrhea and vomiting. The patient is MOST at risk for which of the following conditions? 1. Bradycardia 2. Euvolemia 3. Shortness of breath 4. Orthostatic hypotension

4. Orthostatic hypotension 1. Dehydration is likely owing to fluid loss associated with emesis and diarrhea. Tachycardia is more likely with dehydration. 2. Euvolemia is an indication of normal fluid balance, which is not likely to be present in a patient who has weight loss associated with diarrhea and vomiting. 3. Shortness of breath is not associated with body fluid loss. Significant postural hypotension results from extracellular fluid volume depletion. 4. Body fluid loss associated with weight loss, excessive diarrhea, and vomiting may cause electrolyte imbalances, which can result in orthostatic changes in blood pressure.

The parent of a 2-year-old child reports that while holding the child's hand when walking down a street, the child fell down. The parent tried to hold the child up while the child regained balance. The child now holds the arm at the side with the palm facing down. Which of the following conditions is MOST likely present? 1. Radial tunnel syndrome 2. Necrosis of the capitellum 3. Valgus extension overload syndrome 4. Partial slippage of the annular ligament

4. Partial slippage of the annular ligament 1. In radial tunnel syndrome, there is pain over the extensor muscle mass of the forearm distal to the lateral epicondyle. In addition, long finger extension testing would produce pain, and there could be weakness of finger and thumb (1st digit) extensors and the extensor carpi ulnaris. Typically, this syndrome results from overuse and not from a traumatic event. 2. Necrosis of the capitellum typically affects the dominant elbow of children, mainly boys, between the ages of 5 and 10 years. It is a diagnosis that is often associated with throwing in a young child (i.e., little-league elbow). 3. It is highly unlikely that a 2-year-old child would have valgus extension overload syndrome, because it typically is seen in throwing athletes in whom the repetitive stresses of throwing lead to progressive changes within the elbow joint, which cause pain and athletic impairment. Findings include pain and tenderness around the tip of the olecranon, pain with forced passive elbow extension, and increased valgus laxity. 4. A diagnosis of partial slippage of the annular ligament is made from the history of longitudinal traction on an extended elbow, as with a child whose arm is pulled by a parent when walking. This injury typically occurs in children age 2-3 years.

An 18-year-old patient who has osteogenesis imperfecta has been receiving physical therapy services through the school system. The patient is independent in transfers, wheelchair mobility with a motorized wheelchair, and self-care and has been utilizing public transportation. Which of the following courses of action is MOST appropriate in planning for the patient? 1. Continue physical therapy services with a modified goal to maintain independence with mobility. 2. Continue physical therapy services under the care of a physical therapist assistant. 3. Recommend that the patient participate in a support group. 4. Refer the patient to vocational services.

4. Refer the patient to vocational services. 1. The patient has achieved the goals necessary for independent mobility, so there is no need to continue with a mobility goal. 2. Continuing physical therapy services, even by a physical therapist assistant, is inappropriate when the patient has achieved the goals and expected prognosis at this stage of life. 3. There is no indication in this question that the person requires any social support. 4. This 18-year-old patient is independent for activities of daily living and is developmentally appropriate for the workforce or further academic study. A vocational counselor will provide the best guidance and support for the next setting/stage.

A 4-year-old child who received a diagnosis of spinal muscular atrophy at age 9 months is referred for home physical therapy. The child is unable to sit without upper extremity support but rolls independently. The child has bilateral hip and knee flexion contractures that make use of the child's standing frame uncomfortable. Which of the following interventions are MOST appropriate for physical therapy? 1. Teach parents airway clearance techniques, encourage upper extremity strengthening to prepare for wheelchair self-propulsion, and switch to using a modified prone stander. 2. Teach parents lower extremity stretching and strengthening exercises, adapt the standing frame to accommodate contractures, and encourage supported walking. 3. Teach parents proper transfers, facilitate upright positioning in kneeling and standing positions, and refer to an orthopedist for serial casting to address contractures. 4. Teach the parents range of motion exercises and positioning, encourage play in prone and sitting positions, and order customized seating insert for a power wheeled mobility device.

4. Teach the parents range of motion exercises and positioning, encourage play in prone and sitting positions, and order customized seating insert for a power wheeled mobility device. 1. Upper extremity strengthening is not realistic, considering the progressive weakness expected with this child. The skill level would be too high. Manual propulsion is not functional over the long term. 2. The literature supports that children who have spinal muscular atrophy and who do not develop sitting ability are unlikely to walk and will require power mobility. They may become independent in a power wheelchair by age 1-2 years. 3. These skills are too advanced. The literature supports that children who have spinal muscular atrophy and who do not develop sitting ability are unlikely to walk and will require power mobility. 4. These interventions are most appropriate to the patient's impairments and functional limitations. The literature supports that children who have spinal muscular atrophy and who do not develop sitting ability are unlikely to walk and will require power mobility.

A physical therapist is teaching a patient and her husband about positioning for prevention of contractures. The husband becomes agitated and states that he is unable to help his wife. The therapist is MOST likely to facilitate effective communication with the husband by: 1. offering to leave and come back later when the patient and her husband are calmer. 2. apologizing and asking the husband to repeat what he said. 3. asking the husband to put his concerns in writing so that the problems can be better understood. 4. verbalizing the husband's feelings and asking him if his intended meaning was reflected correctly.

4. verbalizing the husband's feelings and asking him if his intended meaning was reflected correctly. 1. Avoidance is not recommended. Active listening at the time of intense emotion is an effective means of diffusing emotion. 2. Apologies do not help to diffuse emotion-laden exchanges. 3. Listening skills are needed to diffuse emotion. Writing the problem down will not help diffuse the agitation. 4. Active listening/reflection, rather than reacting to the person's words, helps to clarify what the person is saying.

A PT is performing cranial nerve testing on a 58 year old male patient. The patient has ptosis, lateral strabismus, and a dilated pupil in the right eye. Which of the following cranial nerve test results is MOST likely to be abnormal? A. Oculomotor nerve (CN III) B. Optic nerve (CN II) C. Facial nerve (CN VII) D. Trigeminal nerve (CN V)

A. Oculomotor nerve (CN III) CN3- Ptosis, Lateral strabismus and loss of pupillary reflex CN 3 supplies the medial rectus which if absent causes lateral strabismus supplies most of eye muscles except superior oblique (CN 4) and lateral rectus (CN 6) LR6SO4O3 The letters represent the extraocular muscles and numbers represent their respective cranial nerve supply: LR6: lateral rectus, innervated by the 6th (abducens) nerve SO4: superior oblique, innervated by the 4th (trochlear) nerve O3: other muscles (superior, inferior, medial recti and inferior oblique), innervated by the 3 rd (oculomotor) nerve

A patient presents to a clinic with Subacromial impingement syndrome. The PT chooses to use ultrasound for deep heating of the rotator cuff muscles. Which set of US parameters will be MOST effective at increasing the temperature of tissue 4.0 centimeters deep? A. 1.0 MHz, 100% duty cycle with 5 cm sound head B. 3.0 MHz, 100% duty cycle with 2 cm sound head C. 1.0 MHz, 50% duty cycle with 2 cm sound head D. 3.0 MHz, 50% duty cycle with 5 cm sound head

A. 1.0 MHz, 100% duty cycle with 5 cm sound head 1 Mhz for deep (3 cm or more is deep) 3 Mhz for superficial (<3 is superficial) you want continuous US to heat so duty cycle should be ON at all times so you want 100% duty cycle 1 MHz: 1 looks like a SWORD. Think of stabbing your food with a SWORD, it goes DEEP. (2 cm or more) 3 MHz: 3 looks like a FORK. Think of stabbing your food with a FORK, it goes SUPERFICIAL. Intensity 1.5 Watts —> Chronic Less than 0.8 Wats —> Acute Freq: -3 Megahertz = Superficial -1 Megahertz = Deep -Continuous: 100% = Chronic -Pulsed: 20% = Acute US Head: -2 cm = Smaller areas: hand/fingers -5 cm = Standard (regular areas)

A 82-year-old patient suffered a right sided CVA about a week ago. The therapist is educating him on various positioning strategies. Which of the following is the MOST appropriate while lying on the left side? A. Head/neck: neutral, left Scapular protracted; L arm in slight abduction and external rotation; elbow extended, forearm supinated, wrist neutral, fingers extended, and thumb abducted. B. Head/neck: neutral, left Scapular retracted; L arm in slight abduction and internal rotation; elbow extended, forearm pronated, wrist neutral, fingers extended, and thumb adducted C. Head/neck: neutral, right Scapular retracted; L arm in slight abduction and internal rotation; elbow extended, forearm pronated, wrist neutral, fingers extended, and thumb adducted D. Head/neck: neutral, left Scapular protracted; L arm in slight adduction and external rotation; elbow flexed, forearm supinated, wrist extended, fingers flexed, and thumb abducted.

A. Head/neck: neutral, left Scapular protracted; L arm in slight abduction and external rotation; elbow extended, forearm supinated, wrist neutral, fingers extended, and thumb abducted. Key words: scap protracted elbow extended

A 32-year-old female patient arrived at a clinic with the following skin disorder. Which of the following is the MOST appropriately describe these findings? (look at slide 15 FF #14) A. Herpes simplex virus type 1 B. Herpes Zoster C. Acne vulgaris D. Herpes Simplex type 2

A. Herpes simplex virus type 1 (cold sore) type 1 is ABOVE THE WAIST LINE (usually on the face/lip area) type 2 is BELOW THE WASTLINE Herpes simplex is preceded by itching and soreness followed by vesicular eruptions. A good example is a cold sore. Ans: A Rationale: Cold sores caused by HSV-1 (also known as recurrent herpes labialis; "fever blister") are found on the lip or the skin near the mouth. HSV-1 generally only infects areas above the waistline and occurs when oral secretions or mucous membranes infected with HSV come in contact with a break in the skin (e.g., torn cuticle, skin abrasion). HSV 2, also known as "genital herpes" can cause cold sores but usually does not. Shingles is an outbreak of a rash or blisters (vesicles with an erythematous base) on the skin that may be associated with severe pain. The pain is associated with the involved nerve root and associated dermatome and generally presents on one side of the body or face in a pattern characteristic for the involved site. Acne vulgaris is the formation of comedones, papules, pustules, nodules, and/or cysts as a result of obstruction and inflammation of pilosebaceous units

A PT is training a 65 year old to walk with a quadripod (4 point cane). He had severe arthritis of the R hip and was operated 2 months ago. What is the MOST appropriate way the PT should instruct the patient to use the assistive device? A. Hold the cane in your L hand and move L hand and R leg together B. Hold cane in your R hand and move R hand and L leg together C. Hold cane in your L hand and move L hand and L leg together D. Hold cane in your R hand and move R hand and R leg together

A. Hold the cane in your L hand and move L hand and R leg together What type of Point Gait is this? -2 Point Gait

A PT Aide was treating an CVA patient and asked the patient to complete bridging exercises that were not in the original POC. During the bridging exercises, the patient suffered a back injury. Who will need to take the responsibility for the injury? A. Physical therapist B. Physical therapy assistant C. Physical therapy aide D. Physical therapy student

A. Physical therapist Licensed person has to take the responsibility. A PT aide does not have a license and works under the PT. If the question was phrased as "a PTA was treating" than it would be the PTA that takes the responsibility because they're licensed

PT finds that right ASIS is higher than the left ASIS, and right PSIS is lower than the left PSIS. While performing the long sitting special test, the right limb is shorter in supine and appears to get longer in sitting. Based on the examination, what is the MOST appropriate treatment: A. Stretch the Right Hip Extensors B. Stretch the Right Hip Flexors C. Strengthen the Right Hip Flexors D. Strengthen the Right Hip Extensors

A. Stretch the Right Hip Extensors

A patient present to a clinic with complaints of dizziness and vertigo in some positions. On examination, the PT found that pt has his head tilted to one side, conjugate eye torsion, and an abnormal weight shift to one side. These signs are MOST consistent with what condition? A. Unilateral vestibular hypofunction B. Otitis media C. Ototoxicity D. Malingering

A. Unilateral vestibular hypofunction -head tilt and abnormal weight shift to one side is common in UVH pts -C not correct because they are not talking about chemicals or antibiotics -B not correct because they don't talk about the middle ear -D not correct because malingering is a psychological c/o pain

A patient presents with difficulty maintaining balance especially when walking late at night in her house on carpet. She reports feeling dizziness and visual blurriness when scanning the room to the left and right to navigate her surroundings. The Physical Therapist should MOST likely suspect the patient has which of the following diagnosis? A. Vestibular Hypofunction B. Horizontal canalithiasis C. Vestibular Hyperfunction D. Otitis media

A. Vestibular Hypofunction B not correct because it doesn't mention anything about nystagmus

Which of the following is LEAST likely to be followed with respect to contact isolation precautions in a patient with Clostridium difficile infection? A. Wearing N95 masks B. Washing hands with chlorhexidine gluconate soap upon leaving the room. C. Wearing gloves upon entering the room D. Wearing a gown when in direct contact with the patient.

A. Wearing N95 masks Contact: use gloves and gown Droplet: use mask Airborne: use N-95 mask

A PT is teaching a patient with Stage 5 Parkinson disease on how to wheelchair transfer to the commode. The patient has very low endurance. What motor learning strategy would work best for this patient to learn this task? A. Random order and distributed practice B. Blocked and distributed practice C. Random and massed practice D. Blocked and massed practice

B. Blocked and distributed practice Stage 5 -cognitive issues -low endurance so they need more rest Stage 0 -No signs of disease Stage 1 -Unilateral disease Stage 2 -Bilateral disease, without impairment of balance Stage 2.5 -Mild bilateral disease, with recovery on pull test Stage 3 -Mild to moderate bilateral disease; some postural instability; physically independent Stage 4 -Severe disability; still able to walk or stand unassisted Stage 5 -Wheelchair bound or bedridden unless aided

A 54-year-old patient reports to an outpatient clinic with history of insidious onset of pain and paresthesias in the medial forearm and hand. Which of the following is the MOST likely diagnosis? A. C6 radiculopathy B. C8 radiculopathy C. Median nerve entrapment D. Radial nerve entrapment

B. C8 radiculopathy -The radial nerve distribution covers the dorsal and lateral hand. The lateral forearm and hand represents the C6 dermatome. Medial forearm is covered by C8 dermatome

A physical therapist is developing an exercise program for a patient who has upper extremity lymphedema. Which of the following exercises should the patient perform FIRST? A. Elbow flexion B. Cervical rotation C. Shoulder abduction D. Shoulder circumduction

B. Cervical rotation start most proximal Exercise: (Perform first) Cervical rotation —> Shoulder circumduction —> Shoulder abduction —> elbow flexion —> Wrist flexion (perform last)

A 68 year old patient is performing a Bruce protocol on an inclined treadmill. The PT is monitoring the patient using ECG leads. During the exercise, the PT sees the first degree AV block ECG pattern shown. The PT's BEST response should be A. Stop the treadmill session immediately and call the cardiologist B. Continue without any modifications and monitor ECG C. Reduce the treadmill speed and monitor ECG D. Stop the treadmill, have the patient rest, and then resume at a lower intensity

B. Continue without any modifications and monitor ECG No need to stop exercise due to 1st degree AV block isn't as serious C is not correct because Bruce protocol is training the maximum capacity of the heart, to reduce the treadmill speed would be incorrectly performing the Bruce protocol (all in or none)

In establishing an exercise program for a patient with type 2 diabetes, the therapist should be aware that regular exercise should: A. Decrease blood glucose levels and decrease the insulin sensitivity B. Decrease blood glucose levels and increase the insulin sensitivity C. Increase blood glucose levels and decrease the insulin sensitivity D. Increase blood glucose levels and increase the insulin sensitivity

B. Decrease blood glucose levels and increase the insulin sensitivity Sensitivity in terms of insulin and diabetes means how effective it is. The higher the sensitivity the more effective insulin is. The dosage/amount of insulin will be lowered if there is a higher sensitivity

A PT is working on rehabilitating a 29 year old ACL repair patient using aquatic therapy. The patient is immersed to the level of sternoclavicular notch. Which of the following is the MOST expected response during aquatic therapy A. Decreased cardiac output B. Decreased systolic blood pressure C. Increased heart rate D. Increased V02 max

B. Decreased systolic blood pressure Relaxed/happy in the water: -HR decreases -stress free so BP decreases -CO increases -SV increase: heart is more efficient so it increases more blood

A PT is reviewing the lab values of a 58 year old male patient. Which of the following about glucose monitoring is LEAST appropriate? A. Hypoglycemia is documented for fasting blood glucose value of 40mg/dL B. Diabetes mellitus is when fasting blood glucose levels are between 80 and 110 mg/dL measured on 2 separate days C. Glycosylated Hemoglobin normal reference range is 4-6% D. A1C level above 10% requires immediate insulin therapy

B. Diabetes mellitus is when fasting blood glucose levels are between 80 and 110 mg/dL measured on 2 separate days -Not a correct way to dx DM -blood glucose levels between 80-110 is normal -correct dx of DM = >125 blood glucose level on 2 separate days Impaired glucose tolerance-100-125mg/dL Glycosylated Hemoglobin normal reference range- 4-6% A1C level above 10% require immediate insulin therapy

A 65 year old patient taking levodopa exhibits typical end-of dose akinesia during exercise. Which of the following courses of action is NOT appropriate for a physical therapist? A. Schedule physical therapy for 1 hour after antiparkinsonian medications are given. B. Discontinue physical therapy sessions until medications are modified. C. Educate the patient on effects of medication and encourage regular exercise. D. Maintain regular exercise intensity to avoid complication of the Parkinson's disease.

B. Discontinue physical therapy sessions until medications are modified. -Don't have to stop exercise DYSKINESIA when ON the medication. - dysk"I"Nesia: "I"n the vial you have medication, so you are ON the medication. DYSTONIA when OFF the medication. - dyst"O"nia: "O"ut of medication in the vial, so off the medication.

A 54 year old patient presents to an outpatient clinic after a partial rotator cuff repair. Which of the following describes the optimal parameters for duty cycle and MVIC to address the injured rotator cuff muscles? A. Duty cycle of 30%; current intensity to produce 30% MVIC B. Duty cycle of 20%; current intensity to produce 40% MVIC C. Duty cycle of 60%; current intensity to produce 40% MVIC D. Duty cycle of 80%; current intensity to produce 20% MVIC

B. Duty cycle of 20%; current intensity to produce 40% MVIC -Duty cycle should never be over 50% for FES because the on period should always be less than the off period, so eliminate C/D -40% MVIC is the better answer because we want to produce the most MVIC as possible -40% should be the max MVIC for an injured muscle, we don't want 100% MVIC because it's too aggressive

The patient's medical history includes hypercholesterolemia, and type 2 diabetes. The patient's SBP is between 120 129 and DBP is less than 80. Which of the following categories MOST appropriately describes the type of hypertension? A. Normal B. Elevated C. Pre hypertension D. Stage 1

B. Elevated Normal: SBP<120/DBP<80 Elevated: SBP between 120-129 AND DBP <80 Stage 1: SBP between 130-139 OR DBP between 80-89 Stage 2: SBP at least 140 OR DBP at least 90 Hypertensive crisis: SBP over 180 AND/OR DBP over 120

A 58 year old patient has chronic obstructive pulmonary disease. Which of these pulmonary test results will NOT be increased when compared with those of a 58 year old healthy individual? A. Total lung capacity. B. FEV1/FVC ratio. C. Residual volume. D. Functional residual capacity.

B. FEV1/FVC ratio. The FEV1/FVC ratio is the amount of air exhaled in the first second divided by all of the air exhaled during a maximal exhalation.

In PT session, the therapist commands the patient to wear a shirt, but the patient is unable to complete the task due to inability to find buttons on his shirt. Which of the following is the MOST APPROPRIATE diagnosis and the best possible strategy to address this deficit? A. Form discrimination, patient should be encouraged to use touch to identify objects. B. Figure-ground discrimination, patient should be encouraged to use touch to identify objects. C. Position in space impairment, orienting the patient with one object in relation to another D. Ideational apraxia, having the patient perform one part of the task at a time and guiding the patient with the task if needed.

B. Figure-ground discrimination, patient should be encouraged to use touch to identify objects. -Goal is to distinguish between different objects (button from shirt) ex: pt should know what its something they have to do in terms of discriminating between 2 textures Form discrimination—> similar objects such as pens, but they have different thickness -Goal is to distinguish between the same objects with different textures of the same objects (different pens/balls)

The physical therapist is performing the shown technique on a patient to treat his vertigo. What would be the most likely diagnosis for this patient? (image on slide 13 FF#22) A. Posterior semicircular Canal BPPV in Right Ear B. Horizontal semicircular Canal BPPV in left Ear C. Horizontal semicircular Canal BPPV in Right Ear D. Posterior Semicircular Canal BPPV in Left Ear

B. Horizontal semicircular Canal BPPV in left Ear -BBQ roll treatment for horizontal canalithiasis, and they roll the pt to the Left initially, treating the Left side

A PT is treating a pediatric patient with cerebral palsy. The patient is seen in standing with a toe in posture. Which of the following postural strategies most accurately correlates with the observed foot position? A. Internal tibial torsion, increased femoral retroversion B. Internal tibial torsion, increased femoral anteversion C. External tibial torsion, increased femoral retroversion D. External tibial torsion, increased femoral anteversion

B. Internal tibial torsion, increased femoral anteversion

A 28 year old professional mountain bike rider presents to cardiac rehabilitation after suffering a MI 6 months ago. Which exercise test would be MOST beneficial in order to assess aerobic capacity and create an aerobic conditioning program? A. Bruce protocol B. Lower extremity ergometry C. Six Minute Walk test D. Step test

B. Lower extremity ergometry Key words: bike rider want to use a functional test (bike rider) Bruce protocol: helps achieve VO2 max -pt is on treadmill and you increase the speed and incline every 5 minutes (you can think of Bruce Wayne (batman) increasing his speed and climbing buildings (increasing incline) until he can't anymore) 6 MWT: good for identifying pt's endurance

A physical therapist transfers a 50 year old patient from the commode to a wheelchair. The physical therapist provides 30% of the effort to complete the transfer. Which of the following levels of physical assistance should be documented? A. Minimal assist B. Moderate assist C. Maximal assist D. Dependent

B. Moderate assist -pt is putting in 70% min = greater or equal to 75% mod = 50-74% max = 25-49% dependent = <25%

A PT measures leg length on a patient with low back pain. While supine, the patient's right leg is 2 cm shorter than the left. Upon moving into long sitting the right leg becomes longer than the left. Which of the following interventions is the MOST appropriate? A. Muscle energy technique to facilitate right innominate posterior rotation B. Muscle energy technique to facilitate right innominate anterior rotation C. Muscle energy technique to facilitate left innominate anterior rotation D. Backwards lunge stepping posterior with left lower extremity

B. Muscle energy technique to facilitate right innominate anterior rotation -the R innominate is in posterior rotation so we need to promote right innominate rotation shorter in supine = posterior pelvic tilt ALPS in supine -Anterior Long = excessive anterior innominate rotation when LE is longer in supine -Posterior Short = excessive posterior innominate rotation when LE is short in supine

The 54 year old male patient's chart states that they have been taking beta blockers for the past 5 years. Prior to starting an exercise training program, the patient should receive an explanation of the: A. Greater benefits from cardiovascular exercise to be achieved at lower SBP rather than at higher SBP levels. B. Need to use measures other than heart rate to determine intensity of exercise. C. Greater benefits from cardiovascular exercise to be achieved at lower HR than at higher HR levels. D. Need for longer warm up periods and cool down periods during exercise sessions.

B. Need to use measures other than heart rate to determine intensity of exercise.

A 63 year old suffered from a CVA 3 month ago. During ascending stairs, the patient can keep the involved foot on the step above but is unable to transfer the weight to the next stair level due to weak quadriceps. The BEST intervention to solve the problem is A. Bridging, holding B. Standing, partial wall squats C. Supine, knee flexion with hip extension D. Standing, side steps

B. Standing, partial wall squats

A 44 year old male patient is being evaluated by a physical therapist. The patient underwent a hernia repair 3 weeks ago and should AVOID which of the following activities? A. Walking at a metabolic equivalent of 4 after 3 weeks after surgery B. Stretching of the anterior spinal and hip musculature before the incision is fully healed C. Stretching of the posterior spinal and hip musculature before the incision is fully healed D. Wall sits performed in an upright position

B. Stretching of the anterior spinal and hip musculature before the incision is fully healed

All of the following options are exercise considerations when planning a treatment session for a patient with Multiple Sclerosis EXCEPT A. time of day the appointment is scheduled B. exercise at maximal intensity C. have an appropriate balance of exercise and rest D. fatigue level

B. exercise at maximal intensity -Sub-max exercise is ideal for MS pts -Always consider HEAT SENSITIVITY AND FATIGUE with MS pts

To prevent contractures in a patient with an above knee transfemoral ) amputation, emphasis should be placed on designing a positioning program that maintains range of motion in hip: A. flexion and abduction. B. extension and adduction. C. adduction and lateral (external) rotation. D. flexion and medial (internal) rotation

B. extension and adduction.

Abnormal End Feels

Bone to Bone ( Bony) End Feel -occurs when one would not expect to find a bone to bone end feel - hard, unyielding end feel. Restriction occurs before the normal end of range movement caused by osteophytes, degenerative joint disease, mal-union of joint following a fracture etc. Muscle Spasm End Feel -sudden and hard dramatic arrest of movement accompanied by pain which is invoked by the movement. Springy, rebound end feel -Reflexive (protective) muscle guarding designed to prevent further injury. Empty End Feel -No physical restriction to the movement but with considerable pain. Maybe full range of motion but with pain. Example: acute bursitis, joint inflammation. Springy Block End Feel (Internal Derangement) -springy or rebound sensation in a non-capsular pattern. Usually occurs before the end of the normal ROM. Example: loose cartilage, meniscal tissue within joints (eg. torn meniscus won't be able to extend knee fully) Leathery End Feel (Capsular Stretch End Feel) -similar to tissue stretch but occurs when the ROM is reduced. -Hard Capsular: thick quality and the limitation comes on abruptly. -Soft Capsular: more often seen with acute conditions-stiffness occurs early in the range, increasing until the end of the range is reached. Boggy or Soft End Feel -occurs if you have a joint effusion or edema - mushy with soft quality to it -may indicate acute inflammation. Example: acute moderate to severe sprain, ligamentous injury

A PT is educating one of their patients regarding the effects of pregnancy and the implication for positioning and posture. Which of the following is the LEAST appropriate recommendation by the therapist? A. Patient with pregnancy induced hypertension must not do high intensity exercises B. Breath holding exercising, and Valsalva's maneuver should be avoided C. A small wedge must be used under the left hip while sleeping in supine in the second semester D. Borg's scale score of 12-14 during exercise is acceptable for uncomplicated pregnancy

C. A small wedge must be used under the left hip while sleeping in supine in the second semester Rationale: -For supine exercise, place a small wedge or rolled towel under the right hip to lessen the effects of uterine compression on abdominal vessels and to improve cardiac output. The wedge turns the patient slightly toward the left. (Right sidelying causes compression of IVC leading to decreased CO) -Conventional (age-based) target heart rate zones may be too aggressive for the average pregnant patient. -Use of the Borg scale of perceived exertion is more appropriate in this population, with exertion between 12 and 14 suggested during uncomplicated pregnancy. -Therapist must discourage breath-holding and avoid activities that tend to elicit Valsalva's maneuver because this may lead to un-desirable downward forces on the uterus and pelvic floor. In addition, breath-holding causes stress to the cardiovascular system in terms of blood pressure and heart rate. -Absolute contraindications to exercising during pregnancy include pre-eclampsia, type 1 diabetes, intrauterine growth retardation. 11-13 on BORG is normal for Pregnancy. 12-14 is acceptable (close enough to the normal range) Borg. SHVEM—> 13= somewhat hard, 15= hard, 17= very had, 19= extremely hard, 20= maximal exertion

A male patient complains of having significant weight gain in abdomen and in the face. The labs show high levels of cortisol and elevated blood glucose and high levels of ACTH coming from pituitary gland. The patient MOST LIKELY has this diagnosis: A. Addison Disease B. Hasimoto's Disease C. Cushing's Disease D. Parathyroidism

C. Cushing's Disease Cushion/bigger/moon face appearance/gain weight Addison Disease = decreased cortisol/ Hashimoto's Disease = hypothyroid issue Cushing Disease is the diagnosis as the affect is coming from the pituitary gland. The patient experiences high levels of cortisol and elevated blood glucose levels. The patient demonstrates having a distended abdomen, weight gain in trunk and face. Cushings syndrome results as the affect mainly comes from the adrenal gland. The adrenal gland releases excess amount of cortisol causing similar signs and symptoms as cushing disease which include: distended abdomen, weight gain in abdomen and face, bruises and stretch marks, osteoporosis, high levels of cortisol and elevated blood glucose levels. A patient with addison disease has low levels of cortisol released from the adrenal gland (aka primary adrenal insufficiency). Addison's disease is caused by an autoimmune response which attacks the outer cortex of the adrenal gland. Some signs and symptoms of addison's disease include: abdominal pain, abnormal menstrual periods, dehydration, depression, diarrhea, dizziness, loss of appetite, hypoglycemia, hypotension, muscle weakness, nausea, patches of dark skin around scars/skin fold and joints, sensitivity to cold, unexplained weight loss, weight loss, worsening fatigue. Hashimotos disease occurs from when the immune system attacks the thyroid gland causing decrease function of the gland (hypothyroidism). Some signs and symptoms in hypothyroidism include: skin dryness/roughness, hair loss, constipation, depression, joint stiffness, sensitivity to cold, slow heart rate, swelling in extremities, weight gain

Active insufficiency of right iliopsoas will be caused by? A. Hip extension and right lateral flexion of the trunk B. Hip extension and left lateral flexion of the trunk C. Hip flexion and right lateral flexion of the trunk D. Hip flexion and left lateral flexion of the trunk

C. Hip flexion and right lateral flexion of the trunk

A 58 year old patient has chronic obstructive pulmonary disease. During an examination, a PT finds that a patient has a weak wet cough. Which of the following is MOST appropriate to help this patient clear secretions? A. Assisted coughing in supine position B. Postural drainage in side lying position C. Huffing D. Manual or mechanical percussion

C. Huffing Huff coughing is a gentler form of coughing because you keep an open glottis (opening between the vocal cords) while exhaling. This makes it easier to move mucus out of the lungs. A incorrect due to pt able to cough B incorrect need more information as to where the secretion may be D incorrect Assisted coughing helps with neurological patients such as SCI with a weak cough reflex. Postural drainage facilitates drainage of secretions to the level of the segmental bronchus only. In addition, a cough is needed to clear secretions. Huffing helps stabilize collapsible airways present with chronic obstructive pulmonary disease Percussion helps mobilize secretions from the periphery of the lungs; however, it does not improve the strength of the patient's cough.

A 7 years old complains of a vague ache in the groin that radiates to the medial thigh and inner aspect of the knee. On examination, the PT notices decreased abduction and internal rotation. What is the MOST appropriate intervention for this child? A. Improving the strength of abductor muscle strength using closed chain exercises B. Improving the flexibility of abductor and external rotators C. Improving the containment of the femoral head in acetabulum using the brace D. Improving the flexibility of adductor and internal rotators

C. Improving the containment of the femoral head in acetabulum using the brace Rationale: Condition is Legg-Calve-Perthes disease An important aspect in the intervention is the containment of the femoral head in the acetabulum. This is ensured by maintaining the hip in abduction and mild internal rotation for an extended period, using the Atlanta Scottish-Rite Hospital Orthosis. Although this brace does not use internal rotation-like casts as some other orthotics do, it does maintain the hip in abduction and permits approximately 90 degrees of hip flexion Physical therapy may be provided at home, school, or in the clinic. Treatment methods include observation only; ROM exercises in all planes of hip motion (especially internal rotation and abduction); bracing; casting; Gait training with brace, aquatic therapy.

A physical therapist observes gait of a 34 year old male patient. The PT suspects a leg length discrepancy. Which of the following gait deviations is MOST likely seen by the therapist? A. Increased dorsiflexion of the short limb during swing and increased plantar flexion of the long limb during stance B. Decreased knee flexion and increased dorsiflexion of the long limb during stance and increased dorsiflexion of the short limb during swing C. Increased dorsiflexion with early heel rise of the long limb at heel off and increased plantar flexion of the short limb during stance D. Increased plantar flexion of the long limb at heel strike and decreased knee flexion of the short limb during heel off

C. Increased dorsiflexion with early heel rise of the long limb at heel off and increased plantar flexion of the short limb during stance Draw it out! -you'd want to increase the height of the short leg and decrease the height of the long leg

A patient demonstrated painful ROM in horizontal abduction movement during range of motion examination of shoulder. Which of the following joint mobilization techniques is MOST appropriate for the patient? A. Large amplitude oscillations performed at the beginning of the range of motion in an anterior-inferior direction B. Small amplitude oscillations into tissue resistance up to the limit of available motion in an posterior-superior direction C. Large amplitude oscillations within the available range of motion in an anterior-inferior direction D. Small amplitude oscillations into tissue resistance at the limit of available joint motion in a postero-inferior direction

C. Large amplitude oscillations within the available range of motion in an anterior-inferior direction (grade 2) Why not A? SMALL amplitude oscillations are performed at the beginning of the ROM (grade 1)

A physical therapist is treating a patient with the complains of a vague ache in the lower extremities. As shown in the picture below, what is the MOST likely structural malalignment present? (Image on slide 13 on FF#25) A. Lateral tibial torsion with lateral rotation of femur B. Medial tibial torsion with lateral rotation of femur C. Lateral tibial torsion with medial rotation of femur D. Medial tibial torsion with medial rotation of femur

C. Lateral tibial torsion with medial rotation of femur -if the ankle is going laterally, its lateral tibial torsion

A PT is evaluating an 86 years old female, who sustained a right CVA due to a MCA infarction 1 week ago. Patient has the classical signs and symptoms of MCA CVA. During the Initial evaluation, the PT performs reflex testing on the patient. PT would expect? A. Right side hyperreflexia and - Babinski B. Right side hyporeflexia and + Babinski C. Left side hyperreflexia and + Babinski D. Left side hyporeflexia and - Babinski

C. Left side hyperreflexia and + Babinski Stroke/CVA is an UMN LESION (stroking the foot = babinski)

A 44 year old male who consumes excessive amounts of alcohol is referred to PT clinic for knee pain. During treatment, patient reports right sided shoulder pain. Which of the following structures is the MOST likely source of the referred pain? A. Prostate B. Appendix C. Liver D. Kidney

C. Liver -causes referred pain to R shoulder

A PT is examining a patient in an outpatient clinic and notices swelling on patient's right foot as shown in picture. Which of the following conditions can the PT suspect? (look at image on slide 19 FF#15) A. Heart failure B. Lipidema C. Lymphedema D. Chronic Venous insufficiency

C. Lymphedema Not A because we needed to know which side heart failure (RHF would have been a correct answer for peripheral edema) Not D because need to see something at the medial aspect of the distal leg (VENMO)

A 34-year-old male has been diagnosed with a complete C5 spinal cord injury. Patient used a power chair to commute from the parking lot to the PT clinic. While performing a transfer, the MOST important consideration is to A. Make sure he is using his shoulder extensors to support his upper body B. Make sure his legs are supported by PT aide C. Make sure his power chair is turned off D. Make sure to use the sliding board

C. Make sure his power chair is turned off

A review of the patient's medical chart reveals blood pressure (BP) of 165/90, triglyceride level of 160 mg/dL, and a fasting blood glucose level of 115 mg/ dL. Patients BMI is 40 kg/m2 and with a 54-inch waistline. These findings are suggestive of: A. Chronic heart disease. B. Type 2 diabetes. C. Metabolic syndrome. D. Cushing's syndrome.

C. Metabolic syndrome. If 3/5 are above norm ranges its metabolic syndrome Metabolic syndrome is present if three or more of the following five criteria are met: -waist circumference over 40 inches (men) or 35 inches (women), -blood pressure greater or equal to 130/85 mmHg -fasting triglyceride (TG) level over 150 mg/dl -fasting high-density lipoprotein (HDL) cholesterol level less than 40 mg/dl (men) or 50 mg/dl (women) -fasting blood sugar greater or equal to 100 mg/dl. Women Be The Hot Bitches

A patient is getting treated for pelvis pain and is accompanied by his wife to the PT session. While treating, the patient expressed dissatisfaction in his martial life and says he has regular suicidal thoughts. What would be the BEST PT action? A. Ask the patient to seek an appointment with a mental health practitioner B. Disagree with the patient and say positive things about living life C. Refer the patient immediately to mental health practitioner and stay there until help arrives D. Inform the patient's wife that this problem is outside the scope of physical therapy practice

C. Refer the patient immediately to mental health practitioner and stay there until help arrives Suicide is a red flag A: is a suggestion while C: is an action

A 55 year old obese male patient is taking Lipitor (a cholesterol lowering drug). Which of the following side effects should the physical therapist be MOST concerned about? A. Easy bruising B. Sleep disturbance C. Rhabdomyolysis D. Diarrhea

C. Rhabdomyolysis -breakdown of muscle tissue that causes dark colored urine and muscle pain/soreness/stiffness Rhabdomyolysis is a serious syndrome due to a direct or indirect muscle injury. It results from the death of muscle fibers and release of their contents into the bloodstream. This can lead to serious complications such as renal (kidney) failure. This means the kidneys cannot remove waste and concentrated urine. Tip = look for the most dangerous/unique word if you have no clue with SE of drugs

An 45-year-old obese male (BMI 33kg/m2) with type 2 DM is working out on the treadmill in the PT gym. While exercise, the patient suddenly develops light-headedness, dizziness, and instability. The MOST appropriate action is: A. Stop the treadmill and call the primary care physician B. Keep the treadmill moving and have the patient drink orange juice C. Stop the treadmill and have the nurse check blood glucose levels D. Slow the treadmill speed and have the patient drink cold water

C. Stop the treadmill and have the nurse check blood glucose levels safe for the pt so stop treadmill safe for you so ask nurse to check their glucose level

A patient presents with complains of pain and numbness in the right arm extending to the thumb. The PT performs the special test as shown in the picture below. If the test is positive, which is the MOST appropriate intervention? (Look at image on slide 19 on FF#25) A. Strengthen the right levator scapulae and sternocleidomastoid muscles. B. Perform (Posterior Anterior) PA glide of the C6 vertebrae with right side bending and rotation. C. Stretch the right pectoralis minor and sternocleidomastoid muscles. D. Perform grade 3 manipulation of the first and second ribs.

C. Stretch the right pectoralis minor and sternocleidomastoid muscles. ST is Costoclavicular Syndrome (Military Brace) Test for TOS The correct answer is C. Stretch the right pectoralis minor and sternocleidomastoid muscles. The special test shown in the picture is Costoclavicular Syndrome test. A positive test is indicated by an absence of the pulse and implies possible thoracic outlet syndrome. The most appropriate intervention would be to stretch the pectoralis muscles, SCM and scalenes. Mobilizing can also be used to decrease muscle guarding and pain but Grade V (not grade III) manipulation of the first rib (not 2nd) will be the appropriate intervention. D - Costoclavicular syndrome(Military brace ) test:-Subclavian artery and vein and, the brachial plexus are compressed between the clavicle and first rib. The first rib elevate against the clavicle. Restoring mobility to the first rib can increase the costa clavicular space and reduce the imposed load on the neuro vascular structure in the thoracic outlet.

An 89 year old male patient suffered a stroke and was recently transferred to a skilled nursing facility from the hospital. The patient was found to have the following wound at admission. Which of the following is the MOST appropriate staging for this type of wound? (image on slide 13 FF#30) A. Stage I pressure ulcer B. Unstageable ulcer C. Suspected deep tissue injury D. Stage II pressure ulcer

C. Suspected deep tissue injury -Suspected deep tissue injury is described as purple or maroon localized area of discolored intact skin or blood-filled blister due to damage of underlying soft tissue from pressure and/or shear. The unstageable pressure ulcer is defined as Full-thickness tissue loss in which the base of the ulcer is covered by slough (yellow, tan, gray, green, or brown) and/or eschar (tan, brown, or black) in the wound.

A physical therapist is called to assess a patient with traumatic brain injury. The therapist documents that the patient has decerebrate posturing pattern. Which of the following is MOST likely to be seen in this patient? A. Sustained contraction and posturing of the trunk and lower limbs in extension, and the upper limbs in flexion, fists clenched B. Sustained contraction and posturing of the trunk and lower limbs in flexion, and the upper limbs in extension, fists clenched C. Sustained contraction and posturing of the trunk and limbs in a position of full extension D. Sustained contraction and posturing of the trunk and limbs in a position of full flexion

C. Sustained contraction and posturing of the trunk and limbs in a position of full extension

A physical therapist is planning to start his private PT practice and is working on making his clinic wheelchair accessible. Which of the following is NOT appropriate for a wheelchair accessible plan? A. Ramp slope: 1:12 B. Ramp width: 36 inches C. Tile or hardwood floors D. Door width should be 22 inches

D. Door width should be 22 inches Rationale: For wheelchair accessibility, the door width must be at least 32in. Other guidelines for wheelchair accessibility include: Open floor plan •Wheelchair access to bathroom •Toilet seat height same as wheelchair seat height •Adequate clearance under sinks •Insulated pipes •Roll-in shower •Ramp landings every 30 ft (9.1 m) •No thresholds through doorways • Lever-type door handles. Ref: Physical rehabilitation.

A 46 year old patient with shoulder pain is referred to physical therapy. The patient reports 4/10 pain at rest and 10/10 with any movement. When testing passive shoulder range of motion, which of the following end-feels would you MOST likely encounter? A. Soft end feel B. Firm end feel C. Hard end feel D. Empty end feel

D. Empty end feel -Pts ROM is limited and painful so it's empty -All other 3 options are NORMAL end feels

A patient complaints of lower back pain due to an abnormal gait pattern. The PT diagnoses L2 L3 nerve root compression. Which of the following is the LEAST likely gait deviation to be associated with this pathology? A. Trunk lurches backward during swing phase B. Posterior tilt of the pelvis during initial swing C. Circumduction of the hip during swing phase D. Excessive lumbar lordosis in terminal stance

D. Excessive lumbar lordosis in terminal stance Excessive lordosis (anterior pelvic tilt) is caused by hip flexor TIGHTNESS. The tight hip flexors are pulling the pelvic anteriorly causing the excessive lordosis Ant pelvic tilt is hip flexor tightness, post pelvic tilt is due to hip extensor tightness Rationale: L2-L3 nerve root compression leads to hip flexor weakness. Hip flexor weakness causes a backward trunk lurch during swing as the hip flexion is passively generated by backward movement of the trunk. It can also cause circumduction of lower extremity combining hip flexion, hip abduction, and forward rotation of the pelvis during swing phase. Hip flexor weakness causes a posterior tilt of the pelvis during initial swing as abdominals are used during initial swing to advance the swing leg. Excessive lumbar lordosis in terminal stance is seen with hip flexion contracture as the lack of hip extension in terminal stance is compensated for by increased lordosis

A patient complains of sudden onset of mild hearing loss on the left side. Weber's test findings show sound lateralized to the right ear. Rinne test was consistent with air conduction greater than bone conduction on both sides. Which of the following is the MOST likely? A. Right side sensorineural hearing loss B. Left side conduction hearing loss C. Right side conduction hearing loss D. Left side sensorineural hearing loss

D. Left side sensorineural hearing loss Lateralization = sound is louder on one side Walk through question: Where is the problem? Problem is on left side What does Weber test say? louder (lateralization) sound on the R ear, so there is a problem on the L ear Rinne: AC>BC on both sides (1) Where is problem? Problem to the Left: (2) What is Weber showing? Louder to the Right (lateralized to right). Issue with LEFT. (3) Rinne: Left ear had conduction proper. AC more than BC is normal. *Problem is in the LEFT side (mild hearing loss). Leads us to OPTIONS B or D since they deal with the LEFT side. Answer is D due to sensorineural hearing loss and NOT conduction hearing loss as we found out during Rinne.

A 58 year old male is performing a Bruce protocol in your clinic with ECG leads attached. During the protocol, the PT sees the picture shown below. What is the MOST likely diagnosis and intervention? (look at slide 22 of FF #9) A. Myocardial ischemia that has a elevation of the ST segment greater than 1 mm and the PT should stop the protocol B. Myocardial infarction that has a elevation of the ST segment less than 1 mm and the PT should stop the protocol and call 911 C. Myocardial ischemia that has a elevation of the ST segment less than 1 mm and the PT should stop the protocol D. Myocardial infarction that has a elevation of the ST segment greater than 1 mm and the PT should stop the protocol and call 911

D. Myocardial infarction that has a elevation of the ST segment greater than 1 mm and the PT should stop the protocol and call 911 Less than 1 mm of ST elevation is okay and continue with exercise More than 2 mm of ST elevation is dangerous and you should stop exercise

When checking vital signs of a patient with a blood disorder, the PT notices that the patient's BP is around 150/85 mm Hg. Which of the following conditions is MOST likely associated with the observed BP changes? Select one: A. Anemia B. Thrombocytopenia C. Leukopenia D. Polycythemia

D. Polycythemia Polycythemia is characterized by increases in both the number of red blood cells and the concentration of hemoglobin. People with polycythemia have increased whole blood viscosity and increased blood volume which can cause elevated blood pressure Anemia is a reduction in the oxygen-carrying capacity of the blood as a result of an abnormality in the quantity or quality of erythrocytes. Thrombocytopenia is decrease in platelet levels and Leukopenia is decrease in WBC levels. Options A, B&C do not cause an increase in SBP.

A 58 year old patient comes to PT clinic for a gait and posture evaluation. The patient has a right sided transtibial amputation and in unable to fully flex the right knee in stance phase. What is the MOST likely cause for this gait deviation is: A. Anterior displacement of the socket relative to the foot. B. The knee bolt is rotated externally C. The prosthesis is too light D. Posterior displacement of the socket relative to the foot.

D. Posterior displacement of the socket relative to the foot. limited flex too much ext posterior displacement

A 13 year old girl has been diagnosed with structural idiopathic scoliosis. The physical therapist notices a left thoracic, right lumbar structural scoliosis. Which postural deviation would be expected in this patient: A. The body of the thoracic vertebrae rotated to the right B. High right shoulder C. Spinous processes of thoracic spine rotated to the left D. Posteriorly protruding left scapulae

D. Posteriorly protruding left scapulae

A researcher is collecting ROM data on volleyball players during shoulder abduction motion. The results of the tests are BEST categorized as which of the following types of data? A. Interval data B. Ordinal data C. Nominal data D. Ratio data

D. Ratio data

A 68-year-old male patient comes to the clinic with a history of cognitive deficits and Alzheimer disease. The patients chief complain is urinary incontinence. Which of the following is MOST appropriate intervention for this patient? A. Use of absorbent pads B. Pelvic floor exercises targeting levator ani muscles C. Breathing exercises to develop sphincter control D. Scheduled toileting and prompted voiding

D. Scheduled toileting and prompted voiding

A patient arrived at an outpatient clinic with Rt shoulder pain. Upon further evaluation, patient described of having increased difficulty brushing back of head from past 3 months with increased pain at rest. The PT observed that ROM is limited in all directions. Which of the following stages and movement patterns would MOST LIKELY be associated with this scenario? A. Stage III, IR>Abd > ER B. Stage I, ER > Abd > IR C. Stage IV, IR >Abd >ER D. Stage II, ER >Abd> IR

D. Stage II, ER >Abd> IR -Frozen shoulder/Adhesive Capsulitis capsular pattern is EABI -LAM: Lateral ROtation , Abduction and Medial roatation Rationale: Answer D. -Stage 2 (freezing stage) with signs of persistent and intense pain even at rest. The motion is limited in all directions and cannot be fully restored with an intra-articular injection with duration 3-9 months. -Stage 1: signs of gradual onset of pain as it increases with movement and is present at night and exhibiting loss of external rotation with intact rotator cuff strength. The duration is typically less than 3 months, capsular patterns affected ER > ABD >IR. -Stage 3 (frozen stage) with signs of having pain only with movement, significant adhesions and limited GH motions. Patient also exhibits signs of excessive scapulothoracic movement as compensation pattern. Patient would also present with signs of atrophy of deltoid, rotator cuff, biceps and triceps brachii muscles with duration between 9-15 months after onset. -Stage 4 (thawing stage) with signs of minimal pain and non-synovitis with significant capsular restrictions from adhesions. The motion may gradually improve. The duration type Which endocrine condition associated with AC? -Hyperthyroidism Which sclerotic scar agent used to treat AC? -iodine/negative ion

A patient has a superficial partial thickness wound resulting from an abrasion. The wound bed is red and moist and with minimal exudate. Which of the following wound dressings is MOST appropriate to use? A. Wet to dry gauze B. Foams C. Calcium alginate D. Transparent films

D. Transparent films red = granulating properly so go with transparent film -if it didn't mention redness and focused more on minimal exudate than hydrogel/hydrocolloid would be a better answer Foams = moderate exudate Calcium alginate = heavy exudate

What is an AV block?

Delayed or blocked communication between the Atria and the Ventricle The Story of the AV Block Family: Wife = P Wave Husband = QRS Complex Pacer = Counseling Normal Sinus Rhythm Normal: The wife (P wave) waits at home for the husband (QRS). The husband (QRS) comes home on time every night. [PR Interval is less than 0.2 seconds (One large block on the EKG paper), there is one P wave per QRS complex, and the rhythm is regular.] 1st Degree AV Block: The wife (P wave) is waiting at home. The Husband (QRS) comes home late every night, but he always comes home, and it is the same time every night. [Impulse is held at AV node, and PR Interval is greater than 0.2 seconds, or one large box on the EKG paper.] 2nd Degree Block, Type 1 ("Wenckebach"): The wife (P wave) is waiting at home. The husband (QRS) comes home later and later every night until one night he doesn't come home at all. Note: Husband (QRS) must come home at least 2 nights in a row to see this pattern. [PR interval will become greater and greater until there will be a dropped QRS complex and there are two consecutive P waves

Donn/Doff steps?

Doffing/Donning -d"ON"ning: ON Think of Cinderella getting ready to go to the ball. 1) Gown (Sparkling gown) 2) Mask (Make-up) 3) Goggles (Glasses or contacts) 4) Gloves (to keep her hands clean when going out) -d"OFF"ing: OFF Alphabetical order 1) GLoves 2) GOGgles 3) GOWn 4) Mask -Airborne precaution use with TB, Chicken pox (varicella), shingles (herpes zoster), measles (rubeola) = "95 Airborne Measly Chickens Tee-peed the Shingles". -Contact: DISH (c. Diff, Impetigo, Scabies, Hepatitis) Droplet: MaSk for MS (Mumps, strep. A)

Difference between Preeclampsia/Eclampsia/Ectopoc Pregnancy/Gestational Diabetes

Eclampsia -onset of tonic clonic seizures in a woman with preeclampsia -increased risk include upper right abdominal pain/severe headace/vision/mental status changes Preeclampsia -HTN/protein in urine/severe fluid retention -can progress to maternal convulsions/coma/death if it becomes severe (eclampsia) Gestational diabetes -high blood sugar affecting pregnant women Ectopic Pregnancy -live pregnancy that takes place outside the uterus

Flexion synergy for UE Extension synergy for UE Flexion synergy for LE Extension synergy for LE

Flexion Synergy for UE -Scap: RETRACTION/ELEVATION or HYPEREXTENSION -Shoulder: ABD/ER -Elbow: FLEX -Forearm: SUPINATION -Wrist/Finger: FLEX "Draymond Green flexing" Extension Synergy for UE -opposite of flexion synergy -Wrist/Finger: FLEX Flexion Synergy for LE -Hip: FABER -Knee: FLEX -Ankle: DF -Toe: FLEX "Indian sitting position" Extension Synergy for LE -opposite of extension synergy

Hypothyroidism vs Hyperthyroidism?

Hypothyroidism • Symptoms-cold intolerance, excessive fatigue, headaches, weight gain, dry skin, peripheral edema, peripheral neuropathy, proximal weakness, hypoactive reflexes -Hashimoto's Disease "Think of a lazy bed ridden kid named Hashimoto. He never goes out so he has peripheral edema/neuropathy, has dry skin and is weak and is hypoactive with his reflexes because he's so lazy" Hyperthyroidism • Symptoms-Tachycardia, increased sweating, heat intolerance, increased appetite, dyspnea, weight loss, inability to gain weight, hyperactive reflexes -Graves Disease HYPERthyroidism: Think of a HYPER kid named Graves running around. They'll have tachycardia (from running), weight loss (from running), worked up an appetite (from running), sweating (from running), inability to gain weight (since they're HYPER and will go running around after). They can't tolerate the heat of competition if you want to race the HYPER kid (heat intolerance). **Symptoms of hyperthyroidism "SWEATING" • Sweating • Weight loss • Emotional lability • Appetite increased • Tremor / tachycardia • Intolerance of heat/ Irregular menstruation/ Irritability • Nervousness • GI problems (diarrhea) Additional: Increased DTR's , heat insensitivity

Difference between Independent sample T-test vs Paired T-test?

Independent sample T test = difference between 2 independent groups (general definition of T test) ex: PTs vs ATC Paired T test = compares difference between 2 matched sample ex: everyone is a PT, you as PT vs me as PT

Normal Breath Sounds

Look at slide 15 FF #10 Breath Sounds: INTENSITY - Vesicular: VESSELS on the ocean have to be SOFT to float and it's for sailing/riding beginners. - Broncho-vesicular: When riding a BRONCo in a VESSEL on the ocean, it's NOT for beginners but also not a hard level for sailing/riders, so it'll be INTERMEDIATE - Bronchial: Riding a BRONCo is LOUD and for those only prepared since it can get crazy - Tracheal: A TRAKE (DRAKE) concerts gets VERY LOUD no matter who you are, so you know TRAKE (DRAKE) is VERY LOUD Breath Sounds: LOCATION - Vesicular: When you wear a VESt (VESicular), you are covering most of the lungs - Broncho-Vesicular: If Vesicular is the 1st stage and Bronchial is the 2nd stage, then Broncho-Vesicular is going to be BETWEEN 1st and 2nd stage (interspace). - Bronchial: BRONCH:GRONK (GRONKowski) is the MAN (MANubrium). - Tracheal: TRACHEA you associate with the neck, so you know TRACHEAL is OVER THE TRACHEA in the NECK SUMMARY : Vesicular - Inspiratory longer than expiratory Bronchovesicular- Inspiratory & expiratory sounds are equal Bronchial - Expiratory sounds longer than inspiratory ones Tracheal- Both Inspiratory and Expiratory are equal SUMMARY: tracheal: heard over trachea Bronchial= heard in large airways in anterior chest near 2nd and 3rd intercostal space Bronchovesicular= posterior chest between scapula Vesicular= most lung fields

DD for Multiple Sclerosis vs Myasthenia Gravis

MS ● Acquired demyelinating disease with an immunological cause ● Weakness, paresthesia ● Charcots triad- Intentional tremor, scanning speech, nytagmus ( cerebellar involvement) ● spasticity, hypertonicity, hyperreflexia, positive Babinski sign ● incoordination, optic neuritis, ataxia, vertigo, dysarthria ● diplopia, bladder incontinence, tremor, balance deficits, falls, and cognitive deficits. MG ● Neuromuscular junction disorder. ● caused by autoimmune mediated acetylcholine receptor damage, resulting in a deficit in neuromuscular transmission. ● weakness that worsens during periods of activity and improves after periods of rest. ● variable degrees of weakness, ranging from ptosis or diplopia to critical respiratory weakness. ● Muscles that control speech, facial expression, mastication, swallowing, breathing, neck and MG recovery is quicker (hours) than MS recovery (days)

What is plagiocephaly?

Plagiocephaly, also known as flat head syndrome, is a condition characterized by an asymmetrical distortion (flattening of one side) of the skull. A mild and widespread form is characterized by a flat spot on the back or one side of the head caused by remaining in a supine position for prolonged periods.

Most important RLA level to know? What type of interventions should you provide?

RLA level 4 Confused/Agitated Interventions for RLA level 4 -quiet room -remove possible distractions -remove objects that could be thrown or used aggressibely -provide simple instructions with time to process -watch frustration and know when to stop or change activities -provide safe choices for the pt. This allows the pt to feel that he or she has some control over the situation Motor problems RLA level 4 -Prepare multiple activities -Give pt choices Behavioral problems RLA level 4 -Be calm -Be consistent (same treatment time, therapist, location) -Provide orientation -Know when to stop or change activities Don't give yes/no questions Give them options where both of the activities will benefit the pt and be realistic for them Note: Give control to the pt when it is safe and appropriate. Control can be given while maintaining focus on the therapeutic goals by phrasing questions as "Would you rather play ball or go for a walk?" This prevents situations where the pt chooses an undesirable or unrealistic activity if asked, "What would you like to do?" or the case where the pt simply answers "No" with a yes/no question

Neuro Tracts

Summary of tracts: DCML - fine touch, two-point discrimination, proprioception, vibration Anterior Corticospinal - motor Lateral Corticospinal - motor Anterior Spinothalamic - crude touch Lateral Spinothalamic - pain and temp Ascending (sensory) Tracts start in spine and are relayed to brain = SPINO_______ -DCML= 4 Letters - 4 purposes: Light touch, Proprioception, Vibration, 2 point discrimination -Spino"TH"alamic - Think of THermometer - Know Temperature and Pain are on same tract -Spino"cerebellar"- Muscle spindles and GTOs provide info to cerebellum to allow for controlled vol. mvmt. -Spinore"TIC"ular - If left untreated (CHRONIC), tics burry DEEP into dog fur and cause PAIN Descending (motor) tracts start brain and relayed to spine = ______Spinal - CortiCospinal - "C" on athlete's jersey = Captain VOLUNTARILY takes charge/make important moves - RUBROspinal - Captain doesn't like to move down field alone, he needs ASSISTANCE from his buddy who likes to RUB the ball before scoring - "VEST"ibulospinal - Vest covers trunk = Sit up straight (think Muscle tone, Posture, Antigravity muscles) - TecTospinal - "T" = Turn head to locate stimuli

Sensory Examination: Superficial Sensation Deep Sensation Cortical Sensation

Superficial Sensation -Exteroceptors receive stimuli from the external environment via skin and subcutaneous tissue -Pain, temperature, and light touch Deep Sensation -Proprioceptors receive stimuli from mmuscles, tendons, fascia -position sense and awareness of joints at rest, kinesthesia, and vibration Cortical Sensation -Combination of both superficial and deep sensory mechanism -Stereognosis, 2 point discrimination, barognosis, graphestesia, tactile localization, recognition of texture, and double simultaneous stimulation

Position Strategies to Reduce Common Malalignments: Supine Sitting in an Armchair or WC

Supine -Head/neck = neutral and symmetrical supported on pillow -Trunk aligned in midline More Affected UE -Scap = PROTRACTED -Shoulder = FORWARD/SLIGHTLY ABDUCTED, arm supported on pillow -Elbow = EXTENDED with hand resting on pillow -Wrist = NEUTRAL -Fingers = EXTENDED -Thumb = ABDUCTED "arm/fingers sticking OUT and FORWARD" More Affected LE -Hip = forward (pelvis protracted) -Knee = on small pillow to PREVENT HYPEREXTENSION, nothing against soles of feet -for persistent PF, a splint can be used to position the foot and ankle in neutral position "how I like to lie down when I come home from work"

A 45 year old female is referred to physical therapy with rheumatoid arthritis. The physical therapist notes increased flexion at the proximal interphalangeal joints and hyperextension at the distal interphalangeal joints. The MOST likely diagnosis will be: a. Boutonniere deformity b. Mallet finger c. Swan neck deformity d. Ulnar drift

a. Boutonniere deformity Boutonniere Deformity: Rupture of the central band (central slip) of the extensor hood results in the lateral bands of the extensor apparatus (extensor hood) slipping in a volar direction to the PIP joint, causing PIP flexion and DIP extension. Swan Neck Deformity: Laxity of the PIP joint with an over- stretched palmar plate and bowstringing of the lateral bands of the extensor hood result in hyperextension of the PIP and flexion of the DIP joints. Ulnar drift: Ulnar deviation of the digits because of weakening of the capsuloligamentous structures of the metacarpophalangeal joints and the accompanying "bowstring" effect of the extensor communis tendons. Mallet finger: It is the result of a rupture or avulsion of the extensor tendon where it inserts into the distal phalanx of the finger. The distal phalanx rests in a flexed position.

A 14 year old girl presents with a chief complaint of pain in the medial three fingers of the dominant hand, especially during gripping activities. The pain lasts for a while and subsides on its own after resting. She likes to play tennis on weekends but has a sedentary lifestyle otherwise. During postural assessment, the therapist noticed significant winging of the scapula on the same side. Strength of shoulder flexors: 4/5 and abductors is 4+/5 on the affected side. Strength of intrinsic muscles is intact. Which of the following nerves is MOST LIKELY involved? Select one: a. Long thoracic nerve b. Median nerve c. Axillary nerve d. Suprascapular nerve

a. Long thoracic nerve Rationale: Presence of scapular winging is indicative of serratus anterior weakness. The long thoracic nerve (C5-C7) is a motor nerve that supplies the serratus anterior muscle. The long thoracic nerve can be injured due to carrying heavy loads on the shoulder (e.g. backpacks) or when serving while playing tennis. Due to the weakness of scapular muscles, shoulder elevation may also be affected. Long thoracic nerve injury can be associated with inability/pain when fully flexing an extended arm. An axillary nerve injury would lead to weakness of shoulder abduction and ER with square shoulder deformity. The median nerve mainly supplies the forearm and hand muscles, which would not cause winging of the scapula. The suprascapular nerve supplies the supraspinatus and infraspinatus which function to abduct and ER the arm, respectively. It would also cause pain with cervical rotation to the opposite side.

An 18-year-old female volleyball player complains of right forearm and hand pain during overhead activities. Upon examination, the physical therapist finds the following range of motion values: Right elbow flexion 10-100, forearm supination 0-75, forearm pronation 0-45, wrist extension 0-75 and wrist radial deviation 0-20 degrees. What is the MOST appropriate intervention to help increase range of motion on the right side? a. Posterior glide of the radius at the proximal radio-ulnar joint b. Dorsal glide of ulna at the distal radio-ulnar joint c. Dorsal glide of radius at the distal radio-ulnar joint d. Medial glide of the radius at the radio-carpal joint

a. Posterior glide of the radius at the proximal radio-ulnar joint As per the given range of motions, the forearm pronation is the limited movement. To improve pronation at proximal radio-ulnar joint, a posterior/dorsal glide of radius is given. To improve pronation at distal radio-ulnar joint, a volar glide of radius is given.

A 68-year-old patient suffering from Rheumatoid arthritis comes for a physical therapy evaluation. You observe a Boutonniere deformity. Which of the following characteristics the Boutonniere deformity? a. Ruptured central band b. Flexed DIP c. Hyperextended PIP d. Hyperflexed DIP

a. Ruptured central band Boutonniere Deformity: Rupture of the central band (central slip) of the extensor hood results in the lateral bands of the extensor apparatus (extensor hood) slipping in a volar direction to the PIP joint, causing PIP flexion and DIP extension.

You are assessing a patient with a CVA who presents with right sided body weakness, inability to understand commands, weakness of the R half of the face and homonymous hemianopsia. Which of the following will MOST LIKELY be absent in this patient? Select one: a. Topographic disorientation b. Lack of fluency while talking c. Loss of sensation on the right side of body d. Limb-kinetic apraxia

a. Topographic disorientation Rationale: The symptoms of weakness of R body and face, inability to understand commands (also known as Wernicke's Aphasia), and homonymous hemianopsia are suggestive of an L MCA infarct. With an L MCA infarct, Broca's aphasia, limb-kinetic apraxia and loss of sensation of the right UE and face are commonly seen. Topographic disorientation is usually seen with lesions involving non-dominant primary visual areas.

An acute care physical therapist is reading the reports of a patient with chronic kidney disease. The patient appears to be confused and lethargic. His body is warm to touch and he is in bradycardia. His lab findings are as follows: pH: 7.20; HCO3: 21mEq/L; PaCO2: 34 mm Hg. Which of the following is the MOST APPROPRIATE diagnosis for this patient? Select one: a. Uncompensated metabolic alkalosis b. Partially compensated metabolic acidosis c. Compensated metabolic acidosis d. Uncompensated metabolic acidosis

b. Partially compensated metabolic acidosis Rationale: Chronic kidney disease with renal failure results in acidosis because the failing kidney not only is unable to rid the body of excess acids but also cannot produce necessary bicarbonate. Normal values for pH = 7.35-7.45, PaCO2 = 35-45 mm Hg; and HCO3- = 22-26 mEq/L. Fully compensated = pH is within normal range Uncompensated = other value is normal Partially compensate = other value is not within normal and pH not within normal range

A 35-year-old female presents to the clinic with a chief complaint of pain in the mid-back. The pain is on and off throughout the day, but is certainly more when she wakes up in the morning and doesn't subside for an hour. It is only when she is sitting in her chair having breakfast that she notices that the pain is gone. She is an elementary school teacher by profession and must stand for prolonged hours. She attributes the pain to prolonged standing. She is trying to lose weight for her wedding and is practicing intermittent fasting. She has had episodes of mild stomach pain, lightheadedness, fainting, and vomiting. Which of the following is the MOST LIKELY cause of her symptoms? Select one: a. Gastric ulcer b. Mechanical low back pain c. Duodenal ulcer d. Ulcerative colitis

c. Duodenal ulcer Rationale: -Option C: Pain associated with duodenal ulcers is prominent when the stomach is empty, such as between meals and in the early morning. The pain may last from minutes to hours and may be relieved by antacids. Other symptoms may include nausea, vomiting and bloody stools. -Option A: Gastric ulcers cause pain with the presence of food. -Option B: Mechanical lower back pain will not be affected with meals and will not have other symptoms such as stomach pain, lightheadedness and fainting. -Option D: Pain associated with the large intestine or colon may be referred to the sacrum. This pain may be relieved after defecating or passing gas.

A patient with subluxation of the shoulder presents to the PT clinic. During evaluation, the therapist notices atrophy of the flexor surface of the upper arm along with weakness of elbow flexion with a supinated forearm. Which of the following findings will MOST LIKELY be associated with this finding? Select one: a. Loss of 2-point discrimination on the lateral aspect of the upper arm b. Loss of 2-point discrimination on the posterior aspect of the arm c. Loss of 2-point discrimination on the lateral aspect of the forearm d. Loss of 2-point discrimination on the posterior aspect of the forearm

c. Loss of 2-point discrimination on the lateral aspect of the forearm Rationale: With an injury to the musculocutaneous nerve, the patient is unable to flex the elbow with the forearm supinated and may have some instability in the shoulder with atrophy of the flexor surface of the upper arm. The sensory supply of the musculocutaneous nerve is to the lateral aspect of the forearm.

A physical therapist is evaluating a patient, who works as a maid in a hotel. Her work requires her to make beds, vacuum, clean bathrooms, and manipulate a fairly heavy cart with cleaning supplies and linen. She is right hand dominant and suffered a severed biceps tendon on the right side (which has not been repaired) in an incident involving a boyfriend, another woman, and a knife. As a result of this incident, she has experienced a secondary inflammation of the interosseous membrane (between the radius & ulna) that causes pain when it is under moderate tension. During the physical examination, the physical therapist is likely to see a. An increase in range of motion and strength for elbow flexion b. A decrease in strength and increase in range of motion for elbow flexion c. A decrease in strength but no decrease in active range of motion for elbow flexion d. A decrease in strength and active range of motion for elbow flexion

d. A decrease in strength and active range of motion for elbow flexion The function of the biceps muscle is elbow flexion and forearm supination. When the tendon is ruptured, the patient will have weak or absent elbow flexion and supination, swelling, ecchymosis, palpable gap in the biceps tendon.

While assessing a patient with a R CVA, the therapist asks the patient to fold a piece of paper into half and notices that the patient does not use the L hand at all. On asking the patient what is wrong with the L hand, she says that she slept funny so the hand is not moving for now and that it should be moving by the end of the day. Which of the following is the MOST LIKELY medical diagnosis for this presentation? Select one: a. Right-left discrimination disorder b. Somatoagnosia c. Ideational apraxia d. Anosognosia

d. Anosognosia Anosognosia is the denial or lack of awareness of the presence or severity of one's paralysis. Right-left discrimination is the inability to identify the right and left sides of one's own body. Somatoagnosia is a lack of awareness of the body structure and relationship of body parts to oneself or to others. Ideational apraxia is the inability to perform a task on motor or on command.

A 6-month-old child was referred to physical therapy for right torticollis. The MOST effective method to stretch the muscle is by positioning the head and neck into a. Flexion, left side-bending, and left rotation. b. Extension, right side-bending, and left rotation. c. Flexion, right side-bending, and left rotation. d. Extension, left side-bending, and right rotation

d. Extension, left side-bending, and right rotation In torticollis, sternocleidomastoid muscle is most commonly affected and it becomes short. The main action of SCM, is same side bending and opposite side rotation. So, to stretch right SCM, left side bending, right rotation and extension will be the ideal position.

A patient complains of gradual onset of dyspnea with exertion. He is a teacher by occupation and noticed that he needs to take frequent breaks while teaching/talking because he gets fatigued easily. He has been a smoker for over 30 years and reports an increase in cough with expectoration. He is suspected to have obstructive pulmonary disease. Which of the following will most likely NOT be associated with this condition? Select one: a. Hypertrophy of accessory muscles of respiration b. Obliteration of Lovibond angle c. Bluish discoloration of the mucous membrane d. Increase in the amount of exhalation in 1 sec

d. Increase in the amount of exhalation in 1 sec Rationale: The symptoms of COPD include: Hypertrophy of accessory muscles of ventilation, pursed-lip breathing, cyanosis (bluish discoloration of skin or mucous membrane), and digital clubbing (obliteration of Lovibond angle is grade 2 of stages of clubbing). Spirometry is the GOLD standard test for diagnosis of COPD which indicates a decrease in the expiratory flow rates (especially FEV1 which is the amount of air exhaled in 1 sec).

A physical therapist is teaching a patient an exercise in standing with the patient's right side of the body next to the wall and the patient's right arm resting on the wall for support. The patient is told to place the right leg behind the left in the extended, adducted and externally rotated position with the pelvis shifted to the right and a slight trunk lean to the left. Which of the following tests was MOST LIKELY positive for the therapist to recommend this intervention? Select one: a. Piriformis test on the left b. Phelp's test on the left c. Ely's test on the right d. Ober's test on the right

d. Ober's test on the right Rationale: The intervention described is a stretch of the ITB/tensor fascia lata on the right. Ober's test is used to assess the tightness/contracture of the tensor fascia lata. Ely's test is for a tight rectus femoris and Phelp's test is positive in the case of a tight gracilis muscle.

· Which nerve roots dues the ulnar nerve come from?

o C8/T1

What are the motor impairments related to cerebellar pathology?

"Dizzy ANT balancing 3 dys'es (dishes)" -DIZZIness -Ataxia -Nystagmus -Tremor -imBALANCE -DYSdiadochokinesia -DYSarthria -DYSmetria Asthenia: muscle weakness Dysarthria: speech Dysdiadochokinesia: impaired rapid alternating movements Dysmetria: can't judge distance Dyssynergia: no smoothness in movement, robot-like Gait ataxia Hypotonia: decreased muscle tone, flaccid/soft Nystagmus Rebound phenomenon: no control with isometric contraction Tremor

What is DHP used to assess for and which side is affected when performing?

-Diagnostic test (NOT INTERVENTION) used to detect torsional nystagmus -Which ever side the pt's head is rotated towards

What are the symptoms of Huntington's Disease?

-35 to 55 y.o -affects muscle coordination and leads to cognitive decline, psychiatric problem -involuntary jerking or writhing movements (Huntington's chorea) -Muscle problems, such as rigidity or muscle contracture (dystonia) -slow or abnormal eye movements -impaired gait, posture, and balance -difficulty with the physical production of speech or swallowing

What is atrial fibrillation?

-400-600 BPM -Multiple erratic atrial contractions (P waves) before ventricle contraction (QRS complex) (look at slide 25 on FF#9)

Cardiac Rehab: Strength Training in Phase 3 Guidelines

-Begin with the use of elastic bands and light hand weights (1-3#) or 50% of max weight used to complete 1RM -Progress to moderate loads, 12-15 reps -Avoid UE resistance as soft tissue is still healing

CVA at ACA symptoms

-Contralateral motor AND sensory loss greater in the LE>UE -Memory and behavioral impairments due to frontal lobe involvement -Urinary incontinence -Problems with imitation and bi-manual tasks, apraxia "Imagine you have a pt named Amy and she has LE sensory and motor loss, she also has bad memory, urinary incontinence, and problems with imitating movement (apraxia)"

Grading scale of edema?

1+ -mild indentation -<0.25 pitting 2+ -moderate indentation, -returns to normal within 15 seconds -0.25-0.50 inches pitting 3+ -severe indentation -returns to normal 15-30 seconds -0.50-1.0 inches pitting 4+ -very severe -depression lasts for >30 seconds ->1.0 inch

Gait related impairments of PD

-Festinating gait: progressive increase in speed with a shortening of stride. Can be anteropulsive (forward) or retropulsive (backward) -Freezing of gait (FOG): sudden abrupt inability to initiate any movement (Doorways/turns) -difficulty turning: increased steps per turn -reduced stride length: increased step to step variability -reduced speed of walking -increased time: double limb support -insufficient hip, knee, and ankle flexion: shuffling steps -insufficient heel strike with increased forefoot loading -reduced trunk rotation: decreased or absent arm swing -difficulty with dual tasking: simultaneous motor and/or cognitive tasks -difficulty with attentional demands of complex environment

RLA Level 6 Interventions

-Highly structured functional tasks -Decrease cues/external direction -Decrease use of AD -Increase speed and complexity of tasks as able

What is Appendicitis? Special tests associated with appendicitis?

-Inflammation of the vermiform appendix -Progression can lead to a swollen/gangrenous appendix -If perforated, it can lead to peritonitis S/S -pain in the RL quadrant -pain comes in waves progressing to steady -anorexia -nausea/vomiting -elevated temperature -Leukocytosis -fever -tenderness at McBurney's point -Rovsing's sign for pain migration -Blumberg's sign for rebound tenderness MEDICAL EMERGENCY IF FOUND

What is Kehr's sign?

-L shoulder pain with pressure on the upper abdomen -caused by free air or blood in the abdominal cavity -3 associated causes (SLR) Stomach ulcer Laparoscopy Ruptured spleen "Steve Kehr had L shoulder pain because of a stomach ulcer, laparoscopy, and ruptured spleen (SLR)"

Interventions for BVH?

-Need to involve gaze stability during head motion, dysequilibrium, and gait ataxia -Same VOR exercises but in standing then progress to walking to challenge

Rules for Airborne Precautions

-Private room with monitored air pressure -Room door should remain closed with pt within the room -Respiratory protection worn when entering the room -Limit pts transport outside of the room for only essential purposes: pt should wear a mask during transport -Negative pressure room: so its difficult for air to go outside from inside (you don't want the infected air inside the room to go outside) Ex: measles, varicella, tuberculosis Airborne precaution use with TB, Chicken pox (varicella), shingles (herpes zoster), measles (rubeola) = "95 Airborne Measly Chickens Tee-peed the Shingles".

Specific symptoms for L CVA

-R side weakness/paralysis -aphasia (damage of Wernicke's/Broca's area) -personality changes: cautious, compulsive, disorganized -(NEED TO MOTIVATE/PUSH THEM MORE TO EXERCISE BECAUSE THEY'RE SO CAUTIOUS) -difficulty with new information (decreased memory/difficulty generalizing or conceptualizing) "oLd person": cautious in making judgements

List the DTR grading system

0 = Absent 1+ = Slight reflex, present but depressed, low normal 2+ = Normal, typical reflex 3+ = Brisk reflex, possibly but not necessarily abnormal 4+ = Very brisk, abnormal, clonus

A mother reports that her 6-month-old infant has had vomiting and diarrhea over the last 2 days. Which of the following signs would be the MOST accurate indication of severe dehydration? 1. Decreased respiratory rate 2. Sunken fontanelle 3. Warm hands and feet 4. Loud crying when touched

2. Sunken fontanelle 1. Increased respiratory rate is a sign of severe dehydration (not decreased rate). 2. The fontanelle will be sunken in infants who are dehydrated. 3. Cold hands and feet are present in severe dehydration. 4. In severe dehydration, the infant would be unable to cry.

A therapist examines a 69-year-old woman who has suffered a recent stroke. The therapist needs to focus on pre gait activities. Which PNF diagonal best encourages normal gait? A. D1 B. D2 C. PNF is contraindicated D. pelvic PNF patterns only

A. D1 D1 flex/ext mimics gait should be used as a pre-gait activity

Which of the following findings is MOST consistent with the presence of rebound tenderness noted during palpation of the abdomen of a patient who has low back pain? 1. Muscle guarding 2. Superficial reflex 3. Muscle soreness 4. Peritoneal irritation

4. Peritoneal irritation 1. Rebound tenderness at the abdomen is not consistent with muscle guarding (pp. 320-321). 2. Rebound tenderness is not consistent with a superficial reflex of the umbilicus when assessing for a neurologic impairment (pp. 320-321). 3. Rebound tenderness is not consistent with muscle soreness but rather is a classic sign of peritonitis (pp. 320-321). 4. Pain on release of pressure confirms rebound tenderness, a reliable sign of peritoneal inflammation (pp. 320-321).

To palpate the insertion of the supraspinatus tendon, a physical therapist should place a patient's arm in which of the following positions? 1. Shoulder flexion and lateral (external) rotation 2. Shoulder flexion and medial (internal) rotation 3. Shoulder extension and lateral (external) rotation 4. Shoulder extension and medial (internal) rotation

4. Shoulder extension and medial (internal) rotation 1. The supraspinatus tendon is most accessible for palpation when the shoulder is medially (internally) rotated. 2. The supraspinatus tendon is most accessible for palpation when the shoulder is extended. 3. The supraspinatus tendon is most accessible for palpation when the shoulder is medially (internally) rotated. 4. Extending and medially (internally) rotating the shoulder places the supraspinatus tendon in the optimal position for palpation.

Which of the following exercise programs is most appropriate for a 67 years old postmenopausal female patient with MS who has poor balance and decreased strength? A. Stretching, posture and balance program with strengthening exercise as tolerated B. Stretching and progressive hill training on treadmill C. Warm water aquatic training D. Progressive hill training on treadmill with plyometric training

A. Stretching, posture and balance program with strengthening exercise as tolerated

In a research study on obese THA patients, a correlation coefficient (r) of +0.80 was found for the relationship between weight and BMI. Which of the following interpretations of this finding is MOST appropriate? A. Weight and BMI have a high positive correlation. B. 80 percent of the variability in BMI can be accounted for by weight C. 80 percent of the variability in weight can be accounted for by BMI D. There are no significant differences between the weight and BMI levels

A. Weight and BMI have a high positive correlation.

Which of the following lab values are out of its normal value? A. 20% Hematocrit B. Prothrombin time of 12 seconds C. Platelet count of 50,000 cells/mm3 D. INR - 3

ACD Hematocrit norm = 37-52% INR value norm = 0.9-1.1 Platelet count norm = 150,000-400,000 Prothrombin time norm = 11-15 seconds

What is positive for Whispered pectoriloquy?

Able to recognize pt whispering "1, 2, 3" Whisper because the birds are PECtoriloquy PEC123LOQUY

Safety and Precautions: Types of Abuse?

Abuse -Infliction of physical or mental injury or the deprivation of food, shelter, clothing, or services needed to maintain physical or mental health Sexual Abuse -Sexual assault, sexual intercourse without consent, indecent exposure, deviate sexual conduct, or incest adult using a child for sexual gratification without physical contact is considered a sexual abuse Physical Abuse -Physical injury results in pain, impairment or bodily injury of any bodily organ or function, permanent or temporary disfigurement or death Emotional Abuse -Anguish inflicted through threats, intimidation, humiliation, isolation, embarrassing, blaming or rejecting behaviors from adult towards child, withholding love affection and approval Mental Abuse -Impairments of a person's wellbeing, intellectual, or psychological functioning

Distinguish between active insufficiency vs passive insufficiency

Active: inability of a 2 joint muscle to SHORTEN simultaneously at both joints ex: difficult to shorten biceps while in max shoulder flex SAME motion as muscle function Passive: inability of a 2 joint muscle to LENGTHEN simultaneously at both joints ex: difficult to achieve max trunk flexion or hip flex when knee is in full ext with hamstrings OPPOSITE motion of the muscle function

What is COPD?

Airflow limitation that is not fully reversible Air trapped in the lungs due to narrowed airways making it difficult to exhale air Causes increased FRC (RV + ERV)

List the motor impairments related to Basal Ganglia pathology

Akinesia: inability to initiate movement Athetosis: slow involuntary wormlike movement Bradykinesia Chorea: involuntary rapid jerky movements Choreoathetosis Dystonia: involuntary contractions of agonist and antagonist muscles causing abnormal posturing or twisting movements Hemiballismus: large amplitude sudden violent movement Hyperkinesis: abnormally increased muscle activity Rigidity: increase muscle tone Tremor

Complete loss of pain sensitivity?

Analgesia

Inability to recognize the form and shape of objects by touch?

Astereognosis Tactile agnosia

What is a 3rd degree AV block?

Atria is independent of the ventricle No relationship at all of the PR intervals ex: wife and husband are not communicating at all and is an emergency

Difference between atrial tachycardia/atrial flutter/atrial fibrillation?

Atrial tachycardia = 100-250 bpm Atrial flutter = 250-350 bpm Atrial fibrillation = 400-600 bpm (look at slide 24 on FF #9)

The increased metabolic demand placed on the heart during exercise can be best estimated by examining the A. Systolic blood pressure B. Rate product pressure C. Diastolic blood pressure D. Heart rate

B. Rate product pressure RPP = HR x BP Used to determine myocardial O2 demand of a pt at the onset of chest pain symptoms

A young adult underwent right Achilles tendon repair 6 weeks ago and is now able to fully weight bear. The PT is giving him advice on proper shoe modification. Which of the following would be BEST for the patient to utilize? A. Normal shoes B. Shoes with 1.5 cm heel lift C. Shoes with lower than the regular heel D. Shoes with 5 cm heel lift

B. Shoes with 1.5 cm heel lift You want to avoid excessive DF because that will stress the Achilles tendon, so you need a heel lift which promotes PF D is too aggressive

A PT treats a patient diagnosed with Parkinson's disease. When working on controlled mobility, which of the following would best describe the physical therapist's objective? A. Facilitate postural muscle control B. Promote weight shifting and rotation trunk control C. Emphasize reciprocal extremity movement D. Facilitate tone and rigidity

B. Promote weight shifting and rotation trunk control -A incorrect, it's more Stability -C incorrect, it's more Skill -D incorrect because we want to decrease tone and rigidity with PD pts

A 45 year old patient diagnosed with amyotrophic lateral sclerosis disease is being assessed by the PT for the first time. When evaluating a patient with ALS, the PT should suspect which of the following cranial nerve LEAST likely to be involved? A. Glossopharyngeal nerve ( CN IX) B. Vestibulocochlear nerve (CN VIII) C. Vagus nerve (CN X) D. Hypoglossal nerve (CN XII)

B. Vestibulocochlear nerve (CN VIII) ALS effects UMN and LMN Vestibulo is unaffected (balance and coordination) from UMN and LMN lesions

What is Bigeminy and Trigeminy PVC?

Bigeminy = 1 normal beat followed by 1 PVC (not an emergency) Trigeminy = 2 normal beats followed by 1 PVC (not an emergency)

Postural Drainage: What is it? Indications

Bronchus of the involved lung segment is perpendicular to the ground; using gravity, these positions assist the mucociliary transport system in removing excessive secretions from the tree Indication -Pulmonary complications -Weak/elderly pts -Atelectasis, pneumonia, COPD Maintain each position for 5-10 minutes Secretions cleared by coughing or suctioning

Prevent Contracture Position Burn vs Amputation at the Hip

Burn -keep in Ext and ABD "B for Burns" Amputation -keep in Ext and ADD

A patient recently suffered a level IV TBI, and is currently admitted in the hospital. The PT wants participation from the patient during treatment. Which of the following is the MOST appropriate question that the PT should ask the patient to engage them in the treatment session? A. Would you like to walk? B. Would you like to exercise in the hallways today? C. Would you like to work on ball or go for a walk? D. What would you like to do first?

C. Would you like to work on ball or go for a walk? -GIVE CHOICES!!! -no open ended questions

What is each test/treatment used for? -CRM (Canalith Repositioning Maneuver)/Epley -Liberatory/Semont Maneuver -Brandt-Daroff -BBQ roll/Gufani Maneuver/Prolonged Positioning

CRM/Epley -used to TREAT anterior/posterior canalithiasis Liberatory/Semont Maneuver -used to TREAT cupulolithiasis -Brandt-Daroff -used for anterior/posterior canalithiasis HEP BBQ roll/Gufani Maneuver/Prolonged Positioning -used to TREAT horizontal canalithiasis

Cardiac Rehabilitation Phases

Cardiac rehab begins in the hospital and extends indefinitely into the maintenance phase -Acute phase or monitoring phase = Phase 1 -Subacute phase of conditioning phase = Phase 2 -Training phase = Phase 3 -Maintenance period = Phase 4

How to differentiate between Central Vestibular Pathology vs Peripheral Vestibular Disorder?

Central -Nystagmus is pure vertical pendular (single axis/eyes oscillate at equal speeds) -No rotational component with nystagmus -SEVERE ataxia -ABNORMAL smooth pursuit and saccadic eye movement tests (rapid movement of eyes back and forth between 2 index fingers) Peripheral -Nystagmus is up/down beating torsional (vertical with rotational component) or horizontal nystagmus -NORMAL eye movements

Distinguish between Chi square test vs Mann-U Whitney Are they parametric or non-parametric?

Chi square = comparing 2 groups with unequal data ex: comparing running speed of 7 males vs 3 females Mann-U Whitney = comparing 2 groups with unequal data with 2 independent samples ex: FF group invites you in Chicago to celebrate and all go running, all have different running speeds, you divide a line saying people to the left are professional runners and people on the right are amateur runners. There are 2 independent samples but they're all from the same population (PTs from FF). Also there is no clear dividing line between who is a profession and who is an amateur runner Non-parametric data = not equal

What is Myofascial Pain Syndrome?

Chronic regional pain syndrome that is comprised of myofascial trigger points in a muscle that has a specific referred pattern of pain along with sensory, motor, and autonomic symptoms Trigger point is a hyperirritable area in a tight band of muscle described as dull, aching, and deep

What is Fibromyalgia?

Chronic widespread pain that affects multiple body regions (scapula, head, neck, chest, low back) Most commonly seen in older women around age 70-80 y.o

Difference between Colles' fracture vs Smith's fracture?

Colles' fracture: posterior/dorsal displacement of distal radius from FOOSH (fall in wrist ext) Smith's fracture: anterior/volar displacement of distal radius from falling with flexed wrist CE = Colles' Ext SF = Smith's Flex

Types of TENS

Conventional (Sunday) -used for sensory response (chronic LBP) -HIGH pulse frequency (30-150 PPS) -SHORT pulse duration Acupuncture like (Monday) -used for muscle twitching -LOW pulse frequency (2-4 PPS) -LONG pulse duration Brief Intense (Wedensday) -used for wound debridement -HIGH pulse frequency (60-200 PPS) -LONG pulse duration Noxious (Thursday) -used for trigger points -HIGH or LOW pulse frequency -LONG pulse duration TENS: Sufficient Amplitude -Conventional: Your SENSES go wild at a convention because so much happening -Acupuncture: Thinking of a them jamming a needle into you gives you the chills and makes your MUSCLEs TWITCH --Burst tens can be seen with acupuncture tens. Brief tens is used only for a brief time before procedures, High is used for acute pain -Brief Intense: Since brief you know it's STRONG because you don't want STRONG long lasting -Noxious: If you can't tolerate something, it makes you NAUSEOUS

Occupational Safety and Health Administration (OSHA) is responsible for which of the following? A. Determining the how much compensation the therapist should get yearly B. Assuring minimum standard are met at the hospital to maintain accreditation and prevent negligence in patient care. C. Assuring minimum standard are met by the rehabilitation clinics to maintain accreditation and safety of patients. D. Ensuring adequate steps taken to prevent exposure to harmful radiation to employees at a X-ray center.

D. Ensuring adequate steps taken to prevent exposure to harmful radiation to employees at a X-ray center.

What are the 2 hallmark signs of Left sided heart failure?

Cough and SOB

Side effects of Levodopa: Dyskinesia vs Dystonia

Dyskinesia -dynamic involuntary choreoathetotic movements occurring at peak of Levodopa dose -Initially you have facial grimacing with twitching of the lips and tongue protrusion -Severe involvement limbs, trunk, and neck -associated with "on" periods Dystonia -prolonged involuntary contraction that causes twisting or torsion of body segments -clawing of the toes or fingers, or cramping of the calf, neck, face, or paraspinal muscles -associated with pain during "off" periods DYSKINESIA when ON the medication. - dysk"I"Nesia: "I"n the vial you have medication, so you are ON the medication DYSTONIA when OFF the medication. - dyst"O"nia: "O"ut of medication in the vial, so off the medication.

Augmented Feedback for Motor Learning

Early in learning phase -focus feedback on correct aspects of performance Later in learning -focus feedback on errors as they become consistent Use random or serial practice order rather than blocked practice to improve retention -random/serial practice for retention

How would you treat a R posterior SCC canalithiasis?

Epley's Maneuver/Canalith Repositioning -DHP assessment should find up beating nystagmus when pt's head is rotated to the right Steps -Maintain each position for 1-2 minutes or until vertigo and nystagmus has stopped to ensure otoconia low through the canal -Rotate pt's head 45 degrees to the right -Lay supine in same head position and ext pt's head to 30 degrees below horizontal -Rotate head 45 degrees towards other side and again 30 degrees of ext -Roll pt sidelying with nose pointed toward ground -Slowly sit pt up, maintaining head on flexed (chin tucked) position and still rotated -Slowly return head to upright and remain sitting 3-4 minutes, then repeat until no symptoms are seen

What is the function of CN5?

Facial sensation Corneal sensation Anterior tongue sensation Muscles of mastication Dampens sound (lensor tympani)

3 things associated with heat sensitivity with MS patients?

Fatigue: -hard for MS pts to measure their own fatigue so give them frequent rest breaks Weakness Spasticity

Homonymous Hemianopsia vs Hemineglect

Homonymous Hemianopsia -Visual field loss on the SAME side of BOTH eyes -Left homonymous hemianopsia is due to RIGHT occipital lobe lesion causing vision loss in the L temporal and R nasal sides Hemineglect -Pts fail to be aware of items to one side of space -It is a perceptual disease so it ONLY effects the R hemisphere (non-dominant hemisphere) which would only effect the L side -Stroke or damage to the right hemisphere = Left hemineglect -Ex: lady put make up only on the R side of her face Interventions for Hemineglect -strategies that encourage awareness and use of the environment on the hemiparetic side and use of the hemiparetic limb -Teach active visual scanning: movements through turning of the head and axial trunk rotation to the more involved side. Cueing (visual/verbal/motor) is used to direct the pt's attention

How does RV hypertrophy or cor pulmonale occur from COPD?

Hypoxemia (decreased oxygen in the arterial blood to the tissues) occurs. As COPD progresses and more areas of the lungs become involved, hypoxemia will worsen and hypercapnea (increased amount of CO2 within the arterial blood) will develop. RV hypertrophy/cor pulmonale occurs from increased pulmonary vascular resistance secondary to capillary wall damage and reflex vasoconstriction in the presence of hypoxemia

Cardiac Rehab: Phase 1 and 2

Inpatient Phase 1 can be initiated after 24 hours of being stable Subacute Phase 2 can be initiated after discharge No resistance training FITT F = Frequency -short session 2-3x a day I = Intensity -50-70% HR max T = Time -10-15 mins (phase 1) and 30 mins (phase 2) per session T = Type -ADLs, supervised ambulation

What is the corneal reflex and which nerves does it test?

Involuntary blinking in response to corneal stimulation Afferent (sensory): opthalmic branch (V1) of CN 5 Efferent (motor): CN 7

What is the clinical course/feature of CRPS?

Involved limb evolves from an acute warm phase (limb is sensitive, swollen, and displays an elevated temperature) to a chronic cold phase (resolution of inflammatory appearance, decreased temperature, pain and disability persist). Also s/s travel from distal to proximal

Rules for Pupillary Light Reflex (MS pts)? (look at slide 27 FF#17)

Ipsilateral/CN3/Direct response Contralateral/CN3/Consensual response Ipsilateral/CN2/Both direct and consensual response ICI 332 BCD (backwards)

Which nerve roots does the obturator nerve stem from?

L2-L4

What are the different Accreditation Agencies and each of their purposes?

JACHO -American non-profit organization which evaluates health care organizations that voluntarily seek accreditation for hospitals, home care entities, behavioral health care organizations, laboratory, ambulatory care, and long term care services. Also accredits health care networks and health care plans. (Hospitals/SNF/Home Health Agencies/PPO/HMO/Mental Institutions CARF -An international program. Non-profit organization which provides accreditation services worldwide at the request of health and human providers. Accreditation for rehabilitation for disability, addiction and substance abuse, home and community services, retirement living, or other health and human services. CMS -Determines what and how much will be reimbursed by Medicare for pt care OSHA -Provide a workplace free from serious recognized hazards and comply with standards, rules and regulations issued under the OSH Act. Examine workplace conditions to make sure they conform to applicable OSHA standards. Make sure employees have and use safe tools and equipment and properly maintain this equipment.

Augmented Feedback: Knowledge of Results vs Knowledge of Performance

Knowledge of results -telling the result of what just happened Knowledge of performance -analyzing their form/technique of what they did Ex: KR = Michael Scott misses a basketball shot KP = He didn't bend his knees KR = you walked 10 ft today KP = you should bend your knees more as you walk

S/S of Diabetic Ketoacidosis?

Kussmaul respirations progressing to hyperosmolar coma (polyuria, thirst, neuro abnormalities, and stupor) HA Acetone breath Dehydration Weak and rapid pulse "Kuzma HAD Weak and rapid pulse"

Which nerve root affected with a weak knee jerk DTR?

L4

Rule for Latera inset/Medial outset for below knee amputee (BKA)

Lateral Inset. Medial Outset —> LIMO - thrusts Lateral thrust with foot too far inset Medial thrust with foot too far outset Foot that goes inset/knee will want to go lateral thrust Foot that goes outset/knee will want to go medial thrust

What are the shoulder ext muscles?

Lats Teres major Posterior deltoid Triceps (all heads)

Lead-pipe rigidity vs Cogwheel rigidity And what structure in the brain is it associated with?

Lead-pipe: increased/uniform resistance through the whole motion Cogwheel: increased, ratchet-like resistance Basal Ganglia

List the Gross Motor Classification of CP

Level 1 - Patient will walk without restrictions but will have limitations in more advanced gross motor skills. Level 2 - Patient will walk without assistive device with limitations in walking outdoors and in the community. Level 3 - Patient will walk with assistive device with limitations in walking outdoors and in the community. Level 4 - Patient self mobility will be severely limited; children are transported or use power mobility outdoors and in the community. Level 5 - Patient self mobility will be severely limited, even with the use of assistive technology. "I remember level 4 as 4 wheeled=wheelchair. level 3= 2 feet and an assistive device. Level 2= 2 feet."

How to treat cupulolithiasis? How do you perform the technique?

Liberatory (Semont) Maneuver -Hold each position for 1 minute -Pt rotates head 45 degrees toward UNAFFECTED side -Move pt in affected S/L position, head still in same position -Quickly move pt to nonaffected S/L position, head still in same position -Move pt in sitting position with head still in starting position

Type of compression therapy for lymphedema?

Low(short) compression -Phase 1 low stretch bandage provides low resistance on a limb but a high working pressure -can be worn during day and night -used also during active reduction phase of treatment -you want the lymph fluid to move around so low is better than high, high compression won't allow the fluid to move around -High stretch sport bandage like Ace wraps should NOT be used -can use high stretch in later stages of lymph rehab

Which part of the lung is cleared in a L/R sidelying 1/4 turned on a DECLINED bed position with arm raised?

Lower lobes: Lateral basal segment

Which part of the lung is cleared in a PRONE position on a DECLINED bed position with arm raised?

Lower lobes: Posterior basal segment

Muscle action of Lumbricals

MCP flex IP ext

What is Claw Fingers?

MCP hyperextension PIP/DIP flexion (think of a cat)

Different Types of Motor Learning

Massed practice -work > rest Distributed practice -rest equal to or greater than work Blocked practice -repeated task (AAAA) Serial practice -different tasks that are predictable (ABCABCABC) Random practice -random task (ABASDFWE)

Cardiac Rehab: Phase 4

Maintenance phase (pt is back in the community) Location -community centers, YMCA, or clinical facilities Exercise -clinically stable angina, medically controlled arrhythmias during exercise Progression -50-80% of functional capacity -3-4x/week -45 minutes or more/session Discharge -typically in 6-12 months

Pathologies related to ABG

Metabolic Acidosis: -Caused by Renal failure, Shock, Sepsis, DB Ketoacidosis Respiratory Acidosis: -Caused by Hypoventilation due to OD, Chest Trauma, L CHF, Airway Obstruction, COPD Metabolic Alkalosis: -Caused by loss of gastric secretions (vomit), antacid (GERD) Respiratory Alkalosis: -Caused by hyperventilation due to Anxiety, High Altitude, Pregnancy, Fever, Hypoxia

Position Strategies to Reduce Common Malalignments: S/L on less affected side

More Affected UE -Scap = PROTRACTED -Shoulder = FORWARD/SLIGHTLY ABDUCTED, arm supported on pillow -Elbow = EXTENDED -Wrist = NEUTRAL -Fingers = EXTENDED -Thumb = ABDUCTED

Difference between: -Murphy's Sign -McBurney's Point -Rovsing's Sign -Blumberg Sign/Rebound tenderness

Murphy's Sign -Cholecystitis -gall stones in gall bladder -inflamed gall bladder McBurney's Point -Appendicitis -palpate RLQ -pain at RLQ Rovsing's Sign -Appendicitis -palpate LLQ -pain at RLQ Blumberg Sign/Rebound Tenderness -Peritonitis -palpate RLQ and let go quickly -pain at RLQ

Parameters ESTIM for muscle contraction -Muscle strengthening -Muscles spasm reduction -Edema reduction

Muscle strengthening -pulse frequency = 35-80 pps -pulse duration = 150-200 (small muscles) and 200-350 (large muscles) -on:off ratio = 1:5 -ramp time = at least 2 seconds Muscle spasm reduction/Edema reduction -pulse frequency = 35-50 pps -pulse duration = 150-200 (small muscles) and 200-350 (large muscles) -on:off ratio = 1:1 (equal) -ramp time = at least 1 second (everything same except for frequency/on:off ratio/ramp time)

What is the rule for pelvic drop?

R glute med = weak R add magnus = tight L add magnus = weak L glute med = tight L glute med = weak L add magnus = tight R add magnus = weak R glute med = tight

Appendicitis: 2 more special tests

Obturator sign = pain with passive hip IR with flexed thigh and flexed knee Psoas sign = pain with passive hip ext in S/L

DD: Orthostatic Hypotension vs Autonomic Dysreflexia

Orthostatic Hypotension -common in pts with cervical injuries -dizziness or light headedness and ringing in the ears when in a vertical position like sitting or standing Autonomic Dysreflexia -common in SCI pts T6 or above -results in increased BP and pounding HA

DD: Jumper's Knee vs Osgood Slaughter

Osgood Schlatter: -inflammation of the patellar ligament at the Tibial Tuberosity (apophysitis) -due to growing pains (growing too fast) "osgood schlaTTers" Jumpers Knee: -patellar tendinitis, seen with runners and has no avulsion fracture

Cardiac Outpatient Rehab Phase 3: FITT

Outpatient Phase 3 patients commonly undergo a symptom limited maximal stress test (ETT) at 4 to 6 weeks after MI (phase 2 can be inpatient or outpatient) F = 2-3 sessions/week I = 70% to 85% of the peak achieved on the test (HR max) T = 30-60 minutes with 5-10 min of warm up and cool down T = single mode of training (walking) or multiple modes using (treadmill, cycle, ergometer)

Functions of different types of breathing

Paced breathing -used when a person becomes dyspneic during an activity Pursed lip breathing -used to decrease respiratory, dyspnea, and facilitate relaxation for COPD pts Lateral costal breathing -used to expand one segment of the lung Diaphragmatic breathing -used for relaxation

Loss or absence of sensibility to vibration

Pallanesthesia

Difference between parametric data vs non-parametric data

Parametric = equally distributed data (t-test/1 tailed/2 tailed) ex: equal bell shaped curve Non-parametric = not equal distributed or skewed data (Chi square/Mann-U Whitney) ex: unequal/skewed bell shaped curve ex: PT vs ATs in the NBA, there are more ATs in the NBA than PTs so its non-parametric data

Difference between Integrated and Persistent for neonatal terminology?

Persistent: a reflex that has NOT integrated and indicative of CNS damage P = Problem Integrated: when the reflex disappears to allow for normal development

What are the lab values associated with terminating exercise?

Platelet count: <20,000 WBC count: <5,000 Hematocrit: <25% Hemoglobin: <8g

What is a PVC?

Premature ventricular contraction -no P wave -ventricles start contracting before the atria -you see the QRS complex overtake the P wave STOP EXERCISE IMMEDIATELY: ventricles are not receiving any blood from the atria

What are the functions associated with parietal lobe lesion?

Primary sensory cortex Sensory loss Taste affected Perceptual disorders like tactile agnosia types if lesion of Rt hemisphere PARietal lobe: PARent lobe. A PARent senses everything when it comes to their kid. This is primary sensory cortex. PT TP: Pain Temperature Touch Proprioception —> Parietal Lobe

What are the functions associated with occipital lobe lesion?

Primary visual cortex Visual loss Contralateral homonymous hemianopsia Inability to identify previous known objects HH hemianopia is loss of one sided vision. A right HH will have affection of Right temporal field and Left nasal field affecting right sided loss of vision Where would the lesion be for Right sided HH? L optic tract

Cheat sheet key words for Diagnosis/Girth measurement for Lymphedema? Proximal Distal (UE/LE) Pre-post surgery Lymphatic insufficiency

Proximal = Girth measurement Distal (UE/LE) = Volumetric Pre-post surgery = Bioelectric impedance Lymphatic insufficiency = Lymphoscintigraphy

Breathing Exercises

Pursed lip breathing -involve an unresisted inspiration followed by an active oral exhalation throughout the narrow (pursed) mouth opening -use for dyspnea and wheezing Diaphragmatic breathing -hypoxemia, tachypnea, atelectasis, anxiety, excess pulmonary secretions Paced breathing -low endurance (ambulation/stairs), dyspnea, fatigue, anxiety, tachypnes Assisted cough -PT hand or fist force behind pt exhaled air, used when pt abdominal muscles cannot generate cough (SCI) Tracheal stimulation -PT finger or thumb is placed above suprasternal notch and quick inward and downward pressure on trachea elicit cough reflex

What are the s/s of each ABG

Respiratory Acidosis S&S "CARBS" - the pH is less than 7.35, the PaCO2 is >45mmHg Confusion Agitation/Anxiety Restlessness Blurred Vision Seizures Respiratory Alkalosis S&S "NO CARDS" - pH is more than 7.35, the PaCO2 is <35mmHg Numbness and Tingling Orthostatic Hypotensive Confusion Anxious Rapid breathing Dizziness Seizures Metabolic Acidosis S&S "SHAMED" - pH is less than 7.35 and HCO3 (Base) is <22 meq/L Stupor Hyperkalemia Arrhythmias Muscle twitching Emesis Decreased CO/contractility Metabolic Alkalosis S&S "TTTT" - pH is more than 7.45 and the HCO3 is >26 meq/L Tetany Tachycardia Tremors Tingling Respiratory Alkalosis is caused by HYPERVENTILATION. Now that you know this, Respiratory Acidosis CAN'T be HYPERVENTILATION, so you know that it HAS TO BE Metabolic Acidosis as the HYPERVENTILATION. You also know that Respiratory Acidosis AND Metabolic Alkalosis is HYPOVENTILATION. *In remembering that respiratory alkalosis as anxiety and being caused by hyperventilation then you can figure out the rest Respiratory Alkalosis is caused by HYPERVENTILATION. Now that you know this, Respiratory Acidosis CAN'T be HYPERVENTILATION, so you know that it HAS TO BE Metabolic Acidosis as the HYPERVENTILATION. You also know that Respiratory Acidosis AND Metabolic Alkalosis is HYPOVENTILATION. *In remembering that respiratory alkalosis as anxiety and being caused by hyperventilation then you can figure out the rest

List the RPE scale What level is expected for pregnant/cancer/obese pts?

Resting HR - 60 —> Remove "0" and it's RPE of "6". That's no exertion at all. Maximal exertion is 200 (200-age_ = 220 - 20 = 200 HR). Move the "0" of "200" to get "20" max exertion on RPE TO remember: Start at lucky 13: Somewhat Hard. Then spell SHVEM. 13: "S"omewhat hard 15: "H"ard 17: "V"ery hard 19: "E"xtremely hard 20: "M"aximal exertion. SHVEM sounds like something you get on you in the ICU. For the other half of the chart it's just the opposite. Yuck I got some SHVEM on me. RPE: Patient with Cancer: 11-13. RPE: Obesity go up to 17 RPE: Pregnancy: 11-13

Rigidity vs Spasticity

Rigidity: -increased resistance to passive movement that affects BOTH agonist and antagonist muscles at the same time -NOT velocity dependent Spasticity: -velocity dependent and involves either agonist OR antagonist muscle at one time

Scleroderma vs Lupus vs Warts vs Psoriasis

Scleroderma -inflammation and fibrosis of many parts of the body, including the skin, blood vessels, synovium, skeletal muscle, and certain internal organs like kidneys, lungs, heart, and GI tract Warts -benign infection caused by human papilloma virus seen on skin like hands/fingers/pressure points of feet Psoriasis -autoimmune disease of skin characterized by silvery scale plaques causing itching, dryness over knees, scalp, elbows, genitals Lupus -chronic autoimmune disorder affecting skin/joints/kidney/nervous system -characteristic butterfly rash across bridge of nose

Distinguish between sensitivity vs specificity

Sensitivity: SNOUT -high sensitivity test comes out Negative, so rule OUT -ex: metal detector at TSA, you go through it, and comes out negative, so they rule you out as a threat Specificity: SPIN -high specificity test come out Positive, so rule IN -ex: pregnancy test is positive, so it rules in that you are pregnant Formula -Sensitivity = TP / (TP + FN) -Specificity = TN / (FP + TN) REMEMBER HOW TO DRAW THE SQUARE FOR THE FORMULA

Irritable Bowel Syndrome

Spastic, nervous, or irritable colon LL quadrant pain Causes: emotional stress, anxiety, high fat, lactose foods (DPT student) -pain is relieved by defecation -sharp cramps in the morning or after eating -N/V, bloating, foul breath, diarrhea -symptoms disappear while sleeping Treatment -stress reduction -dietary modification -exercise

4 classifications of Cerebral Palsy?

Spastic: velocity dependent resistance of a muscle to stretch Dystonic: tonic hold of muscles and contributes to passive joint stiffness Ataxia: disorder of coordination, force, and timing, associated with cerebellar involvement Athetosis: disorder of basal ganglia, characterized by involuntary movements that are slow and writhing

Distinguish between spondys

Spondylosis: degeneration of the IV disc Spondylolysis: defect in pars interarticularis or the arch of the vertebra (Scotty dog) Spondylolisthesis: forward displacement of one vertebra over another Retrolisthesis: backward displacement of one vertebra on another

Assistive Devices: Standard Crutches (Axillary Crutches) Forearm Crutches (Lofstrand Crutches) Cane

Standard Crutches (Axillary Crutches) -Crutch is 6" lateral and 6" anterior to the pt's foot -Axilla space of 2-3" Forearm Crutches (Lofstrand Crutches) -Top of forearm cuff is just distal to elbow Cane -Always goes on the opposite side of effected side -Ascending = Good foot goes up first followed by bad foot and cane -Descending = Bad foot and cane first followed by good foot "Good foot goes to heaven, Bad foot goes to hell"

What is the purpose of the following lumbar special tests? Stork Standing test Gillet Test Van Gelderen Bicycle Test Quadrant Test

Stork Standing Test = spondylolisthesis Gillet = SI cluster Van Gelderen Bicycle Test = spinal stenosis Quadrant Test = facets

4 Types of Urinary Incontinence

Stress incontinence -Involuntary leakage of urine during cough, sneezing, or exertion Urge incontinence -Involuntary contraction of the detrusor muscle with a strong desire to void (urgency) -the urge is there to urinate -ex: a familiar sound triggers your urge to urinate Functional incontinence -Incontinence due to mobility, dexterity, or cognitive deficits Overflow incontinence -Incontinence caused by neurological condition that causes a disruption of the activity of the detrusor muscle

Interventions for different types of Incontinence?

Stress incontinence -strengthen pelvic floor muscles (weak muscles) Urge incontinence -treat infections (UTI/bacteria) Functional incontinence -clear clutter and prompted voiding (unable to reach the bathroom due to cognition or bathroom is physically too far) Overflow incontinence -medication and catherization (DM/overflow of urine)

Rules for Lumbar Traction

Supine position -with pillow under knee: for intervertebral joints, facet joints, muscle elongation Prone position -for posterior disc herniation Increase IV space of -L5/S1 = 45-60 degree hip flex -L3/L4 = 75-90 degree hip flex Disc protrusion, spasm, elongation -25% body weight Joint distraction -50 lbs or 50% body weight (7-10% body weight for cervical)

How to TEST/DX for horizontal canalithiasis vs cupulolithiasis?

Supine roll test -(+) = horizontal nystagmus WITHOUT torsion Canalithiasis = Geotropic nystagmus (toward the Ground) -involved ear to treat is the side with the STRONGER nystagmus Cupulolithiasis = Ageotropic nystagmus (Away from the ground) -involved ear to treat is the side with the WEAKER nystagmus

Muscles that do ankle inversion/DF

TA EHL (Deep peroneal nerve)

When should you terminate exercise in regards to pt vitals?

TERMINATE EXERCISE HR: Sudden drop >15bpm, change from regular to irregular rhythm or exceeds HR maximum OR acute cardiac pt. If HR raises >20 bmp or if bmp if on BETA BLOCKERS SBP >200 mmHg, decrease to <90 mmHg >10 mmHg from resting or with increased exercise OR acute cardiac pt if SBP raises >20 mmHg DBP >110 mmHg OR in an acute cardiac pt if DBP raises >20 mmHg or drops >10 mmHG

Muscles that do ankle inversion/PF? Nerve supply?

TP FHL (Tibial nerve)

4 cardinal signs of PD?

TRAP T = Tremors (pin rolling) R = Rigidity (cogwheel or leadpipe) A = Akinesia, Bradykinesia P = Postural instability Early signs -loss of sense of smell -constipation -sleep disorders Other s/s -forward head posture -backward leaning when upright (impaired balance and coordination)

Types of Aphasia

TWARF • Temporal Lobe • Wernicke's • Aphasia • Receptive • Fluent FBEAN • Frontal Lobe • Broca's • Expressive • Aphasia • Nonfluent

What is tachycardia and bradycardia ranges?

Tachycardia: HR above 100 Bradycardia: HR below 60

Vascular lesion of the thalamus resulting in sensory disturbances and partial or complete paralysis of one side of the body, associated with severe, boring-type pain; sensory stimuli may produce an exaggerated, prolonged, or painful response?

Thalamic syndrome

What is cardiac output? What is VO2 max?

The amount of blood coming into your body SV x HR = CO SV = how much blood is being pumped with one heartbeat HR = how many times the heart is beating VO2 max = volume of oxygen consumed

Coronary Artery Bypass Graft

The usage of a donor vessel to bypass the lesion and establish an alternate improved blood supply -vessels used: radial artery/saphenous vein/internal mammary artery Prolonged time in the crucifix position during surgery can cause the pt to develop ulnar nerve palsy -Examination of sensation and MMT is indicated to rule out brachial plexus injury PT Intervention -ROM with awareness of soreness/discomfort pt may have -If sternal wound is present, posture/scapula retraction and functional shoulder movements are indicated

Inability to perceive sensations of heat and cold?

Thermanesthesia

Increased sensitivity to temperature?

Thermhyperesthesia

Decreased temperature sensibility?

Thermhypesthesia

Which tendons does the post tib nerve travel with behind the medial malleolus?

Tib post FDL FHL (TDH)

What is Inspiratory Capacity?

Tidal volume + IRV Volume of air that is inspired beginning from a tidal exhalation

What is upper crossed syndrome?

Tight/Short -levator scap -upper trap -pec major/minor -SCM Weak -deep neck flexors -lower scap stabilizers (low traps/rhomboids) Promotes forward head posture and scapular winging

What is lower crossed syndrome?

Tight/short -erector spinae -iliopsoas Weak -abdominals -glutes Promotes anterior pelvic tilt and lumbar lordosis

High fall risk scores for OM?

Tinetti (POMA) = <19 Functional Reach Test (FRT) = <10 TUG = >30 seconds Berg Balance = 45 or lower "19 TINy PuMAs ate 10 FRuiTs and played TUG a war for 30 seconds while BALANCing on 45 BuRGers"

Position Strategies to Reduce Common Malalignments: Sitting in an Armchair or WC

Trunk = Spine EXT Pelvis = aligned in neutral with WB on both buttocks More Affected UE -Scap = PROTRACTED -Shoulder = FORWARD -Elbow = supported on arm trough or lapboard -Wrist = NEUTRAL -Fingers = EXTENDED -Thumb = ABDUCTED Both LEs: -Hips = FLEXED to 90 degrees and neutral rotation

What are the rules for Trunk lean during stance phase?

Trunk lean is towards the side of weakness on stance phase Backward trunk lean = hip ext weakness Forward trunk lean = knee ext weakness Lateral trunk lean = g med weakness (trunk lean during swing phase is opposite)

Distinguish UMN vs LMN characteristics

UMN -Stroke/TBI/SCI -Hypertonic -Hyperreflexia/clonus/+Babinski -Muscle spasms -Muscle weakness: unilateral (stroke) or bilateral (SCI) LMN -Polio, GBS, Peripheral nerve injury/neuropathy/radiculopathy -Hypotonic/flaccid -Hyporeflexia -Fasciculations (muscle twitch) -Muscle weakness: unilateral only

Inflammatory Bowel Disease: Ulcerative Colitis vs Crohn's Disease

Ulcerative Colitis -occurs in the large intestine and rectum -continuous lesions (lesions formed throughout the tract) -LL quadrant pain = uLcerative coLitis Symptoms: -rectal pain -bleeding -LBP -fecal urgency -bloody diarrhea with mucus/pus -weight loss Crohn's Disease -occurs anywhere in the GI tract -skip lesions present (lesions formed in intervals along the tract) -RL quadrant pain = cRohn's disease Symptoms: -pain relieved by farting -joint arthritis -abdominal pain -weight loss

Difference between venous and arterial insufficiency? (location/appearance/symptoms)

Venous: location = proximal to MEDIAL MALLEOLI appearance = irregular, shallow appearance/flaking, dry skin, brown discoloration symptoms = mild to mod pain/elevation decrease pain/ gravity dependent VENMO (VENous is Medial malleoli) Arterial: location = lower 1/3 of leg, toe, LATERAL MALLEOLI appearance = smooth edges, well defined, tend to be deep/thin and shiny, hair loss, yellow nails symptoms = severe pain/elevation increase pain/ gravity independent ALARM (Arterial is LAteral malleoli)

BPPV symptoms?

Vertigo -with change in head position such as when turning over in bed, getting into or out of bed, or when bending over/coming up Nausea -with or without vomiting Nystagmus -involuntary, rapid, and repetitive movement of the eyes -Most important symptom

What is the order of recruitment for balance systems (CTSIB) from greatest to least?

Vision Somatosensory Vestibular

What is RV (residual volume)?

Volume of air left in the lungs after ERV

What are the changes associated with pregnancy?

Weight gain : -20-30lbs. Essential for baby's nourishment. MSK system: -Postural changes - Forward head, Kyphosis, increased lordosis, Anterior pelvic tilt. -Postural stress continues even post partum due to lifting and carrying of baby. Treatment : -Postural education, stretching of tight muscles and strengthen weaker ones, pelvic stabilization exercises, Pelvic tilts (No supine lying in 3rd trimester) -Blood pressure is low in first and second trimester and may increase in the last trimester. -Supine lying can cause compression of inferior vena cava (after 4th month). This declines CO and may cause supine hypotensive syndrome. -Left side lying is considered the best as it decreases compression of IVC, maximizes CO, decreases GERD as internal organs are relaxed and improves maternal and fetal circulation.

Brain: CT vs MRI

X-ray = cheap CT scan = 3D X-ray MRI = expensive -T1 vs T2 T1 images - 1 tissue type is bright - FAT T2 images - 2 tissue types are bright - FAT and WATER "World War 2: Water is White in T2 scan (MRI). T1 scan (MRI) shows fat as being white." Myelogram = spine

Loud sudden noise causes blows to remain flexed and hands closed a. Startle reflex b. Plantar grasp reflex c. Rooting reflex d. Moro reflex

a. Startle reflex 28 weeks of gestation to 5 months

Head elevation to 45 degress in prone, prone on elbows with elbows behind shoulders Head bobs in supported sitting Does not accept weight on LEs Responds to friendly handling? Which developmental age? a. 1 month b. 2 month c. 4 month d. 6 month

b. 2 months

The patient suffered an injury on the right side and is having difficulty in pronation of the right arm. Which nerve is MOST likely injured a. Ulnar nerve b. Median nerve c. Radial nerve d. Suprascapular nerve

b. Median nerve The pronator muscles of forearm i.e. pronator quadratus, pronator teres are supplied by median nerve. So, injury to median nerve will result in weak pronation.

Rule of 9's percentage for kids (under 14 y.o) head and unilateral LE

head = 8.5% LE = 6.5%

Herpes zoster: 4 topics covered

herpes zoster has a dermatomal pattern and can be location anywhere on the body -dermatomes -integumentary -other systems -non-systems: TENS pain control

4 things to examine with wounds

location characteristics wound edges wound drainage and odor

A PT was assessing a patient's balance response to perturbations. The perturbation given was a slow- paced, small amplitude posterior perturbation. The patient showed a normal balance response and was able to maintain stability. For a normal balance response, what would be the order of muscle activation? A. Tibialis Anterior, Quadriceps, Abdominals B. Paraspinals, Hamstrings, Gastrocnemius C. Gastrocnemius, Hamstrings, Paraspinals D. Abdominals, Quadriceps, Tibialis Anterior.

A. Tibialis Anterior, Quadriceps, Abdominals Distal to proximal for ankle Proximal to distal for hip

A study is looking at effects of different exercise options for a patient with knee OA. The study compared pool exercise, treadmill walking, and ankle weights on knee pain in 30 adults An appropriate statistical test to assess the effectiveness of treatment in three groups is: A. Spearman's rho B. Analysis of covariance C. ANOVA D. Chi square

C. ANOVA ANCOVA would be correct if a height factor was involved (variant)

What is asthma?

Common chronic pulmonary disease characterized by chronic airway inflammation associated with airway hypersensitivity resulting in bronchospasm. Leads to airway narrowing and airflow resistance causing air to be trapped

What are the shoulder abd muscles?

Deltoid Supraspinatus

Distinguish between Ulcerative colitis/Diverticular Disease/Crohn's Disease/IBS

IBS -functional motility disorder of small/large intestines -abdominal pain/discomfort relieved with defecation and pain disappears at night while sleeping -pain pattern is a LL (lower left) quadrant abdominal pain/constipation/diarrhea Crohn's disease -Pain at LR (lower right) quadrant Ulcerative colitis -inflammation and ulceration of inner lining of large intestine and rectum -symptoms are diarrhea/rectal bleeding Diverticulitis -infection and inflammation that accompany a microperforation of one of the diverticula

What is Vital Capacity/Forced Vital Capacity? (VC or FVC)

IRV + TV + ERV All of the possible volume of air that is under volitional control

Referred pain for: -L shoulder -R shoulder -Mid-back/scapula -Pelvis/low back/sacrum?

L shoulder -heart -tail of pancreas -diaphragm -spleen R shoulder -gall bladder -liver -head of pancreas -peptic ulcers "Good Luck Have Pun" Mid-back/scapula -pancreas -esophagus -gall bladder -stomach "feed PEGS in my mid back/scap region" Pelvis/Low Back/Sacrum -colon -appendix -pelvic viscera "there's a CAP in my low back"

How to distinguish between L3 or L4 or L5 nerve root lesion?

L3: weak quads sluggish knee jerk DTR L4: weak ankle DF/big toe ext weak knee jerk DTR L5: weak big toe ext/ankle EV/hip abd and ER weak ankle jerk DTR

Which nerve root does the Tibial/Post tib nerve come from?

L4-S3

Weight Reduction Guidelines for Obese Adults

Minimum of 250-300 min/week is required Frequency -3-5 days/week to maximize caloric expenditure Intensity -Initially moderate at 40-60% -Progression to 50-75% Time -from 45-60 min/day Type -moderate exercise

What are 2 unique interventions for CRPS?

Mirror Therapy: used to correct the sensorimotor incongruence by visualizing the unaffected limb in the felt position of the affected limb Graded motor imagery: targets activation of different brain regions in a graded manner (L/R discrimination, motor imagery rehearsal, mirror therapy)

Muffled Lung Sounds Mnemonic

Mnemonic: Muffled lung sounds are considered normal Increased or clear sounds are abnormal and signs of fluid in the lungs Whispered Petriloquy- saying 123 in a whisper and being able to hear it Pectus + Loquor =Chest speak -Increased resonance so whispered sound presents as audible Egophony- Saying E and hearing A Mneumonic- I have such a big ego teacher tells me you got an E but all I hear is A Bronchophony-Saying 99 and hearing 99= 99 Bronchos

Which muscles perform the following muscle actions of the TMJ? Mouth Opening Mouth Closing Protrusion of Mandible Retraction of Mandible Lateral Deviation of Mandible

Mouth Opening -lateral pterygoid -norm ROM = 35-55mm Mouth Closing -temporalis -masseter -medial pterygoid Protrusion of Mandible -medial pterygoid -lateral pterygoid Retraction of Mandible -temporalis (posterior fibers) Lateral Deviation of Mandible -lateral pterygoid (ipsilateral muscle) -medial pterygoid (contralateral muscle) "LA LA LAAAAAAteral pterygoid —> As if you were OPENING your mouth to sing. MMMMMMMMedial pterygoid —> To make the sound "MMMM" your mouth is CLOSED." "When you say the Alphabet L - you open the mouth and say L so lateral pterygoid to open the mouth When you close your mouth you say Alphabet M - so medial pterygoid closes the mouth."

What is Multifocal PVC and Couplet?

Multifocal PVC = more than 1 PVC is present and 2 do not appear similar in configuration Couplet = 2 consecutive PVCs together with no normal beat between them = STOP EXERCISE!!! -couplet will most likely lead to 3 PVCs (v-tach) (look at slide 29 FF#9)

What is the ranking of levels of evidence from strongest to weakest?

My: Meta-analysis System: Systematic Really: Randomized Controlled Trials Helps: CoHort Control :Case control aCross: Cross-sectional studies world Series: Case Series Reports: Case Reports

Distinguish between OA vs RA

OA -usually after age 40 -develops slowly over many years in response to mechanical stress -cartilage degradation, altered joint architecture, osteophyte formation -usually asymmetrical -morning stiffness, increased pain with activity, crepitus and loss of ROM RA -usually between age 15-50 -may develop suddenly, within weeks or months -inflammatory synovitis and irreversible structural damage to cartilage and bone -usually bilateral -redness, warmth, swelling, and prolonged morning stiffness -increased pain with activity -general feeling of sickness and fatigue, weight loss, and fever; may develop rheumatoid nodules; may have ocular, respiratory, hematological, and cardiac symptoms

Muscle action for Flexor Digitorum Superficialis (FDS)?

PIP flex

Convert RPE to Max HR

RPE vs Max HR 43224 These numbers stand for how many numbers in the RPE scale are being included in that section. RPE 4(6,7,8,9): Max HR 50-60% ————— RPE 3 (10,11,12):Max HR 60-70% ____ RPE 2(13,14): Max HR 70-80% ____ RPE 2(15,16): Max HR 80-90% _____ RPE 4(17,18,19,20): Max HR 90-100% ** I just used another trick to match RPE and HR max: Here it goes. Start at 9 RPE being 50% and then add 5% for every number increase . 9= 50% 10=55% 11=60% 12=65% 13=70% 14=75% 15=80% 16=85% 17=90% 18=95% 19=100%

Distinguish between Swan Neck Deformity vs Boutonniere Deformity What condition is it related to?

Swan Neck = DIP flex/PIP ext Boutonniere = PIP flex/DIP ext RA

A 29-year-old patient presents to the clinic with pain on the lateral aspect of the shoulder. The physical therapist performs a manual muscle tests and finds weak shoulder abductors (3+/5). Based on patient presentation, which of the following nerve is MOST likely affected? a. Long Thoracic nerve b. Suprascapular nerve c. Thoraco-dorsal nerve d. Median nerve

b. Suprascapular nerve The suprascapular nerve provides the motor supply to the supraspinatus and infraspinatus muscles which help in shoulder abduction movement. So, if this nerve is injured the patient can have weak shoulder abduction. Long thoracic nerve supplies Serratus Anterior muscle. Thoraco-dorsal nerve supplies Latissimus dorsi

A rock climber injured his right foot and right hand. He comes to a PT clinic and reports a bowstring injury. Which of the following structures are most likely to be injured which muscles? a. The Cruciform pulleys b. The Annular pulleys c. Lumbrical muscles d. Wrist extensors crossing the elbow

b. The Annular pulleys Annular pulleys along with other flexor pulleys share a similar function of holding the underlying tendons at a relatively close distance to the joints. With injury to these structures, the force of a strong contraction of the extrinsic finger flexors causes the tendon to pull away from the joint's axis of rotation, a phenomenon referred to as "bowstringing" of the tendon.

A patient presents to the clinic four weeks after a reverse arthroplasty of the right shoulder. Which of the following movements would you perform while treating the patient at this stage? Select one: a. Shoulder ER beyond 20 degree b. Shoulder extension beyond 0 degree c. Shoulder elevation in scapular plane up to 90 deg d. Shoulder IR up to 30 degree

c. Shoulder elevation in scapular plane up to 90 deg Rationale: Phase 1 of rehabilitation post reverse shoulder arthroplasty lasts for up to 6 weeks. The motions that are restricted during this period include: -No GH extension or internal rotation past neutral -No combined GH extension, adduction, and internal rotation -0°-20° external rotation -Up to 90°-120° arm elevation in scapular plane

Prone position results in body and extremities in flex Supine position results in body and extremities in ext a. Symmetrical tone neck reflex b. Asymmetrical tonic neck reflex c. Tonic labyrinthine reflex d. Galant reflex

c. Tonic labyrinthine reflex Birth to 6 months

A physician is assessing how well a baby is doing immediately after birth and how well the newborn is doing outside the womb. Which of the following findings indicate the BEST result? Select one: a. Crying baby, HR<100 bpm, some muscle tone, grimacing present, pink body with blue extremities b. Entire body is pink, active motion of extremities, grimacing present, HR <100, Irregular and slow respiration c. Crying baby, HR< 100 bpm, pink body, flexed arms and legs, grimacing present and a cough d. Crying Baby, HR >100 bpm, pink body with blue extremities, prompt response to stimulus, active movements of extremities

d. Crying Baby, HR >100 bpm, pink body with blue extremities, prompt response to stimulus, active movements of extremities Rationale: (look at Apgar Scoring Table) Option A- 2+1+1+1+1=6/10 Option B-2+2+1+1+1=7/10 Option C: 2+1+2+1+2=8/10 Option D: 2+2+1+2+2=9/10

A patient presents to the clinic with a chief complaint of pain on the sole of the foot. The patient's symptoms were gradual in onset with no history of trauma. Weight-bearing on the affected extremity and hyperextending the great toe aggravate the symptoms. Tenderness is present in the web spaces of the toes with pressure. The patient has full pain-free AROM and 5/5 strength with no increase in symptoms during testing. Which of the following is the MOST LIKELY diagnosis for this condition? Select one: a. Plantar fasciitis b. Midfoot sprain c. Retrocalcaneal bursitis d. Morton's neuroma

d. Morton's neuroma Rationale: Morton's Neuroma is gradual in onset with no known cause. The symptoms include pain on the sole of the foot which increases with WB with overpressure into toe extension during passive ROM. AROM is full and pain free with intact strength. The foot may be pronated with low arches with pain present in the web spaces of the toes. Morton's test is usually positive i.e. squeezing the metatarsal head reproduces the pain. Pain in plantar fasciitis is present on the insertion of the plantar fascia on the calcaneus and weak intrinsic muscles of the foot. Retrocalcaneal bursitis pain is usually just above the insertion site of the Achilles tendon on the calcaneus. Midfoot sprain involves a history of trauma or high-impact activity with generalized pain on the midfoot.

· What does the musculocutaneous nerve become as it travels distally?

o Lateral cutaneous nerve of the forearm

· Which cord does the axillary nerve come from?

o Posterior

Diagnostic characteristics with ALS?

• Lou Gehrig's disease • Most common and devastatingly fatal Motor Neuron Disease among adults • It presents with denervated muscle Note: Cervical extensor weakness is typical and finger dexterity issues (difficulty with zippers/buttoning)

Different types of dressings?

• Very mild exudate- Transparent films • Minimal exudate- Hydrogel dressing, Hydrocolloid • Moderate exudate- Foams • Heavy exudate- Alginates (max capacity) The = Transparent = very mild Hot = Hydrogel/Hydrocolloid = min Fire = Foams = mod Ah! = Alginate = heavy Wound Dressing Occlusive Best to Worst: Happy = Hydrocolloid Hour = Hydrogel For = Foam Friends = Film IS = Impregnated Gauze Always = Alginate Good = Gauze macerated wound = too much moisture like when you're in the water too long and your skin gets wrinkly

What is Multiple Scerlosis (MS)?

• is an autoimmune disease characterized by inflammation, selective demyelination, and gliosis. • Chronic demyelinating disease of the central nervous system • Any area of the CNS can be affected: cortical, subcortical, cerebellum, spinal cord (UMN disease) • Occurs more in females (2:1) • Onset 20-40, peak age 30 • Occurs more in Caucasians White females around 30 y.o affected

Safety and Precautions: Central Venous Pressure Catheter vs Arterial Line vs Chest Drainage Tube

"Hickman's catheter is a central venous catheter most often used for the administration of chemotherapy or other medications, as well as for the withdrawal of blood for analysis and can also be used for measuring central venous pressure. Hickman lines are inserted through internal jugular or cephalic veins and may remain in place for extended periods and are used when long-term intravenous access is required. >Swan Ganz catheter is a pulmonary artery catheter. Pulmonary artery catheterization is the insertion of a catheter into a pulmonary artery. Its purpose is mainly diagnostic; it is used to detect heart failure or sepsis, monitor therapy, and evaluate the effects of drugs. >Arterial lines are inserted into an artery and are usually used for continuous monitoring of blood pressure and blood sample collection. >CVP is central venous pressure catheter mainly used to measure pressures in right atrium and superior vena cava. Reference: Patient care Pierson and Fairchild's pg. 273-275"

Treatment for GERD?

-Maintain upright positions -Eat meals at least 3-4 hours before sleep -AVOID supine: tends to straighten the esophagus -Sleep on the LEFT side preventing nocturnal reflex -Exercise must be completed 2-3 hours after eating or before meals -AVOID spicy, chocolate, and fatty food Drugs -Antacids -H2 receptor blockers -Proton pump inhibitors

A physical therapist is completing an examination of an inpatient with multiple comorbidities who had a total knee arthroplasty 2 days ago. The therapist observes a bluish discoloration of the nail beds of the toes on the operative extremity. This finding is MOST often associated with which of the following conditions? 1. Decreased peripheral blood flow 2. Deep vein thrombosis 3. Lymphedema 4. Aneurysm

1. Decreased peripheral blood flow 1. Cyanosis is described in the stem. Peripheral cyanosis is associated with decreased peripheral blood flow (p. 166). 2. Deep vein thrombosis is associated with edema and localized tenderness (p. 257). 3. Lymphedema is associated with swelling of the feet (p. 257). 4. Aneurysms are associated with a palpable, pulsing mass (p. 200).

A patient with which of the following injuries has the BEST prognosis for nonsurgical recovery? 1. Neurapraxia 2. Axonotmesis 3. Neurotmesis 4. Complete nerve severance

1. Neurapraxia 1. Neurapraxia involves blockage that stops or slows conduction across that point in the nerve. Conduction above and below the blockage is usually normal. Recovery is possible. 2. With axonotmesis, the neural tube is intact, but axonal damage has occurred with Wallerian degeneration. Surgical intervention may be required. 3. Neurotmesis involves total loss of axonal function, with disruption of the neural tube. Surgical intervention is usually required. 4. The nerve is completely separated. Surgical intervention is required.

Which of the following conditions would MOST likely be experienced by a 7-year-old patient who has a tethered spinal cord? 1. Upper extremity hypertonicity 2. Bladder dysfunction 3. Blurred vision 4. Headache

2. Bladder dysfunction 1. Upper extremity hypertonicity is consistent with shunt dysfunction, not tethered cord (Palisano, p. 554). 2. A tethered spinal cord occurs when adhesions anchor the spinal cord at the site of the lesion as individuals with spina bifida grow. The tethering of the spinal cord can result in rapidly progressive scoliosis, hypertonicity in the lower extremities, changes in gait, and changes in urologic function. (Tecklin, p. 285) 3. Blurred vision is consistent with shunt dysfunction, not tethered cord (Palisano, p. 554). 4. Headache is consistent with shunt dysfunction, not tethered cord (Palisano, p. 554).

A child with trisomy 21 is MOST likely to have which of the following conditions? 1. Athetosis 2. Seizures 3. Hypotonia 4. Hydrocephalus

3. Hypotonia 1. Athetosis is not associated with trisomy 21 (Down syndrome) but is characteristic of a type of cerebral palsy (pp. 193, 384). 2. Seizures are uncommon in children with trisomy 21 (Down syndrome) but are associated with cerebral palsy (pp. 190, 384). 3. Children with trisomy 21 (Down syndrome) have hypotonia (p. 392). 4. Hydrocephalus is uncommon in trisomy 21 (Down syndrome) but is commonly associated with spina bifida (pp. 249, 384).

In a patient with breast cancer, which of the following sites are the MOST common sites of metastasis? 1. Central and peripheral nervous systems 2. Liver and gastrointestinal tract 3. Lungs and bony skeleton 4. Spleen and pituitary gland

3. Lungs and bony skeleton 1. The central nervous system is a common metastatic site, but not the peripheral nervous system. 2. The liver is a common metastatic site, but not the gastrointestinal tract. 3. Lungs and bones are common metastatic sites for breast cancer. 4. Neither the spleen nor the pituitary gland is a common metastatic site for breast cancer.

A 58 year old male underwent a total hip arthroplasty on the left side using a postero-lateral approach. The physical therapist is working on a gait training program for the patient. The therapist should instruct the patient to hold the crutch or cane in the: 1. Left hand to decrease activity in the left hip abductors. 2. Left hand to facilitate activity in the left hip abductors. 3. Right hand to decrease activity in the left hip abductors. 4. Right hand to facilitate activity in the left hip abductors.

3. Right hand to decrease activity in the left hip abductors.

A patient who has a grade III ligamentous sprain of the radial collateral ligament of the metacarpophalangeal joint of the thumb (1st digit) is MOST likely to exhibit pain during which of the following tests? 1. Passive range of motion into thumb (1st digit) metacarpophalangeal joint flexion 2. Valgus stress testing of the metacarpophalangeal joint 3. Resisted isometric testing of the abductor pollicis longus 4. Palpation of the radial portion of the thumb (1st digit) metacarpophalangeal joint

4. Palpation of the radial portion of the thumb (1st digit) metacarpophalangeal joint 1. Passive range of motion into thumb (1st digit) metacarpophalangeal joint flexion should not stretch the involved ligament (p. 431). 2. Valgus stress testing of the metacarpophalangeal joint does not stress the injury (p. 431). 3. Resisted isometric testing of the abductor pollicis longus should not stretch the affected ligament (p. 431). 4. The radial portion of the thumb (1st digit) metacarpophalangeal joint is the affected structure and would be tender to palpation (p. 32).

For a patient with a lesion proximal to the dorsal root ganglion, a sensory nerve conduction velocity test will reveal that conduction times are: 1. decreased in amplitude. 2. markedly decreased. 3. markedly increased. 4. within normal limits.

4. within normal limits. 1. A sensory nerve conduction test only measures the distal component of a peripheral nerve. The technique would not be able to detect abnormalities in a lesion proximal to the dorsal root ganglion. If only sensory roots are injured, no electromyogram changes occur. 2. A sensory nerve conduction test only measures the distal component of a peripheral nerve. The technique would not be able to detect abnormalities in a lesion proximal to the dorsal root ganglion. If only sensory roots are injured, no electromyogram changes occur. 3. A sensory nerve conduction test only measures the distal component of a peripheral nerve. The technique would not be able to detect abnormalities in a lesion proximal to the dorsal root ganglion. If only sensory roots are injured, no electromyogram changes occur. 4. A sensory nerve conduction test only measures the distal component of a peripheral nerve. The technique would not be able to detect abnormalities in a lesion proximal to the dorsal root ganglion. If only sensory roots are injured, no electromyogram changes occur.

The patient whose hands are shown in the photograph exhibits a capsular pattern and firm end-feel of the left wrist. While stabilizing the distal radius and ulna, a physical therapist performs small-amplitude oscillations to the proximal carpal bones at the limit of the available motion and into tissue resistance. In which directions should the therapist move the carpals to improve the patient's ability to lift a heavy suitcase? (picture shows limited L wrist ext) 1. Dorsal and ulnar 2. Dorsal and radial 3. Volar (palmar) and ulnar 4. Volar (palmar) and radial

4. Volar (palmar) and radial 1. The power grip produces the greatest amount of force for hand grip. The power grip of the hand requires range of motion of slight extension and ulnar deviation (Magee, p. 452). The dorsal mobilization is used for improving flexion, while the ulnar mobilization is used for improving radial deviation. Therefore, both of these mobilizations would not help improve the motions required for the power grip. 2. The power grip produces the greatest amount of force for hand grip. The power grip of the hand requires range of motion of slight extension and ulnar deviation (Magee, p. 452). The dorsal mobilization would improve flexion, while the radial mobilization would improve ulnar deviation. Therefore, these would not be the most effective mobilization directions. 3. The power grip produces the greatest amount of force for hand grip. The power grip of the hand requires range of motion of slight extension and ulnar deviation (Magee, p. 452). Although mobilization in a volar (palmar) direction would improve extension, the ulnar glide would increase radial deviation, not ulnar deviation. 4. The power grip produces the greatest amount of force for hand grip. The power grip of the hand requires range of motion of slight extension and ulnar deviation (Magee, p. 452). While stabilizing the distal radius and ulna, mobilizing the proximal carpal rows in a volar (palmar) direction will increase extension, while a radial mobilization will increase ulnar deviation (Dutton). Therefore, a volar (palmar) glide followed by a radial glide would improve both extension and ulnar deviation, resulting in an improved power grip.

A 58-year-old female patient with uncomplicated MI (Myocardial Infarction) has been discharged from the acute care. Which of following activities would be MOST appropriate during early phase 2 rehabilitation? A. 30 minutes walking at 4 mph with no incline. B. 15 minutes of elastic band training with yellow elastic bands. C. 15 minutes upper limb workout with 2 pound hand weights. D. 30 minutes walking at 4 mph with 5% incline.

A. 30 minutes walking at 4 mph with no incline. -least stressful to the CV system -if using RPE scale you want 11-13 for phase 1 and 2

A patient has a pressure ulcer with bone visible. The wound has necrotic tissue and exudate present. Which of the following is the BEST wound care option? (look at slide 26 FF #14) A. Calcium Alginate B. Hydrocolloid dressing C. Hydrogel dressings D. Transparent films

A. Calcium Alginate this pressure ulcer has some necrotic tissue so should use wound irritation non-selective debridement

In the gait lab, the PT notices that the patient deviates the trunk to the left during the swing phase of the right leg. Which of the following is the MOST LIKELY cause for this deviation? A. Contracture of the right ankle plantarflexors B. Contracture of the left hip flexors C. Shorter limb length of the right leg D. Weakness of the right knee extensors

A. Contracture of the right ankle plantarflexors

A PT is performing phonophoresis at 1.5 W/cm², 1.0 MHz and 20% duty cycle on the distal hamstrings. During treatment, the patient reports feeling pain over distal hamstrings. The PT's FIRST appropriate course of action is to: A. Decrease the intensity to 1.2 W/cm² and continue moving the head at same speed B. Keep the same intensity at 1.5 W/cm²and change the setting to 50% duty cycle C. Increase the intensity to 2.0 W/cm² and add more coupling medium D. Turn off the ultrasound and immediately call the physician

A. Decrease the intensity to 1.2 W/cm² and continue moving the head at same speed

A PT is considering implementing electrical stimulation in the patient's plan of care to promote healing during proliferative phase of a non infected wound. Which of the following will be MOST effective for this patient? A. High voltage pulsed waveform on positive electrode in wound, 110 PPS. B. High voltage pulsed waveform on negative electrode in wound, 10 PPS. C. High voltage pulsed waveform on positive electrode in wound, 10 PPS. D. High voltage pulsed waveform on negative electrode in wound, 110 PPS.

A. High voltage pulsed waveform on positive electrode in wound, 110 PPS. -Pulse frequency for both negative (infected/inflamed) and positive (non-infected/non-inflamed) electrodes is always between 60-125 PPS

A 49-year-old male was diagnosed with Pneumonia. The PT suspects a lot of secretions before starting pulmonary rehabilitation. Which of the following is MOST likely to be seen in the examination finding ? A. Increased fremitus B. Decreased fremitus C. Hyper resonant percussion D. Increased breath sounds

A. Increased fremitus

A 69 year old patient has symptoms of lightheadedness, along with numbness and tingling of arms and legs. His blood gas report is: PH - 7.48, PaCO2 - 30 mm Hg, HCO3 - 26 Which of the following is the most INAPPROPRIATE treatment? A. Intravenous injection of sodium bicarbonate B. CO2 administration C. Pain control D. Using Rebreathing mask/paper bag

A. Intravenous injection of sodium bicarbonate Pain can cause hyperventilation and hyperventilation can cause respiratory alkalosis, if we control the pain this would be an APPROPRIATE treatment

When observing a patient ambulating, a PT notices that the patient's gait has the following characteristics : impaired trunk control and ataxic gait. There is also circumduction to assist with foot clearance due to weakness of dorsiflexors of the right foot. This gait pattern is often observed in patients with diagnosis of: A. Multiple Sclerosis B. Parkinson's Disease C. Duchenne Muscular Dystrophy D. Guillain Barre Syndrome (GBS)

A. Multiple Sclerosis GBS is BILATERAL involvement

A 37-year-old male patient presents to the clinic with complaints of low-grade, fever, nausea, and vomiting. The patient also mentions that he has a loss of appetite and a "dull pain" near the navel area. The PT suspects appendicitis; which test is LEAST likely confirm this diagnosis? A. Murphy's sign B. Psoas sign C. Obturator sign D. Blumberg's rebound sign

A. Murphy's sign -used for cholecystitis at the R subcostal margin for gall stone causing RUQ pain -this is a medical emergency "eddie Murphy 'C'oming to america to get cholecystitis"

A PT is treating a patient who was originally diagnosed with a pressure injury stage 3. The patient presents to clinic with ulcer shown in the picture. Which of the following findings is MOST accurate? (look at slide 12 FF #14) A. No change has occurred in the pressure injury B. The pressure injury has improved to Stage 1 C. The pressure injury has improved to Stage 2 D. The pressure injury has become unstageable

A. No change has occurred in the pressure injury Documentation: can't change the stage if you notice it improved, you just say it has improved but still stage 3 -you would say "healing has started in stage 3" -the original stage is needed under documentation

After a stroke, a patient's visual field is as shown in the picture. Which of the following is the MOST LIKELY location of the lesion? (image is pt only can see inner L and R halves of vision) Select one: a. Optic chiasm b. R optic tract c. L optic nerve d. L occipital lobe

A. Optic chiasm Rationale: The deficit shown in the image is bitemporal hemianopsia which is caused due to a lesion in the optic chiasm. L optic nerve damage would cause monocular vision. R optic tract lesion would cause C/L homonymous hemianopsia. Lesion in the L occipital lobe would cause C/L homonymous hemianopia with macular sparing.

A PT is developing a treatment plan for a 32 year old male patient 4 weeks post arthroscopic debridement of the L knee joint. The PT decides to use FES to improve the strength of the quadriceps. Which of the following parameters is the MOST APPROPRIATE? A. Pulse duration 250-300 microsec ; 10 seconds on, 50 seconds off B. Pulse duration 150-170 microsec ; 10 seconds on, 20 seconds off C. Pulse duration 250-300 microsec ; 10 seconds on, 20 seconds off D. Pulse duration 150-170 microsec ; 10 seconds on, 50 seconds off

A. Pulse duration 250-300 microsec ; 10 seconds on, 50 seconds off -for strengthening large group muscles you want a higher pulse duration (more time) and large ratio for duty cycle to allow the muscle to rest Rationale: -Muscle strengthening: Pulse duration for large muscle group = 250-300 microsec Pulse duration for small muscle group = 150-200 microsec -On:Off time = 1:5 and may be reduced gradually; -Pulse frequency: 35-80pps -Ramp time: at least 2 sec.

A two-year-old child with Down syndrome is being treated by a physical therapist. The child has moderate developmental delay and exhibits hypo tonicity. The MOST appropriate physical therapy treatment should include: A. Pushing a toy cart while standing B. Locomotor training using body weight support C. Rolling and somersault activities D. Rhythmic stabilization of postural extensors in sitting

A. Pushing a toy cart while standing -toy cart good for motor control and standing is good for weightbearing B not correct due to not full weightbearing C not correct due to AA joint laxity D not correct due to not weight bearing while sitting

A 52 year old female has a history of multiple sclerosis and described that the symptoms initially would appear for a few days and disappear but have now become continuous and do not disappear. This patient MOST likely has which of the following type of MS: A. Secondary-Progressive B. Primary-Progressive C. Progressive-relapsing D. Relapsing-remitting

A. Secondary-Progressive -water temperature for MS pts should be below 85 (Uhthoff's Phenomenon) -exercise in the morning preferred

A PT is creating an exercise program for a patient who underwent a an anterior approach total hip arthroplasty 3 weeks ago. Which of the following exercises would MOST safely promote gluteal strengthening? A. Sit to stand transfer exercises B. Standing hip extension exercises C. Standing external rotator strengthening using a theraband D. Sitting external rotator strengthening using an ankle weight

A. Sit to stand transfer exercises -Ant approach hip precaution = FABER and also isolated ext (ant fibers are recovering) -you want to do an closed chain approach

During the initial evaluation, PT asks the stroke patient to lift her left arm as high as she is able. Patient is able to lift her arm off of her lap; however, she is only able to move it with elbow flexion, shoulder elevation, and wrist flexion with significant increased tone. Which of the following Brunnstrom's Stages of Recovery is MOST appropriate description for this patient? A. Stage 3 B. Stage 4 C. Stage 5 D. Stage 6

A. Stage 3 -Pt is exhibiting movements within the synergy pattern Stage 4 would be anything outside of the synergy pattern

A PT is evaluating a 26 year old female basketball player with a vague diagnosis of right knee pain. The patient is performing a jump landing as shown. The PT treatment should MOST likely focus on: (look at slide 8 on FF #12) A. Strengthening of the right gluteus medius B. Strengthening of the right adductor magnus C. Strengthening of the right vastus medialis D. Strengthening of the right tibialis anterior

A. Strengthening of the right gluteus medius Proximal stability is more important and would help stabilize the knee

A 50 year old male patient presents to an outpatient clinic with complains of medial knee pain. The picture represents the position of the foot during normal quiet standing. What is the MOST appropriate diagnosis of this condition? A. Uncompensated forefoot valgus B. Compensated forefoot valgus C. Uncompensated forefoot varus D. Compensated forefoot varus

A. Uncompensated forefoot valgus Rationale: -Uncompensated situation is when the toes are NOT in line Forefoot valgus -When the heel is NEUTRAL and the forefoot is in valgus Forefoot varus -When the heel is NEUTRAL and the forefoot is an UNCOMENSATED FOREFOOT VARUS Compensate Forefoot valgus -When the heel is in varus to compensate for the forefoot valgus, forefoot is now neutral/flat Heel and forefoot are opposite

Functional Muscle Slings: Push-up Sling 4 muscles involved in order to perform a proper push-up?

All these muscles have to work together in order to perform a proper push-up -Cervical extensors -Serratus anterior -Abdominals -Quads

A 45 year old female patient is recovering from a left fibula fracture and is restricted to partial (25%) weightbearing on the left side. What is the best assistive device? A. Cane on the left side B. 2 axillary crutches C. Cane on the right side D. Forearm crutch on left

B. 2 axillary crutches Rationale -Partial WB component so pt needs more than a cane -Pt still puts weight through their extremity with a cane -Cane is full WB -You can start using Forearm crutches when Partial WB is >50%

An 75 year old individual has limited endurance. There is no history of cardiorespiratory problems. After an exercise tolerance test, which was negative for coronary heart disease, the BEST initial exercise prescription for this individual would be: A. 80-90% HR max. B. 60-70% HR max. C. 40-50% HR max. D. 30-40% of HR max.

B. 60-70% HR max. 220-75 = 145 50% of 145 = 72.5 which is too low for endurance exercise and is basically a Resting HR (60-100)

A patient presents to the clinic with history of burns on the left hip and thigh region causing tight hip flexors. What gait abnormality is MOST likely to be demonstrated by the patient? A. A shorter step length with the left lower extremity B. A shorter step length with the right lower extremity C. Backward lean during stance phase on left lower extremity D. Lateral lean during swing phase on right lower extremity

B. A shorter step length with the right lower extremity A tight hip flexor would limit hip ext ROM because it doesn't want to extend If you have tight hip flexors it would limit terminal stance also impacting step length on the opposite leg Hip extensor tightness —> Affects SAME side step length. Hip flexor tightness —> Affects OPPOSITE side step length

A PT is performing sensory examination of a patient post SCI. When PT strokes lightly on the patient's arm with a camel brush, the patient doesn't recognize the sensation. On pricking the patient with a neurological pin, the sensation is recognized. What other sensation is most likely recognized by the patient? A. Letters drawn on the hand. B. A warm test tube placed on the skin. C. The shape of an object placed in his hand D. Vibration from a tuning fork.

B. A warm test tube placed on the skin. -DCML is gone -Spinothalamic is working Summary of tracts: DCML - fine touch, two-point discrimination, proprioception, vibration Anterior Corticospinal - motor Lateral Corticospinal - motor Anterior Spinothalamic - crude touch Lateral Spinothalamic - pain and temp

A physical therapist is working with a patient who has tuberculosis, Which of the following options BEST describes the appropriate precautions and type of personal protective equipment that a physical therapist should wear? A. Contact precautions, N-95 respirator B. Airborne precautions, N-95 respirator C. Contact precautions, gloves and gown D. Airborne precautions, gloves and gown

B. Airborne precautions, N-95 respirator -C is for Clostridium Difficile (C. diff): contact precautions with gloves and gown should be used Airborne precaution use with TB, Chicken pox (varicella), shingles (herpes zoster), measles (rubeola) = "95 Airborne Measly Chickens Tee-peed the Shingles".

A patient presented to the outpatient physical therapy department with complains of dizziness and frequent falls. On examination, the therapist noted abnormal smooth pursuit, vertical pendular nystagmus, and abnormal saccadic eye movements. The MOST likely cause of the symptoms is: A. Vertebrobasilar artery insufficiency B. Central vestibular system pathology C. Peripheral vestibular system pathology D. Optic Neuritis

B. Central vestibular system pathology -abnormal smooth pursuit/vertical pendular nystagmus/abnormal saccadic eye movements are all common in Central Vestibular system pathology

A 68-year-old patient is performing a Bruce protocol on an inclined treadmill in the cardiac unit. The PT is monitoring the patient using ECG leads. During the exercise, the PT sees the ECG pattern shown. The PT's BEST response should be: (image on slide 60 #29) A. Stop the treadmill session immediately and call the cardiologist B. Continue without any modifications and monitor ECG C. Reduce the treadmill speed and monitor ECG D. Stop the treadmill, have the patient rest, and then resume at a lower intensity

B. Continue without any modifications and monitor ECG LOOK CAREFULLY AT THE EKG CHART!!!) -prolonged P wave noted -this is a 2nd degree Type 1 AV Block -Bruce protocol doesn't allow you to reduce, its all or nothing so you either stop exercise or continue exercise

A 76 year old patient has been transferred to an acute care unit with symptoms of CHF. During auscultation, the PT finds S3 heart sound. Which of the following breath sound is MOST likely associated with this finding? A. Stridor B. Crackles C. Wheeze D. Pleural rub

B. Crackles

Which of the following burn wound classifications is MOST likely to produce the keloid scar shown in the picture? (look at slide 19 FF#14) A. Superficial burn B. Deep partial-thickness burn C. Superficial partial-thickness burn D. Subdermal burn

B. Deep partial-thickness burn

Which of the following statements regarding changes related to denervation post heart transplant is LEAST appropriate for the therapist to follow while exercising the patient? A. HR, RPE and BP must be measured while exercising the patient B. Due to cardiac denervation, patient will have a reduced resting HR to achieve normal cardiac output C. Patient will have delayed elevation of HR with exercise D. Patient will have slower decrease in HR values during recovery phase of exercise.

B. Due to cardiac denervation, patient will have a reduced resting HR to achieve normal cardiac output -pt will have a higher resting HR after surgery "when you have a new hearts, its out of control so you have a higher resting HR after surgery and takes longer to recover" Clinically, the patient with a denervated heart following transplantation will present with elevated resting HRs to achieve normal cardiac output, delayed elevation in HRs with exercise due to circulating catecholamines, decreased maximum HR responses, and slower decreases in HR values during the recovery phase of exercise.

A patient presents complains of loss of sensation over the deltoid, radial forearm and hand. On evaluation, there is weakness of shoulder and elbow muscles however the intrinsic of the hand seems uninvolved. Which of the following is the MOST LIKELY diagnosis? A. Radial nerve palsy B. Erb's palsy C. Klumpke's palsy D. C6 radiculopathy

B. Erb's palsy

To prevent contractures in a patient with an above knee (transfemoral) amputation, emphasis should be placed on designing a positioning program that maintains range of motion in hip: A. Flexion and abduction. B. Extension and adduction. C. Adduction and lateral rotation. D. Flexion and medial rotation

B. Extension and adduction. AKA tend to go in FLEX and ABD so we need to put them in EXT and ADD BKA tend to go into knee flex so you should also put it in EXT

A patient comes into an outpatient clinic with symptoms of anxiety, hyperventilation, pain and cramping of the hand. Upon evaluation, the physical therapist respiratory rate of 40. Based on the symptoms and findings, what is the MOST appropriate treatment? A. Use a bronchodilator to decrease respiratory acidosis B. Have the patient to breath into a re-breathing mask to decrease respiratory alkalosis C. Paced breathing to decrease respiratory alkalosis D. Use a mechanical ventilator to decrease respiratory acidosis

B. Have the patient to breath into a re-breathing mask to decrease respiratory alkalosis Pt is hyperventilating which means he's going in to respiratory alkalosis. He needs more CO2 so have him breathe in to paper bag/mask

20 year old female, after an uncomplicated delivery has a 3 cm diastasis recti and weak abdominal muscles. Which of the following exercises is MOST appropriate for the patient? A. Sitting on a Swiss Ball B. Head lifts with arms bracing the abdomen C. Double leg lifts D. Deep breathing exercises in bridging position

B. Head lifts with arms bracing the abdomen -not the same as abdominal curls -<2 cm okay to do strengthening exercises -3 cm can do head lifts, but avoid aggressive exercise -4cm or more avoid ANY exercise, no head lifts, ONLY perform deep breathing exercise

Patient is a 8 weeks post op case of Achilles tendon repair. Patient has recently started discontinuing the CAM boot and unable to bear weight with complains of pain with weight bearing. The PT should FIRST emphasize: A. Stretching of posterior compartment muscles in standing B. Heel lift C. Continue with CAM boot D. Notify the doctor

B. Heel lift 1.0-1.5 cm ideal for heel lift

A physical therapist is examining a 54 year old patient and diagnoses a right pelvic drop during walking at self selected walking speed. Which of the following is a potential cause of right pelvic drop? A. Weak hip abductors on right side B. Hip abductor contracture on right side C. Weak hip adductors on left side D. Hip abductors contracture on left side

B. Hip abductor contracture on right side Either a L hip abd weakness OR a R hip abd contracture SUMMARY: Left Pelvic Drop: - Weak right GM, Tight right Add Mag - Weak left Add Mag, Tight left GM Right Pelvic Drop: - Weak left GM, Tight left Add Mag - Weak right Add Mag; Tight right GM

A 31-year-old pregnant obese female, in her third trimester, is diagnosed with damage of pudendal nerve. The patient is referred to PT clinic for pelvic floor strengthening. What is the BEST initial physical therapy intervention? A. Supine and perform a maximum of 5 contractions held for 3 seconds each B. Left side lying and perform a maximum of 10 contractions held for 5 seconds each C. Right side lying and perform a maximum of 10 contractions held for 5 seconds each D. Sitting and perform a maximum of 10 contractions held for 10 seconds each

B. Left side lying and perform a maximum of 10 contractions held for 5 seconds each -sitting would be a progression exercise -R sidelying not correct due to IVC on R side of body -R sidelying compresses IVC and decreases CO causing Hypotension (you want to avoid a Hypotensive Syndrome)

A therapist is treating a patient with Central Cord Syndrome that resulted from motor vehicle accident. The therapist's examination should reveal: A. Loss of motor function and pain and temperature sensation, with preservation of light touch and position sense below the level of the lesion. B. Loss of upper extremity function (cervical tract involvement), with preservation of lower extremity function (lumbosacral tract involvement). C. Sparing of tracts to sacral segments, with preservation of perianal sensation and active toe flexion . D. Ipsilateral weakness and loss of position sense and vibration below the lesion level, with contralateral loss of pain and temperature sensation.

B. Loss of upper extremity function (cervical tract involvement), with preservation of lower extremity function (lumbosacral tract involvement). -A is Anterior cord syndrome -C is Sacral sparing -D is Brown-Sequard MUD-E (mnemonic for central cord syndrome) -Motor more involved than sensory -UE > involvement than LE -Distal > proximal -E = cervical Ext injury

A patient that is eight days status post ACL reconstruction (patellar tendon autograft) is being examined by a physical therapist. What is the MOST appropriate exercise to implement into the patient's home exercise program? A. Leg press on operated side B. Mini squats C. Limited range isokinetic at 30 degrees D. Active knee extension in short sitting

B. Mini squats -Isokinetic = same speed exercise that causes too much shear force on the ACL -For ACL: AVOID END RANGE TEMRINAL KNEE EXTENSION IN OPEN CHAIN as it increases pressure/stress on the newly grafted ligament

A patient recovering from a traumatic brain injury seems disoriented and unable to correctly state the time of the day and his location. Which of the following is the MOST appropriate action for the PT to take? A. Immediately notify the patient's neurologist on call B. Orient the patient to the correct time and location. C. Tell caregiver to give patient a low dose of medicine to help with recalls D. Document dementia in the patient's chart

B. Orient the patient to the correct time and location.

A PT is examining a 10-year-old patient diagnosed as level 3 on gross motor classification of CP. According to the gross motor classification of CP what is the MOST likely ambulation status of this patient? A. Patient will walk without restrictions but will have limitations in more advanced gross motor skills. B. Patient will walk with assistive device with limitations in walking outdoors and in the community. C. Patient self mobility will be severely limited, even with the use of assistive technology. D. Patient will walk without assistive device with limitations in walking outdoors and in the community.

B. Patient will walk with assistive device with limitations in walking outdoors and in the community. A is level 1 C is level 5 D is level 2

A physical therapist is evaluating the sensitivity of a balance test in a group of older adults. Which of the following results indicates a true positive finding? A. Patients with a history of falls test negatively. B. Patients with a history of falls test positively. C. Patients with no history of falls test negatively. D. Patients with no history of falls test positively.

B. Patients with a history of falls test positively. A is false negative C is true negative D is false positive

A patient is unable to correctly perform the push up due to weakness in the push up sling. Based on the principal of muscle slings, Which muscle group should be strengthened to help in correcting the push up technique? (Look at image on slide 8 on FF#26) A. Cervical Flexors B. Serratus Anterior C. Hamstrings D.Ankle dorsiflexors

B. Serratus Anterior

A 21 year female player has limited ROM and pain on the right shoulder. The PT plans an intervention program but the patient refuses any treatment. Patient agrees that without intervention the symptoms might worsen. Which is MOST appropriate PT response? A. Call the coach of the team and report that the injury might effect the players performance B. Respect the player's decision to not use physical therapy services C. Start the treatment as the patient will most likely join in once she sees the benefits of the treatment. D. Refer the patient to another therapist who is a sports specialist

B. Respect the player's decision to not use physical therapy services "consent is sexy" If pt was a minor (under 18), you need to involve the parent/guardian If it was a minor with no parent/guardian, a social worker should be involved

Three DPT students are looking at effects of drugs on falls. The students are using data from previous patient medical records to compare drug usage in fallers and non fallers. Which of the following will be the MOST appropriate study design? A. Clinical case report B. Retrospective cohort C. Randomized controlled trials D. Prospective cohort

B. Retrospective cohort

A PT was testing gag reflex in a 45 old male complaining of sudden change of voice quality. When PT asks the patient to say " Ahh ", the PT finds the patient presentation picture below, Which cranial nerve is MOST likely affected? (picture of uvula deviating to the left) A. Left vagus nerve B. Right vagus nerve C. Left glossopharyngeal nerve D. Right glossopharyngeal nerve

B. Right vagus nerve Gag reflex (use swab method to cause gag reflex) Sensory = glossopharyngeal nerve (CN 9) Motor = vagus nerve (CN 10) CN 10 is for uvula and goes the opposite side of lesion. "U"vula : "U" Turn. It's making a U-Turn away from the lesion. CN 10: vagus injury uvula deviates to contralateral side CN 12: hypoglossal injury tongue deviates to ipsilateral side Rule of 17: for deviation CN 5 & CN 12= SAME (5+12=17); CN 7 & CN 10= OPPOSITE (7+10=17)

A PT is considering implementing electrical stimulation in the patient's POC to promote healing on a non-infected wound. Which is of the following is the BEST technique with regards to pad placement and polarity? A. Pad should be placed first on wound bed follow by placing saline-soaked gauze with negative polarity B. Saline-soaked gauze placed first on wound bed follow by placing pad with positive polarity C. Pad should be placed first on wound bed follow by placing saline-soaked gauze with positive polarity D. Saline-soaked gauze placed first on wound bed follow by placing pad with negative polarity

B. Saline-soaked gauze placed first on wound bed follow by placing pad with positive polarity non-infected = positive polarity (anode) -proliferation is good/positive infection = negative polarity (cathode) -infection is bad/negative -we want to protect the wound with saline soaked gauze first to not let any bacteria in

A PT examines the gait of a 62 year old male patient. The patient exhibits right early heel off during stance phase of gait as shown in the picture. Which of the following impairments is MOST likely associated with this finding? (look at slide 24 on FF#11) A. Shortening of the hamstrings B. Shortening of the gastrocnemius C. Weakness of the tibialis anterior D. Weakness of the iliopsoas

B. Shortening of the gastrocnemius Weakness will cause reduction in ROM Shortening will cause...early activation of muscle

A patient develops a Stage 2 pressure injury over the sacrum and is referred to physical therapy for wound care. Which of the following is the MOST appropriate initial application to clean the wound? A. Povidone-iodine solution B. Sterile normal saline C. Zinc oxide cream D. Nitrofurazone solution

B. Sterile normal saline key words: clean the wound/initial application Cleaning would be sterile normal saline. Providone-iodine solution would be for INFECTION to clean the area, especially before surgical procedure. Zinc oxide can be used for dental fillings, but mostly CHRONIC wounds (not initial). Nitrofurazone solution would be used for an INFECTED wound but NOT initial application as well

53-year-old patient has ulcer on the foot as shown in the picture below. What is the MOST likely diagnosis : (look at slide 10 FF#14) A. Arterial ulcer B. Venous ulcer C. Diabetic foot ulcer D. Pressure ulcer

B. Venous ulcer medial malleoli swelling (Arterial doesn't have swelling due to lack of blood flow)

Documenting the care provided to the patient is essential and must be completed in a timely manner. Which of the following is NOT appropriate with respect to documentation? A. When a charting error is made, it must be clearly indicated that a change was made without deleting the original record B. When a charting error is made, use white-out material to correct the text C. Mistakes must be crossed out with a single line through the error, and then both initialed and dated by the therapist D. Medically approved symbols or abbreviations can be used for documentation

B. When a charting error is made, use white-out material to correct the text

A 60 years old male is undergoing cardiac rehabilitation post MI in the hospital. On Day 3, patient has progressed to walking. Which of the following is MOST appropriate statement to when documenting the activity ? A Patient ambulated for 20 minutes with RPE 10/20 B Patient ambulated 300 ft with RPE 4/20 C Patient ambulated 500 ft in 6 minutes with RPE 10/20 D Patient ambulated in the ICU hallway with RPE 10/20

C Patient ambulated 500 ft in 6 minutes with RPE 10/20 Documentation (the more details the better) -How much distance -How much time -RPE B not correct due to RPE scale is 6-20 and does not tell you time

A physical therapist is evaluating the specificity of the Phalen test in a group of computer programmers. Which of the following results indicates a true negative finding? A. Patients with a history of wrist pain and carpal tunnel syndrome will test negatively. B. Patients with a history of wrist pain and carpal tunnel syndrome will test positively. C. Patients with no history of wrist pain and carpal tunnel syndrome will test negatively. D. Patients with no history of wrist pain and carpal tunnel syndrome will test positively.

C. Patients with no history of wrist pain and carpal tunnel syndrome will test negatively. A = false negative B = true positive C = true negative D = false positive

A physical therapist is teaching a patient who suffered from a spinal cord injury to negotiate a 4 inch curb with a wheelchair. The MOST appropriate instruction that the therapist gives to the patient is: A. Hook the arms around push handle and descend backward B. Ascend backward with large wheels first. C. Ascend in a wheelie position by lifting the front casters D. Place the front casters down first during descent.

C. Ascend in a wheelie position by lifting the front casters -Ascending always castor first, descending always large first (perform wheelie)

A PT is educating a patient about changes seen during pregnancy. Which of the following is MOST appropriate regarding physiological and anatomical changes related to pregnancy? A. Heart rate decreases about 20 beats per minute B. Cardiac output decreases during pregnancy C. Basal metabolic rate increases during pregnancy D. Center of gravity shifts inferiorly and posteriorly

C. Basal metabolic rate increases during pregnancy -increases because they consume more energy for the baby also causing increased heat production "Moms are hot" -CO and HR would increase because the mother is supplying more blood to the baby -BP would increase -SV stays the same

A 57 year old male patient is having difficulty with walking. The PT documents the knee extension strength to be 2 --/5 and incorporates EMG biofeedback to help increase motor recruitment. Which of the following protocol is the MOST appropriate to increase motor recruitment for the knee extensors? A. Begin with electrodes close together and biofeedback sensitivity is low B. Begin with electrodes widely spaced and biofeedback sensitivity is low C. Begin with electrodes widely spaced and biofeedback sensitivity is high D. Begin with electrodes close together and biofeedback sensitivity is high

C. Begin with electrodes widely spaced and biofeedback sensitivity is high strengthen muscles = high sensitivity spastic muscle = low sensitivity BIOFEEDBACK SUMMARY : Weak Muscle - Wide apart and High sensitivity. Tight Muscle - closely placed and low sensitivity spaced "W"ide apart, high intensity = "W"eak muscles spaced "C"lose, low intensity = "C"onstricted muscles (tight) Weak: "WH"eak: "W"ide space, "H"igh intensity Tight: some T L C (Tight Low Close)

A patient presents to clinic with a posterior herniated nucleus pulposus at L2 3. During the traction treatment in supine, the patient reports increased localized back pain. Which of the following actions is MOST appropriate for the PT to take? A. Change the position of the patient to side lying B. Refer the patient back to the physician C. Change the position of the patient to prone D. Reduce the traction force to 15% of body weight

C. Change the position of the patient to prone Supine position puts pressure on the POSTERIOR herniated nucleus pulposus so put them in PRONE

The therapist is assessing a cyclist complains of lateral knee pain. The therapist is assessing the cycle of the athlete. Which of the following factors can cause the lateral knee pain due to ITB tightness? A. Low seat height B. Seat set too forward C. Cleats are excessively internally rotated D. Cleats are excessively externally rotated

C. Cleats are excessively internally rotated ITB is a frontal plane issue, "stick to the plane" -IR causes valgus stress on the knee, causing pain to the ITB Rationale: -ITBFS is a repetitive stress injury that results from friction of the ITB as it slides over the prominent lateral femoral condyle at approximately 30 degrees of knee flexion. ITBFS is also common in cyclists. This is thought to be due to the pedaling stroke, which causes the ITB to be pulled anteriorly on the downstroke and posteriorly on the upstroke. Extrinsic factors include excessive bike seat height or cleat position on the pedal. If the cleats are excessively internally rotated on the pedal, the tibia also internally rotates, resulting in a valgus force on the knee and increased tension of the ITB. ST used with a bike? -Van Gelderen -used for stenosis -If they flex down and can ride longer/better then stenosis. If flexing doesn't make a difference then claudication

To assess an anterior tibiofibular ligament injury, which of the following tests is MOST appropriate? A. Talar tilt with the ankle in neutral dorsiflexion B. Anterior drawer at the ankle with the ankle in neutral dorsiflexion C. Compression of the shafts of the tibia and fibula at mid calf D. Squeezing the calf with the ankle in neutral dorsiflexion

C. Compression of the shafts of the tibia and fibula at mid calf -talar tilt = calcaneofibular ligament (CFL) -anterior drawer = ATFL -squeezing calf = thompson test for achilles tear

A geriatric patient recently developed CHF. The patient has symptoms of nausea, vomiting, and GI irritability. The patient also presents with mental confusion and frustration. Based on these symptoms, what is the clinical condition? A. Alzheimer's disease B. Dementia C. Digoxin toxicity D. Age related symptoms

C. Digoxin toxicity D"igoxin —> I "D"ig Star Wars —> "D"igoxins will increase the FORCE of heart contractility SE of digoxin: -GI disturbance -loss of appetite -confusion -blurred vision -irregular pulse -Hypokalemia

A patient recovering from TBI is functioning at stage IV on the Rancho Los Amigos Level of Cognitive Functioning scale. During the therapist's initial examination, the patient becomes agitated and tries to bite the therapist. The BEST course of action is to: A. Postpone the examination until later in the day when the patient calms down B. Postpone the examination for 1 week and then try again C. Document the behaviors and engage in a calming activity D. Restructure the formal exam so the therapist can complete it in three very short sessions.

C. Document the behaviors and engage in a calming activity -Documentation is extremely important with this type of behavior

A newborn is examined at birth using the APGAR screening test. The following scores are reported at 5 min: heart rate (2), respiration (2), reflex irritability (1), muscle tone (2) and color (2). The PT should: A. Monitor the APGAR score again at 10 minutes B. Expect respiratory complications due to low score on respiration C. Expect good heart rate, good respiration and normal reflex responses D. Expect neurological complications due to low score on reflex irritability

C. Expect good heart rate, good respiration and normal reflex responses Score of 7 or less requires testing again

The patient presents with acute inflammatory demyelinating polyradiculoneuropathy (AIDP), 4 weeks following a relatively benign respiratory illness with complaints of finger dysesthesias and proximal muscle weakness of the lower extremities. The weakness may progress over hours to days to involve the arms, truncal muscles, cranial nerves, and muscles of respiration. These characteristics are for what condition? A. Amyotrophic lateral Sclerosis B. Multiple Sclerosis C. Guillain Barre Syndrome D. Myasthenia Gravis

C. Guillain Barre Syndrome -AIDP associated with GBS -Going from distal to proximal -Effects muscles of respiration

A 50-year-old patient complains of restriction of shoulder movements during overhead activities. The PT notes hyperactive reflexes. The patient is taking antiinflammatory drugs since last 4 weeks, but has had no resolution of symptoms. Which condition is MOST commonly associated with the above findings? A. Hypothyroidism B. Biceps tendinitis C. Hyperthyroidism D. Supraspinatus tendinitis

C. Hyperthyroidism No resolution of symptoms with NSAIDS = not MSK issue Key words: -hyperactive reflexes -ROM limitiations The correct answer is C: Hyperthyroidism. Rationale- Hyperthyroidism is associated with increased calcium depositions leading to periarticular or tendinous calcification. Chronic periarthritis is also associated with hyperthyroidism causing pain and reduced range of motion, commonly in shoulder. Hyperactive reflexes is also a common sign. Hypothyroidism (Option A) causes decreased reflexes. Biceps (Option B) and supraspinatus tendinitis (Option D) does not alter reflexes. **Symptoms of hyperthyroidism "SWEATING" • Sweating • Weight loss • Emotional lability • Appetite increased • Tremor / tachycardia • Intolerance of heat/ Irregular menstruation/ Irritability • Nervousness • GI problems (diarrhoea) Additional: Increased DTR's , heat insensitivity

Which of the following is the MOST INAPPROPRIATE advice for foot care for a diabetic patient? A. Cut toenails straight across; do not cut into corners. B. Never try to treat corns, calluses, or toenails with sharp instruments, home remedies, or store-bought foot care products. C. If feet get cold at bedtime, use a hot water bottle or heating pad to warm the feet. D. Shop for shoes in the afternoon when your feet are the largest.

C. If feet get cold at bedtime, use a hot water bottle or heating pad to warm the feet. -they have diabetic neuropathy so avoid temperature changes, this is a contraindication

A patient performs one sit up with arms crossed across the chest and one with arm behind his head. The relationship between EMG and force indicates that EMG amplitude (mv) of the core abdominal muscles A. Remains unchanged as the muscle moment arm doesn't change B. Is higher when the arms are across the chest C. Is higher when the arms are behind the head D. Remains unchanged as student is putting similar effort

C. Is higher when the arms are behind the head

A patient 1 month post myocardial infarction is being seen in a clinic. When increasing the patient to 5 MET's of workload, the therapist notes that the ECG reads 1 unifocal PVC. The PT's IMMEDIATE action should be: A. Continue to exercise and increase the intensity B. Stop exercise because patient is undergoing ischemia C. Keep exercising at a lower intensity, consultation with physician is not required here. D. Stop exercise and consult with physician before starting any exercise

C. Keep exercising at a lower intensity, consultation with physician is not required here. B not correct due to needing more information on ST segment to classify as ischemia 1 PVC = is not serious 2 PVC (couplet) = stop exercise 3 PVC (v-tach) = emergency

A posterior approach was used for a case of revised total hip arthroplasty. Which of the following should NOT be included while educating the patient in order to prevent posterior dislocation? A. Transfer to the sound side from chair to bed B. When descending stairs, lead with the operated leg. C. Keep the knees higher than the hips when sitting D. Avoid standing activities that involve rotating the body towards the operated extremity

C. Keep the knees higher than the hips when sitting -Posterior approach: avoid FADIR -Hips should NOT cross 90 degrees of flexion

The PT student is given a task by their clinical instructor to provide the appropriate wheelchair measurements. Which of the following options provide the correct measurement for seat depth? A. Measure the length from posterior buttock to posterior aspect of popliteal fossa and add 2 inches to the measurement. B. Measure the total area of both the hips and add 2 inches to the measurement. C. Measure the total area from posterior buttock to posterior of popliteal fossa and subtract 2 inches from the measurement. D. Measure the trunk length and subtract it from lower leg length and add 2 inches to the measurement.

C. Measure the total area from posterior buttock to posterior of popliteal fossa and subtract 2 inches from the measurement.

The patient arrives to a PT clinic looking pale and is sweating profusely. His pulse is weak and his breath is shallow and rapid. The PT suspects heat exhaustion. Which of the following is NOT an appropriate response? A. Offer the patient some water B. Offer ice pack for forehead and the neck C. Offer the patient some salt tablets for electrolyte balance D. Ask the patient to remove any outer layer of clothing

C. Offer the patient some salt tablets for electrolyte balance -giving them salt tablets would cause them to be more dehydrated, thirsty, and cause an electrolyte balance

A PT is looking at squat activity in 20 obese and 20 normal weight females. Both groups performed 10 bilateral squats at 100 degrees of knee flexion. The PT hypothesizes that knee moments will be higher in obese. Which is the MOST appropriate test? A. Chi Square test B. Mann Whitney U. test C. One tailed t test D. Two tailed t test

C. One tailed t test because there is a biased/directional hypothesis (knee moment higher in obese) A. Chi square test: this would be the answer if the data was unequal

A 49 year old patient presents to a clinic with swollen legs. More swelling is seen in the right lower extremity than the left lower extremity. The PT suspects lipedema as a diagnosis. Which of the following is NOT likely seen as patient's presentation? A. Patient is susceptible to bruising of the affected area. B. Patient would have a negative stemmer sign. C. Patient has a high likelihood of developing cellulitis. D. Patient would report pain on pressure.

C. Patient has a high likelihood of developing cellulitis.

A PT is working with a 18-year old patient who has a complete T5 spinal cord injury. The patient has a stage 2 sacral pressure ulcer and was admitted for management of bowel and bladder incontinence. Which is the MOST appropriate intervention for the patient: A. Apply low-volt monophasic pulsed current to manage the ulcer B. Perform independent curl ups and bridging exercises. C. Perform independent rolling from side to side D. Administer diet and medications to manage a flaccid bowel

C. Perform independent rolling from side to side -A incorrect, you want HIGH volt monophasic -B not possible due to SCI level -D incorrect, diet and meds should be used to manage a SPASTIC bowel

A therapist is examining a patient's vestibular dysfunction. The patient is asked to assume a long sitting position. The patient's head is turned toward her left side while she is in a sitting position. The therapist then quickly moves the patient backward so that the head is extended over the end of the table approx.30 degrees below horizontal. This maneuver causes dizziness and vertigo. The repeat test to the right side did not produce any symptoms. The therapist reports these findings as: A. Positive right Dix Hallpike test B. Positive sharpened Romberg test C. Positive left Dix Hallpike test D. Positive Positional test

C. Positive left Dix Hallpike test

A patient has symptoms of muscle aches, cramps, soreness and pain in back and extremities. Creatine kinase level is 10 times more than normal and the urine is tea colored. Which of the following is MOST consistent with the clinical findings? A. Hyponatremia B. Hypokalemia C. Rhabdomyolysis D. Myasthenia gravis

C. Rhabdomyolysis -kinase levels are high means muscles are breaking down

A 48 year old patient with multiple sclerosis presents with dysmetria in both upper extremities. Which of the following interventions is the BEST choice to deal with this problem? A. 8-lb weight cuffs to wrists during activities of daily living training. B. Pool exercises using water temperatures greater than 88 degrees F. C. Proprioceptive neuromuscular facilitation patterns using dynamic reversals with carefully graded resistance. D. Isokinetic training using low resistance and fast movement speeds.

C. Proprioceptive neuromuscular facilitation patterns using dynamic reversals with carefully graded resistance. -A is too heavy -B is too hot, needs to be under 85 degrees and may cause Uhthoff's Phenomenon

A 28 year old male patient with a left above knee amputation is referred to an outpatient amputee clinic. During gait analysis, the PT observes a medial heel whip during swing phase. Which of the following is the MOST likely cause of this deviation? (look at slide 14 FF#13) A. Taut extension aid B. Inadequate medial rotation of the knee joint. C. Prosthetic knee bolt is externally rotated. D. Short prosthesis

C. Prosthetic knee bolt is externally rotated. Way to remember: -When you ER your hip with your knee bent the heel is on the medial side -When you IR your hip with your knee bent, the heel is on the lateral side LIME Lateral heal/ IR bolt Medial heal/ ER bolt

A physical therapist positions a patient with Parkinson's disease in long sitting on a mat table. The PT's goal is to improve the trunk control by facilitating co-contraction of the antagonists. Which of the following PNF techniques would be MOST useful? Select one: A. Agonistic reversal B. Hold relax C. Rhythmic stabilization D. Contract relax

C. Rhythmic stabilization Correct Answer - C Rhythmic stabilization is used to promote stability through co-contraction of the proximal stabilizing musculature of the trunk as well as the shoulder and pelvic girdle regions of the body. Performed in weight-bearing positions to incorporate joint approximation into the procedure, hence further facilitating co-contraction. Hold relax and contract relax are PNF stretching procedures to increase the ROM of tight muscles. Agonist reversals are mainly used for weak anti-gravity muscles for postural control.

A patient with an above knee prosthetic limb is displaying right lateral trunk bending while ambulating. Which of the following would be the MOST likely cause of this gait abnormality? A. Right anterior wall is too high B. Right medial wall too low C. Right lateral wall too low D. Right posterior wall is too high

C. Right lateral wall too low R lateral trunk R hip abd side weak R lateral side low

During the evaluation, the patient reports general muscle aches, pain, and soreness, along with increased frequency of dark colored urine. The PT noticed that the patient has hyperlipidemia. Which of the following medications is LEAST likely related to this patient's clinical scenario? A. Atorvastatin (Lipitor) B. Pravastatin (Pravachol) C. Ropinirole (Requip) D. Simvastatin (Zocor)

C. Ropinirole (Requip) -used for PD, a dopamine agonist for PD disease management

A 45-year-old man with lymphedema of the left leg is being examined by a PT. The PT determines that it is a stage 2 lymphedema and pitting scale grade is 3+. Which of the following statements in MOST likely clinical presentation? A. Elevation of the foot reduces the swelling. B. The indentation on finger pressure last for less than 15 seconds C. Skin is hardening due to fibrosis. D. The indentation produced is less than 1/2 inch

C. Skin is hardening due to fibrosis.

A PT is assessing a patient's lymph nodes 6 months post-chemotherapy treatment. When assessing the lymph nodes, which presentations LEAST likely require referral to physician? A. Hard and immobile lymph nodes less than 1cm in diameter B. Rubbery and firm lymph nodes more than 1 cm in diameter C. Soft and non tender lymph nodes less than 1 cm in diameter D. Palpable and tender lymph nodes more than 1 cm in diameter

C. Soft and non tender lymph nodes less than 1 cm in diameter Normal Lymph Nodes - supple, non-tender, mobile Infected or Inflamed Lymph Nodes - Firm and enlarged, tender, warm, mobile Malignant Lymph Nodes - Firm and gradually growing in size, non-tender, matted or immobile

A patient presents with limited mouth opening of 25 mm due to pain. There is no complain of clicking sound or mouth deviation when he opens his mouth. The patient is unable to completely close his mouth with teeth clenched together. What is the MOST likely diagnosis? A. Hypomobility B. Disc displacement with reduction C. Synovitis D. Capsulitis

C. Synovitis -decreased ROM -PAIN with NO deviation Capsulitis -decreased ROM -PAIN with DEVIATION Hypomobility -limited ROM -<35mm and NO pain Disc displacement with reduction -click sound -NO PAIN Disc displacement Without reduction -no click -NO PAIN

A 54 year old patient is admitted to the hospital for lung congestion. The patient's daughter who works in the same hospital as a respiratory therapist, visits him in the hospital. She wants to look at her father's medical record. The PT should: A. Tell her to ask the consultant physician for permission. B. Tell her she cannot see the chart because she could misinterpret the information. C. Tell her that she must have the permission of her father before she can look at the chart. D. Give her the chart and let her read it as she may have some insights.

C. Tell her that she must have the permission of her father before she can look at the chart.

A 2-year-old patient is being treated for gait training in outpatient physical therapy. The patient has calluses on the dorsal aspect of the toes and is unable to perform MTP extension. Which of the following is MOST likely true? A. The patient has a persistent ATNR B. The patient has an integrated STNR C. The patient has a persistent plantar grasp reflex D. The patient has an integrated plantar grasp reflex

C. The patient has a persistent plantar grasp reflex

A 85-year-old patient with chronic congestive heart failure was recently admitted to the hospital. Which of the following plans for prophylactic respiratory care is LEAST appropriate? A. Frequent turning and positioning every 2 waking hours B. Gentle percussion 2 times per day C. Vigorous chest vibrations, with the foot of the bed elevated, 4 times per day D. Gentle coughing and deep breathing exercises every 4 hours

C. Vigorous chest vibrations, with the foot of the bed elevated, 4 times per day -prophylactic = preventative -elevating the foot of the bed is a bad idea because you don't want the heart to work harder to get to the LEs that are elevated -When it comes to "LEAST appropriate" exercises look for the more aggressive words

A 74 year old contractor has a prominent forward lean when he is in stance on the right lower extremity. From this observation, PT would hypothesize that the MOST LIKELY cause is: A. Weak hip extensors on the left B. Weak hip flexors on the left C. Weak knee extensors on the right D. Weak knee flexors on the right

C. Weak knee extensors on the right Person leans TOWARDS the side of the weakness COWS= Contracture opp side step length reduced, weakness see side step length reduced. Applicable only for hip flexors Summary: Trunk lean is usually towards side of weakness on stance phase Backward trunk lean= hip ext weakness Forward trunk lean= quads weakness. Lateral trunk lean- g med weakness

What is a t-test? Is a t-test parametric or non-parametric?

Compares differences between 2 independent groups (parametric = balanced) ex: Test to compare athletic trainers vs PT with ACL rehab

A 31 year old female presents to a physical therapy clinic with agonizing, nauseating, exhausting, tiring and unbearable pain. What pain pattern is MOST related to the symptoms? A. Vascular B. Neurogenic C. Musculoskeletal D. Emotional

D. Emotional

Garment Compression Classification for Lymphedema (image on slide 50 FF#30)

LOOK AT CHART -pressure should not be more than DBP because we want the vessels to be able to relax and promote movement

A PT collected data on THA patients and plotted the relationship of trunk flexion with waist circumference. The regression plot showed a R square value of 0.64 (See picture). What is the best way to describe the slope? A. Weak positive slope B. Weak negative slope C. Strong positive slope D. Strong negative slope Look at image on slide 25, 06 FF PDF

D. Strong negative slope Weak = not steep Strong = steep

A PT is performing an exercise stress test on a 45 year old male patient. The patient resting values are: BP 130/90 mm Hg. HR 75 bpm and RR 24 breaths/min. Which of the following is an ABNORMAL response to vigorous aerobic exercise? A. Diastolic blood decreases to 88 mm Hg B. Diastolic blood pressure increases to 100 mm Hg C. Respiratory rate increases to 34 breaths/minute D. Systolic blood pressure decreases to 100 mm Hg

D. Systolic blood pressure decreases to 100 mm Hg DBP: Increase/decrease of 10 at most is normal

All of the following are guidelines in treating patients with GERD EXCEPT A. Any intervention requiring a supine position should be scheduled before meals and avoided just after eating B. Encourage the patient to sleep on left side for nocturnal reflex C. Modification of position towards a more upright posture may be required if symptoms persists during therapy D. Encourage the patient to sleep on right side for nocturnal reflex

D. Encourage the patient to sleep on right side for nocturnal reflex -pregnancy, hernia, and GERD are ALL preferred LEFT side lying

A single 22-year-old woman who is 3 months' pregnant arrives at a therapist's private practice complaining of shoulder and leg pain. She has a black eye and some bruising at the wrists. The BEST course of action for the therapist is: A. Administer massage for bruising, TENS, and ice modalities for pain, as indicated by the examination findings. B. Direct the patient to the nearest ambulatory care center for physician evaluation. C. Refuse to examine the patient and send her to the nearest emergency room. D. Examine the patient, and if abuse is suspected, report the findings to the appropriate authorities.

D. Examine the patient, and if abuse is suspected, report the findings to the appropriate authorities.

A patient presents to the clinic with complains of knee pain. On evaluation the PT notices that the patient has excessive ankle plantar flexion along with excessive anterior pelvic tilt. What could be the correlated motion at the knee joint? A. Genu Valgum B. Genu Varum C. Medial tibial torsion D. Genu recurvatum

D. Genu recurvatum

A physical therapist is working on rehabilitating a 58-yearold client using aquatic therapy. All the following are cardiovascular and respiratory effects of full immersion hydrotherapy (up to neck) EXCEPT: A. Reduced heart rate B. Reduced systolic blood pressure C. Increased cardiac output D. Increased vital capacity

D. Increased vital capacity -lungs can't expand due to hydrostatic pressure so you have DECREASED vital capacity with aquatic therapy

A 34 year old male presents to an outpatient clinic with jaw pain. What is the resting position of the temporomandibular joint (TMJ) and its functional ROM available? A. Mouth closed, 55 mm B. Mouth open, 25 mm C. Jaw slightly closed, 25 mm D. Jaw slightly open, 40 mm

D. Jaw slightly open, 40 mm Answer is D. Normal opening of mouth is 40 mm, less than 35 is hypo mobility and greater than 50 is hyper mobility. The resting position of mouth is slightly open. Mouth closed is the closed pack position.

Which of the following instruments is MOST appropriate for measuring patients' quality of life? A. Dynamic Gait Index (DGI) B. Oswestry low back pain disability index (ODI) C. Timed up and go test (TUG) D. Medical Outcomes Study 36 item short form (SF 36)

D. Medical Outcomes Study 36 item short form (SF 36) The Medical Outcomes Study (MOS) resulted in development of the 36-item short form for measuring patient's quality of life. It is a self-report that covers eight domains of physical functioning, role limitation due to physical problems, role limitations due to emotional problems, fatigue, and general health perceptions.

A patient with a transfemoral amputation is unable to wear a total contact prosthesis for the past 4 days. The patient reports shooting pain at the end of the residual limb. Examination of the residual limb does not show any erythema. The MOST LIKELY cause is: A. Cellulitis B. Dermatitis C. Impetigo D. Neuroma

D. Neuroma -shooting pain at end of residual limb -axons have been cut and don't have anywhere to go which causes the sharp pain Different from Phantom Limb Pain Phantom limb pain (PLP) refers to ongoing painful sensations that seem to be coming from the part of the limb that is no longer there. The limb is gone, but the pain is real. The onset of this pain most often occurs soon after surgery. Dermatitis: Itching ,redness skin, dry Cellulitis H -hot E- edematous R- red Impetigo = bacterial infection

A PT is working with a 49-year old patient who has a complete T4 SCI. The patient has a stage 3 sacral pressure ulcer. To document that the skin care has been properly addressed, the PT should record that the A. Patient can perform independent curl ups and bridging exercises B. Patient has good strength of the scapular depressors and elbow extensors C. Patient is able to perform 50 push-ups in the wheelchair. D. Patient can perform independent rolling from side to side

D. Patient can perform independent rolling from side to side -root of the question is focusing on skin care not strength -D addresses skin car and all the othe answeres involve strengthening

A 65 year old patient presents to the clinic post CVA. The PT reviews the angiogram and identifies a blockage of the upper division of left MCA. Which of the following conditions is the PT MOST likely to note upon performing an evaluation? A. Weakness in the left upper extremity B. Presence of non expressive aphasia C. Left homonymous hemianopia D. Presence of expressive aphasia

D. Presence of expressive aphasia (BEN Franklin) Broca's aphasia Expressive aphasia Non-Fluent aphasia A and C effects R side

After gait training a patient with a new below knee prosthesis, you notice redness along the patellar tendon and medial tibial flare. This would indicate A. The socket is too small and the residual limb is not seated properly B. The socket is too large and pistoning is occurring C. Improper weight distribution during stance D. Pressure tolerant weight bearing during stance

D. Pressure tolerant weight bearing during stance Pistoning occurs when the residual limb moves up and down inside the socket, usually due to inadequate suspension. Pistoning is also referred to as vertical displacement. -large socket causes pistoning

A physical therapist examines the viscosity and color of a sputum sample after completing postural drainage activities. The sputum is yellowish greenish color and is very thick. The PT can best describe the sputum as: A. Fetid B. Frothy C. Mucoid D. Purulent

D. Purulent Fetid: foul smelling Mucoid: thick related to COPD bronchitis Purulent: infection Purulent is infection, yellow-greenish in color, & very thick * Frothy is Pulmonary Edema, Serous, Air bubbles. Pink * Mucoid is COPD, Bronchitis, Asthma. Retained mucous. THICK * Fetid is foul smelling

A PT student is constantly on the new phone and is having difficulty swiping upwards with their thumb as shown in the picture below. What is the BEST mobilization to improve the thumb range of motion so that the student can swipe happily thereafter! A. Inferior glide at the CMC joint B. Superior glide at the CMC joint C. Ulnar glide at the CMC joint D. Radial glide at the CMC joint

D. Radial glide at the CMC joint

A 38 year old male patient has been diagnosed with atherosclerosis by his cardiologist, and wants to avoid primary risk factors. Which of the following factors is NOT considered to be a primary risk factor for atherosclerosis? A. High blood pressure B. Increased lipids C. Cigarette smoking D. Sedentary lifestyle

D. Sedentary lifestyle -Primary is a direct effect -Secondary are indirect effects Primary risk factors: high blood pressure, diabetes, high cholesterol/lipids levels, family history, gender, and smoking Secondary risk factors: Stress, Obesity, Sedentary Lifestyle

Excessive upward rotation of the left scapula is noted as the patient attempts shoulder abduction. Which of the following exercises is MOST appropriate to help correct the excessive scapular rotation? A. Forearm wall slides to strengthen serratus anterior B. Shoulder shrugs to strengthen upper trapezius C. Standing wall push-ups to strengthen serratus anterior D. Standing rows to strengthen the rhomboids

D. Standing rows to strengthen the rhomboids Need to strengthen the downward rotation muscles

A patient complains of loss of hand grip strength 11 weeks post Colles fracture. PT notices that the patient has a stronger grip with wrist in extension, but the grip strength decreases as the patient starts flexing the wrist. What is the MOST appropriate intervention to improve the ROM? A. Stretch the flexor digitorum and strengthen extensor digitorum B. Strengthen extensor digitorum. C. Stretch both flexor digitorum and extensor digitorum D. Stretch extensor digitorum and strengthen flexor digitorum

D. Stretch extensor digitorum and strengthen flexor digitorum

A patient presents to the clinic with a full thickness burn on the forearm. When the PT tries to touch it, patient doesn't complain of pain on pressure or light touch. Which of the following is the MOST LIKELY presentation of this wound? A. Mixed red waxy white appearance with blanching and slow capillary refill B. Erythematous pink with no blisters C. Erythematous with blanching and quick capillary refill D. Tanned appearance with no blanching

D. Tanned appearance with no blanching Full thickness has NO BLANCHING and NO PAIN -A is deep partial -B is superficial -C is superficial partial Rationale: Full thickness burns are white, tan, or charred and are usually anesthetic with poor circulation and absence of blanching. A- Deep partial thickness burns have broken blisters and are insensitive to light touch but sensitive to pressure and blanch with slow capillary refill. B- Epidermal burns are erythematous with no blisters C- superficial partial thickness burns are erythematous and blanch with quick capillary refill

How to find out which canal is affected with BPPV?

DHP assessment Vertical torsion/rotation: -Up beating nystagmus = posterior canal -Down beating nystagmus = anterior canal

What is the rule to determine a false/true, negative/positive statement?

Decide if it is a true or a false story, and then match the last word of the question with the last word of the answer choice

Difference between Decorticate rigidity vs Decerebrate rigidity?

Decorticate = UE in flex LE in ext Decerebrate = all limbs in ext "there are 4 Es in decerebrate (4 E's 4 limbs in ext)"

Tone Abnormalities: Decortivate vs Decerebrate vs Opisthotonus

Decorticate: abnormale flexor response -UE in flex -LE in ext "Decorticate: hands on the core (flexed arms)" Decerebrate: abnormal extensor response -all EXT "Tip- dEcErEbratE- abnormal Extensor response-lots of E" Opisthotonus: strong extensor tone -sustained contraction of the extensor muscles of the neck and trunk "looks like possessed demon"

How would pulmonary function tests look with Restrictive Lung Disease?

Decreased TV, IRV, ERV, VC, TLC, FRC Increase or same RV (look at day 5 picture)

SE of Levodopa

Dyskinesias -dynamic involuntary choreoathetotic movements occurring at peak of Levodopa dose -initially: facial grimacing with twitching of the lips, tongue protrusion -severe: involves limbs, trunk, and neck Dystonia -prolonged involuntary contraction that causes twisting or torsion of body segments -clawing of the toes or fingers, or cramping of the calf, neck, face, or paraspinal muscles -associated with pain during "off" periods

Hyperparathyroidism vs Hypoparathyroidism?

Hyperparathyroidism • Elevated calcium and decreased serum phosphate • Can demineralize bone=bone weakness and decreased density • Symptoms-Prox. Weakness, fatigue, drowsiness, myalgias, depression, glove/stocking sensory loss, osteopenia, confusion, gout. "you have more calcium in the blood than the bone which leads to bone weakness, general weakness, fatigue, drowsiness, muscle pain, and because you're so weak, you're also depressed. The depression gets worse which leads to other conditions like glove/stocking sensory loss, osteopenia (loss of bone mass/weak bones), confusion, and gout" Hypoparathyroidism • Low calcium and high phosphorus • Symptoms-Neck stiffness, muscle cramps, seizures, irritability, depression, muscle twitching, cardiac arrhythmias, paresthesia's of finger tips and mouth. "you have more calcium in the bone than the blood which leads to too much calcium in the bone which can lead to neck stiffness. There's so much stiffness that it leads to muscle cramps/twitching, seizures, irritability, cardiac arrhythmias, and finger tip and mouth paresthesia" (Grace Nuna)

Dementia

Involves gradual decline of cognitive functioning Symptoms become apparent after age 60 -decreased functional capabilities -poor education carryover -impaired activities of daily living Dementia can involve multiple parts of the brain -emotional changes -decreased motivation -speaking/processing difficulties Alzheimer's can cause Dementia

Which nerve root does the sciatic nerve come from?

L4-S3

Which nerve root affected with decreased ankle jerk DTR?

L5

Which nerve root affected with compression at the L4/L5 vertebrae?

L5 nerve root

Peripheral Nerve Lesions (look at chart on slide 55 FF#29)

Lateral cutaneous nerve of thigh -sensory only to lateral thigh -L2-L3 -MERALGIA PARESTHETICA Posterior cutaneous nerve -sensory only to posterior thigh -S1-S3 Obturator nerve -sensory to medial thigh -motor for hip ADD -L2-4 Femoral nerve -sensory for anteromedial thigh and leg -motor for hip flex and knee ext -L2-4 Saphenous branch of femoral nerve -sensory only for anteromedial knee and medial leg -L2-4 Sciatic nerve -sensory for anterior/posterior leg, sole, and dorsum of foot -sensory loss of entire foot -motor loss for foot DF/IV/PF and knee flex -L4-S1

Abnormal Respiratory Sounds

Look at slide 17 FF#10 Ronchi: snoronchi rhonchi "gurgling or snore-like"- conditions associated are bronchitis, bronchiectasis, pneumonia, and CHF Wheezes - whistling high pitch: conditions asthma, bronchiectasis, bronchitis, emphysema Crackles/Rales - "popping/crackling" found in pulmonary edema - wet crackle, CHF, chronic bronchitis, pulmonary fibrosis - dry crackles, pneumonia Pleural Friction Rub - "scratching, grating noise heard during inspiration and expiration. Found in pleural effusion, pleurisy, pneumonia

What are the functions associated with a frontal lobe lesion?

Primary motor cortex Motor loss Olfaction affected Broca's aphasia Apraxia Emotional, behavior control affected

Typical pain patterns by Quadrant?

RUQ: -peptic ulcers -gall bladder pathology -head of pancreas RLQ: -appendix -Crohn's Disease LUQ: -Tail of pancreas -spleen pathology LLQ: -diverticulitis -ulcerative colitis -IBS

How would COPD affect pulmonary function tests?

RV and FRC increased due to air trapping FEV1 decreased

Different categories of MS?

Relapse remitting MS (RRMS): discrete attacks of neuro deficits (relapses) with either full or partial recovery (remission) Primary progressive MS (PPMS): disease progression and a deterioration in function from onset; may experience modest fluctuations in neuro disability but discrete attacks do not occur Secondary progressive MS (SPMS): initial relapsing remitting course, followed by a change to a progressive course with a steady decline in function, with or without continued acute attacks Progressive relapsing MS (PRMS): steady deterioration in disease from onset (similar to PPMS) but with occasional acute attacks; intervals between attacks are characterized by continuing disease progression Clinically isolated syndrome (CIS): first episode of demyelination in the CNS that could become MS if additional activity occurs; can progress to RRMS

LCPD vs SCFE (Slipped Capital Femoral Epiphysis)

SCFE (slipped ice cream on a cone) -Age = male 10-17 y.o and female 8-15 y.o, obese -Males > Females -Blacks > Whites -Initial traumatic episode can be as minimal as turning over in bed -Acute onset = unable to bear weight -S/S = pain in the groin, medial thigh, or knee along with limping and ER of the leg while walking -ROM limitation = FABER -X-ray = shows displacement of femoral epiphysis "F in SCFE for flexion" LCPD (abnormal shaped ice cream) -Age = 2-13 y.o (average onset at 6 y.o) -Boys (3-5 times) > Girls -Initial sign is limp -(+) Trendelenburg sign (hip drop) -Muscle spasm = hip adductors and iliopsoas -Pain referred to the groin, medial thigh, or inner aspect of knee -ROM limitation = EABIR -Hip flexion contracture may be present (limited ext) -MRI = (+) Bony Crescent Sign

Glascow Coma Scale (Look at slide 46 on FF#21)

Scores 3-8: Severely abnormal 9-12: Moderately abnormal 13-14: Mildly abnormal 15: Normal EYES- 4 letters- so it is out of 4 EVM - eyes 4, Verbal 5 and Motor 6 motor is 5 letters and verbal is 6 letters but you REVERSE it. so motor is out of 6 and verbal is out of 5 ** GCS Mnemonics EVM 456- Eye Opening 4 Verbal response 5 Motor Response 6 ESPN- Eye Opening E eye opening spontaneously 4 S eye opening to sound 3 P eye opening to pain 2 N No response 1 Only Cows In India - Verbal Response 5 Oriented - 5 Confused - 4 Inappropriate words - 3 Incomprehensible sounds - 2 No response -1 Only Can Live With Friends Except Nancy - Motor Response 6 Obeys Commands- 6 Localizes pain - 5 Withdraws from Pain - 4 Flexion to pain (decorticate)- 3 Extension to pain (decerebrate) - 2 None- 1

Sensory and Motor symptoms associated with GBS?

Sensory • Initially bilateral and symmetrical "glove and stocking" distribution (hands/feet will feel numb) • Pain in large muscles of the body such as the back, thighs and buttocks • Stiffness, cramping, and "deep aching" pain • Burning, tingling, pins and needles, "ants under the skin", and numbness Motor • Decreased signal strength of peripheral nerves • Muscle weakness and atrophy • DTR - diminished • Partial or full paralysis (functional dependence) • Respiratory muscle involvement - mechanical ventilation • Facial and oral-motor weakness - vision, speech and swallowing problems

Scapular Winging Types: Serratus Anterior Trapezius

Serratus Anterior = Open book palsy/Medial winging Trapezius = Sliding door palsy/Lateral winging - watch that SLIDING DOOR, it's a TRAP! —> Sliding door palsy (lateral winging) is TRAPezius. - OPEN BOOK test? Sir gets an A. —> OPEN BOOK palsy (medial winging) is (Sir) SERratus (A)nterior.

What is biventricular failure?

Severe LV pathology, fluid from the LV backs up into the lungs, increasing PA pressure and causing fluid to back up into the R side of the heart and the systemic venous vasculature. Pts with biventricular failure will present with both pulmonary and systemic signs of heart failure.

5 Different types of burns?

Superficial Burn Involves only the epidermis, Area may be red with slight edema. Healing occurs with evidence of scarring in 2-5 days. Superficial Partial- Thickness Burn Involves the epidermis and the upper portion of the dermis. The involved area may be extremely painful and exhibit blisters. Healing occurs with minimal to no scarring in 5-21 days. (most painful due to part of the nerve endings destroyed) Deep Partial- Thickness Burn Involves complete destruction of the epidermis and the majority of the dermis. Hypertrophic or keloid scarring may occur. In the absence of infection, healing will occur in 21-35 days. (not as painful due to majority of nerve endings destroyed? review video) Full thickness burn Involves complete destruction of the epidermis and dermis along with partial damage to the subcutaneous fat layer. Healing varies within weeks, with or without grafting, and/or months to heal. Subdermal Burn Involves the complete destruction of the epidermis, dermis, and subcutaneous tissue. Subdermal burns may involve muscle and bone and as a result, often require multiple surgical interventions and extensive healing time. 5/21/35

What 2 systems control the nervous system regulation of HR?

Sympathetic: flight and flight ex: seeing a pretty girl and the heart is pumping (increased HR) Parasympathetic: rest and digest ex: realizing you need to relax (decreased HR)

What is Phonophoresis?

The use of US for transdermal delivery of medication -anti-inflammatory (dexamethasone) -analgesic (methyl salicylate, lidocaine) Duty cycle: Pulsed Treatment time: 5-10 minutes

You are testing a patient's deep tendon reflexes during an evaluation for low back pain in outpatient physical therapy. Which of the following is MOST likely to be consistent with a false negative patellar reflex? Select one: a. 2+ reflex and the patient has a confirmed L4 nerve root syndrome b. 1+ reflex and the patient has a confirmed S1 nerve root syndrome c. 3+ reflex and the patient has a confirmed L4 nerve root syndrome d. 2+ reflex and the patient does not have L4 nerve root syndrome

a. 2+ reflex and the patient has a confirmed L4 nerve root syndrome Rationale: 2+ reflexes are considered normal according to the reflex grading system. A false negative indicates that the test is normal but the patient has pathology. The correct answer states that the reflexes are normal but the patient has a pathology that affects the L4 nerve root.

A PT student is studying the effects and characteristics of ultrasound. Which of the following statements regarding therapeutic ultrasound is true? Select one: a. A 40% duty cycle would have an off time of 6msec and an on time of 4msec b. A 40% duty cycle would have an off time of 10msec and an on time of 4 msec c. A 40% duty cycle would have an off time of 4msec and an on time of 6msec d. A 40% duty cycle would have an off time of 4msec and an on time of 10msec

a. A 40% duty cycle would have an off time of 6msec and an on time of 4msec Rationale: Duty cycle is the fraction of time the US energy is on over one pulse period (time on + time off). Duty cycle= Duration of pulse/Pulse period x 100 Therefore, a 40% duty cycle would have an on time of 4msec and on off time of 6msec. Duty cycle = On time/whole duration

You are assessing a patient in the ICU who opens eyes to pain, has an abnormal flexion response to motor stimuli, and is oriented verbally. You would MOST likely categorize this patient on the Glasgow Coma Scale as having what type of brain injury? Select one: a. Moderate b. Mild c. Severe d. Very severe

a. Moderate Rationale: This patient would score a 10 on the Glasgow Coma Scale. A score of less than 8 is considered a severe brain injury, 9-12 is considered a moderate brain injury, and a score of 13-15 is considered a mild brain injury.

A 22 year old patient complains of a gradual onset of thoracic and sacroiliac pain. Lumbar ROM is limited to 50%. Patient reports his stiffness is worse in the morning and gradually improves. He recently started a new job at Comcast and has been sitting for long periods of time. Which of the following would you MOST likely suspect? Select one: a. Thoracic hypomobility b. Ankylosing spondylitis c. Spinal tumor d. Pectoral muscle adaptive shortening

b. Ankylosing spondylitis Rationale: Ankylosing spondylitis is a rheumatic disease characterized by chronic inflammation of ligaments in the lumbar spine, sacroiliac joint, thoracic spine, and shoulders that usually occur in the teenage years and 20's. Radiographs will show a bamboo spine. (hunched forward posture)

A 65 year old patient with diabetic neuropathy presents to the clinic with a complaint of frequent falls especially at night. Which of the following conditions of the Sensory Organization Test is this patient MOST LIKELY to score poorly? Select one: a. Conditions 4 and 6 b. Conditions 5 and 6 c. Conditions 1 and 3 d. Conditions 2 and 4

b. Conditions 5 and 6 Rationale: Due to peripheral neuropathy, this patient is more dependent on their vision and vestibular systems. Patients depending on vision become unstable in conditions 2, 3, 5 & 6 where we either close the eyes, or have a conflict between vision and the vestibular system. With conditions 1 and 4, the patient will have the opportunity to rely on the visual system and maintain balance.

Pressure to base of toes causes toe flexion a. Palmar grasp reflex b. Plantar grasp reflex c. Rooting reflex d. Moro reflex

b. Plantar grasp reflex 28 weeks of gestation to 9 months

Weight placed on balls of feet when upright causes stiffening of legs and trunk into extension a. Palmar grasp reflex b. Positive support reflex c. Rooting reflex d. Moro reflex

b. Positive support reflex 35 weeks of gestation to 2 months

You are treating a patient with spastic cerebral palsy in outpatient physical therapy clinic. The patient has difficulty with ambulation in the community and the patient's mother reports concern about falling. When quickly extending the right knee joint, there is a catch followed by minimal resistance through the rest of the remaining range (about 25% of the total range of motion). Which of the following statements is MOST ACCURATE? Select one: a. 1 on the Modified Ashworth Scale and increased tone in the quadriceps b. 3 on the Modified Ashworth Scale and increased tone in the hamstrings c. 1+ on the Modified Ashworth Scale and increased tone in the hamstrings d. 3 on the Modified Ashworth Scale and increased tone in the quadriceps

c. 1+ on the Modified Ashworth Scale and increased tone in the hamstrings Rationale: The Modified Ashworth Scale is a tool used to grade spasticity. A slight increase in muscle tone manifested by a catch, followed by minimal resistance throughout the remainder of movement (less than half of the range of motion) is classified as 1+.

9) Nancy suffered a shoulder injury and had been immobilized for 4 weeks. She came for her first physical therapy appointment after removal of the cast and the physical therapist needs to check the functioning of GH ligaments. In order to stretch the inferior ligament of GH joint a. Arm should be elevated to 45 degrees b. Arm should be extended to 30 degree c. Arm should be elevated to 90 degrees d. Arm should be externally rotated to 45 degrees

c. Arm should be elevated to 90 degrees Inferior glenohumeral ligament is taut/ stretched with 90 degrees of shoulder abduction. Middle glenohumeral ligament is taut between 45-60 degrees of abduction.

· What are the Myotome tests?

o C3 = SB o C4 = shoulder shrug o C5 = shoulder abd o C6 = elbow flex/wrist ext o C7 = elbow ext/wrist flex o C8 = thumb ext o T1 = finger abd o L1/2 = hip flex o L3 = knee ext o L4 = ankle DF o L5 = toe ext o S1 = ankle EV/PF and hip ext o S2 = knee flex o S3 = intrinsic foot muscles (except abd hallucis)

· Which muscles are innervated by the musculocutaneous nerve?

o Coracobrachialis o Biceps brachii o Brachialis

· What causes partial claw hand deformity?

o Guyon's canal trauma/ulnar nerve entrapment (pisiform and hook of hamate)

· Which cord does the musculocutaneous nerve come from?

o Lateral cord

Values for arterial blood gas? Acronym to determine ABG interpretation?

pH = 7.35-7.45 CO2 (respiratory system) = 35-45 HCO3 (metabolic system)= 22-26 ROME = Respiratory opposite, Metabolic equal Fully compensated = pH is within normal range Partially compensated = pH AND other value NOT within normal range -"Good/active friend is willing to step out of his comfort zone/his norm range to help you, you will be Partially Compensated by your friend" Uncompensated = other value is within normal range -"Lazy/unactive friend is NOT willing to step out of his comfort zone/his norm range to help you, you will be Uncompensated by your friend"

What is Cholecystitis? What is the special test associated with this condition?

"Inflammation of the gall bladder" Blockage or impaction of gallstones (rotting teeth) in the cystic duct resulting in inflammation of the gallbladder -Causes pain in the RU quadrant, radiating to the R scapula -Nausea, vomiting, low grade fever Special test: Murphy's Sign -palpate near the right subcostal margin, tell pt to take a deep breath -(+) if pain and tenderness with inspiration Murphy's sign is elicited in patients with acute cholecystitis by asking the patient to take in and hold a deep breath while palpating the right subcostal area. Deep tenderness at McBurney's point, known as McBurney's sign, is a sign of acute appendicitis.

List the MAS scale for spasticity

0 = no resistance 1 = catch, with resistance at end 1+ = catch, with resistance at <1/2 ROM 2 = resistance throughout ROM, easily moveable 3 = harder resistance, PROM difficult 4 = rigid

Stages of lymphedema?

0 = no signs/tissue appears normal 1 = reversible/gravity dependent/no stemmer's sign 2 = irreversible/gravity independent/positive stemmer's sign/brawny edema 3 = elephantiasis/same as 2 but has more infection "lymph03dema"

Difference between 1 tailed t test vs 2 tailed t test Are they parametric or non-parametric?

1 tailed is biased (directional hypothesis) ex: 3rd party thinks one group is better than the other 2 tailed is objective (non-directional hypothesis) ex: chemo therapy can go either way, they are objective Parametric

A patient presents with moderate pain in the elbow after a fall. The radiograph is negative for a fracture. Which of the following mobilizations is MOST appropriate for decreasing the pain? 1. Small-amplitude oscillations before the onset of tissue resistance 2. Small-amplitude oscillations into tissue resistance 3. Large-amplitude oscillations into tissue resistance 4. Large-amplitude oscillations at the end of tissue resistance

1. Small-amplitude oscillations before the onset of tissue resistance 1. Small-amplitude oscillations before the onset of tissue resistance are appropriate for pain modulation. (grade 1) 2. Small-amplitude oscillations into tissue resistance are more appropriate for joint stiffness, not pain. (grade 4) 3. Large-amplitude oscillations into tissue resistance are more appropriate for joint stiffness. (grade 3) 4. Large-amplitude oscillations at the end of tissue resistance are for end range joint restrictions and are too aggressive for patients who have pain. (grade 3)

A patient with a focal lesion in the region of the left basal ganglia will MOST likely demonstrate: 1. clasp knife resistance to active range of motion and hypotonia in the left extremities. 2. cogwheel resistance to passive range of motion and tremor in the right extremities. 3. clasp knife resistance to passive range of motion and tremor in the right extremities. 4. cogwheel resistance to active range of motion and hypotonia in the left extremities.

2. cogwheel resistance to passive range of motion and tremor in the right extremities. 1. The clasp-knife phenomenon occurs as a result of injury to descending motor pathways from the cortex or brainstem, not the basal ganglia. 2. Cogwheel resistance to passive manipulation results from lesions of the basal ganglia. Tremor is also seen with injury to the basal ganglia. 3. The clasp-knife phenomenon occurs as a result of injury to descending motor pathways from the cortex or brainstem, not the basal ganglia. 4. Rigidity rather than hypotonia is seen with lesions of the basal ganglia.

Which of the following muscular responses would be expected during normal micturition? 1. Relaxation of the detrusor and relaxation of the pelvic floor 2. Relaxation of the detrusor and contraction of the pelvic floor 3. Contraction of the detrusor and relaxation of the pelvic floor 4. Contraction of the detrusor and contraction of the pelvic floor

3. Contraction of the detrusor and relaxation of the pelvic floor 1. If the detrusor does not contract, the result is incomplete bladder emptying. 2. Neither relaxation of the detrusor nor the contraction of the pelvic floor would allow for complete bladder emptying as expected in normal micturition. 3. The contraction of the detrusor acts to empty the bladder, while the relaxation of pelvic floor allows the urine to pass. 4. Contraction of the pelvic floor would limit the flow of urine.

A 30 years old male client visits a town which is 9000 feet above sea level. What are the INITIAL cardiovascular responses during his first few days in town? A. Increased BP, increased cardiac output with tachycardia and no significant changes in SV. B. Decreased BP, decreased cardiac output with bradycardia and increased SV. C. Increased BP, decreased cardiac output with bradycardia and increased SV. D. Decreased BP, increased cardiac output with tachycardia and increased SV.

A. Increased BP, increased cardiac output with tachycardia and no significant changes in SV. Key words: INITIAL, first few days ex: first date jitters, increased HR, high BP, need more blood supplier so CO increases CO = HR x SV -HR usually reacts first with the demands of CO and SV responds later Why doesn't SV increase? SV is volume of blood pumped with each heart beat HR (the rate/speed) is increasing the amount of blood NOT the amount of each stroke of the heart ** SUMMARY for altitude : Initial changes : HR increase, BP increases , Cardiac Output increases, RR increases, SV remains same . Acclimatization (body gets used to it) changes (later at high altitude): HR increases, BP normal, CO normal, SV decreases. Late changes (coming back to plains) : HR and BP stabilize , Higher CO and Higher SV.

A patient is referred with a diagnosis of chronic periarthritis of the shoulder. The patient feels tired all the time and has noticed recent weight loss in spite of an increased appetite. Which of the following would be seen along with these symptoms if his thyroid levels are elevated? A. Increased DTR, Increased HR, low BP, Heat intolerance B. Decreases DTR, Increased HR, high BP, cold intolerance C. Increased DTR, Increased HR, low BP, cold intolerance D. Decreases DTR, Increased HR, high BP, Heat intolerance

A. Increased DTR, Increased HR, low BP, Heat intolerance

A patient has reddish brown papules and plaques around the mouth. The area is warm and tender to the touch. Which of the following conditions does the patient MOST likely have? (image on slide 14 FF#28) A. Cryoglobulinemia B. Cutaneous sarcoidosis C. Granulomatosis D. Vasculitis

B. Cutaneous sarcoidosis -Cutaneous (face) Sarcoidosis. Sarcoidosis is an autoimmune disease in which clumps of immune cells, called granulomas, form in different tissues and organs. These clumps can lead to scarring and interfere with how well an affected organ works. -Cryoglobulinemia is a form of vasculitis—a family of rare disorders characterized by inflammation of the blood vessels, which can restrict blood flow and damage vital organs and tissues. -Granulomatosis with polyangiitis is an uncommon disorder that causes inflammation of the blood vessels in your nose, sinuses, throat, lungs and kidneys. Formerly called Wegener's granulomatosis, this condition is one of a group of blood vessel disorders called vasculitis. It slows blood flow to some of your organs. -Vasculitis is an inflammation of the blood vessels. It happens when the body's immune system attacks the blood vessel by mistake. It can happen because of an infection, a medicine, or another disease. The cause is often unknown. Vasculitis can affect arteries, veins and capillaries.

During assessment skin sensation, which of the following structures are responsible for transmission of the cold sensation? A. Meissner corpuscles B. Krause end bulbs C. Golgi tendon organs D. Ruffini endings

B. Krause end bulbs Santa Krause (cold) Ruffini endings = detect heat Ruffini: A Dog goes Ruff Ruff → RuffRuff. Ruffini = Dog's breath is HOT and WARM Meissner corpuscles = detect touch/vibration ex: touching different textures of fabric at Joann's Pacinian corpuscles are responsible for pressure and vibration sensory modalities. Golgi tendon organs are sensitive to muscle contraction force. Free nerve endings are sensitive to pain and itch.

A 55-year-old female presents with chief complaint of heaviness in her right arm. Patient has history of breast cancer and radiation 8 months ago. The PT confirms lymphedema as a diagnosis. Which of the following is the MOST appropriate intervention? A. Use short-stretch compression bandages with more layers proximally than distally and educate the patient to spent more than 15 minutes in the hot tub B. Use short-stretch compression bandages with more layers distally than proximally and educate the patient about taking BP on the left arm C. Use ace bandages with more layers proximally than distally and talk to the patient about keeping the extremity clean and dry D. Use ace bandages with more layers distally than proximally and avoid using sunscreen when in the sun.

B. Use short-stretch compression bandages with more layers distally than proximally and educate the patient about taking BP on the left arm -ace wrap bandages (high stretch) are not good for lymphedema, only used for acute injuries, lymphedema is a chronic/gradual condition

A patient with LBP comes to PT clinic with a history of cardiac surgery. Pain is localized over the sacroiliac joint and X ray and bone scans show the destructive changes in SI joint resulting in seeding of the joint by septic emboli. The PT will suspect A. Pericarditis B. SI joint dysfunction C. Lumbar stenosis D. Endocarditis

D. Endocarditis "seeding of joint" = seeds of infection 2 systems (cardiac/MSK) -Cardiac surgery -clot sneaked out -emboli caused "seeding of infection" to the SIJ causing pain Rationale: -Septic emboli are a common complication of infective endocarditis. Septic emboli could affect multiple organs and cause variable insults. Blood cultures are usually positive in patients with septic emboli. Septic emboli cause tissue injury by two different mechanisms: ischemia and infection. Pericarditis -inflammation of the pericardium, a sac-like structure with two thin layers of tissue that surround the heart to hold it in place and help it work. A small amount of fluid keeps the layers separate so there's less friction between them as the heart beats. Endocarditis -inflammation of your heart's inner lining, called the endocardium. It's usually caused by bacteria. When the inflammation is caused by infection, the condition is called infective endocarditis. Endocarditis is uncommon in people with healthy hearts.

A patient complains of lack of grip strength. PT notices that the patient has a stronger grip with wrist in extension, but the grip strength decreases as the patient starts flexing the wrist. What is the MOST appropriate intervention to improve the ROM? A. Stretch the flexor digitorum and strengthen extensor digitorum B. Strengthen extensor digitorum and put a flexion splint C. Stretch flexor digitorum and extensor digitorum D. Stretch extensor digitorum and strengthen flexor digitorum

D. Stretch extensor digitorum and strengthen flexor digitorum Grip strengthen decreases as the pt starts to FLEX the wrist Active insufficiency of the flexor digitorum: need to strengthen Passive insufficiency of the extensor digitorum: need to stretch

A Physical Therapist has determined that a patient has unilateral vestibular hypofunction. What would be the MOST appropriate INITIAL home exercise to provide to the patient? A. Brandt-Daroff Exercises B. VOR x2 Walking C. Walking towards a busy wallpaper D. VOR x1 sitting

D. VOR x1 sitting

What is Restrictive Lung Disease?

Group of diseases that result in difficulty expanding the lungs and a reduction in lung volumes (can't take in air)

What is Rate Product Pressure?

HR x SBP

What happens to HR/CO/SBP/DBP with increased work with exercise?

HR/CO/SBP all increase DBP stays fairly constant/stable

A 32-year-old patient has been referred for a L4 nerve root lesion. Which of the following special tests would MOST LIKELY be a positive test? Select one: a. Gaenslen's test b. Flamingo test c. Gillet's test d. Thigh thrust test

a. Gaenslen's test Rationale: -Gaenslen's test involves the patient lying in the supine or sidelying position. In sidelying, the lower leg is flexed and the upper leg is hyperextended at the hip. In supine position, the patient draws both legs up onto the chest and then slowly lowers the test leg into extension beyond the edge of the table. The other leg is tested in a similar fashion for comparison. Pain indicates a positive test. The pain may be caused by an ipsilateral sacroiliac joint lesion, hip pathology, or an L4 nerve root lesion. -Thigh thrust test and flamingo test, if positive, are indicative of SI joint pain. -Gillet's test is used for assessing an ipsilateral posterior innominate rotation.

The following EKG pictured below MOST LIKELY demonstrates which cardiac condition? (image is of more then 3 PVCs) Select one: a. Ventricular tachycardia b. Supraventricular tachycardia c. This is a normal electrocardiogram, there is no pathology present d. Atrial fibrillation

a. Ventricular tachycardia Rationale: Increased Periventricular contractions puts stress on the ventricular musculature. PVC's that are present as a triplet or more are considered ventricular tachycardia.

What is tidal volume?

amount of air inspired or expired during normal resting ventilation

Rolls from prone to supine Weight shifting from one forearm to the other in prone Head control in supported sitting a. 3 month b. 5 months c. 7 months d. 9 months

b. 5 months

In a PT clinic, a 2-year-old child collapses to the ground while playing. There is only one PT in the clinic that day and he runs to the site and starts looking at the child for breathing and a pulse. He finds that the child is gasping and does not have a pulse. What should be the FIRST course of action by the PT, as he has not witnessed the collapse? Select one: a. Retrieve the AED or defibrillator b. Begin compressions and breath cycles with a ratio of 30:2 c. Wait for the paramedics team to arrive d. Activate the emergency response system

b. Begin compressions and breath cycles with a ratio of 30:2 Rationale: -When the rescuer is alone and has noted no breathing or only gasping and no pulse, he should first begin cycles of 30 compressions and 2 breaths. After about 2 minutes of the 30:2 cycles, if still alone, activate the emergency response system and retrieve the AED. Attaching the AHA recommended algorithm for a single rescuer paediatric BLS (2015 update).

Prone on hands with elbows extended, weight shifting from hand to hand Rolls supine to prone Independent sitting Pulls to stand with hands held, bounces a. 2 month b. 4 months c. 6 months d. 8 months

c. 6 months

You are treating a patient with a cerebellar infarct that is affecting the UE. The patient has trouble coordinating rapid, alternating movements of the UE. Finger to nose testing reveals minimal aberrations, and toe tapping is within functional limits. What BEST describes the aforementioned symptoms? Select one: a. Resting Tremor b. Dysmetria c. Dysdiadochokinesia d. Asthenia

c. Dysdiadochokinesia Rationale: A resting tremor is observed when the patient is at rest and will diminish during movement. Dysmetria occurs when the patient is unable to perform activity with good accuracy; typically, they will overshoot or undershoot movement. Asthenia is fixation or position holding and is tested with application of manual resistance. Dysdiadochokinesia is tested with alternating movement and is what is affected in this patient.

What are the 3 primary peripheral nerves of the lumbosacral plexus?

femoral, obturator, sciatic nerve

Specific symptoms for R CVA

-L side weakness/paralysis -L neglect, spatial perceptual problems -poor judgement, impulsivity -(DON'T LEAVE THE CLIENT AND BE ON GUARD WITH THIS PT) -short attention span and short term memory loss -communication problems (due to weakness of facial muscles) -cognitive problems "baby in a Rocker": impulsive makes decisions to quickly

Medical Management of PD

-Levodopa/carbidopa (Sinemet) is the gold standard drug therapy for PD -L-dopa (almost 99%) is metabolized before reaching the brain, requiring administration of high doses -L-dopa is commonly administered with carbidopa, a decarboxylase inhibitor that allows a higher percentage of L-dopa to enter the brain. Thus, lower doses of L-dopa can be used with fewer adverse side effects -Sinemet is available in immediate-release (IR) and controlled-release (CR) formulations

ACSM Guidelines for Cancer Pts

-Low to Mod exercise for 20-60 min -Resistive exercises at least 1 set of 12-15 reps -Focus on ADL training Contraindications for cancer rehab: -severe nausea/vomiting/diarrhea within 24-36 hours -acute infection and fever above 100F -chest pain -dizziness, light headedness, disorientation, confusion, blurred vision, and ataxia

Common signs and symptoms of GI disorders?

-Nausea/vomiting occurs from irritation of the GI tract -Diarrhea caused by infection such as Diverticulitis, IBS, HYPERthyroidism "diaRRhea—> hypeRthyRoidism" -Constipation caused by lack of fiber/fluids, HYPOthyroidism, splinting, muscle guarding, also causes referred pain to the lower abdomen, anterior hip, groin/thigh "cOnstipation-hypOthyroidism-Obese-cOld intolerance" -Abdominal pain is from inflammation, ischemia, or stretching -GI bleeding: gastritis, peptic ulcers, ulcerative colitis -Heart burn: aggravated by caffeine, spicy, or peppermint

What is DMD?

-Neuro condition that only effects boys due to missing gene that causes destruction of muscle cells -Appearance of thick calf muscles from hypertrophy of fat Duchenne muscular dystrophy (DMD) and Becker muscular dystrophy (BMD) are X-linked recessive disorders caused by mutations of the DMD gene located at Xp21. In DMD patients, dystrophin is virtually absent; whereas BMD patients have 10% to 40% of the normal amount. Goal for DMD pts: -Maintain mobility as long as possible -Maintain joint ROM with active/passive

What is Huntington's Disease (HD)?

-Neurodegenerative genetic disease -involves degeneration of the basal ganglia and cerebral cortex -an autosomal dominant illness caused by an expansion of the CAG repeats on the short arm of chromosome 4 -characterized by a combination of movement disorders, cognitive decline, and behavioral changes

Sarcoidosis

-Systemic disease of unknown cause -Triggering agent may be genetic, infectious (bacterial or viral), immunologic, or toxic -Characterized by granulomatous inflammation in nodular form -Asymptomatic state to a progressive, life-threatening condition Symptoms -Dyspnea, cough, fever, malaise, weight loss, skin lesions, erythema nodosum, dry cough, chest pain, hemoptysis, and pneumothorax -Muscle weakness, aches, tenderness, and fatigue that reduces the functional capacity

Hoehn and Yahr Stages of PD?

1 - Minimal or absent Symptoms on only one side of the body if present. (slight tremor, some stiffness and slowing of movement. 2 - Minimal bilateral/ midline involvement. Balance not impaired. Symptoms on both sides of the body and no difficulty walking (as above with speech and face masking appearing) 3 - Impaired righting reflex Symptoms on both sides of the body and minimal difficulty walking (Some activities are restricted, but patient can live independently and continue some forms of employment. ) 4 - All symptoms present and severe Symptoms on both sides of the body and moderate difficulty walking (as above with increase assist on ADL ) 5 - Symptoms on both sides of the body and unable to walk (wheelchair dependent) 3 and 4 most likely questioned on the exam

Difference between one way ANOVA vs 2 way ANOVA?

1 way ANOVA: 2 or more groups compared on 1 intervention ex: effects of stretching for anterior/posterior/lateral approach THA 2 way ANOVA: 2 or more independent groups compared on 2 intervention ex: effects of stretching and strengthening for anterior/posterior/lateral approach THA

A patient with a transtibial amputation is being treated by a physical therapist for gait training with a prosthesis. The patient reports tingling and shooting pain at the end of the residual limb. The pain occurs whether or not the patient is wearing the prosthesis. The pain is MOST likely caused by which of the following? 1. A neuroma 2. Inadequate prosthetic tibial relief 3. Distal soft-tissue adhesions 4. Osteomyelitis

1. A neuroma 1. A neuroma is a collection of axons and fibrous tissue that can cause sharp, shooting, and localized pain. Localized hypersensitivity may be an indicator that a neuroma has developed. (Chui, pp. 479, 676) 2. Inadequate prosthetic tibial relief may result in skin breakdown, which would be seen and is not indicated in the stem (Chui, p. 684). 3. Adherent scar tissue near the end of the bone is a particular problem that may lead to skin breakdown, which is not indicated in the stem (Chui, p. 676). 4. Clinical manifestation of pain with osteomyelitis may be described as deep, constant, and increasing with weight-bearing when present in the lower extremity. Patients may report local pain and swelling, which is not indicated in the stem. (Goodman, p. 1236)

A 62 year old male with a 20 year history of smoking, DM II, HTN stage 1 and COPD presents with cough and expectoration. On auscultation the therapist finds presence of crepitus. Which of the following lobes can be drained MOST EFFICIENTLY in the sitting position? Select one: a. L Apical Segment of Upper lobe and R Posterior Segment of Upper lobes b. L Apical Segment of Upper lobe and R Middle lobe c. R Apical Segments of Upper lobe and R Superior Segments of Lower lobes d. R Apical segment of Upper lobe and R Anterior segment of Upper lobe

a. L Apical Segment of Upper lobe and R Posterior Segment of Upper lobes Rationale: Apical and Posterior segments of the upper lobes can be drained in the sitting position. Anterior segment is drained in supine. Superior segment is drained in prone. R middle lobe is drained in the side lying position.

During shoulder flexion, the physical therapist observes that the scapulae is moving abnormally on the right side. To check the proper functioning of the lower trapezius, a physical therapist resists which of the following sets of movements? a. Scapular depression, adduction, and upward rotation b. Scapular depression, abduction, and downward rotation c. Scapular elevation, abduction, and upward rotation d. Scapular elevation, adduction, and downward rotation

a. Scapular depression, adduction, and upward rotation The lower trapezius muscle acts as a scapular depressor, upward rotator (along with upper trapezius) and adductor (retractor) of the scapula. So, these movements can be resisted to check the proper functioning of this muscle.

A therapist is observing a patient with a transtibial prosthesis and notices "drop off" during the late stance phase of gait. Which of the following prosthetic/anatomical causes would be the LEAST LIKELY cause of this gait deviation? Select one: a. Socket too far posterior b. Knee flexion contracture c. Insufficient plantarflexion d. Keel too short

a. Socket too far posterior Rationale: An early knee flexion during the late stance phase of gait is also known as "drop off". It occurs due to the following causes: Prosthetic causes: High shoe heel, Insufficient plantar flexion, Keel too short, Dorsiflexion stop too soft, Socket too far anterior, Socket excessively flexed, Cuff tabs too posterior. Anatomical causes: Knee flexion contracture. A socket placed too far posterior would cause delayed knee flexion.

You are evaluating a 30 year old patient with right elbow pain radiating into the forearm. The patient works as a bagger at a grocery store and reports his pain began 6 months ago after carrying a heavy box at work. The patient reports his pain is located on the medial side of the elbow and is worsened with prolonged gripping and carrying grocery bags. Which of the following diagnoses is MOST likely to be accurate for this patient? Select one: a. Tendinopathy of the wrist flexors b. Lateral epicondylalgia c. Biceps tendinopathy at proximal insertion d. Biceps tendinopathy at distal insertion

a. Tendinopathy of the wrist flexors Rationale: The patient's pain is in the medial elbow which is the anatomical location of the flexor tendon. Flexor tendinopathy is also known as "golfers elbow" and is caused by repetitive wrist flexion. Lateral epicondylalgia is a tendinopathy of the extensor tendon and biceps tendinopathy would be incorrect secondary to the location of pain specified in the question.

A physical therapist is treating a patient with a spinal cord injury in the inpatient rehab unit at the hospital. On assessment, the therapist found the following muscles of respiration to be intact: cervical extensors, upper trapezius, levator scapulae with loss of innervation to the pectoralis major, serratus anterior and latissimus dorsi. Which of the following is the MOST LIKELY neurologic level of this spinal cord injury? Select one: a. C1-C2 b. C3-C4 c. C5-C8 d. T1-T5

b. C3-C4 Rationale: -With high spinal cord lesions at C1 and C2, phrenic nerve innervation and spontaneous respiration are lost. The only muscles of respiration that are intact are accessory muscles: sternocleidomastoid, upper trapezius, and cervical extensors. -C3-C4 level injuries have partial diaphragm innervation, as well as scalenes, levator scapulae, and trapezius. -Injuries at C5-C8 have a fully innervated diaphragm, as well as many accessory muscles. Some cough ability is preserved; however, it is usually weak.

A physical therapist is reading the lab report of a patient admitted in the hospital. In which of the following scenarios would the patient be advised to NOT exercise? Select one: a. Platelet count: 22,000/mm3; Hb: 9g/dL; WBC: 6000/mm3 b. Hb: 7g/dL; Hematocrit: 23%; WBC: 4800/mm3 c. Platelet count: 29,000/mm3; Hb: 8g/dL; Hematocrit: 30% d. Hematocrit: 25%; Platelet count: 20,000/mm3; WBC: 5200/mm3

b. Hb: 7g/dL; Hematocrit: 23%; WBC: 4800/mm3 Rationale: Guidelines for exercise based on laboratory values suggest no exercises for the following findings: WBC <5000/mm3; Hematocrit <25%; Hb <8g/dL; Platelets <20,000- AAROM or AROM but no antigravity or resisted exercise.

A physical therapist who was working in an outpatient clinic is leaving the practice for better career opportunities. He is the only lymphedema certified specialist at that location and single-handedly manages all the patients with lymphedema. Which of the following will be the BEST course of action for the physical therapist toward patients requiring continued care? Select one: a. Therapist is not allowed to terminate provider relationship if the patient requires continued care b. Provide information to patients about alternative options to obtain care c. Discharge the patients before leaving d. Find a replacement for the position before leaving

b. Provide information to patients about alternative options to obtain care Rationale: Per the Code of Ethics for Physical Therapists: Principle #5: Physical therapists shall fulfill their legal and professional obligations. (Core Values: Professional Duty, Accountability). Physical therapists shall provide notice and information about alternatives for obtaining care in the event the physical therapist terminates the provider relationship while the patient or client continues to need physical therapist services.

During exams week, the physical therapist observes the posture of a 29-year-old female patient. The PT notices that she has a forward head posture. Which of the following characterizes forward head posture? a. Extension of lower cervical spine b. Flexion of upper cervical spine c. Increase in the resistance moment arm of the head d. Decrease in the resistance moment arm of the head

c. Increase in the resistance moment arm of the head With a forward head posture, the patient has his lower cervical spine flexed along with upper cervical extension. This causes the head to translate anteriorly and the load of the weight of the head (load arm) on the long cervical extensors increases resulting in an increase in the external moment arm (resistance arm). (The external moment arm is the perpendicular distance between the axis of rotation and the external force.)

During a postural screen, the therapist observes medial winging of the scapulae during a push up motion. The winging of the scapulae is MOSTLY due to a. Weakness of upper trapezius b. Shortening of middle and lower trapezius c. Weakness of serratus anterior d. Shortening of rhomboids

c. Weakness of serratus anterior If the serratus anterior is weak or paralyzed, the winging of the scapula occurs on abduction and forward flexion (especially with a "punch out" forward against resistance). It causes the scapula to elevate and move medially with the inferior angle rotating medially. If the trapezius is weak or paralyzed, the winging of the scapula occurs before 90° abduction, and there is little winging on forward flexion. It causes the scapula to depress and move laterally with the inferior angle rotated laterally. Which nerve is it innervated by?

Which of the following is LEAST LIKELY to be present in a patient with congestive heart failure? Select one: a. Weight gain b. Orthopnea c. Weight loss d. Peripheral edema

c. Weight loss Rationale: Weight gain, orthopnea (shortness of breath in the supine position) and peripheral edema are all signs consistent with Congestive heart failure.

5 Glands of the endocrine system?

• Hypothalamus responsible for regulation of the ANS (body temperature, appetite, sweating, thirst, sexual behavior, rage, fear, blood pressure, sleep) "Hypothalamus: HIPPO-THE-LLAMAS. Think of a FEARFUL HIPPOPOTAMUS spraying 2 THIRSTY LLAMAS with water to quench their THIRST and COOL THEM DOWN in the ANS jungle (ANS = Autonomic Nervous System). Meaning = Hypothalamus regulates ANS (body temperature, thirst, sweating, fear just to name a few)" • Pituitary gland secretes endorphins and reduces a person's sensitivity to pain. It controls ovulation and works as a catalyst for the testes and ovaries to create sex hormones "Pituitary Gland: When you're in the "PIT" fighting versus the NPTE, you will cause that NPTE PAIN. You also don't want to go into the "PIT" versus a sexually hormonal "CAT" (CATalyst) because that can be quick to attack and are dangerous. Meaning = Pituitary gland reduces sensitivity to pain, works as CATalyst for creation of sex hormone." • Thyroid gland produces hormones that act to control the rate at which cells burn the fuel from food "Chicken thy(roid) = food/fuel" • Parathyroid gland regulate calcium and phosphate metabolism "drink your parathyroid to improve your bones!" • Adrenal gland produces corticosteroids that will regulate water and sodium balance, the body's response to stress, the immune system, and metabolism. "Adrenal: AD for STEROIDS (corticosteroids) that you need to drink with SALT WATER (regulates water and sodium balance) which ultimately helps you in the gym to improve STRESS, IMMUNE SYSTEM, and METABOLISM"


Set pelajaran terkait

Legal Environment of Business Unit 7 Test

View Set

Ch.20 The Cardiovascular System: The Heart

View Set

Unit 3 Quiz - Biological Bases of Behavior (Unit 2 on My AP Classroom)

View Set

Mastering Biology Chapter 7, CHAPTER 7: Membranes & Cell Transport, Chapter 7 Practice Quiz, Ch. 7 Membrane Structure and Function

View Set

Chapter 15 - Anti discrimination Laws and Other Marketing Regulations

View Set

Managerial economics Salvatore 8th Edition, chapter 1

View Set

2. Exercise for Exploring Linux Command-Line Tools, Linux Essentials 010-150 Part 2, Chapter 14, Linux Ch 7, 1.12 Regular Expressions, 211 Final, Quiz 2 (Linux), LINUX TEST 3, Chapter 10: Editing Files (Review Questions), Chapter 9, Liinux Study, Hom...

View Set